You are on page 1of 118

INSURANCE Page 1

ABOITIZ SHIPPING CORPORATION V NEW - Petitioner elevated the case to the Court of presumed to be negligent since it is tasked with the
INDIA ASSURANCE COMPANY, LTD. Appeals, which in turn, affirmed in toto the trial maintenance of its vessel. Though this duty can be
[CITATION] court’s decision. delegated, still, the shipowner must exercise close
QUISUMBING; May 2, 2006 - Petitioner moved for reconsideration but the same supervision over its men.
was denied. - In the present case, petitioner has the burden of
NATURE - Hence, this petition for review showing that it exercised extraordinary diligence in
Petition for review on certiorari Petitioner’s Claim the transport of the goods it had on board in order to
> Petitioner contends that respondent’s claim for invoke the limited liability doctrine. Differently put,
FACTS damages should only be against the insurance to limit its liability to the amount of the insurance
- Societe Francaise Des Colloides loaded a cargo of proceeds and limited to its pro-rata share in view of proceeds, petitioner has the burden of proving that
textiles and auxiliary chemicals from France on board the doctrine of limited liability the unseaworthiness of its vessel was not due to its
a vessel owned by Franco-Belgian Services, Inc. Respondent’s Comments fault or negligence.
- The cargo was consigned to General Textile, Inc., > Respondent counters that the doctrine of real and - Considering the evidence presented and the
in Manila and insured by respondent New India hypothecary nature of maritime law is not applicable circumstances obtaining in this case, we find that
Assurance Company, Ltd. in the present case because petitioner was found to petitioner failed to discharge this burden. Both the
- While in Hongkong, the cargo was transferred to have been negligent. Hence, according to trial and the appellate courts, in this case, found that
M/V P. Aboitiz for transshipment to Manila. respondent, petitioner should be held liable for the the sinking was not due to the typhoon but to its
- Before departing, the vessel was advised that it total value of the lost cargo unseaworthiness. Evidence on record showed that
was safe to travel to its destination, but while at sea, the weather was moderate when the vessel sank.
the vessel received a report of a typhoon moving ISSUE These factual findings of the Court of Appeals,
within its path. WON the limited liability doctrine applies in this case affirming those of the trial court are not to be
- To avoid the typhoon, the vessel changed its disturbed on appeal, but must be accorded great
course. However, it was still at the fringe of the HELD weight. These findings are conclusive not only on
typhoon when its hull leaked. NO the parties but on this Court as well.
- On October 31, 1980, the vessel sank, but the Ratio Where the shipowner fails to overcome the Disposition Petition is denied for lack of merit.
captain and his crew were saved. presumption of negligence, the doctrine of limited
- On November 3, 1980, the captain of M/V P. liability cannot be applied. PHILAMCARE HEALTH SYSTEMS, INC. V CA
Aboitiz filed his “Marine Protest”, stating that the (TRINOS)
wind force was at 10 to 15 knots at the time the ship Reasoning 379 SCRA 357
foundered and described the weather as “moderate - From the nature of their business and for reasons YNARES-SANTIAGO; March 18, 2002
breeze, small waves, becoming longer, fairly of public policy, common carriers are bound to
frequent white horse observe extraordinary diligence over the goods they NATURE
- Petitioner notified the consignee of the total loss of transport according to all the circumstances of each Petition for review of CA decision
the vessel and all of its cargoes. case. In the event of loss, destruction or
- General Textile, lodged a claim with respondent for deterioration of the insured goods, common carriers FACTS
the amount of its loss. are responsible, unless they can prove that the loss, - Ernani TRINOS, deceased husband of respondent
- Respondent paid General Textile and was destruction or deterioration was brought about by Julita, applied for a health care coverage with
subrogated to the rights of the latter. the causes specified in Article 17341 of the Civil Philamcare Health Systems, Inc. In the standard
- Respondent hired a surveyor to investigate, and Code. In all other cases, common carriers are application form, he answered no to the question:
the same concluded that the cause was the flooding presumed to have been at fault or to have acted “Have you or any of your family members ever
of the holds brought about by the vessel’s negligently, unless they prove that they observed consulted or been treated for high blood pressure,
questionable seaworthiness. extraordinary diligence. Moreover, where the vessel heart trouble, diabetes, cancer, liver disease,
- Respondent filed a complaint for damages against is found unseaworthy, the shipowner is also asthma or peptic ulcer? (If Yes, give details).”
petitioner Aboitiz, Franco-Belgian Services and the - The application was approved for period of one
latter’s local agent, F.E. Zuellig, Inc. (Zuellig) 1 year; upon termination, it was extended for another
Art. 1734. Common carriers are responsible for the loss, destruction,
- On November 20, 1989, the trial court ruled in 2 years. Amount of coverage was increased to a
or deterioration of the goods, unless the same is due to any of the
favor of respondent and held petitioner Aboitiz liable following causes only: maximum sum of P75T per disability.
for the total value of the lost cargo plus legal interest (1) Flood, storm, earthquake, lightning, or other natural disaster or - During this period, Ernani suffered a HEART
calamity;
- The complaint with respect to Franco and Zuellig (2) Act of the public enemy in war, whether international or civil;
ATTACK and was confined at the Manila Medical
was dismissed (3) Act of omission of the shipper or owner of the goods; Center (MMC) for one month. While her husband was
(4) The character of the goods or defects in the packing or in the in the hospital, Julita tried to claim the
containers;
(5) Order or act of competent public authority. hospitalization benefits.
INSURANCE Page 2

- Petitioner treated the Health Care Agreement Ratio Every person has an insurable interest in the (b) Notice must be based on the occurrence after
(HCA) as void since there was a concealment life and health of himself2. The health care effective date of the policy of one or more of the
regarding Ernani’s medical history. Doctors at agreement was in the nature of non-life insurance, grounds mentioned;
the MMC allegedly discovered at the time of his which is primarily a contract of indemnity. Once the (c) Must be in writing, mailed or delivered to the
confinement, he was hypertensive, diabetic and member incurs hospital, medical or any other insured at the address shown in the policy;
asthmatic. Julita then paid the hospitalization expense arising from sickness, injury or other (d) Must state the grounds relied upon provided in
expenses herself, amounting to about P76T. stipulated contingent, the health care provider must Section 64 of the Insurance Code and upon request
- After her husband died, Julita instituted action for pay for the same to the extent agreed upon under of insured, to furnish facts on which cancellation is
damages against Philamcare and its Pres. After the contract. based.
trial, the lower court ruled in her favor and ordered Reasoning - These conditions have not been met. When the
Philamcare to reimburse medical and hospital - A contract of insurance3 is an agreement whereby terms of insurance contract contain limitations on
coverage amounting to P76T plus interest, until fully one undertakes for a consideration to indemnify liability, courts should construe them in such a way
paid; pay moral damages of P10T; pay exemplary another against loss, damage or liability arising from as to preclude insurer from non-compliance of
damages of P10T; atty’s fees of P20T. an unknown or contingent event. obligation. Being a contract of adhesion, terms of an
- CA affirmed the decision of the trial court but - An insurance contract exists where the following insurance contract are to be construed strictly
deleted all awards for damages and absolved elements concur: against the party which prepared it – the insurer.
petitioner Reverente. (a) The insured has an insurable interest; - Also, Philamcare had 12 months from the date of
Petitioner’s Claims (b) The insured is subject to a risk of loss by the issuance of the Agreement within which to contest
(1) Agreement grants “living benefits” such as happening of the peril; the membership of the patient if he had previous
medical check-ups and hospitalization which a (c) The insurer assumes the risk; ailment of asthma, and six months from the issuance
member may immediately enjoy so long as he is (d) Such assumption of risk is part of a general of the agreement if the patient was sick of diabetes
alive upon effectivity of the agreement until its scheme to distribute actual losses among a large or hypertension.
expiration. group of persons bearing a similar risk; and * The health care agreement is in the nature of a
(2) Only medical and hospitalization benefits are (e) In consideration of the insurer’s promise, the contract of indemnity. Hence, payment should be
given under the agreement without any insured pays a premium. made to the party who incurred the expenses. It is
indemnification, unlike in an insurance contract 2. NO clear that respondent paid all the hospital and
where the insured is indemnified for his loss. Ratio Where matters of opinion or judgment are medical bills; thus, she is entitled to reimbursement.
(3) HCAs are only for a period of one year; called for, answers made in good faith and without Disposition Petition DENIED.
therefore, incontestability clause does not apply, as intent to deceive will not avoid a policy even though
it requires effectivity period of at least 2 yrs. they are untrue; since in such case the insurer is not PINEDA V CA (INSULAR LIFE INSURANCE
(4) It is not an insurance company, governed by justified in relying upon such statement, but is COMPANY)
Insurance Commission, but a Health Maintenance obligated to make further inquiry. 226 SCRA 755
Organization under the authority of DOH. Reasoning DAVIDE; September 27, 1993
(5) Trinos concealed a material fact in his - The fraudulent intent on the part of the insured
application. must be established to warrant rescission of the NATURE
(6) Julita was not the legal wife since at the time insurance contract. The right to rescind should be Appeal by certiorari for review and set aside the
of their marriage, the deceased was previously exercised previous to the commencement of an Decision of the public respondent Court of Appeals
married to another woman who was still alive.* action on the contract. No rescission was made. and its Resolution denying the petitioners' motion for
Besides, the cancellation of health care agreements reconsideration
ISSUES as in insurance policies requires:
1. WON a health care agreement is an insurance (a) Prior notice of cancellation to insured; FACTS
contract (If so, “incontestability clause” under the - In 1983, Prime Marine Services, Inc. (PMSI)
Insurance Code is applicable) procured a group policy from Insular Life to provide
2. WON the HCA can be invalidated on the basis of 2 life insurance coverage to its sea-based employees
alleged concealment Sec.10. Every person has an insurable interest in the life and enrolled under the plan. During the effectivity of the
health:
(1) of himself, of his spouse and of his children;
policy, 6 covered employees perished at sea. They
HELD (2) of any person on whom he depends wholly or in part for education were survived by complainants-appellees, the
YES or support, or in whom he has a pecuniary interest; beneficiaries under the policy.
(3) of any person under a legal obligation to him for the payment of
money, respecting property or service, of which death or illness might - complainants-appellees sought to claim death
delay or prevent the performance; and benefits due them and approached Capt. Roberto
(4) of any person upon whose life any estate or interest vested in him
depends.
Nuval, President and GM of PMSI, then executed
3 special powers of attorney authorizing Capt. Nuval to
Section 2 (1) of the Insurance Code
INSURANCE Page 3

“follow up, ask, demand, collect and receive” for of Ayo and LOntok, even if no bond was posted as - the labor of the employees is the true source
their benefit indemnities of sums of money due required of the benefits, which are a form of additional
them…” compensation to them.
- Unknown to the complainants, PMSI filed with HELD - the employer is the agent of the insurer in
Insular Life claims for and in behalf of them through 1. YES performing the duties of administering group
Capt. Nuval, even using the 5 special powers of Ratio Third persons deal with agents at their peril insurance policies. It cannot be said that the
attorney that they executed as documents. Insular and are bound to inquire as to the extent of the employer acts entirely for its own benefit or for the
Life then released 6 checks, payable to the order of power of the agent with whom they contract. The benefit of its employees in undertaking
the complainant-appellees, to the treasurer of PMSI person dealing with an agent must also act administrative functions. While a reduced premium
(who happened to be Capt. Nuval’s son-in-law). with ordinary prudence and reasonable may result if the employer relieves the insurer of
Capt. Nuval then endorsed and deposited these diligence. Obviously, if he knows or has good these tasks, and this, of course, is advantageous to
checks (which were for the complainants) in his bank reason to believe that the agent is exceeding his both the employer and the employees, the insurer
account. authority, he cannot claim protection. So if the also enjoys significant advantages from the
- 3 years after, the complainants-appellees found out suggestions of probable limitations be of such a clear arrangement. The reduction in the premium which
that they were entitled, as beneficiaries, to life and reasonable quality, or if the character assumed results from employer-administration permits the
insular benefits under a group policy with by the agent is of such a suspicious or unreasonable insurer to realize a larger volume of sales, insurer to
respondent-appellant so they sought to recover nature, or if the authority which he seeks to exercise realize a larger volume of sales, and at the same
these benefits from Insular Life. Insular Life denied is of such an unusual or improbable character, as time the insurer's own administrative costs are
the claim, saying that their liability to complainants would suffice to put an ordinarily prudent man upon markedly reduced.
was already extinguished upon delivery to and his guard, the party dealing with him may not shut - the employee has no knowledge of or control over
receipt by PMSI of the 6 checks issued in the his eyes to the real state of the case, but should the employer's actions in handling the policy or its
complainants’ names. Complainants filed case with either refuse to deal with the agent at all, or should administration. An agency relationship is based upon
the Insurance Commission which decided in their ascertain from the principal the true condition of consent by one person that another shall act in his
favor. affairs. behalf and be subject to his control. It is clear from
- Insurance Commission held that the special powers Reasoning the evidence regarding procedural techniques here
of attorney executed by complainants in favor of the - The execution by the principals of special powers of that the insurer-employer relationship meets this
complainants do not contain in unequivocal and clear attorney, which clearly appeared to be in prepared agency test with regard to the administration of the
terms authority to Capt. Nuval to obtain, receive, forms and only had to be filled up with their names, policy, whereas that between the employer and its
receipt from respondent company insurance residences, dates of execution, dates of employees fails to reflect true agency. The insurer
proceeds arising from the death of the seaman- acknowledgement and others, excludes any intent to directs the performance of the employer's
insured; also, that Insular Life did not convincingly grant a general power of attorney or to constitute a administrative acts, and if these duties are not
refuted the claim of Mrs. Alarcon that neither she nor universal agency. Being special powers of attorney, undertaken properly the insurer is in a position to
her husband executed a special power of authority in they must be strictly construed. Insular Life knew exercise more constricted control over the
favor of Capt. Nuval (and therefore, the company that a power of attorney in favor of Capt. Nuval for employer's conduct.
should have not released the check to Capt. Nuval- the collection and receipt of such proceeds was a - ON GROUP INSURANCE: Group insurance is
PMSI); and that it did not observe Sec 180(3), as deviation from its practice with respect to group essentially a single insurance contract that
repealed by Art. 225 of the Family Code, when it policies (that the employer-policyholder is the agent provides coverage for many individuals. In its
released the benefits due to the minor children of of the insurer). original and most common form, group insurance
Ayo and Lontok, when the said complainants did not - The employer acts as a functionary in the collection provides life or health insurance coverage for
post a bond as required and payment of premiums and in performing related the employees of one employer. The coverage
- Insular Life appealed to the CA; CA modified the duties. Likewise falling within the ambit of terms for group insurance are usually stated in a
decision of the Insurance Commission, eliminating administration of a group policy is the disbursement master agreement or policy that is issued by the
the award to the Lontoks and Ayo of insurance payments by the employer to the insurer to a representative of the group or to an
employees. Most policies, such as the one in this administrator of the insurance program, such as an
ISSUES case, require an employee to pay a portion of the employer. Although the employer may be the titular
1. WON Insular Life should be liable to the premium, which the employer deducts from wages or named insured, the insurance is actually
complainants when they relied on the special powers while the remainder is paid by the employer. This is related to the life and health of the employee.
of attorney, which Capt. Nuval presented as known as a contributory plan as compared to a Indeed, the employee is in the position of a real
documents, when they released the checks to the non-contributory plan where the premiums are solely party to the master policy, and even in a non-
latter paid by the employer. contributory plan, the payment by the employer of
2. WON Insular Life should be liable to the the entire premium is a part of the total
complainants when they released the check in favor compensation paid for the services of the employee.
INSURANCE Page 4

2. YES CEBU SHIPYARD ENGINEERING WORKS, INC. V shall itself be subject to the following overriding
Ratio Regardless of the value of the unemancipated WILLIAM LINES, INC. and PRUDENTIAL limitations and exceptions, namely:
common child's property, the father and mother ipso GUARANTEE and ASSURANCE COMPANY, INC. (a) The total liability of the Contractor to the
jure become the legal guardian of the child's [CITATION] Customer (over and above the liability to replace
property. However, if the market value of the PURISIMA; May 5, 1999 under Clause 10) or of any sub-contractor shall be
property or the annual income of the child exceeds limited in respect of any defect or event (and a
P50,000,00, a bond has to be posted by the parents NATURE series of accidents arising out of the same defect or
concerned to guarantee the performance of the Petition for review on certiorari event shall constitute one defect or event) to the
obligations of a general guardian. sum of Pesos Philippine Currency One Million only.
Reasoning FACTS x x x
- Sec 180, Insurance Code: 'In the absence of a - Cebu Shipyard and Engineering Works, Inc. 20. The insurance on the vessel should be
judicial guardian, the father, or in the latter's (CSEW) is engaged in the business of dry-docking maintained by the customer and/or owner of the
absence or incapacity, the mother of any minor, who and repairing of marine vessels while the Prudential vessel during the period the contract is in effect.”
is an insured or a beneficiary under a contract of life, Guarantee and Assurance, Inc. (Prudential) is in the - While the M/V Manila City was undergoing dry-
health or accident insurance, may exercise, in behalf non-life insurance business. docking and repairs within the premises of CSEW,
of said minor, any right under the policy, without - William Lines, Inc. is in the shipping business. It the master, officers and crew of M/V Manila City
necessity of court authority or the giving of a bond was the owner of M/V Manila City, a luxury stayed in the vessel, using their cabins as living
where the interest of the minor in the particular act passenger-cargo vessel, which caught fire and sank quarters. Other employees hired by William Lines to
involved does not exceed twenty thousand pesos " on Feb. 16, 1991. At the time of the unfortunate do repairs and maintenance work on the vessel were
- …repealed by Art. 225, Family Code: "ART. 225. occurrence sued upon, subject vessel was insured also present during the dry-docking.
The father and the mother shall jointly exercise legal with Prudential for P45M for hull and machinery. The - On February 16, 1991, after subject vessel was
guardianship over the property of their Hull Policy included an “Additional Perils transferred to the docking quay, it caught fire and
unemancipated common child without the necessity (INCHMAREE)” Clause covering loss of or damage to sank, resulting to its eventual total loss.
of a court appointment. In case of disagreement, the the vessel through the negligence of, among others, - On February 21, 1991, William Lines, Inc. filed a
father's decision shall prevail, unless there is judicial ship repairmen complaint for damages against CSEW, alleging that
order to the contrary. - Petitioner CSEW was also insured by Prudential for the fire which broke out in M/V Manila City was
Where the market value of the property or the third party liability under a Shiprepairer’s Legal caused by CSEW’s negligence and lack of care.
annual income of the child exceeds P50,000, the Liability Insurance Policy. The policy was for P10 - On July 15, 1991 was filed an Amended Complaint
parent concerned shall be required to furnish a bond million only, under the limited liability clause, to wit: impleading Prudential as co-plaintiff, after the latter
in such amount as the court may determine, but not - On Feb. 5, 1991, William Lines, Inc. brought its had paid William Lines, Inc. the value of the hull and
less than ten per centum (10%) of the value of the vessel, M/V Manila City, to the Cebu Shipyard in machinery insurance on the M/V Manila City. As a
property or annual income, to guarantee the Lapulapu City for annual dry-docking and repair. result of such payment Prudential was subrogated to
performance of the obligations prescribed for general - On Feb. 6, 1991, an arrival conference was held the claim of P45 million, representing the value of
guardians." between representatives of William Lines, Inc. and the said insurance it paid.
-"market value of the property or the annual CSEW to discuss the work to be undertaken on the On June 10, 1994, the trial court a quo came out
income of the child": the aggregate of the child's M/V Manila City. The contracts, denominated as with a judgment against CSEW:
property or annual income; if this exceeds Work Orders, were signed thereafter., with the 1. To pay unto plaintiff Prudential Guarantee and
P50,000.00, a bond is required - There is no following stipulations: Assurance, Inc., the subrogee, the amount of P45M,
evidence that the share of each of the minors in the “10. The Contractor shall replace at its own work and with interest at the legal rate until full payment is
proceeds of the group policy in question is the at its own cost any work or material which can be made; the amount of P56,715,000 representing loss
minor's only property. Without such evidence, it shown to be defective and which is communicated in of income of M/V MANILA CITY, with interest at the
would not be safe to conclude that, indeed, that is writing within one (1) month of redelivery of the legal rate until full payment is made;
his only property. vessel or if the vessel was not in the Contractor’s 2. To pay unto plaintiff, William Lines, Inc. the
Disposition the instant petition is GRANTED. The Possession, the withdrawal of the Contractor’s amount of P11M as payment, in addition to what it
Decision of 10 October 1991 and the Resolution of workmen, or at its option to pay a sum equal to the received from the insurance company to fully cover
19 May 1992 of the public respondent in CA-G.R. SP cost of such replacement at its own works. These the injury or loss, in order to replace the M/V
No. 22950 are SET ASIDE and the Decision of the conditions shall apply to any such replacements. MANILA CITY, with interest at the legal rate until full
Insurance Commission in IC Case No. RD-058 is 11. Save as provided in Clause 10, the Contractor payment is made; the sum of P927,039 for the loss
REINSTATED. Costs against the private respondent. shall not be under any liability to the Customer of fuel and lub oil on board the vessel when she was
SO ORDERED. either in contract or for delict or quasi-delict or completely gutted by fire at defendant, Cebu
otherwise except for negligence and such liability Shipyard’s quay, with interest at the legal rate until
full payment is made; the sum of P3,054,677.95 as
INSURANCE Page 5

payment for the spare parts and materials used in - When M/V Manila City capsized, steel and angle - The that factual findings by the CA are conclusive
the M/V MANILA CITY during dry-docking with bars were noticed to have been newly welded along on the parties and are not reviewable by this Court.
interest at the legal rate until full payment is made; the port side of the hull of the vessel, at the level of They are entitled to great weight and respect, even
P500,000 in moral damages;the amount of P10Min the crew cabins. William Lines did not previously finality, especially when, as in this case, the CA
attorney’s fees; and to pay the costs of this suit. apply for a permit to do hotworks on the said portion affirmed the factual findings arrived at by the trial
- On September 3, 1997, the Court of Appeals of the ship as it should have done pursuant to its court. When supported by sufficient evidence,
affirmed the appealed decision of the trial court, work order with CSEW. findings of fact by the CA affirming those of the trial
ordering CSEW to pay Prudential, the subrogee, the Prudential’s version court, are not to be disturbed on appeal. The
sum of P45 Million, with interest at the legal rate > At around 7AM of Feb. 16, 1991, the Chief Mate of rationale behind this doctrine is that review of the
until full payment is made. M/V Manila City was inspecting the various works findings of fact of the CA is not a function that the
CSEW’s version: being done by CSEW on the vessel, when he saw Supreme Court normally undertakes.
On Feb. 13, 1991, the CSEW completed the that some workers of CSEW were cropping out steel - The CA and the Cebu RTC are agreed that the fire
drydocking of M/V Manila City at its grave dock. It plates on Tank Top No. 12 using acetylene, oxygen which caused the total loss of subject M/V Manila
was then transferred to the docking quay of CSEW and welding torch. He also observed that the rubber City was due to the negligence of the employees and
where the remaining repair to be done was the insulation wire coming out of the air-conditioning workers of CSEW. Both courts found that the M/V
replating of the top of Water Ballast Tank No. 12 unit was already burning, prompting him to scold the Manila City was under the custody and control of
which was subcontracted by CSEW to JNB General workers. petitioner CSEW, when the ill-fated vessel caught
Services. Tank Top No. 12 was at the rear section of > At 2:45 PM of the same day, witnesses saw smoke fire. The decisions of both the lower court and the CA
the vessel, on level with the flooring of the crew coming from Tank No. 12. The vessel’s reeferman set forth clearly the evidence sustaining their finding
cabins located on the vessel’s second deck. reported such occurence to the Chief Mate who of actionable negligence on the part of CSEW. This
At around 7AM of Feb. 16, 1991, the JNB workers immediately assembled the crew members to put factual finding is accorded great weight and is
trimmed and cleaned the tank top framing which out the fire. When it was too hot for them to stay on conclusive on the parties. The court discerns no basis
involved minor hotworks (welding/cutting works). board and seeing that the fire cannot be controlled, for disturbing such finding firmly anchored on
The said work was completed at about 10AM. The the vessel’s crew were forced to withdraw from enough evidence.
JNB workers then proceeded to rig the steel plates, CSEW’s docking quay. - Furthermore, in petitions for review on certiorari,
after which they had their lunch break. The rigging - In the morning of Feb. 17, 1991, M/V Manila City only questions of law may be put into issue.
was resumed at 1PM sank. As the vessel was insured with Prudential Questions of fact cannot be entertained. The finding
While in the process of rigging the second steel Guarantee, William Lines filed a claim for of negligence by the CA is a question which this
plate, the JNB workers noticed smoke coming from constructive total loss, and after a thorough Court cannot look into as it would entail going into
the passageway along the crew cabins. When one of investigation of the surrounding circumstances of the factual matters on which the finding of negligence
the workers, Mr. Casas, proceeded to the tragedy, Prudential found the said insurance claim to was based. Such an approach cannot be allowed by
passageway to ascertain the origin of the smoke, he be meritorious and issued a check in favor of this Court in the absence of clear showing that the
noticed that smoke was gathering on the ceiling of William Lines in the amount of P45 million pesos case falls under any of the exceptions to the well-
the passageway but did not see any fire as the crew representing the total value of M/V Manila City’s hull established principle.
cabins on either side of the passageway were locked. and machinery insurance. The finding by the trial court and the Court of
He immediately sought out the proprietor of JNB, Appeals that M/V Manila City caught fire and sank by
Mr. Buenavista, and the Safety Officer of CSEW, Mr. ISSUES reason of the negligence of the workers of CSEW,
Aves, who sounded the fire alarm. CSEW’s fire 1. WON CSEW had “management and supervisory when the said vessel was under the exclusive
brigade immediately responded as well as the other control“ of the m/v manila city at the time the fire custody and control of CSEW is accordingly upheld.
fire fighting units in Metro Cebu. However, there broke out 2. YES
were no WLI representative, officer or crew to guide 2. WON the doctrine of res ipsa loquitur applies - For the doctrine of res ipsa loquitur to apply to a
the firemen inside the vessel. against the crew given situation, the following conditions must
- Despite the combined efforts of the firemen of the 3. WON CSEW’S expert evidence is admissible or of concur: (1) the accident was of a kind which does
Lapulapu City Fire Dept., Mandaue Fire Dept., probative value not ordinarily occur unless someone is negligent;
Cordova Fire Dept. Emergency Rescue Unit 4. WON Prudential has the right of subrogation and (2) that the instrumentality or agency which
Foundation, and fire brigade of CSEW, the fire was against its own insured THE CONTRACTUAL 5. 5. 5. caused the injury was under the exclusive control of
not controlled until 2AM of the following day. 5. WON the provisions limiting csew’s liability for the person charged with negligence.
- On the early morning of Feb. 17, 1991, gusty winds negligence to a maximum of p1 million are valid The facts and evidence on record reveal the
rekindled the flames on the vessel and fire again concurrence of said conditions in the case under
broke out. Then the huge amounts of water pumped HELD scrutiny. First, the fire that occurred and consumed
into the vessel, coupled with the strong current, 1. YES M/V Manila City would not have happened in the
caused the vessel to tilt until it capsized and sank ordinary course of things if reasonable care and
INSURANCE Page 6

diligence had been exercised. In other words, some during the fire, the latter are of more probative 20. The insurance on the vessel should be
negligence must have occurred. Second, the agency value. maintained by the customer and/or owner of the
charged with negligence, as found by the trial court vessel during the period the contract is in effect.
and the CA and as shown by the records, is the - According to petitioner, under the aforecited
herein petitioner, CSEW, which had control over clause, William Lines, Inc., agreed to assume the risk
subject vessel when it was docked for annual repairs. 4. YES of loss of the vessel while under drydock or repair
So also, as found by the RTC, “other responsible - Petitioner contends that Prudential is not entitled to and to such extent, it is benefited and effectively
causes, including the conduct of the plaintiff, and be subrogated to the rights of William Lines, Inc., constituted as a co-assured under the policy.
third persons, are sufficiently eliminated by the theorizing that (1) the fire which gutted M/V Manila - This theory of petitioner is devoid of sustainable
evidence.” City was an excluded risk and (2) it is a co-assured merit. Clause 20 of the Work Order in question is
What is more, in the present case the trial court under the Marine Hull Insurance Policy. clear in the sense that it requires William Lines to
found direct evidence to prove that the workers - It is petitioner’s submission that the loss of M/V maintain insurance on the vessel during the period of
and/or employees of CSEW were remiss in their duty Manila City or damage thereto is expressly excluded dry-docking or repair. Concededly, such a
of exercising due diligence in the care of subject from the coverage of the insurance because the stipulation works to the benefit of CSEW as the
vessel. The direct evidence substantiates the same resulted from “want of due diligence by the shiprepairer. However, the fact that CSEW benefits
conclusion that CSEW was really negligent. Thus, Assured, Owners or Managers” which is not included from the said stipulation does not automatically
even without applying the doctrine of res ipsa in the risks insured against. Again, this theory of make it as a co-assured of William Lines. The
loquitur, in light of the direct evidence on record, the petitioner is bereft of any factual or legal basis. It intention of the parties to make each other a co-
ineluctable conclusion is that CSEW was negligent proceeds from a wrong premise that the fire which assured under an insurance policy is to be gleaned
and consequently liable for damages to the gutted subject vessel was caused by the negligence principally from the insurance contract or policy itself
respondent, William Lines, Inc. of the employees of William Lines, Inc. To repeat, and not from any other contract or agreement
3. NO the issue of who between the parties was negligent because the insurance policy denominates the
- Petitioner maintains that the CA erred in has already been resolved against CSEW. Upon proof assured and the beneficiaries of the insurance. The
disregarding the testimonies of the fire experts, of payment by Prudential to William Lines, Inc., the hull and machinery insurance procured by William
Messrs. David Grey and Gregory Michael Southeard, former was subrogated to the right of the latter to Lines, Inc. from Prudential named only “William
who testified on the probable origin of the fire in M/V indemnification from CSEW. As aptly ruled by the Lines, Inc.” as the assured. There was no
Manila City. Petitioner avers that since the said fire Court of Appeals, the law on the matter is succinct manifestation of any intention of William Lines, Inc.
experts were one in their opinion that the fire did not and clear, to wit: to constitute CSEW as a co-assured under subject
originate in the area of Tank Top No. 12 where the - Art. 2207. If the plaintiff’s property has been policy. It is axiomatic that when the terms of a
JNB workers were doing hotworks but on the crew insured, and he has received indemnity from the contract are clear its stipulations control.i] Thus,
accommodation cabins on the portside No. 2 deck, insurance company for the injury or loss arising out when the insurance policy involved named only
the RTC and the CA should have given weight to of the wrong or breach of contract complained of, William Lines, Inc. as the assured thereunder, the
such finding based on the testimonies of fire experts; the insurance company shall be subrogated to the claim of CSEW that it is a co-assured is unfounded.
petitioner argues. rights of the insured against the wrongdoer or the - Then too, in the Additional Perils Clause of the
But courts are not bound by the testimonies of person who has violated the contract. If the amount same Marine Insurance Policy, it is provided that:
expert witnesses. Although they may have probative paid by the insurance company does not fully cover Subject to the conditions of this Policy, this
value, reception in evidence of expert testimonies is the injury or loss, the aggrieved party shall be insurance also covers loss of or damage to vessel
within the discretion of the court, under Section 49, entitled to recover the deficiency from the person directly caused by the following:
Rule 130 of the Revised Rules of Court. It is never causing the loss or injury. xxx
mandatory for judges to give substantial weight to - Thus, when Prudential, after due verification of the Negligence of Charterers and/or Repairers, provided
expert testimonies. If from the facts and evidence on merit and validity of the insurance claim of William such Charterers and/or Repairers are not an
record, a conclusion is readily ascertainable, there is Lines, Inc., paid the latter the total amount covered Assured hereunder.
no need for the judge to resort to expert opinion by its insurance policy, it was subrogated to the right - As correctly pointed out by respondent Prudential,
evidence. In the case under consideration, the of the latter to recover the insured loss from the if CSEW were deemed a co-assured under the policy,
testimonies of the fire experts were not the only liable party, CSEW. it would nullify any claim of William Lines, Inc. from
available evidence on the probable cause and origin - Petitioner theorizes further that there can be no Prudential for any loss or damage caused by the
of the fire. There were witnesses who were actually right of subrogation as it is deemed a co-assured negligence of CSEW. Certainly, no shipowner would
on board the vessel when the fire occurred. under the subject insurance policy. To buttress its agree to make a shiprepairer a co-assured under
Between the testimonies of the fire experts who stance that it is a co-assured, petitioner placed such insurance policy; otherwise, any claim for loss
merely based their findings and opinions on reliance on Clause 20 of the Work Order which or damage under the policy would be invalidated.
interviews and the testimonies of those present states: Such result could not have been intended by William
Lines, Inc.
INSURANCE Page 7

5. NO Respondent’s comments
- Although in this jurisdiction, contracts of adhesion FACTS > Western Guaranty Corporation through Claims
have been consistently upheld as valid per se; as - The antecedents of this case show that Julian Sy Manager Bernard S. Razon told the plaintiff that his
binding as an ordinary contract, the Court recognizes and Jose Sy Bang have formed a business claim 'is denied for breach of policy conditions.'
instances when reliance on such contracts cannot be partnership in the City of Lucena. Under the business Reliance Insurance purveyed the same message as
favored especially where the facts and circumstances name of New Life Enterprises, the partnership well as Equitable Insurance Corporation.
warrant that subject stipulations be disregarded. engaged in the sale of construction materials at its - The said policy in question follows:
Thus, in ruling on the validity and applicability of the place of business, a two storey building situated at "The insured shall give notice to the Company of any
stipulation limiting the liability of CSEW for Iyam, Lucena City. The facts show that Julian Sy insurance or insurances already effected, or which
negligence to P1M only, the facts and circumstances insured the stocks in trade of New Life Enterprises may subsequently be effected, covering any of the
vis-a-vis the nature of the provision sought to be with Western Guaranty Corporation, Reliance Surety property or properties consisting of stocks in trade,
enforced should be considered, bearing in mind the and Insurance Co. Inc., and Equitable Insurance goods in process and/or inventories only hereby
principles of equity and fair play. Corporation. insured, and unless such notice be given and the
- It is worthy to note that M/V Manila City was - On May 15, 1981, Western Guaranty Corporation particulars of such insurance or insurances be stated
insured with Prudential for P45M. To determine the issued Fire Insurance Policy No. 37201 in the amount therein or endorsed on this policy pursuant to
validity and sustainability of the claim of William of P350,000.00. This policy was renewed on May 13, Section 50 of the Insurance Code, by or on behalf of
Lines, Inc., for a total loss, Prudential conducted its 1982. the Company before the occurrence of any loss or
own inquiry. Upon thorough investigation by its hull - On July 30, 1981, Reliance Surety and Insurance damage, all benefits under this policy shall be
surveyor, M/V Manila City was found to be beyond Co., Inc. issued Fire Insurance Policy No. 69135 in deemed forfeited, provided however, that this
economical salvage and repair. The evaluation of the the amount of P300,000.00 (Renewed under condition shall not apply when the total insurance or
average adjuster also reported a constructive total Renewal Certificate No. 41997). An additional insurances in force at the time of loss or damage is
loss. The said claim of William Lines, Inc., was then insurance was issued by the same company on not more than P200,000.00."
found to be valid and compensable such that November 12, 1981 under Fire Insurance Policy No. Petitioner’s comments
Prudential paid the latter the total value of its 71547 in the amount of P700,000.00. > Petitioners contend that they are not to be blamed
insurance claim. Furthermore, it was ascertained - On February 8, 1982, Equitable Insurance for the omissions, alleging that insurance agent Leon
that the replacement cost of the vessel (the price of Corporation issued Fire Insurance Policy No. 39328 Alvarez (for Western) and Yap Kam Chuan (for
a vessel similar to M/V Manila City), amounts to in the amount of P200,000.00. Reliance and Equitable) knew about the existence of
P55M. - Thus when the building occupied by the New Life the additional insurance coverage and that they were
- Considering the aforestated circumstances, let Enterprises was gutted by fire at about 2:00 o'clock not informed about the requirement that such other
alone the fact that negligence on the part of in the morning of October 19, 1982, the stocks in or additional insurance should be stated in the
petitioner has been sufficiently proven, it would trade inside said building were insured against fire in policy, as they have not even read said policies.
indeed be unfair and inequitable to limit the liability the total amount of P1,550,000.00. According to the
of petitioner to One Million Pesos only. As aptly held certification issued by the Headquarters, Philippine ISSUE
by the trial court, “it is rather unconscionable if not Constabulary/Integrated National Police, Camp WON New Life Enterprises’ claim for payment be
overstrained.” To allow CSEW to limit its liability to Crame, the cause of fire was electrical in nature. denied
P1M notwithstanding the fact that the total loss According to the plaintiffs, the building and the
suffered by the assured and paid for by Prudential stocks inside were burned. After the fire, Julian Sy HELD
amounted to P45M would sanction the exercise of a went to the agent of Reliance Insurance whom he YES
degree of diligence short of what is ordinarily asked to accompany him to the office of the Ratio Furthermore, when the words and language of
required because, then, it would not be difficult for company so that he can file his claim. He averred documents are clear and plain or readily
petitioner to escape liability by the simple expedient that in support of his claim, he submitted the fire understandable by an ordinary reader thereof, there
of paying an amount very much lower than the clearance, the insurance policies and inventory of is absolutely no room for interpretation or
actual damage or loss suffered by William Lines, Inc. stocks. construction anymore. Courts are not allowed to
Disposition Petition is DENIED. Resolution of the He further testified that the three insurance make contracts for the parties; rather, they will
CA is AFFIRMED. companies are sister companies, and as a matter of intervene only when the terms of the policy are
fact when he was following-up his claim with ambiguous, equivocal, or uncertain. The parties
NEW LIFE ENTERPRISES V CA Equitable Insurance, the Claims Manager told him to must abide by the terms of the contract because
207 SCRA 669 go first to Reliance Insurance and if said company such terms constitute the measure of the insurer's
REGALADO; March 31, 1992 agrees to pay, they would also pay. The same liability and compliance therewith is a condition
treatment was given him by the other insurance precedent to the insured's right of recovery from the
NATURE companies. Ultimately, the three insurance insurer.
Appeal by certiorari companies denied plaintiffs' claim for payment.
INSURANCE Page 8

- While it is a cardinal principle of insurance law that pace, it was taking several passengers, all of whom the insurance contract on the limit of the insurer's
a policy or contract of insurance is to be construed managed to board the bus while it was already at the liability.
liberally in favor of the insured and strictly against bus stop; plaintiff was the last one to board the bus.
the insurer company, yet contracts of insurance, like While the plaintiff was still on the bus with his hand ISSUE
other contracts, are to be construed according to the on the bus door, the slowly moving bus sped forward WON the CA erred in the interpretation of the
sense and meaning of the terms which the parties at a high speed, as a result of which, the plaintiff lost insurance contract on the limit of the insurer’s
themselves have used. If such terms are clear and balance and fell from the bus. As plaintiff clung liability
unambiguous, they must be taken and understood in instinctively to the handle bar, he was dragged by
their plain, ordinary and popular sense. Moreover, the bus along the asphalted road. The bus driver, Gil HELD
obligations arising from contracts have the force of Agpalo, abruptly stopped the bus. Then fled from the YES
law between the contracting parties and should be scene, leaving the bus and the injured plaintiff - The insurance policy clearly placed the maximum
complied with in good faith. behind. limit of the petitioner's liability for damages arising
Reasoning - The plaintiff was brought to the Manila Sanitarium from death or bodily injury at P12,000.00 per
a. The terms of the contract are clear and and Hospital where the doctors performed 2 major passenger and its maximum liability per accident at
unambiguous. The insured is specifically required to surgical operations on plaintiffs right leg. (P50,000.00. Since only one passenger was injured
disclose to the insurer any other insurance and its - Plaintiff was confined at the hospital for (40) days, in the accident, the insurer's liability for the damages
particulars which he may have effected on the same from June 10, 1984 to August 26, 1984. Medical suffered by said passenger is pegged to the amount
subject matter. The knowledge of such insurance by expenses totaled the amount of P69,444.41. of P12,000.00 only.
the insurer's agents, even assuming the acquisition Plaintiff’s medical expenses were advanced by his - The limit of P50,000.00 per accident means that
thereof by the former, is not the "notice" that would employer Maglines but he was required to reimburse the insurer's maximum liability for any single
stop the insurers from denying the claim. Besides, Maglines on a staggered basis by way of salary accident will not exceed P50,000.00 regardless of the
the so-called theory of imputed knowledge, that is, deductions. After his release from the hospital, he number of passengers killed or injured therein.
knowledge of the agent is knowledge of the principal, returned to the hospital for further treatment and The bus company may not recover from the
aside from being of dubious applicability here has checkup. The injuries had left plaintiff with a huge insurance company more than P12,000.00 per
likewise been roundly refuted by respondent court scar on his right leg. Also, the plaintiff incurred lost passenger killed or injured, or (P50,000.00) per
whose factual findings we find acceptable. earning by way of unearned salaries amounting to accident even if under the judgment of the court, the
b. Petitioners should be aware of the fact that a P7,500.00 due to said physical injuries and the erring bus operator will have to pay more than
party is not relieved of the duty to exercise the consequent hospital confinement. P12,000.00 to each injured passenger. The trial
ordinary care and prudence that would be exacted in - Plaintiff filed on June 26, 1985 the complaint court's interpretation of the insurance contract was
relation to other contracts. The conformity of the against DMTC and its driver. Agpalo was later the correct interpretation.
insured to the terms of the policy is implied from his dropped as a party defendant because he could not Disposition petition for review is GRANTED. The
failure to express any disagreement with what is be served with summons. Upon filing its answer, decision promulgated by the CA, ordering the third
provided for. defendant DMTC filed a thirdparty complaint against party defendent, First Quezon City Insurance Co.,
First Quezon City Insurance Co., Inc. September 17, Inc. to indemnify theI private respondent, (DMTC),
FIRST QUEZON CITY INSURANCE CO. v. CA (DE 1985, third-party defendant filed its answer to the the sum of P50,000.00 for the damages of the
DIOS MARIKINA TRANSPORT CO) third-party complaint. passenger, Jose V. Del Rosario, is hereby modified
218 SCRA 526 - TC held DMTC complaint dismissed for lack of merit by reducing the award to 12,000.00 only. Costs
GRINO-AQUINO; February 28, 1993 and as regards the third-party complaint First against the private respondent De Dios Marikina
Quezon City Insurance Co., Inc. was to indemnify Transportation Co., Inc.
NATURE third-party plaintiff DMTC in the sum of P12,000.00
PETITION for review of the decision of the Court of with interest. There being no satisfactory warrant the TY V FIRST NATIONAL SURETY
Appeals. FQCIC seeks to limit to P12000, the amount court dismissed the rest of the claims in the 1 SCRA 1324
specified in the insurance contract, it’s liability to complaint and third-party complaint. LABRADOR; April 29, 1961
indemnify the respomdemt DMTC, for the damages - The bus company appealed to the CA, which
suffered by a passenger, who accidentally fell off the modified the dispositive as regards the third-party FACTS
bug. complaint, that the third-party defendant First - At different times within a period of two months
Quezon City Insurance Co., Inc. be ordered to prior to 24 December 1953, Diosdado C. Ty,
FACTS indemnify third-party plaintiff DMTC the SUM of employed as operator mechanic foreman in the
- After sending off certain seamen at the departure P50,000.00 with legal interest. Insurance company Broadway Cotton Factory insured himself in 18 local
area of MIA, Jose V. del Rosario proceeded to the filed a MFR which was denied. insurance companies, among which being the 8
public utility bus stop. While at the bus stop, the Hence, this petition for review, assailing the above-named defendants, which issued to him
plaintiff saw a DMTC bus. While moving at a crawling appellate courts' interpretation of the provision of personal accident policies. Plaintiff’s beneficiary was
INSURANCE Page 9

his employer, Broadway Cotton Factory, which paid through the bones of the wrist.” There was no such not dim its light. The crankcase and flywheel housing
the insurance premiums. On 24 December 1953, a amputation in the case at bar. of the car broke when it hit a hollow block lying
fire broke out which totally destroyed the Broadway - The Supreme Court affirmed the appealed decision, alongside the water hole. The car was towed and
Cotton Factory. Fighting his way out of the factory, with costs against the plaintiff-appellant. repaired by Morosi Motors at a total cost of P302.27.
plaintiff was injured on the left hand by a heavy - When the repairs on the car had already been
object. He was brought to the Manila Central MISAMIS LUMBER V CAPITAL INSURANCE made, Misamis made a report of the accident to
University hospital, and after receiving first-aid, he 17 SCRA 288 Capital Insurance.
went to the National Orthopedic Hospital for REYES; May 20, 1966 - Since Capital refused to pay for the total cost of to
treatment of his injuries (fractures in index, middle, wage and repairs, suit was filed in the municipal
fourth, and fifth fingers of left hand). From 26 NATURE court originally.
December 1953 to 8 February 1954, he underwent Direct appeal on a point of law from the judgment of - The defendant-appellant admits liability in the
medical treatment in the hospital. The above- the Court of First Instance of Manila amount of P150, but not for any excess thereof. The
described physical injuries have caused temporary lower court did not exonerate the said appellant for
total disability of plaintiff’s left hand. Plaintiff filed FACTS the excess because the company's absolution would
the corresponding notice of accident and notice of - Misamis Lumber Corporation, under its former render the insurance contract one-sided and that the
claim with all of the above-named defendants to name, Lanao Timber Mills, Inc., insured its Ford said insurer had not shown that the cost of repairs in
recover indemnity. Defendants rejected plaintiff’s Falcon motor car for the amount of P14,000 with the sum of P302.27 is unreasonable, excessive or
claim for indemnity for the reason that there being Capital Insurance & Surety Company, Inc. The padded, nor had it shown that it could have
no severance of amputation of the left hand, the pertinent provisions of the policy provided, as undertaken the repairs itself at less expense.
disability suffered by him was not covered by his follows:
policy. 1. The Company will subject to the Limits of ISSUE
- Plaintiff sued the defendants in the Municipality Liability indemnify the Insured against loss or WON Capital Insurance can be made to pay more
Court of this City, which dismissed his complaints. damage to the Motor Vehicle and its accessories than P150
Thereafter, the plaintiff appealed to the Court of First and spare parts whilst thereon.
Instance Manila, presided by Judge Gregorio S. 2. (a) by accidental collision or overturning or HELD
Narvasa, which absolved the defendants from the collision or overturning consequent when NO
complaints. Hence, the appeal. mechanical breakdown or consequent upon wear - The insurance policy stipulated in paragraph 4 that
and tear. if the insured authorizes the repair the liability of the
ISSUE 3. At its option, the Company may pay in cash the insurer, per its sub-paragraph (a), is limited to
WON Diosdado Ty is entitled to indemnity under the amount of the loss or damage or may repair, P150.00. The literal meaning of this stipulation must
insurance policy for the disability of his left hand reinstate or replace the Motor Vehicle or any part control, it being the actual contract, expressly and
thereof or its accessories or spare parts. The plainly provided for in the policy.
HELD liability of the Company shall not exceed the value - Recourse to legal hermeneutics is not called for
- The agreement contained in the insurance policies of the parts lost or damaged and the reasonable because paragraph 4 of the policy is clear and
is the law between the parties. As the terms of the cost of fitting such parts or the value of the Motor specific and leaves no room for interpretation.
policies are clear, express and specific that only Vehicle at the time of the loss or damage - The option to undertake the repairs is accorded to
amputation of the left hand should be considered as whichever is the loss. The Insured's estimate of the insurance company per paragraph 2. The said
a loss thereof, an interpretation that would include value stated in the schedule shall be the maximum company was deprived of the option because the
the mere fracture or other temporary disability not amount payable by the Company in respect of any insured took it upon itself to have the repairs made,
covered by the policies would certainly be claim for loss or damage. and only notified the insurer when the repairs were
unwarranted. In the case at bar, due to the clarity of 4. The Insured may authorize the repair of the done. As a consequence, paragraph 4, which limits
the stipulation, distinction between “temporary Motor Vehicle necessitated by damage for which the company's liability to P150.00, applies.
disability” and “total disability” need not be made in the Company may be liable under this policy - The insurance contract may be rather onerous
relation to one’s occupation means that the condition provided that: ("one-sided", as the lower court put it), but that in
of the insurance is such that common prudence (a) the estimated cost of such repair does not itself does not justify the abrogation of its express
requires him to desist from transacting his business exceed the authorized Repair Limit. terms, terms which the insured accepted or adhered
or renders him incapable of working. While the Court (b) a detailed estimate of the cost is forwarded to and which is the law between the contracting
sympathizes with the plaintiff or his employer, for to the Company without delay and providing parties.
whose benefit the policies were issued, it can not go also that the authorized repair limit is P150.00. - To require the insurer to prove that the cost of the
beyond the clear and express conditions of the - One night, the insured car, while traveling along in repairs ordered by the insured is unreasonable, when
insurance policies, all of which define partial Aurora Boulevard, passed over a water hole which the insurer was not given an opportunity to inspect
disability as loss of either hand by a amputation the driver did not see because an oncoming car did
INSURANCE Page
10

and assess the damage before the repairs were 1. NO Rosete. Driver Magalong was assigned by PRC
made, is contrary to elementary justice and equity. - The SC held that Condition 27 of the Insurance Management Systems.
policy is very clear and free from any doubt or - After an investigation by the Pasay police, driver
SUN INSURANCE OFFICE LTD. V CA (TAN) ambiguity. It has to be taken in its plain, ordinary, Magalong and guard Atiga were charged, together
195 SCRA 193 and popular sense. The rejection letter of February with Batigue , Aquino and John Doe, with violation of
PARAS; March 13, 1991 29, 1984 was clear and plain. The Court noted that P.D. 532 (Anti-Highway Robbery Law)
the one year period is likewise in accord with Section - Demands were made by the Producers upon the
NATURE 23 of the Insurance Code which states that any Fortune to pay the amount of the loss of
Petition for certiorari to review the decision of the CA condition which limits the time for commencing an P725,000.00 but the latter refused to pay as the loss
action to a period of less than one year when the is excluded from the coverage of the insurance policy
FACTS cause of action accrues is void. The right of action, specifically under "General Exceptions"
- Private respondent Emilio Tan took from the according to the SC, accrues at the time that the > The company shall not be liable under this
petitioner a Peso 300,000 property insurance policy claim is rejected at the first instance. A request for policy in respect of x x x (b) any loss caused by
to cover his interest in the electrical insurance store reconsideration of the denial cannot suspend the any dishonest, fraudulent or criminal act of the
of his brother housed in a building in Iloilo City on running of the prescriptive period. The Court noted insured or any officer, employee, partner,
August 15, 1983. Four days after the issuance of the that the rationale for the one year period is to ensure director, trustee or authorized representative of
policy, the building including the insured store that the evidence as to the origin and cause of the the Insured whether acting alone or in
burned. destruction have not yet disappeared. conjunction with others.
- On August 20, 1983, Tan filed his claim for fire 2. NO - Fortune opposes the contention of Producers that
loss. Sun Insurance, on February 29, 1984, wrote - The Court clarified its ruling in Eagle Star Insurance Atiga and Magalong are not its "officer, employee, x
the private respondent denying the claim. On April 3, Co. vs Chia Yu where it ruled that “the cause of x x trustee or authorized representative x x x at the
1984, private respondent wrote another letter to the action in an insurance contract does not accrue until time of the robbery
insurance company requesting reconsideration of the the Insured’s claim is finally rejected by the Insurer” - Trial Court
denial. Tan’s lawyer wrote another letter to the by stating the use of the word “finally” cannot be > On being “EMPLOYEES”
insurance company inquiring about the April 3 letter construed to mean the rejection of a petition for Magalong and Atiga were not employees or
which sought for a reconsideration of the denial. In reconsideration. What the court referred to in effect representatives of Producers as their services as
its reply to the lawyer’s letter, Sun Insurance is the rejection in the first instance as claimed by armored car driver and as security guard having
reiterated its denial of the claim and enclosed therein Sun Insurance been merely offered by PRC Management and by
copies of the two previous denials dated February Disposition The decision of the CA is reversed and Unicorn Security and which latter firms assigned
29, 1984 and May 17, 1985. set aside. The case is dismissed them to plaintiff. The wages and salaries of both
- On November 20, 1985, Tan filed a civil case with Magalong and Atiga are presumably paid by their
the RTC. Petition filed a motion to dismiss on the FORTUNE INSURANCE AND SURETY CO. INC.V respective firms, which alone wields the power to
alleged ground that the action has already prescribed CA (PRODUCERS BANK OF THE PHILIPPINES) dismiss them
based on Condition 27 of the Insurance Policy which 244 SCRA 308 > On being “AUTHORIZED REPRESENTATIVE”
stated that the window to file the appropriate action DAVIDE; May 23, 1995 They were merely an assigned armored car driver
with either the Insurance Commission or in any court and security guard for the money transfer. It was
of competent jurisdiction is twelve months from the NATURE teller Maribeth Alampay who had "custody" of the
rejection of the claim. RTC denied the motion and Petition for Review on certiorari of CA decision P725,000.00 cash being transferred along a specified
the subsequent motion for reconsideration. The CA money route
likewise denied the petition of Sun Insurance. FACTS - Court of Appeals
- Producers Bank of the Philippines filed a complaint > affirmed in toto
ISSUE against Fortune Insurance and Surety Co., Inc. for > A policy or contract of insurance is to be construed
1. WON the court the filing of a motion for recovery of P725,000.00 under the policy issued by liberally in favor of the insured and strictly against
reconsideration interrupts the 12 months prescription Fortune. The sum was allegedly lost on June 29, the insurance company (New Life Enterprises vs.
period to contest the denial of the insurance claim 1987 during a robbery of Producer's armored vehicle Court of Appeals; Sun Insurance Office, Ltd. vs.
2. WON the rejection of the claim shall be deemed while it was in transit to transfer the money from its Court of Appeals). Contracts of insurance, like other
final only if it contains words to the effect that the Pasay City Branch to its head office in Makati under contracts, are to be construed according to the sense
denial is final the custody of its teller, Maribeth Alampay. The and meaning of the terms which the parties
armored car was driven by Benjamin Magalong Y de themselves have used. If such terms are clear and
Vera, escorted by Security Guard Saturnino Atiga Y unambiguous, they must be taken and understood in
HELD
INSURANCE Page
11

their plain, ordinary and popular sense (New Life exercised by Producers. Fortune further insists that contract with Unicorn Security Services which
Enterprises Case; Sun Insurance Office). PRC Management System and Unicorn Security provides that the guards of the latter "are in no
> The language used by Fortune in the policy is Services are but "labor-only" contractors under sense employees of the CLIENT."
plain, ordinary and simple. No other interpretation is Article 106 of the Labor Code which provides:
necessary. The word "employee" should be taken to Art. 106. Contractor or subcontractor. - There is ISSUE
mean in the ordinary sense. The Labor Code is a "labor-only" contracting where the person WON Fortune Insurance and Surety Co. Inc. is liable
special law specifically dealing with/and specifically supplying workers to an employer does not have under the Money, Security, and Payroll Robbery
designed to protect labor and therefore its definition substantial capital or investment in the form of policy it issued to Producers Bank of the Philippines
as to employer-employee relationships insofar as the tools, equipment, machineries, work premises, or WON recovery is precluded under the general
application/enforcement of said Code is concerned among others, and the workers recruited and exceptions clause of the policy
must necessarily be inapplicable to an insurance placed by such persons are performing activities
contract. Had it intended to apply the Labor Code in which are directly related to the principal business HELD
defining what the word "employee" refers to, it of such employer. In such cases, the person or NO
must/ should have so stated expressly in the intermediary shall be considered merely as an Ratio A contract of insurance is a contract of
insurance policy. Said driver and security guard agent of the employer who shall be responsible to adhesion, thus any ambiguity therein should be
cannot be considered as employees of Producers the workers in the same manner and extent as if resolved against the insurer, or it should be
bank because it has no power to hire or to dismiss the latter were directly employed by him. construed liberally in favor of the insured and strictly
said driver and security guard under the contracts > International Timber Corp. vs. NLRC - a "labor- against the insurer. Limitations of liability should be
except only to ask for their replacements from the only" contractor is equivalent to a finding that there regarded with extreme jealousy and must be
contractors. is an employer-employee relationship between the construed in such a way as to preclude the insurer
- Fortune’s Contention owner of the project and the employee of the "labor- from non-compliance with its obligation. It goes
> when Producers commissioned a guard and a only" contractor without saying then that if the terms of the contract
driver to transfer its funds from one branch to - Producer’s Contention are clear and unambiguous, there is no room for
another, they effectively and necessarily became its > Magalong and Atiga were not its employees since construction and such terms cannot be enlarged or
authorized representatives in the care and custody of it had nothing to do with their selection and diminished by judicial construction.
the money. Assuming that they could not be engagement, the payment of their wages, their - An insurance contract is a contract of indemnity
considered authorized representatives, they were, dismissal, and the control of their conduct. upon the terms and conditions specified therein. It is
nevertheless, employees of Producers. It asserts that > International Timber Corp. is not applicable to all settled that the terms of the policy constitute the
the existence of an employer-employee relationship cases but only when it becomes necessary to prevent measure of the insurer's liability. In the absence of
"is determined by law and being such, it cannot be any violation or circumvention of the Labor Code, a statutory prohibition to the contrary, insurance
the subject of agreement." Thus, if there was in social legislation whose provisions may set aside companies have the same rights as individuals to
reality an employer-employee relationship between contracts entered into by parties in order to give limit their liability and to impose whatever conditions
Producers, on the one hand, and Magalong and protection to the working man. they deem best upon their obligations not
Atiga, on the other, the provisions in the contracts of > American President Lines vs. Clave should be inconsistent with public policy.
Producers with PRC Management System for applied which stated Reasoning
Magalong and with Unicorn Security Services for In determining the existence of employer- - It should be noted that the insurance policy
Atiga which state that Producers is not their employee relationship, the following elements are entered into by the parties is a theft or robbery
employer and that it is absolved from any liability as generally considered, namely: (1) the selection insurance policy which is a form of casualty
an employer, would not obliterate the relationship. and engagement of the employee; (2) the insurance. Section 174 of the Insurance Code
> an employer-employee relationship depends upon payment of wages; (3) the power of dismissal; provides:
four standards: and (4) the power to control the employee's Sec. 174. Casualty insurance is insurance covering
(1) the manner of selection and engagement of the conduct. loss or liability arising from accident or mishap,
putative employee - Since under Producers' contract with PRC excluding certain types of loss which by law or
(2) the mode of payment of wages Management Systems it is the latter which assigned custom are considered as failing exclusively within
(3) the presence or absence of a power to dismiss Magalong as the driver of Producers' armored car the scope of insurance such as fire or marine. It
and and was responsible for his faithful discharge of his includes, but is not limited to, employer's liability
(4) the presence and absence of a power to control duties and responsibilities, and since Producers paid insurance, public liability insurance, motor vehicle
the putative employee's conduct. the monthly compensation of P1,400.00 per driver to liability insurance, plate glass insurance, burglary
> Of the four, the right-of-control test has been held PRC Management Systems and not to Magalong, it is and theft insurance, personal accident and health
to be the decisive factor. It asserts that the power of clear that Magalong was not Producers' employee. As insurance as written by non-life insurance
control over Magalong and Atiga was vested in and to Atiga, Producers relies on the provision of its
INSURANCE Page
12

companies, and other substantially similar kinds of employees of the party employing them and not of - RTC: policy was violated by Verendia when it failed
insurance. (italics supplied) the party who supplied them to the employer. to inform Fidelity of his other insurance coverages,
- Except with respect to compulsory motor vehicle - But even granting for the sake of argument that thus no need to pay.
liability insurance, the Insurance Code contains no these contracts were not "labor-only" contracts, and - CA: reversed decision
other provisions applicable to casualty insurance or PRC Management Systems and Unicorn Security
to robbery insurance in particular. These contracts Services were truly independent contractors, we are ISSUE
are, therefore, governed by the general provisions satisfied that Magalong and Atiga were, in respect of (There is a procedural issue involved here, but is
applicable to all types of insurance. Outside of these, the transfer of Producer's money from its Pasay City irrelevant to our discussion. It concerns the filing of
the rights and obligations of the parties must be branch to its head office in Makati, its "authorized a motion for extension of time to file a motion for
determined by the terms of their contract, taking representatives" who served as such with its teller reconsideration, where the court said that although it
into consideration its purpose and always in Maribeth Alampay. Howsoever viewed, Producers now prohibits filing of such motion, the instant
accordance with the general principles of insurance entrusted the three with the specific duty to safely motion was filed before the effectivity of this rule,
law. transfer the money to its head office, with Alampay thus allowing the adjudication of the case)
- With the foregoing principles in mind, it may now to be responsible for its custody in transit; Magalong WON Fidelity was liable to pay Verendia considering
be asked whether Magalong and Atiga qualify as to drive the armored vehicle which would carry the the circumstances
employees or authorized representatives has been money; and Atiga to provide the needed security for
aptly observed that in burglary, robbery, and theft the money, the vehicle, and his two other HELD
insurance, "the opportunity to defraud the insurer - companions. In short, for these particular tasks, the 1. NO
the moral hazard - is so great that insurers have three acted as agents of Producers. A Ratio As the insurance contract is the law between
found it necessary to fill up their policies with "representative" is defined as one who represents or the parties, Verendia is deemed to have forfeited his
countless restrictions, many designed to reduce this stands in the place of another; one who represents right to claim by the misrepresentation he made.
hazard. Seldom does the insurer assume the risk of others or another in a special capacity, as an agent, Reasoning
all losses due to the hazards insured against." and is interchangeable with "agent." - the court reviewed the factual findings of the courts
Persons frequently excluded under such provisions Disposition instant petition is hereby GRANTED. CA below, since it appears that there was a
are those in the insured's service and employment. decision and RTC Makati decision are REVERSED and misapprehension of the facts by the CA.
The purpose of the exception is to guard against SET ASIDE. Civil Case is DISMISSED. - Verendia is found to have concocted the lease
liability should the theft be committed by one having contract to deflect responsibility for the fire towards
unrestricted access to the property. In such cases, VERENDIA V CA (FIDELITY & SURETY CO. OF an alleged lessee, even making it appear that the
the terms specifying the excluded classes are to be THE PHILS) alleged lessee had disappeared, inflated the value of
given their meaning as understood in common 217 SCRA 417 the property, and insured same property with two
speech. The terms "service" and "employment" are MELO; January 22, 1993 other companies.
generally associated with the idea of selection, - An insurance contract is the law between the
control, and compensation. NATURE parties, its terms and conditions constitute the
- There is marked disagreement between the parties Petition to review decision of the CA measure of the insurer’s liability and compliance
on the correct meaning of the terms "employee" therewith is a condition precedent to the insured’s
and "authorized representatives." FACTS right to recovery from the insurer.
It is clear to us that insofar as Fortune is concerned, - Fidelity Co. issued a Fire Insurance Policy covering - As it is also a contract of adhesion, an insurance
it was its intention to exclude and exempt from Verendia’s residential building in the amount of contract should be liberally construed in favor of the
protection and coverage losses arising from P385k. Verendia also insured the same building with insured and strictly against the insurer company
dishonest, fraudulent, or criminal acts of persons two other companies (Country Bankers Insurance for which usually prepares it.
granted or having unrestricted access to Producers' P56k, and Development Insurance for P400k). - Considering, however, the fact that Verendia used
money or payroll. When it used then the term - While all 3 policies were in force, the insured a false lease contract to support his claim, the terms
"employee," it must have had in mind any person property was completely destroyed by fire. Verendia of the policy should be strictly construed against the
who qualifies as such as generally and universally filed a claim against Fidelity, but the latter refused insured. Verendia failed to live by the terms of the
understood, or jurisprudentially established in the payment, thus a complaint was filed in the RTC. policy, specifically Section 13 thereof which is
light of the four standards in the determination of Fidelity’s reason for refusal: the policy was avoided expressed in terms that are clear and unambiguous,
the employer-employee relationship or as statutorily by reason of over-insurance, and that Verendia that all benefits under the policy shall be forfeited “If
declared even in a limited sense as in the case of maliciously represented that the building was under the claim be in any respect fraudulent, or if any false
Article 106 of the Labor Code which considers the lease to a Roberto Garcia, when it was actually a declaration be made or used in support thereof, or if
employees under a "labor-only" contract as Marcelo Garcia who was the lessee. any fraudulent means or devises are used by the
Insured or anyone acting in his behalf to obtain any
INSURANCE Page
13

benefit under the policy”. Verendia, having presented extending the coverage from October 15, 1961 to estoppel undeniably calls for application. After
a false declaration to support his claim for benefits in October 15, 1962. This time Federico Songco's petitioner Fieldmen's Insurance Co., Inc. had led the
the form of a fraudulent lease contract, he forfeited private jeepney carried Plate No. J-68136- insured Federico Songco to believe that he could
all benefits therein by virtue of Section 13 of the Pampanga-1961. ... On October 29, 1961, during the qualify under the common carrier liability insurance
policy in the absence of proof that Fidelity waived effectivity of the renewed policy, the insured vehicle policy, and to enter into contract of insurance paying
such provision. Worse yet, by presenting a false while being driven by Rodolfo Songco, a duly the premiums due, it could not, thereafter, in any
lease contract, Verendia reprehensibly disregarded licensed driver and son of Federico (the vehicle litigation arising out of such representation, be
the principle that insurance contracts are uberrimae owner) collided with a car in the municipality of permitted to change its stand to the detriment of the
fidae and demand the most abundant good faith. Calumpit, province of Bulacan, as a result of which heirs of the insured. As estoppel is primarily based
Disposition Decision of CA reversed, and that of mishap Federico Songco (father) and Rodolfo Songco on the doctrine of good faith and the avoidance of
RTC is reinstated. (son) died, Carlos Songco (another son), the latter's harm that will befall the innocent party due to its
FIELDMEN'S INSURANCE CO. INC V VDA. DE wife, Angelita Songco, and a family friend by the injurious reliance, the failure to apply it in this case
SONGCO name of Jose Manuel sustained physical injuries of would result in a gross travesty of justice.
25 SCRA 20 varying degree." 1 - That is all that needs be said insofar as the first
FERNANDO; 1968 - It was further shown according to the decision of alleged error of respondent Court of Appeals is
respondent Court of Appeals: "Amor Songco, 42- concerned, petitioner being adamant in its far-from-
FACTS year-old son of deceased Federico Songco, testifying reasonable plea that estoppel could not be invoked
- An insurance firm, petitioner Fieldmen's Insurance as witness, declared that when insurance agent by the heirs of the insured as a bar to the alleged
Co., Inc., was not allowed to escape liability under a Benjamin Sambat was inducing his father to insure breach of warranty and condition in the policy. lt
common carrier insurance policy on the pretext that his vehicle, he butted in saying: 'That cannot be, Mr. would now rely on the fact that the insured owned a
what was insured, not once but twice, was a private Sambat, because our vehicle is an "owner" private private vehicle, not a common carrier, something
vehicle and not a common carrier, the policy being vehicle and not for passengers,' to which agent which it knew all along when not once but twice its
issued upon the insistence of its agent who Sambat replied: 'whether our vehicle was an "owner" agent, no doubt without any objection in its part,
discounted fears of the insured that his privately type or for passengers it could be insured because exerted the utmost pressure on the insured, a man
owned vehicle might not fall within its terms, the their company is not owned by the Government and of scant education, to enter into such a contract.
insured moreover being "a man of scant education," the Government has nothing to do with their - Nor is there any merit to the second alleged error
finishing only the first grade. So it was held in a company. So they could do what they please of respondent Court that no legal liability was
decision of the lower court thereafter affirmed by whenever they believe a vehicle is insurable' ... In incurred under the policy by petitioner. Why liability
respondent Court of Appeals. Petitioner in seeking spite of the fact that the present case was filed and under the terms of the policy 5 was inescapable was
the review of the above decision of respondent Court tried in the CFI of Pampanga, the defendant set forth in the decision of respondent Court of
of Appeals cannot be so sanguine as to entertain the company did not even care to rebut Amor Songco's Appeals. Thus: "Since some of the conditions
belief that a different outcome could be expected. To testimony by calling on the witness-stand agent contained in the policy issued by the defendant-
be more explicit, we sustain the Court of Appeals. Benjamin Sambat, its Pampanga Field appellant were impossible to comply with under the
- The facts as found by respondent Court of Appeals, Representative." 2 existing conditions at the time and 'inconsistent with
binding upon us, follow: "This is a peculiar case. - The plaintiffs in the lower court, likewise the known facts,' the insurer 'is estopped from
Federico Songco of Floridablanca, Pampanga, a man respondents here, were the surviving widow and asserting breach of such conditions.' From this
of scant education being only a first grader ..., children of the deceased Federico Songco as well as jurisprudence, we find no valid reason to deviate and
owned a private jeepney with Plate No. 41-289 for the injured passenger Jose Manuel. On the above consequently hold that the decision appealed from
the year 1960. On September 15, 1960, as such facts they prevailed, as had been mentioned, in the should be affirmed. The injured parties, to wit,
private vehicle owner, he was induced by Fieldmen's lower court and in the respondent Court of Carlos Songco, Angelito Songco and Jose Manuel, for
Insurance Company Pampanga agent Benjamin Appeals.1awphîl.nèt whose hospital and medical expenses the defendant
Sambat to apply for a Common Carrier's Liability - The basis for the favorable judgment is the company was being made liable, were passengers of
Insurance Policy covering his motor vehicle ... Upon doctrine announced in Qua Chee Gan v. Law Union the jeepney at the time of the occurrence, and
paying an annual premium of P16.50, defendant and Rock Insurance Co., Ltd., 3 with Justice J. B. L. Rodolfo Songco, for whose burial expenses the
Fieldmen's Insurance Company, Inc. issued on Reyes speaking for the Court. It is now beyond defendant company was also being made liable was
September 19, 1960, Common Carriers Accident question that where inequitable conduct is shown by the driver of the vehicle in question. Except for the
Insurance Policy No. 45-HO- 4254 ... the duration of an insurance firm, it is "estopped from enforcing fact, that they were not fare paying passengers,
which will be for one (1) year, effective September forfeitures in its favor, in order to forestall fraud or their status as beneficiaries under the policy is
15, 1960 to September 15, 1961. On September 22, imposition on the insured." 4 recognized therein." 6
1961, the defendant company, upon payment of the - As much, if not much more so than the Qua Chee - Even if it be assumed that there was an ambiguity,
corresponding premium, renewed the policy by Gan decision, this is a case where the doctrine of an excerpt from the Qua Chee Gan decision would
INSURANCE Page
14

reveal anew the weakness of petitioner's contention. - This is merely to stress that while the morality of South Africa, for US$154.40 per metric ton or a total
Thus: "Moreover, taking into account the well known the business world is not the morality of institutions of P10,304,231.75 due to its perishable nature which
rule that ambiguities or obscurities must be strictly of rectitude like the pulpit and the academe, it could no longer stand a voyage of twenty days to
interpreted against the party that caused them, the cannot descend so low as to be another name for Manila and another twenty days for the discharge
'memo of warranty' invoked by appellant bars the guile or deception. Moreover, should it happen thus, thereof. It reduced its claim to US$448,806.09 (or its
latter from questioning the existence of the no court of justice should allow itself to lend its peso equivalent of P9,879,928.89 at the exchange
appliances called for in the insured premises, since approval and support.1awphîl.nèt rate of P22.0138 per $1.00) representing its loss
its initial expression, 'the undernoted appliances for - We have no choice but to recognize the monetary after the proceeds of the sale were deducted from
the extinction of fire being kept on the premises responsibility of petitioner Fieldmen's Insurance Co., the original claim.Malayan maintained its position
insured hereby, ... it is hereby warranted ...,' admits Inc. It did not succeed in its persistent effort to avoid that the arrest of the vessel by civil authorities on a
of interpretation as an admission of the existence of complying with its obligation in the lower court and question of ownership was an excepted risk under
such appliances which appellant cannot now the Court of Appeals. Much less should it find any the marine insurance policies.
contradict, should the parol evidence rule apply." 7 receptivity from us for its unwarranted and Petitioners Claim
- To the same effect is the following citation from the unjustified plea to escape from its liability. - an arrest by civil authority is not compensable
same leading case: "This rigid application of the rule since the term "arrest" refers to "political or
on ambiguities has become necessary in view of MALAYAN INSURANCE CORP. V CA (TKC executive acts" and does not include a loss caused
current business practices. The courts cannot ignore MARKETING CORP.) by riot or by ordinary judicial process as in this case
that nowadays monopolies, cartels and concentration 270 SCRA 242 - the deletion of the Free from Capture or Seizure
of capital, endowed with overwhelming economic ROMERO; March 20, 1997 Clause would leave the assured covered solely for
power, manage to impose upon parties dealing with the perils specified by the wording of the policy itself
them cunningly prepared 'agreements' that the NATURE - the rationale for the exclusion of an arrest pursuant
weaker party may not change one whit, his Petition for review on certiorari to judicial authorities is to eliminate collusion
participation in the 'agreement' being reduced to the between unscrupulous assured and civil authorities.
alternative to 'take it or leave it' labelled since FACTS - any loss which private respondent may have
Raymond Saleilles 'contracts by adherence' (contrats - TKC Marketing Corp. was the owner/consignee of incurred was in the nature and form of unrecovered
d'adhesion), in contrast to those entered into by some 3,189.171 metric tons of soya bean meal acquisition value brought about by a voluntary
parties bargaining on an equal footing, such which was loaded on board the ship MV Al Kaziemah sacrifice sale and not by arrest, detention or seizure
contracts (of which policies of insurance and for carriage from the port of Rio del Grande, Brazil, of the ship.
international bills of lading are prime examples) to the port of Manila. Said cargo was insured against - its act of rejecting the claim was a result of its
obviously call for greater strictness and vigilance on the risk of loss by petitioner Malayan Insurance honest belief that the arrest of the vessel was not a
the part of courts of justice with a view to protecting Corporation for which it issued two (2) Marine Cargo compensable risk under the policies issued
the weaker party from abuses and imposition, and Policies. Respondents Comments
prevent their becoming traps for the unwary (New - While the vessel was docked in Durban, South - petitioner, being the sole author of the policies,
Civil Code. Article 24; Sent. of Supreme Court of Africa the civil authorities arrested and detained it "arrests" should be strictly interpreted against it
Spain, 13 Dec. 1934, 27 February 1942)." 8 because of a lawsuit on a question of ownership and because the rule is that any ambiguity is to be taken
- The last error assigned which would find fault with possession. TKC Marketing notified Malayan of the contra proferentum. Risk policies should be
the decision of respondent Court of Appeals insofar arrest of the vessel and made a formal claim for the construed reasonably and in a manner as to make
as it affirmed the lower court award for exemplary dollar equivalent on the policies (US$916,886.66) for effective the intentions and expectations of the
damages as well as attorney's fees is, on its face, of non-delivery of the cargo. It likewise sought the parties.
no persuasive force at all. assistance of Malayan on what to do with the cargo. - the policies clearly stipulate that they cover the
- The conclusion that inescapably emerges from the - Malayan replied that the arrest of the vessel by civil risks of non-delivery of an entire package and that it
above is the correctness of the decision of authority was not a peril covered by the policies. TKC was petitioner itself that invited and granted the
respondent Court of Appeals sought to be reviewed. advised Malayan that it might tranship the cargo and extensions and collected premiums thereon.
For, to borrow once again from the language of the requested an extension of the insurance coverage
Qua Chee Gan opinion: "The contract of insurance is until actual transhipment, which extension was ISSUES
one of perfect good faith (uberima fides) not for the approved upon payment of additional premium. The 1. WON the arrest of the vessel was a risk covered
insured alone,but equally so for the insurer; in fact, insurance coverage was extended under the same under the subject insurance policies
it is more so for the latter, since its dominant terms and conditions embodied in the original 2. WON insurance policies should be strictly
bargaining position carries with it stricter policies while in the process of making arrangements construed against the insurer
responsibility." 9 for the transhipment of the cargo from Durban to
Manila. However the cargo was sold in Durban, HELD
INSURANCE Page
15

1.YES company in order to avoid a forfeiture, unless no In a Resolution dated 10 January 1990, the Court of
- With the incorporation of subsection 1.1 of Section other result is possible from the language used. Appeals denied the motion for reconsideration for
1 of the Institute War Clauses, "arrest" caused by - If a marine insurance company desires to limit or lack of merit. Petitioner Western is now before us on
ordinary judicial process is deemed included among restrict the operation of the general provisions of its a Petition for Review alleging that the Court of
the covered risks. This interpretation becomes contract by special proviso, exception, or exemption, Appeals erred in holding petitioner liable to pay
inevitable when subsection 1.1 of Section 1 of the it should express such limitation in clear and beyond the limits set forth in the Schedule
Institute War Clauses provided that "this insurance unmistakable language. Indemnities and in finding Western liable for loss of
covers the risks excluded from the Standard Form of Be that as it may, exceptions to the general earnings, moral damages and attorney's fees.
English Marine Policy by the clause 'Warranted free coverage are construed most strongly against the Succinctly stated, it is petitioner Western's position
of capture, seizure, arrest, etc. x x x'" or the F.C. & company. Even an express exception in a policy is to that it cannot be held liable for loss of earnings,
S. Clause. Jurisprudentially, "arrests" caused by be construed against the underwriters by whom the moral damages and attorney's fees because these
ordinary judicial process is also a risk excluded from policy is framed, and for whose benefit the exception items are not among those included in the Schedule
the Standard Form of English Marine Policy by the is introduced. Indemnities set forth in the insurance policy.
F.C. & S. Clause. - Petitioner Western in effect contends before this
- Petitioner cannot adopt the argument that the WESTERN GUARANTY CORPORATION V CA Court, as it did before the Court of Appeals, that
"arrest" caused by ordinary judicial process is not (RODRIGUEZ, and DE DIOS TRANSPORTATION because the Schedule of Indemnities limits the
included in the covered risk simply because the F.C. CO) amount payable for certain kinds of expenses
& S. Clause under the Institute War Clauses can only 187 SCRA 652 "hospital room", "surgical expenses", "an
be operative in case of hostilities or warlike FELICIANO; July 20, 1990 aesthesiologists' fee", "operating room" and "medical
operations on account of its heading "Institute War expenses" that Schedule should be read as
Clauses." FACTS excluding liability for any other type of expense or
2. YES - At around 4:30 in the afternoon of 27 March 1982, damage or loss even though actually sustained or
Ratio Insurance Policies should be construed while crossing Airport Road on a pedestrian lane on incurred by the third party victim. We are not
liberally in favor of the insured and strictly against her way to work, respondent Priscilla E. Rodriguez persuaded by Western's contention.
the insurer. was struck by a De Dios passenger bus owned by
Reasoning respondent De Dios Transportation Co., Inc., then ISSUE
- An insurance contract should be so interpreted as driven by one Walter Saga y Aspero. The bus driver WON the Schedule of indemnities as stated in the
to carry out the purpose for which the parties disregarded the stop signal given by a traffic insurance policy should be construed strictly to
entered into the contract which is, to insure against policeman to allow pedestrians to cross the road. exclude all others not explicitly stated therein
risks of loss or damage to the goods. Such Priscilla was thrown to the ground, hitting her
interpretation should result from the natural and forehead. She was treated at the Protacio Emergency
reasonable meaning of language in the policy. Where Hospital and later on hospitalized at the San Juan De
restrictive provisions are open to two interpretations, Dios Hospital. Her face was permanently disfigured, HELD
that which is most favorable to the insured is causing her serious anxiety and moral distress. NO
adopted. - Respondent bus company was insured with Ratio An insurance policy being in the nature of an
Indemnity and liability insurance policies are petitioner Western Guaranty Corporation ("Western") adhesion contract is to be strictly construed against
construed in accordance with the general rule of under its Master Policy which enumerated specific the insurer and liberally in favor of the insured.
resolving any ambiguity therein in favor of the liabilities of the insurance company and ended with a Reasoning
insured, where the contract or policy is prepared by clause to clarify the limitations of the amount which - Firstly, the Schedule of Indemnities does not
the insurer. A contract of insurance, being a contract could be granted as indemnity. purport to restrict the kinds of damages that may be
of adhesion, par excellence, any ambiguity therein - Respondent Priscilla Rodriguez filed a complaint for awarded against Western once liability has arisen.
should be resolved against the insurer.Limitations of damages before the Regional Trial Court of Makati Section 1, quoted above, does refer to certain
liability should be regarded with extreme jealousy against De Dios Transportation Co. and Walter A. "Limits of Liability" which in the case of the third
and must be construed in such a way as to preclude Saga. Respondent De Dios Transportation Co., in party liability section of the Master Policy, is
the insurer from noncompliance with its obligations turn, filed a third-party complaint against its apparently P50,000.00 per person per accident.
- It must be borne in mind that such contracts are insurance carrier, petitioner Western. Within this over-all quantitative limit, all kinds of
invariably prepared by the companies and must be - On 6 August 1985, the trial court rendered a damages allowable by law "actual or compensatory
accepted by the insured in the form in which they decision in favor of respondent Priscilla E. Rodriguez, damages"; "moral damages"; "nominal damages";
are written. Any construction of a marine policy - On appeal, the Court of Appeals affirmed in toto "temperate or moderate damages"; "liquidated
rendering it void should be avoided. Such policies the decision of the trial court. Petitioner moved for damages"; and "exemplary damages" may be
will, therefore, be construed strictly against the the reconsideration of the appellate court's decision. awarded by a competent court against the insurer
INSURANCE Page
16

once liability is shown to have arisen, and the REYES; December 17, 1955 Insurance Act from September 26, 1940, until each
essential requisites or conditions for grant of each is paid, with costs against the defendant.
species of damages are present. It appears to us NATURE - In its first assignment of error, the insurance
self-evident that the Schedule of Indemnities was An appeal by defendant insurance company from the company alleges that the trial Court should have
not intended to be an enumeration, much less a decision of CFI in favor of the plaintiff held that the policies were avoided for breach of
closed enumeration, of the specific kinds of damages warranty, specifically the one appearing on a rider
which may be awarded under the Master Policy FACTS pasted (with other similar riders) on the face of the
Western has issued. - before the last war, plaintiff-appellee owned 4 policies.4
- Secondly, the reading urged by Western of the warehouses or bodegas in Tabaco, Albay, used for - It is argued that since the bodegas insured had an
Schedule of Indemnities comes too close to working the storage of stocks of copra and of hemp, baled external wall perimeter of 500 meters or 1,640 feet,
fraud upon both the insured and the third party and loose, in which the appellee dealt extensively. the appellee should have 11 fire hydrants in the
beneficiary of Section 1, quoted above. For They had been, with their contents, insured with the compound, and that he actually had only 2, with a
Western's reading would drastically and without defendant Company since 1937, and the loose made further pair nearby, belonging to the municipality of
warning limit the otherwise unlimited (save for the payable to the Philippine National Bank as mortgage Tabaco.
over-all quantitative limit of liability of P50,000.00 of the hemp and crops, to the extent of its interest.
per person per accident) and comprehensive scope - Fire of undetermined origin that broke out in the ISSUES
of liability assumed by the insurer Western under early morning of July 21, 1940, and lasted almost 1. WON the defendant-appellant can claim the
Section 1: "all sums necessary to discharge liability one week, gutted and completely destroyed Bodegas policies it had issued as void ab initio
of the insured in respect of [bodily injury to a third Nos. 1, 2 and 4, with the merchandise stored 2. WON the insured violated the "Hemp Warranty"
party]". This result which is not essentially therein. Plaintiff-appellee informed the insurer by provisions of Policy No. 2637165 against the storage
different from taking away with the left hand what telegram on the same date; and on the next day, the of gasoline
had been given with the right hand we must avoid fire adjusters engaged by appellant insurance 3. WON the insured connived at the loss and
as obviously repugnant to public policy. If what company arrived and proceeded to examine and fraudulently inflated the quantity of the insured stock
Western now urges is what Western intended to photograph the premises, pored over the books of in the burnt bodegas
achieve by its Schedule of Indemnities, it was the insured and conducted an extensive
incumbent upon Western to use language far more investigation. The plaintiff having submitted the HELD
specific and precise than that used in fact by corresponding fire claims, totalling P398,562.81 (but 1. NO
Western, so that the insured, and potential reduced to the full amount of the insurance, Ratio It is usually held that where the insurer, at the
purchasers of its Master Policy, and the Office of the P370,000), the Insurance Company resisted time of the issuance of a policy of insurance, has
Insurance Commissioner, may be properly informed payment, claiming violation of warranties and knowledge of existing facts which, if insisted on,
and act accordingly. conditions, filing of fraudulent claims, and that the would invalidate the contract from its very inception,
- Petitioner Western would have us construe the fire had been deliberately caused by the insured or such knowledge constitutes a waiver of conditions in
Schedule of Indemnities as comprising contractual by other persons in connivance with him. the contract inconsistent with the facts, and the
limitations of liability which, as already noted, is - Que Chee Gan, with his brother, Qua Chee Pao, insurer is stopped thereafter from asserting the
comprehensively defined in Section 1 "Liability to and some employees of his, were indicted and tried breach of such conditions. The law is charitable
the Public" of the Master Policy. It is well-settled, in 1940 for the crime of arson, it being claimed that enough to assume, in the absence of any showing to
however, that contractual limitations of liability found they had set fire to the destroyed warehouses to the contrary, that an insurance company intends to
in insurance contracts should be regarded by courts collect the insurance. They were, however, acquitted
with a jaundiced eye and extreme care and should by the trial court. 4
Memo. of Warranty. — The undernoted Appliances for the extinction
be so construed as to preclude the insurer from - the civil suit to collect the insurance money of fire being kept on the premises insured hereby, and it being
evading compliance with its just obligations. proceeded to its trial with the CFI holding that: declared and understood that there is an ample and constant water
supply with sufficient pressure available at all seasons for the same, it
- Finally, an insurance contract is a contract of judgment is rendered for the plaintiff and against the is hereby warranted that the said appliances shall be maintained in
adhesion. The rule is well entrenched in our defendant condemning the latter to pay the former efficient working order during the currency of this policy, by reason
jurisprudence that the terms of such contract are to — (a) Under the first cause of action, the sum of whereof a discount of 2 1/2 per cent is allowed on the premium
chargeable under this policy.
be construed strictly against the party which P146,394.48; (b) Under the second cause of action, Hydrants in the compound, not less in number than one for each 150
prepared the contract, which in this case happens to the sum of P150,000; (c) Under the third cause of feet of external wall measurement of building, protected, with not less
than 100 feet of hose piping and nozzles for every two hydrants kept
be petitioner Western. action, the sum of P5,000; (d) Under the fourth under cover in convenient places, the hydrants being supplied with
cause of action, the sum of P15,000; and (e) Under water pressure by a pumping engine, or from some other source,
QUA CHEE GAN V LAW UNION AND ROCK the fifth cause of action, the sum of P40,000; all of capable of discharging at the rate of not less than 200 gallons of water
per minute into the upper story of the highest building protected, and a
INSURANCE CO., LTD. which shall bear interest at the rate of 8% per trained brigade of not less than 20 men to work the same.'
96 PHIL 85 annum in accordance with Section 91 (b) of the
INSURANCE Page
17

executed a valid contract in return for the premium - Receipt of Premiums or Assessments after Cause - Appellee admitted that there were 36 cans of
received; and when the policy contains a condition for Forfeiture Other than Nonpayment. — It is a well gasoline in the building designed. It However,
which renders it voidable at its inception, and this settled rule of law that an insurer which with gasoline is not specifically mentioned among the
result is known to the insurer, it will be presumed to knowledge of facts entitling it to treat a policy as no prohibited articles listed in the so-called "hemp
have intended to waive the conditions and to execute longer in force, receives and accepts a premium on warranty." The cause relied upon by the insurer
a binding contract, rather than to have deceived the the policy, estopped to take advantage of the speaks of "oils (animal and/or vegetable and/or
insured into thinking he is insured when in fact he is forfeiture. It cannot treat the policy as void for the mineral and/or their liquid products having a flash
not, and to have taken his money without purpose of defense to an action to recover for a loss point below 300 Fahrenheit)", and is decidedly
consideration. thereafter occurring and at the same time treat it as ambiguous and uncertain; for in ordinary parlance,
Reasoning valid for the purpose of earning and collecting "Oils" mean "lubricants" and not gasoline or
- The appellant is barred estoppel to claim violation further premiums. kerosene. And how many insured, it may well be
of the so-called fire hydrants warranty, for the - Moreover, taking into account the well known rule wondered, are in a position to understand or
reason that knowing fully all that the number of that ambiguities or obscurities must be strictly determine "flash point below 300 Fahrenheit.
hydrants demanded therein never existed from the interpreted against the party that caused them, the - If the company intended to rely upon a condition of
very beginning, the appellant neverthless issued the "memo of warranty" invoked by appellant bars the that character, it ought to have been plainly
policies in question subject to such warranty, and latter from questioning the existence of the expressed in the policy.
received the corresponding premiums. The insurance appliances called for in the insured premises - The contract of insurance is one of perfect good
company was aware, even before the policies were On the alleged violations of the plaintiff The faith not for the insured alone, but equally so for the
issued, that in the premises insured there were only alleged violation of the warranty of 100 feet of fire insurer; in fact, it is mere so for the latter, since its
two fire hydrants installed by Qua Chee Gan and two hose for every two hydrants, must be equally dominant bargaining position carries with it stricter
others nearby, owned by the municipality of Tabaco, rejected, since the appellant's argument thereon is responsibility.
contrary to the requirements of the warranty in based on the assumption that the insured was bound - Another point that is in favor of the insured is that
question to maintain no less than eleven hydrants, which the gasoline kept in Bodega No. 2 was only incidental
- The plain, human justice of this doctrine is requirement appellant is estopped from enforcing. to his business, being no more than a customary 2
perfectly apparent. To allow a company to - As to maintenance of a trained fire brigade of 20 day's supply for the five or six motor vehicles used
accept one's money for a policy of insurance men, the record is preponderant that the same was for transporting of the stored merchandise. "It is well
which it then knows to be void and of no effect, organized, and drilled, from time to give, although settled that the keeping of inflammable oils on the
though it knows as it must, that the assured not maintained as a permanently separate unit, premises though prohibited by the policy does not
believes it to be valid and binding, is so which the warranty did not require. void it if such keeping is incidental to the business."
contrary to the dictates of honesty and fair 2. NO On the submission of books, voucbers, etc. The
dealing, and so closely related to positive Ratio Here, again, by reason of the exclusive control charge that the insured failed or refused to submit to
fraud, as to the abhorrent to fair-minded men. of the insurance company over the terms and the examiners of the insurer the books, vouchers,
It would be to allow the company to treat the phraseology of the contract, the ambiguity must be etc. demanded by them was found unsubstantiated
policy as valid long enough to get the premium held strictly against the insurer and liberally in favor by the trial Court, and no reason has been shown to
on it, and leave it at liberty to repudiate it the of the insured, specially to avoid a forfeiture. alter this finding. The insured gave the insurance
next moment. This cannot be deemed to be the Insurance is, in its nature, complex and examiner all the date he asked for, and the examiner
real intention of the parties. To hold that a difficult for the layman to understand. Policies even kept and photographed some of the examined
literal construction of the policy expressed the are prepared by experts who know and can books in his possession. What does appear to have
true intention of the company would be to anticipate the hearing and possible been rejected by the insured was the demand that
indict it, for fraudulent purposes and designs complications of every contingency. So long as he should submit "a list of all books, vouchers,
which we cannot believe it to be guilty of. insurance companies insist upon the use of receipts and other records", but the refusal of the
- The appellant company so worded the policies that ambiguous, intricate and technical provisions, insured in this instance was well justified, since the
while exacting the greater number of fire hydrants which conceal rather than frankly disclose, demand for a list of all the vouchers (which were not
and appliances, it kept the premium discount at the their own intentions, the courts must, in in use by the insured) and receipts was positively
minimum of 2 1/2%, thereby giving the insurance fairness to those who purchase insurance, unreasonable, considering that such listing was
company a double benefit. Such abnormal treatment construe every ambiguity in favor of the superfluous because the insurer was not denied
of the insured strongly points at an abuse of the insured. An insurer should not be allowed, by access to the records, that the volume of Qua Chee
insurance company's selection of the words and the use of obscure phrases and exceptions, to Gan's business ran into millions, and that the
terms of the contract, over which it had absolute defeat the very purpose for which the policy demand was made just after the fire when
control. was procured. everything was in turmoil. That the representatives
Reasoning of the insurance company were able to secure all the
INSURANCE Page
18

date they needed is proved by the fact that the - A rider to the Policy contained the following;
adjuster Alexander Stewart was able to prepare his FACTS IV. DROWNING
own balance sheet that did not differ from that - Francisco del Rosario was insured by Equitable It is hereby declared and agreed that
submitted by the insured except for the valuation of Insurance and Casualty Co. Inc under Personal exemption clause Letter (h) in PART VI of
the merchandise, as expressly found by the Court in Accident Policy no. 7136. The Company bound itself the policy is hereby waived by the
the criminal case for arson. to pay P1000 to P3000 as indemnity for the death company, and to form a part of the
3. NO of the insured. provision covered by the policy.
Ratio Both defenses are predicted on the - Under the policy: - Feb 24, 1957, Francisco del Rosario while on board
assumption that the insured was in financial Part I. Indemnity for Death the motor launch ISLAMA, with his beneficiary to the
difficulties and set the fire to defraud the insurance If the insured sustains any bodily injury policy, Remedios Jayme, were forced to jump off
company, presumably in order to pay off the which is effected solely through violent, said launch on account of fire which broke out on
Philippine National Bank, to which most of the external, visible and accidental means, said vessel, resulting in the death by drowning of the
insured hemp and copra was pledged. Both defenses and which shall result, independently of insured and his beneficiary.
are fatally undermined by the established fact that, all other causes and within sixty days - Simeon del Rosario, the insured’s father, filed a
notwithstanding the insurer's refusal to pay the value from the occurrence thereof, in the Death claim for payment with the company. The company
of the policies the extensive resources of the insured of the Insured, the Company shall pay the paid him P1000 pursuant to section 1 Part I of the
enabled him to pay off the National Bank in a short amount set opposite such injury: policy.
time; and if he was able to do so, no motive appears - On the same date, Atty. Francisco wrote to the
for attempt to defraud the insurer. While the Section 1. Injury sustained other than company acknowledging receipt by his client of the
acquittal of the insured in the arson case is not res those specified below unless excepted P1000 but informing said company that said amount
judicata on the present civil action, the insurer's hereinafter P1000 was not the correct one. He claimed that the amount
evidence, to judge from the decision in the criminal Section 2. Injury sustained by the payable should be P1500 under the provision of
case, is practically identical in both cases and must wrecking or disablement of a railroad Section 2 Part I, based on the rule of pari materia.
lead to the same result, since the proof to establish passenger car or street railway car in or - The company referred the matter to the Insurance
the defense of connivance at the fire in order to on which the Insured is traveling as a Coomissioner, who was of the opinion that the
defraud the insurer "cannot be materially less farepaying passenger P1500 liability of the company was only P1000. thus the
convincing than that required in order to convict the Section 3. Injury sustained by the burning company refused to pay more that P1000. Atty.
insured of the crime of arson." of a church, theatre, public library or Francisco wrote a subsequent letter to company
- As to the defense that the burned bodegas could municipal administration building while asking for p3000, which the company refused to pay.
not possibly have contained the quantities of copra the Insured is therein at the - A complaint for recovery of the balance of P2000
and hemp stated in the fire claims, the insurer's case commencement of the fire P2000 was instituted with the CFI Rizal, praying for a
rests almost exclusively on the estimates, inferences Section 4. Injury sustained by the further sum of P10000 as attorney’s fees, expenses
and conclusions of its adjuster investigator who wrecking or disablement of a regular of litigation and costs.
examined the premises during and after the fire. His passenger elevator car in which the - CFI ruled in favor of petitioner, ordering the
testimony, however, was based on inferences from Insured is being conveyed as a passenger company to pay P2000 to del Rosario.
the photographs and traces found after the fire, and (Elevator in mines exluded) P2500
must yield to the contradictory testimony of those Section 5. Injury sustained by a stroke of ISSUE
who actually saw the contents of the bodegas shortly lightning or by a cycloneP3000 How much should the indemnity be
before the fire, while inspecting them for the xxxx xxxx
mortgagee Bank. xxxx HELD
Disposition We find no reversible error in the Part VI. Exceptions - All the parties agree that indemnity has to be paid,
judgment appealed from, wherefore the same is This policy shall not cover disappearance but the conflict centers on how much it should be.
hereby affirmed. of the Insured nor shall it cover Death, - Where there is ambiguity with respect to the terms
Disability, Hospital fees, or Loss of time, and conditions of the policy, the same will be
DEL ROSARIO V EQUITABLE INSURANCE & caused to the insured: resolved against the one responsible thereof.
CASUALTY CO., INC x x x (h) By drowning except as a Generally, the insured has little, if any, participation
8 SCRA 343 consequence of the wrecking or in the preparation of the policy, together with the
PAREDES; June 29, 1963 disablement in the Philippine waters of a drafting of its terms and conditions. The
passenger steam or motor vessel in interpretation of obscure stipulations in a contract
NATURE which the Insured is traveling as a should not favor the party who caused the obscurity.
Appeal from judgment of CFI Rizal farepaying passenger; x x x - SC agreed with the ruling of the lower court:
INSURANCE Page
19

x x x death by drowning is a ground for recovery insurances in force at the time of the loss or damage knew of the existence of the two other policies
apart from the bodily injury because death by is not more than P200,000.00." issued by the PFIC
bodily injury is covered by Part I of the policy -On 27 May 1990, fire of accidental origin broke out
while death by drowning is covered by Part VI at around 7:30 p.m. at the public market of San ISSUES
thereof. But while the policy mentions specific Francisco, Agusan del Sur. The petitioner's insured 1. WON the petitioner had prior knowledge of the
amounts that may be recovered for death for stocks-in-trade were completely destroyed two insurance policies issued by the PFIC when he
bodily injury, yet, there is not specific amount prompting him to file w/ the private respondent a obtained the fire insurance policy from the private
mentioned in the policy for death thru drowning claim under the policy. On 28 Dec 1990, the private respondent, thereby, for not disclosing such fact,
although the latter is, under Part VI of the respondent denied the claim because it found that at violating Condition 3 of the policy
policy, a ground for recovery thereunder. Since the time of the loss the petitioner's stocks-in-trade 2. if he had, WON he is precluded from recovering
the defendant has bound itself to pay P1000 to were likewise covered by two fire insurance policies therefrom
P3000 as indemnity for the death of the insured for P100,000.00 each, issued by the Cebu Branch of
but the policy does not positively state any the Philippines First Insurance Co., Inc. (PFIC). HELD
definite amount that may be recovered in case -The basis of the private respondent's denial was the 1. YES
of death by drowning, there is an ambiguity in petitioner's alleged violation of Condition 3 of the - We agree w/ the CA that the petitioner knew of the
this respect in the policy, which ambiguity must policy. prior policies issued by the PFIC. His letter of 18
be interpreted in favor of the insured and strictly - Geagonia then filed a complaint against the private January 1991 to the private respondent conclusively
against the insurer so as to allow a greater respondent w/ the Insurance Commission for the proves this knowledge. His testimony to the contrary
indemnity. x x x plaintiff is therefore entitled to recovery of P100,000.00 under fire insurance policy, before the Insurance Commissioner and which the
recover P3000. for attorney's fees, and costs of litigation. He claims latter relied upon cannot prevail over a written
Disposition Judgment appealed from is affirmed. that the time he obtained the private respondent's admission made ante litem motam. It was, indeed,
fire insurance policy he knew that the two policies incredible that he did not know about the prior
GEAGONIA v. CA (COUNTRY BANKERS issued by the PFIC were already in existence; policies since these policies were not new or original.
INSURANCE) however, he had no knowledge of the provision in 2. NO
8 SCRA 343 the private respondent's policy requiring him to - It must, however, be underscored that unlike the
DAVIDE; February 6 1995 inform it of the prior policies; this requirement was "other insurance" clauses involved in General
not mentioned to him by the private respondent's Insurance and Surety Corp. vs. Ng Hua or in
FACTS agent; and had it been so mentioned, he would not Pioneer Insurance & Surety Corp. vs. Yap, which
-Geagonia is the owner of Norman's Mart located in have withheld such information. He further asserted read:
the public market of San Francisco, Agusan del Sur. that the total of the amounts claimed under the "The insured shall give notice to the company of any
On 22 Dec 1989, he obtained from the private three policies was below the actual value of his insurance or insurances already effected, or which
respondent fire insurance policy for P100,000.00. stocks at the time of loss, w/c was P1M. may subsequently be effected covering any of the
The period of the policy was from 22 Dec 1989 to 22 - The Insurance Commission found that the property hereby insured, and unless such notice be
Dec 1990 and covered the ff: "Stock-in-trade petitioner did not violate Condition 3 as he had no given and the particulars of such insurance or
consisting principally of dry goods such as RTW's for knowledge of the existence of the two fire insurance insurances be stated in or endorsed on this Policy by
men and women wear and other usual to assured's policies obtained from the PFIC; that it was Cebu or on behalf of the Company before the occurrence
business. Tesing Textiles w/c procured the PFIC policies w/o of any loss or damage, all benefits under this Policy
-The policy contained the following condition: informing him or securing his consent; and that Cebu shall be forfeited." or in the 1930 case of Santa Ana
"3. The insured shall give notice to the Company of Tesing Textile, as his creditor, had insurable interest vs. Commercial Union Assurance Co. which
any insurance or insurances already effected, or on the stocks. These findings were based on the provided "that any outstanding insurance upon the
which may subsequently be effected, covering any of petitioner's testimony that he came to know of the whole or a portion of the objects thereby assured
the property or properties consisting of stocks in PFIC policies only when he filed his claim with the must be declared by the insured in writing and he
trade, goods in process and/or inventories only private respondent and that Cebu Tesing Textile must cause the company to add or insert it in the
hereby insured, and unless notice be given and the obtained them and paid for their premiums w/o policy, without which such policy shall be null and
particulars of such insurance or insurances be stated informing him. The Insurance Commission then void, and the insured will not be entitled to
therein or endorsed in this policy pursuant to Section ordered the respondent company to pay complainant indemnity in case of loss," Condition 3 in the
50 of the Insurance Code, by or on behalf of the the sum of P100,000.00 with legal interest from the private respondent's policy No. F-14622 does
Company before the occurrence of any loss or time the complaint was filed until fully satisfied plus not absolutely declare void any violation
damage, all benefits under this policy shall be the amount of P10,000.00 as attorney's fees. thereof. It expressly provides that the
deemed forfeited, provided however, that this -CA reversed the decision of the Insurance condition "shall not apply when the total
condition shall not apply when the total insurance or Commission because it found that the petitioner insurance or insurances in force at the time of
INSURANCE Page
20

the loss or damage is not more than exceeds the property's value, the insured may have because a gun should always be handled with
P200,000.00." an inducement to destroy the property for the caution.
- Interpretation: It is a cardinal rule on insurance purpose of collecting the insurance. The public as Respondents’ Comments
that a policy or insurance contract is to be well as the insurer is interested in preventing a - Felix believed the gun to be safe because he had
interpreted liberally in favor of the insured and situation in which a fire would be profitable to the removed the magazine.
strictly against the company, the reason being, insured. - He repeatedly assured his secretary that the gun
undoubtedly, to afford the greatest protection which Disposition Petition granted. The decision of the was not loaded.
the insured was endeavoring to secure when he Court of Appeals in CA-G.R. SP No. 31916 is SET
applied for insurance. It is also a cardinal principle of ASIDE and the decision of the Insurance Commission ISSUES
law that forfeitures are not favored and that any in Case No. 3340 is REINSTATED. 1. WON Felix Lim’s death was an accident, thus
construction which would result in the forfeiture of making his widow Nerissa liable to claim the accident
the policy benefits for the person claiming SUN INSURANCE OFFICE, LTD. V CA (LIM) insurance
thereunder, will be avoided, if it is possible to 211 SCRA 554 2. WON the award of damages to Nerissa Lim was
construe the policy in a manner which would permit CRUZ; July 17, 1992 justified
recovery, as, for example, by finding a waiver for
such forfeiture. Stated differently, provisions, NATURE HELD
conditions or exceptions in policies which tend Petition for review from the decision of the Court of 1. YES, Felix Lim’s death was an accident.
to work a forfeiture of insurance policies should Appeals Ratio There is no accident when a deliberate act is
be construed most strictly against those for performed unless some additional, unexpected,
whose benefits they are inserted, and most FACTS independent and unforeseen happening occurs which
favorably toward those against whom they are - Felix Lim was issued a Personal Accident Policy produces or brings bout their injury or death.
intended to operate. The reason for this is that, insurance with petitioner company with a face value Reasoning
except for riders which may later be inserted, the of P200,000. His beneficiary was his wife Nerissa. - An accident has been defined to be that which
insured sees the contract already in its final form and - October 6, 1982 – Felix accidentally shot himself in happens by chance or fortuitously without intention
has had no voice in the selection or arrangement of the head with his own gun.  or design and which is unexpected, unusual and
the words employed therein. On the other hand, the - He was playing with the handgun after he had unforeseen. It an event that takes pace without
language of the contract was carefully chosen and removed the gun’s magazine (kasi naman…). one’s foresight or expectastion – an event that
deliberated upon by experts and legal advisers who - He pointed the gun at his secretary and only proceeds from an unknown cause or is an unusual
had acted exclusively in the interest of the insurers witness Pilar Nalagon as a joke and assured her effect of a known case and therefore not expected.
and the technical language employed therein is that the gun was not loaded (are you sure…). It happens without any human agency, an event
rarely understood by ordinary laymen. - He then put the gun to his temple and fired it which, under the circumstances, is unusual to and
- With these principles in mind, we are of the (haaay, sabi ko na nga ba). not expected by the person to whom it happens.
opinion that Condition 3 of the subject policy is - Both parties are in agreement that there was no - The firing of the gun was deemed to be the
not totally free from ambiguity and must be suicide. unexpected and independent and unforeseen
meticulously analyzed. Such analysis leads us - Nerissa claimed as Felix’s beneficiary but Sun occurrence that led to the insured person’s death.
to conclude that (a) the prohibition applies only Insurance would not grant her claim, saying that her - There was no willful exposure to needless peril for
to double insurance, and (b) the nullity of the husband’s death was not an accident. the part of Felix. Suicide and exposure to needless
policy shall only be to the extent exceeding - Nerissa sued Sun Insurance and won the case. peril are similar in the sense that both signify
P200,000.00 of the total policies obtained. Sun Insurance was ordered to pay her P200,000 disregard for one’s life. Suicide imparts a positive
- Furthermore, by stating within Condition 3 itself representing the face value of the claim along with act of ending one’s life whereas the latter indicates
that such condition shall not apply if the total moral, exemplary and compensatory damages and recklessness that is almost suicidal in intent.
insurance in force at the time of loss does not attorney’s fees. The decision was affirmed by the - Accident insurance policies were never meant to
exceed P200,000.00, the private respondent was CA. reward the insured for his tendency to show off or
amenable to assume a co-insurer's liability up to a Petitioners’ Claim for his miscalculations. They were intended to
loss not exceeding P200,000.00. What it had in mind - Sun Insurance cites one of the four exceptions in provide for contingencies.
was to discourage over-insurance. Indeed, the the contract of insurance which includes bodily injury - Lim was unquestionably negligent but it should not
rationale behind the incorporation of "other consequent upon the insured person attempting to prevent his widow from recovering from the
insurance" clause in fire policies is to prevent over- commit suicide or willfully exposing himself to insurance policy he obtained precisely against
insurance and thus avert the perpetration of fraud. needless peril in an attempt to save a human life. accident.
When a property owner obtains insurance policies - There mere act of pointing the gun to his temple - Insurance contracts are, as a rule, supposed to be
from two or more insurers in a total amount that showed that Felix willfully exposed himself to danger interpreted liberally in favor of the assured.
INSURANCE Page
21

2. NO, the claim for damages should not be granted and/or concrete under galvanized iron roof amusement machines and spare parts stored at
for being unjust. and occupied as hosiery mills, garment and the two-storey building because it (Transworld)
Ratio A person may be made liable to the payment lingerie factory, transistor-stereo assembly had no insurable interest in said goods or items.
of moral damages if his act is wrongful. The adverse plant, offices, warehouse and caretaker's - The Court denied the appeal with finality.
result of an action does not per se make the act quarters. - Petitioner Rizal Insurance and private respondent
wrongful and subject the act or to the payment of 'Bounds in front partly by one-storey concrete Transworld, interposed a Motion for
moral damages. building under galvanized iron roof occupied Reconsideration before the Court of Appeals, which
Reasoning as canteen and guardhouse, partly by building reconsidered its decision of July 15, 1993, as
- Petitioner was acting in good faith when it resisted of two and partly one storey constructed of regards the imposition of interest.
the private respondent’s claim on the ground that concrete below, timber above - Undaunted, petitioner Rizal Surety & Insurance
the death of the insured was covered by the undergalvanized iron roof occupied as garage Company found its way to the Court.
exception. and quarters and partly by open space and/or
- The issue was debatable and was clearly not raised tracking/ packing, beyond which is the ISSUE
only for the purpose of evading a legitimate aforementioned Magdalo Street; on its right WON the fire insurance policy litigated upon
obligation. and left by driveway, thence open spaces, and protected only the contents of the main building
at the rear by open spaces.'" (four-span), and did not include those stored in the
RIZAL SURETY & INSURANCE COMPANY V CA - The same pieces of property insured with the two-storey annex building
(TRANSWORLD KNITTING MILLS, INC.) petitioner were also insured with New India
336 SCRA 12 Assurance Company, Ltd., (New India). HELD
PURISIMA; July 18, 2000 - Fire broke out in the compound of Transworld, NO
razing the middle portion of its four-span building - Resolution of the issue posited hinges on the
NATURE and partly gutting the left and right sections proper interpretation of the stipulation in subject fire
Petition for Review on Certiorari under Rule 45 of the thereof. A two-storey building (behind said four- insurance policy regarding its coverage, which reads:
Rules of Court span building) where fun and amusement "xxx contained and/or stored during the currency of
machines and spare parts were stored, was also this Policy in the premises occupied by them forming
FACTS destroyed by the fire. part of the buildings situate (sic) within own
- Rizal Surety & Insurance Company (Rizal - Transworld filed its insurance claims with Rizal Compound xxx"
Insurance) issued Fire Insurance Policy No. 45727 in Surety & Insurance Company and New India - It can be gleaned unerringly that the fire insurance
favor of Transworld Knitting Mills, Inc. (Transworld). Assurance Company but to no avail. policy in question did not limit its coverage to what
- Pertinent portions of subject policy on the buildings - Private respondent brought against the said were stored in the four-span building. As opined by
insured, and location thereof, read: insurance companies an action for collection of the trial court of origin, two requirements must
"‘On stocks of finished and/or unfinished sum of money and damages. concur in order that the said fun and amusement
products, raw materials and supplies of every - Petitioner Rizal Insurance countered that its fire machines and spare parts would be deemed
kind and description, the properties of the insurance policy sued upon covered only the protected by the fire insurance policy under scrutiny,
Insureds and/or held by them in trust, on contents of the four-span building, which was to wit:
commission or on joint account with others partly burned, and not the damage caused by the "First, said properties must be contained and/or
and/or for which they (sic) responsible in case fire on the two-storey annex building. stored in the areas occupied by Transworld and
of loss whilst contained and/or stored during - The trial court dismissed the case as against The second, said areas must form part of the building
the currency of this Policy in the premises New India Assurance Co., Ltd. but ordered described in the policy xxx"
occupied by them forming part of the defendant Rizal Surety And Insurance Company to - Said building of four-span lofty one storey in height
buildings situate (sic) within own Compound pay Transwrold (sic) Knitting Mills, Inc. with mezzanine portions is constructed of reinforced
at MAGDALO STREET, BARRIO UGONG, - Both the petitioner, Rizal Insurance Company, concrete and hollow blocks and/or concrete under
PASIG, METRO MANILA, PHILIPPINES, BLOCK and private respondent, Transworld Knitting Mills, galvanized iron roof and occupied as hosiery mills,
NO. 601.’ Inc., went to the Court of Appeals, which required garment and lingerie factory, transistor-stereo
x New India Assurance Company to pay plaintiff- assembly plant, offices, ware house and caretaker's
xx............ appellant the amount of P1,818,604.19 while the quarter.
...xxx....... Rizal Surety has to pay the plaintiff-appellant - The Court is mindful of the well-entrenched
........xxx P470,328.67. doctrine that factual findings by the Court of Appeals
‘Said building of four-span lofty one storey in - New India appealed to the Court theorizing inter are conclusive on the parties and not reviewable by
height with mezzanine portions is constructed alia that the private respondent could not be this Court, and the same carry even more weight
of reinforced concrete and hollow blocks compensated for the loss of the fun and
INSURANCE Page
22

when the Court of Appeals has affirmed the findings where forfeiture is involved' and the reason for actually and directly controverted and determined
of fact arrived at by the lower court. this is that the 'insured usually has no voice in and not as to matters merely involved therein."
- In the case under consideration, both the trial court the selection or arrangement of the words Disposition Decision, and the Resolution of the CA
and the Court of Appeals found that the so called employed and that the language of the contract WERE AFFIRMED in toto. No pronouncement as to
"annex " was not an annex building but an integral is selected with great care and deliberation by costs.
and inseparable part of the four-span building experts and legal advisers employed by, and
described in the policy and consequently, the acting exclusively in the interest of, the PAN MALAYAN INSURANCE CORPORATION vs.
machines and spare parts stored therein were insurance company.' " COURT OF APPEALS (ERLINDA FABIE & HER
covered by the fire insurance in dispute. - Equally relevant is the following disquisition of the UNKNOWN DRIVER)
- Verily, the two-storey building involved, a Court in Fieldmen's Insurance Company, Inc. vs. 184 SCRA 55; G.R. No. 81026
permanent structure which adjoins and Vda. De Songco, to wit: CORTES; April 3, 1990
intercommunicates with the "first right span of the "'This rigid application of the rule on ambiguities
lofty storey building", formed part thereof, and has become necessary in view of current FACTS
meets the requisites for compensability under the business practices. The courts cannot ignore - December 10, 1985: PANMALAY filed a complaint
fire insurance policy sued upon. that nowadays monopolies, cartels and for damages with the RTC of Makati against private
- So also, considering that the two-storey building concentration of capital, endowed with respondents Erlinda Fabie and her driver. PANMALAY
aforementioned was already existing when subject overwhelming economic power, manage to averred the following: that it insured a Mitsubishi
fire insurance policy contract was entered into, impose upon parties dealing with them Colt Lancer car registered in the name of Canlubang
petitioner should have specifically excluded the said cunningly prepared 'agreements' that the Automotive Resources Corporation [CANLUBANG];
two-storey building from the coverage of the fire weaker party may not change one whit, his that on May 26, 1985, due to the "carelessness,
insurance if minded to exclude the same but if did participation in the 'agreement' being reduced to recklessness, and imprudence" of the unknown
not, and instead, went on to provide that such fire the alternative to 'take it or leave it' labelled driver of a pick-up, the insured car was hit and
insurance policy covers the products, raw materials since Raymond Saleilles 'contracts by suffered damages in the amount of P42,052.00; that
and supplies stored within the premises of adherence' (contrats [sic] d'adhesion), in PANMALAY defrayed the cost of repair of the insured
respondent Transworld which was an integral part of contrast to these entered into by parties car and, therefore, was subrogated to the rights of
the four-span building occupied by Transworld, bargaining on an equal footing, such contracts CANLUBANG against the driver of the pick-up and his
knowing fully well the existence of such building (of which policies of insurance and international employer, Erlinda Fabie; and that, despite repeated
adjoining and intercommunicating with the right bills of lading are prime example) obviously call demands, defendants, failed and refused to pay the
section of the four-span building. for greater strictness and vigilance on the part claim of PANMALAY. PANMALAY clarified that the
- Indeed, the stipulation as to the coverage of the of courts of justice with a view to protecting the damage caused to the insured car was settled under
fire insurance policy under controversy has created a weaker party from abuses and imposition, and the "own damage", coverage of the insurance policy.
doubt regarding the portions of the building insured prevent their becoming traps for the unwary.'" - Private respondents filed a Motion to Dismiss
thereby. Article 1377 of the New Civil Code provides: - The issue of whether or not Transworld has an alleging that PANMALAY had no cause of action
"Art.1377. The interpretation of obscure words insurable interest in the fun and amusement against them. RTC dismissed PANMALAY's complaint
or stipulations in a contract shall not favor the machines and spare parts, which entitles it to be for no cause of action and denied PANMALAY's
party who caused the obscurity" indemnified for the loss thereof, had been settled in motion for reconsideration. CA affirmed. Hence, this
- Conformably, it stands to reason that the doubt G.R. No. L-111118, entitled New India Assurance petition for review.
should be resolved against the petitioner, Rizal Company, Ltd., vs. Court of Appeals, where the
Surety Insurance Company, whose lawyer or appeal of New India from the decision of the Court of ISSUE
managers drafted the fire insurance policy contract Appeals under review, was denied with finality by WON the insurer PANMALAY may institute an action
under scrutiny. Citing the aforecited provision of law this Court on February 2, 1994. to recover the amount it had paid its assured in
in point, the Court in Landicho vs. Government - The rule on conclusiveness of judgment, which settlement of an insurance claim against private
Service Insurance System, ruled: obtains under the premises, precludes the respondents as the parties allegedly responsible for
"This is particularly true as regards insurance relitigation of a particular fact or issue in another the damage caused to the insured vehicle
policies, in respect of which it is settled that the action between the same parties based on a different
'terms in an insurance policy, which are claim or cause of action. "xxx the judgment in the HELD
ambiguous, equivocal, or uncertain x x x are to prior action operates as estoppel only as to those YES
be construed strictly and most strongly against matters in issue or points controverted, upon the - Article 2207 of the Civil Code is founded on the
the insurer, and liberally in favor of the insured determination of which the finding or judgment was well-settled principle of subrogation. If the insured
so as to effect the dominant purpose of rendered. In fine, the previous judgment is property is destroyed or damaged through the fault
indemnity or payment to the insured, especially conclusive in the second case, only as those matters or negligence of a party other than the assured, then
INSURANCE Page
23

the insurer, upon payment to the assured, will be appellate court to indulge in contract construction, to Policy No. 306-7432324-3 for the period March 1,
subrogated to the rights of the assured to recover apply the ejusdem generis rule, and to ascribe 1991 to 1992.The new oil mill was insured under
from the wrongdoer to the extent that the insurer meaning contrary to the clear intention and Policy No. 306-7432321-9 for the same term. Official
has been obligated to pay. understanding of these parties. receipts indicating payment for the full amount of the
- General Rule: Payment by the insurer to the - Although the terms "accident" or "accidental" as premium were issued by the petitioner's agent.
assured operates as an equitable assignment to the used in insurance contracts have not acquired a - Policy description:
former of all remedies which the latter may have technical meaning, the Court has on several Front: by a driveway thence at 18 meters distance
against the third party whose negligence or wrongful occasions defined these terms to mean that which by Bldg. No. 2.
act caused the loss. The right of subrogation is not takes place "without one's foresight or expectation, Right: by an open space thence by Bldg. No. 4.
dependent upon, nor does it grow out of, any privity an event that proceeds from an unknown cause, or is Left: Adjoining thence an imperfect wall by Bldg.
of contract or upon written assignment of claim. It an unusual effect of a known cause and, therefore, No. 4.
accrues simply upon payment of the insurance claim not expected." The concept "accident" is not Rear: by an open space thence at 8 meters
by the insurer. necessarily synonymous with the concept of "no distance.”
- Exceptions: fault". It may be utilized simply to distinguish - A fire that broke out in the early morning of
a.) if the assured by his own act releases the intentional or malicious acts from negligent or September 30,1991 gutted and consumed the new
wrongdoer or third party liable for the loss or careless acts of man. oil mill. Petitioner rejected respondent’s claim for
damage, from liability - Obiter Dicta: Even if under the above the insurance proceeds on the ground that no policy
b.) where the insurer pays the assured the value of circumstances PANMALAY could not be deemed was issued by it covering the burned oil mill. It
the lost goods without notifying the carrier who has subrogated to the rights of its assured under Article stated that the description of the insured
in good faith settled the assured's claim for loss 2207 of the Civil Code, PANMALAY would still have a establishment referred to another building.
c.) where the insurer pays the assured for a loss cause of action against private respondents. The Petitioner’s Claim
which is not a risk covered by the policy, thereby insurer who may have no rights of subrogation due The policies referred to the old mill, as stated in the
effecting "voluntary payment" to "voluntary" payment may nevertheless recover description contained in the policy.
- None of the exceptions are availing in the present from the third party responsible for the damage to
case. the insured property under Article 1236 of the Civil ISSUE
- When PANMALAY utilized the phrase "own damage" Code. WON new oil mill is insured by fire insurance policy
— a phrase which, incidentally, is not found in the Disposition Petition is GRANTED. Petitioner's
insurance policy — to define the basis for its complaint for damages against private respondents HELD
settlement of CANLUBANG's claim under the policy, it is REINSTATED. Case remanded to the lower court YES, new oil mill is insured.
simply meant that it had assumed to reimburse the for trial on the merits. Ratio In construing the words used descriptive of a
costs for repairing the damage to the insured building insured, the greatest liberality is shown by
vehicle. AMERICAN HOME ASSURANCE COMPANY V the courts in giving effect to the insurance. In view
- It is a basic rule in the interpretation of contracts TANTUCO ENTERPRISES of the custom of insurance agents to examine
that the terms of a contract are to be construed 366 SCRA 740 buildings before writing policies upon them, and
according to the sense and meaning of the terms PUNO; October 8, 2001 since a mistake as to the identity and character of
which the parties thereto have used. In the case of the building is extremely unlikely, the courts are
property insurance policies, the evident intention of NATURE inclined to consider that the policy of insurance
the contracting parties, i.e., the insurer and the Petition for Review on Certiorari assailing the covers any building which the parties manifestly
assured, determine the import of the various terms Decision of the Court of Appeals. intended to insure, however inaccurate the
and provisions embodied in the policy. It is only description may be.
when the terms of the policy are ambiguous, FACTS Reasoning
equivocal or uncertain, such that the parties - Respondent Tantuco Enterprises, Inc. is engaged in - The parties manifestly intended to insure the new
themselves disagree about the meaning of particular the coconut oil milling and refining industry. It owns oil mill. “On machineries and equipment with
provisions, that the courts will intervene. In such an two oil mills. Both are located at its factory complete accessories usual to a coconut oil mill
event, the policy will be construed by the courts compound at Iyam, Lucena City. Respondent including stocks of copra, copra cake and copra mills
liberally in favor of the assured and strictly against commenced its business operations with only one oil whilst contained in the new oil mill building, situate
the insurer. mill. In 1988, it started operating its second oil mill (sic) at UNNO. ALONG NATIONAL HIGH WAY, BO.
- Considering that the very parties to the policy were ( the new oil mill). IYAM, LUCENA CITY UNBLOCKED.”
not shown to be in disagreement regarding the - The two oil mills were separately covered by fire - If the parties really intended to protect the first oil
meaning and coverage of Section III-1, specifically insurance policies issued by petitioner American mill, then there is no need to specify it as new. It
sub-paragraph (a) thereof, it was improper for the Home Assurance Co. The first oil mill was insured would be absurd to assume that respondent would
INSURANCE Page
24

protect its first oil mill for different amounts and - One of them, 19-year old Edgardo Perea, sued - The insurance policy involved explicitly limits
leave uncovered its second one. Milagros Cayas for damages in the CFI of Cavite, petitioner's liability to P12,000.00 per person and to
- As may be gleaned from the testimony of the while three others agreed to a settlement of P50,000.00 per accident.
petitioner’s employee, the source of the discrepancy P4,000.00 each. - In Stokes vs. Malayan Insurance Co., Inc., the
happened during the preparation of the written - After trial, the court rendered a decision in favor of Court held that the terms of the contract constitute
contract. Perea, ordering Cayas to compensate him, with an the measure of the insurer's liability and compliance
- Respondent is not estopped from claiming that the award of exemplary and moral damages, as well as therewith is a condition precedent to the insured's
policy description is wrong. Evidence on record attorney’s fees. ( P32,000 total) right of recovery from the insurer.
reveals that respondent’s operating manager, Mr. - On November 11, 1981, Milagros Cayas filed a - In the case at bar, the insurance policy clearly and
Edison Tantuco, notified the petitioner’s agent with complaint for a sum of money and damages against categorically placed petitioner's liability for all
whom respondent negotiated for the contract about PCSI in the Court of First Instance of Cavite. damages arising out of death or bodily injury
the inaccurate description in the policy. However, - In view of Milagros Cayas' failure to prosecute the sustained by one person as a result of any one
Mr. Borja assured Mr. Tantuco that the use of the case, the court motu propio ordered its dismissal accident at P12,000.00.
adjective new will distinguish the insured property. without prejudice. - Said amount complied with the minimum fixed by
- Regarding policy requirements that fire - Alleging that she had not received a copy of the the law then prevailing, Section 377 of Presidential
extinguishment appliances should be available and in answer to the complaint, and that "out of Decree No. 612, which provided that the liability of
good working condition, warranty did not require sportsmanship", she did not file a motion to hold land transportation vehicle operators for bodily
respondent to provide for all the fire extinguishing PCSI in default, Milagros Cayas moved for the injuries sustained by a passenger arising out of the
appliances enumerated therein. Neither did it reconsideration of the dismissal order. Said motion use of their vehicles shall not be less than P12,000.
require that the appliances are restricted to those for reconsideration was acted upon favorably by the - In other words, under the law, the minimum
mentioned in the warranty. (Within the vicinity of the court. liability is P12,000 per passenger. Petitioner's liability
new oil mill can be found the following devices: - About two months later, Milagros Cayas filed a under the insurance contract not being less than
numerous portable fire extinguishers, two fire hoses, motion to declare PCSI in default for its failure to file P12,000.00, and therefore not contrary to law,
fire hydrant, and an emergency fire engine.) an answer. morals, good customs, public order or public policy,
- The object of the court in construing a contract is - The motion was granted and plaintiff was allowed said stipulation must be upheld as effective, valid
to ascertain the intent of the parties to the contract to adduce evidence ex-parte. and binding as between the parties.
and to enforce the agreement which the parties have - On July 13, 1982, the court rendered judgment by - In like manner, we rule as valid and binding upon
entered into. In determining what the parties default ordering PCSI to pay Milagros Cayas P50,000 private respondent the condition requiring her to
intended, the courts will read and construe the policy as compensation for the injured passengers, P5,000 secure the written permission of petitioner before
as a whole and if possible, give effect to all the parts as moral damages and P5,000 as attorney's fees. effecting any payment in settlement of any claim
of the contract. - Said decision was set aside after the PCSI filed a against her.
Disposition Petition is dismissed. motion therefor. Trial of the case ensued. - There is nothing unreasonable, arbitrary or
- In due course, the court promulgated a decision objectionable in this stipulation as would warrant its
PERLA COMPANIA DE SEGUROS, INC. v, CA ordering defendant Perla Compania de Seguros, Inc. nullification. The same was obviously designed to
(MILAGROS CAYAS) to pay plaintiff Milagros Cayas the sum of safeguard the insurer's interest against collusion
185 SCRA 741 P50,000.00 under its maximum liability as provided between the insured and the claimants.
FERNAN; May 28, 1990 for in the insurance policy; and the sum of P5,000.00 - It being specifically required that petitioner's
as reasonable attorney's fee written consent be first secured before any payment
NATURE - PCSI appealed to the Court of Appeals, which in settlement of any claim could be made, private
Petition for review on certiorari of a decision of the affirmed in toto the lower court's decision. respondent is precluded from seeking reimbursement
Court of Appeals - Its motion for reconsideration having been denied of the payments made to the three other passangers
by said appellate court, PCSI filed this petition in view of her failure to comply with the condition
FACTS contained in the insurance policy.
- Private respondent Milagros Cayas was the ISSUE - Clearly, the fundamental principle that contracts
registered owner of a Mazda bus, insured with Perla WON, as maintained by petitioner, its liability is are respected as the law between the contracting
Compania de Seguros, Inc. (PCSI) under a policy limited only to the payment made by private parties finds application in the present case.
issued on February 3, 1978. respondent to Perea and only up to the amount of - It was error on the part of the trial and appellate
- On December 17, 1978, the bus figured in an P12,000.00 courts to have disregarded the stipulations of the
accident in Naic, Cavite injuring several of its parties and to have substituted their own
passengers. HELD interpretation of the insurance policy.
YES
INSURANCE Page
25

- In Phil. American General Insurance Co., Inc vs. vehicle for foreclosure. It is specifically provided in 2. NO
Mutuc, we ruled that contracts which are the private the promissory note and chattel mortgage that Reasoning
laws of the contracting parties should be fulfilled failure to pay any installment when due shall make - Petitioners failed to show any provision in the
according to the literal sense of their stipulations, if subsequent installments and the entire balance of insurance policy or mortgage contract providing that
their terms are clear and leave no room for doubt as the obligation due and payable. the loss of the mortgaged vehicle extinguishes their
to the intention of the contracting parties, for - After they refused to do so, BPI then filed principal obligation to BPI.
contracts are obligatory, no matter what form they complaint. - While it is true that the proceeds from the
may be, whenever the essential requisites for their - Petitioners claimed that BPI required them to insurance policy over the mortgaged chattel is for
validity are present. obtain a motor vehicle insurance policy from FGU the benefit of BPI, this will result in partial or full
- In Pacific Oxygen & Acetylene Co. vs. Central Insurance Corporation (FGU Insurance). This is a satisfaction of the obligation only if the insurer pays
Bank," it was stated that the first and fundamental sister company of BPI. They had been paying the the mortgagee, BPI, or if the insurance proceeds
duty of the courts is the application of the law monthly installments on the vehicle until it figured in were paid to BPI. In this case, upon the loss of the
according to its express terms, interpretation being an accident where it became a total wreck. Under vehicle due to total wreck, the petitioners filed a
called for only when such literal application is the terms of the insurance policy from FGU claim under the insurance policy, collected and
impossible. Insurance, the vehicle had to be replaced or its value received the proceeds thereof, but did not settle
- We observe that although Milagros Cayas was able paid to them. Due to the failure and refusal of FGU their obligation with BPI which remained outstanding
to prove a total loss of only P44,000.00, petitioner Insurance to replace the vehicle or pay its value, despite the loss of the vehicle.
was made liable for the amount of P50,000.00, the they stopped payment of the monthly installments. Disposition CA decision AFFIRMED with the
maximum liability per accident stipulated in the - RTC ordered POLTANS to pay BPI the said amount. modification that the interest rate be reduced to
policy. This is patent error. An insurance indemnity, - CA reversed and remanded case to RTC for trial on 12% per annum from 24 May 1994 until fully paid,
being merely an assistance or restitution insofar as the merits. and the award of attorney’s fees be reduced to P50T.
can be fairly ascertained, cannot be availed of by any - RTC again ruled in favor of BPI. CA affirmed.
accident victim or claimant as an instrument of FILIPINO MERCHANTS INS. v. CA (CHOA TIEK
enrichment by reason of an accident. ISSUES SENG)
Disposition Petition granted. The decision of the 1. WON contracts presented in evidence by BPI were 179 SCRA 638
Court of Appeals is modified in that petitioner shall unjust and unacceptable contracts of adhesion REGALADO; November 28, 1989
pay Milagros Cayas the amount of Twelve Thousand 2. WON the terms and conditions of the
Pesos (P12,000. 00) plus legal interest from the comprehensive car insurance policy issued by FGU NATURE
promulgation of the decision of the lower court until should be deemed as having automatically operated Review of the decision of the CA
it is fully paid and attorney's fees in the amount of in favor of BPI as the assured mortgagee, and if so,
P5,000.00. it should be deemed as resulting in the FACTS
extinguishment of petitioner’s obligation - Plaintiff insured said shipment with defendant
POLTAN v. BPI & JOHN DOE insurance company under said cargo for the goods
G.R. No. 164307 HELD described as 600 metric tons of fishmeal in new
CHICO-NAZARIO; March 5, 2007 1. NO gunny bags of 90 kilos each from Bangkok, Thailand
Ratio A contract of adhesion is one in which one of to Manila against all risks under warehouse to
NATURE the parties imposes a ready-made form of contract, warehouse terms.
Petition for review of CA decision which the other party may accept or reject, but - Some of the goods arrived in bad condition.
which the latter cannot modify. It is just as binding Plaintiff made a claim against Filipino Merchants
FACTS as ordinary contracts. Insurance Company. The latter refused to pay.
- Petitioners POLTAN obtained a loan evidenced by a Reasoning Plaintiff brought an action against them. The
promissory note from the MANTRADE Dev’t Corp. - Petitioners failed to show that they were under defendant insurance company presented a third
This was secured by a chattel mortgage over a 1-unit duress or forced to sign the loan documents. The party complaint against the vessel and the arrastre
Nissan Sentra vehicle. natural presumption is that one does not sign a contractor.
- With notice to petitioners, MANTRADE assigned to document without first informing himself of its - Judgment was rendered against the insurance
BPI, by way of a Deed of Assignment, all its rights, contents and consequences. company. On the third party complaint, the third
title and interest to the promissory note and chattel - Contracts of adhesion are not entirely prohibited party defendants were ordered to pay the third party
mortgage. even as the courts remain careful in scrutinizing the plaintiffs. The CA affirmed, but modified the same
- Petitioners defaulted and so BPI demanded the factual circumstances underlying each case to with regard to the adjudication of the third-party
whole balance of P286.5T including accrued interest, determine the respective claims of contending complaint
or to return to BPI the possession of the motor parties on their efficacy.
INSURANCE Page
26

ISSUES - Respondent’s interest over the goods is based on delivered by the Insured which are
1. WON some fortuity, casualty or accidental cause is the perfected contract of sale. The perfected contract outstanding at the date of loss for a
needed to be proved despite the “all risks” policy (as of sale between him and the shipper of the goods period in excess of six (6) months from
asserted by the insurance company) operates to vest in him an equitable title even before the date of the covering invoice or
2. WON the respondent has an insurable interest delivery or before be performed the conditions of the actual delivery of the merchandise
sale. whichever shall first occur.
HELD - Further, Article 1523 of the Civil Code provides that 2. Warranted that the Insured shall submit to the
1. NO where, in pursuance of a contract of sale, the seller Company within twelve (12) days after the close of
- The very nature of the term "all risks" must be is authorized or required to send the goods to the every calendar month all amount shown in their
given a broad and comprehensive meaning as buyer, delivery of the goods to a carrier, whether books of accounts as unpaid and thus become
covering any loss other than a willful and fraudulent named by the buyer or not, for, the purpose of receivable item from their customers and dealers. x
act of the insured. 7 This is pursuant to the very transmission to the buyer is deemed to be a delivery xx
purpose of an "all risks" insurance to give protection of the goods to the buyer, the exceptions to said rule - Petitioner is a customer and dealer of the products
to the insured in those cases where difficulties of not obtaining in the present case. The Court has of IMC and LSPI. On February 25, 1991, the Gaisano
logical explanation or some mystery surround the heretofore ruled that the delivery of the goods on Superstore Complex in Cagayan de Oro City, owned
loss or damage to property. board the carrying vessels partake of the nature of by petitioner, was consumed by fire. Included in the
- Generally, the burden of proof is upon the insured actual delivery since, from that time, the foreign items lost or destroyed in the fire were stocks of
to show that a loss arose from a covered peril, but buyers assumed the risks of loss of the goods and ready-made clothing materials sold and delivered by
under an "all risks" policy the burden is not on the paid the insurance premium covering them IMC and LSPI. On February 4, 1992, respondent
insured to prove the precise cause of loss or damage - Moreover, the issue of lack of insurable interest filed a complaint for damages against petitioner. It
for which it seeks compensation. The insured under was not raised in petitioner’s answer. alleges that IMC and LSPI filed with respondent their
an "all risks insurance policy" has the initial burden Disposition Petition denied claims under their respective fire insurance policies
of proving that the cargo was in good condition when with book debt endorsements; that as of February
the policy attached and that the cargo was damaged GAISANO CAGAYAN v. INSURANCE Co. OF 25, 1991, the unpaid accounts of petitioner on the
when unloaded from the vessel; thereafter, the NORTH AMERICA sale and delivery of ready-made clothing materials
burden then shifts to the insurer to show the 490 SCRA 296 with IMC was P2,119,205.00 while with LSPI it was
exception to the coverage. As we held in Paris- Austria-Martinez; June 8, 2006 P535,613.00; that respondent paid the claims of IMC
Manila Perfumery Co. vs. Phoenix Assurance Co., and LSPI and, by virtue thereof, respondent was
Ltd. the basic rule is that the insurance company has NATURE subrogated to their rights against petitioner; that
the burden of proving that the loss is caused by the Petition for review on certiorari of the Decision of the respondent made several demands for payment
risk excepted and for want of such proof, the Court of Appeals upon petitioner but these went unheeded. In its
company is liable. In the present case, there being Answer with Counter Claim dated July 4, 1995,
no showing that the loss was caused by any of the FACTS petitioner contends that it could not be held liable
excepted perils, the insurer is liable under the policy. - Intercapitol Marketing Corporation (IMC) is the because the property covered by the insurance
2. YES maker of Wrangler Blue Jeans. Levi Strauss (Phils.) policies were destroyed due to fortuities event or
- Section 13 of the Insurance Code defines insurable Inc. (LSPI) is the local distributor of products bearing force majeure; that respondent’s right of subrogation
interest in property as every interest in property, trademarks owned by Levi Strauss & Co.. IMC and has no basis inasmuch as there was no breach of
whether real or personal, or any relation thereto, or LSPI separately obtained from respondent fire contract committed by it since the loss was due to
liability in respect thereof, of such nature that a insurance policies with book debt endorsements. fire which it could not prevent or foresee; that IMC
contemplated peril might directly damnify the The insurance policies provide for coverage on “book and LSPI never communicated to it that they insured
insured. In principle, anyone has an insurable debts in connection with ready-made clothing their properties; that it never consented to paying
interest in property who derives a benefit from its materials which have been sold or delivered to the claim of the insured.
existence or would suffer loss from its destruction various customers and dealers of the Insured - At the pre-trial conference the parties failed to
whether he has or has not any title in, or lien upon anywhere in the Philippines.” The policies defined arrive at an amicable settlement. Thus, trial on the
or possession of the property y. 16 Insurable interest book debts as the “unpaid account still appearing in merits ensued. On August 31, 1998, the RTC
in property may consist in (a) an existing interest; the Book of Account of the Insured 45 days after the rendered its decision dismissing respondent’s
(b) an inchoate interest founded on an existing time of the loss covered under this Policy.” The complaint. It held that the fire was purely accidental;
interest; or (c) an expectancy, coupled with an policies also provide for the following conditions: that the cause of the fire was not attributable to the
existing interest in that out of which the expectancy 1. Warranted that the Company shall not negligence of the petitioner; that it has not been
arises. be liable for any unpaid account in established that petitioner is the debtor of IMC and
respect of the merchandise sold and LSPI; that since the sales invoices state that “it is
INSURANCE Page
27

further agreed that merely for purpose of securing which have been sold or delivered to various or personal, or any relation thereto, or liability in
the payment of purchase price, the above-described customers and dealers of the Insured anywhere in respect thereof, of such nature that a contemplated
merchandise remains the property of the vendor the Philippines.”; and defined book debts as the peril might directly damnify the insured.”
until the purchase price is fully paid”, IMC and LSPI “unpaid account still appearing in the Book of Parenthetically, under Section 14 of the same Code,
retained ownership of the delivered goods and must Account of the Insured 45 days after the time of the an insurable interest in property may consist in: (a)
bear the loss. Dissatisfied, petitioner appealed to the loss covered under this Policy.” Nowhere is it an existing interest; (b) an inchoate interest founded
CA. On October 11, 2000, the CA rendered its provided in the questioned insurance policies that on existing interest; or (c) an expectancy, coupled
decision setting aside the decision of the RTC. The the subject of the insurance is the goods sold and with an existing interest in that out of which the
CA held that the sales invoices are proofs of sale, delivered to the customers and dealers of the expectancy arises.
being detailed statements of the nature, quantity insured. - Therefore, an insurable interest in property does
and cost of the thing sold; that loss of the goods in - Indeed, when the terms of the agreement are clear not necessarily imply a property interest in, or a lien
the fire must be borne by petitioner since the and explicit that they do not justify an attempt to upon, or possession of, the subject matter of the
proviso contained in the sales invoices is an read into it any alleged intention of the parties, the insurance, and neither the title nor a beneficial
exception under Article 1504 (1) of the Civil Code, to terms are to be understood literally just as they interest is requisite to the existence of such an
the general rule that if the thing is lost by a appear on the face of the contract. Thus, what were interest, it is sufficient that the insured is so situated
fortuitous event, the risk is borne by the owner of insured against were the accounts of IMC and LSPI with reference to the property that he would be
the thing at the time the loss under the principle of with petitioner which remained unpaid 45 days after liable to loss should it be injured or destroyed by the
res perit domino; that petitioner’s obligation to IMC the loss through fire, and not the loss or destruction peril against which it is insured. Anyone has an
and LSPI is not the delivery of the lost goods but the of the goods delivered. insurable interest in property who derives a benefit
payment of its unpaid account and as such the 2. NO from its existence or would suffer loss from its
obligation to pay is not extinguished, even if the fire - The present case clearly falls under paragraph (1), destruction. Indeed, a vendor or seller retains an
is considered a fortuitous event; that by subrogation, Article 1504 of the Civil Code: insurable interest in the property sold so long as he
the insurer has the right to go against petitioner; ART. 1504. Unless otherwise agreed, the goods has any interest therein, in other words, so long as
that, being a fire insurance with book debt remain at the seller’s risk until the ownership he would suffer by its destruction, as where he has a
endorsements, what was insured was the vendor’s therein is transferred to the buyer, but when the vendor’s lien. In this case, the insurable interest of
interest as a creditor. Petitioner filed a motion for ownership therein is transferred to the buyer the IMC and LSPI pertain to the unpaid accounts
reconsideration but it was denied by the CA in its goods are at the buyer’s risk whether actual appearing in their Books of Account 45 days after the
Resolution dated April 11, 2001. delivery has been made or not, except that: time of the loss covered by the policies.
(1) Where delivery of the goods has been made to 3. YES
ISSUES the buyer or to a bailee for the buyer, in - Petitioner’s argument that it is not liable because
1. WON the CA erred in construing a fire insurance pursuance of the contract and the ownership in the fire is a fortuitous event under Article 1174 of
policy on book debts as one covering the unpaid the goods has been retained by the seller the Civil Code is misplaced. As held earlier,
accounts of IMC and LSPI since such insurance merely to secure performance by the buyer of petitioner bears the loss under Article 1504 (1) of
applies to loss of the ready-made clothing materials his obligations under the contract, the goods the Civil Code.
sold and delivered to petitioner. are at the buyer’s risk from the time of such - Moreover, it must be stressed that the insurance in
2. WON IMC bears the risk of loss because it delivery; (Emphasis supplied) this case is not for loss of goods by fire but for
expressly reserved ownership of the goods by - Thus, when the seller retains ownership only to petitioner’s accounts with IMC and LSPI that
stipulating in the sales invoices that “[i]t is further insure that the buyer will pay its debt, the risk of remained unpaid 45 days after the fire. Accordingly,
agreed that merely for purpose of securing the loss is borne by the buyer. Accordingly, petitioner petitioner’s obligation is for the payment of money.
payment of the purchase price the above described bears the risk of loss of the goods delivered. Where the obligation consists in the payment of
merchandise remains the property of the vendor - IMC and LSPI did not lose complete interest over money, the failure of the debtor to make the
until the purchase price thereof is fully paid.” the goods. They have an insurable interest until full payment even by reason of a fortuitous event shall
3. WON the petitioner liable for the unpaid accounts payment of the value of the delivered goods. Unlike not relieve him of his liability. The rationale for this is
the civil law concept of res perit domino, where that the rule that an obligor should be held exempt
HELD ownership is the basis for consideration of who bears from liability when the loss occurs thru a fortuitous
1. NO the risk of loss, in property insurance, one’s event only holds true when the obligation consists in
- It is well-settled that when the words of a contract interest is not determined by concept of title, the delivery of a determinate thing and there is no
are plain and readily understood, there is no room but whether insured has substantial economic stipulation holding him liable even in case of
for construction. In this case, the questioned interest in the property. fortuitous event. It does not apply when the
insurance policies provide coverage for “book debts - Section 13 of our Insurance Code defines insurable obligation is pecuniary in nature.
in connection with ready-made clothing materials interest as “every interest in property, whether real
INSURANCE Page
28

- Under Article 1263 of the Civil Code, “[i]n an admission of petitioner’s unpaid account with LSPI. P50K and contents thereof for P70K. On July 31,
obligation to deliver a generic thing, the loss or It only confirms the loss of Levi’s products in the 1975, the building and the contents were totally
destruction of anything of the same kind does not amount of P535,613.00 in the fire that razed razed by fire.
extinguish the obligation.” If the obligation is generic petitioner’s building on February 25, 1991. - Based on the computation of the loss, including the
in the sense that the object thereof is designated - Moreover, there is no proof of full settlement of the Travellers, respondents, ZIC, PBAC, and SSS paid
merely by its class or genus without any particular insurance claim of LSPI; no subrogation receipt was their corresponding shares of the loss. Complainants
designation or physical segregation from all others of offered in evidence. Thus, there is no evidence that were paid the following: P41,546.79 by PBAC,
the same class, the loss or destruction of anything of respondent has been subrogated to any right which P11,877.14 by ZIC, and P5,936.57 by SSS. Demand
the same kind even without the debtor’s fault and LSPI may have against petitioner. Failure to was made from respondent Travellers for its share in
before he has incurred in delay will not have the substantiate the claim of subrogation is fatal to the loss but was refused. Hence, complainants
effect of extinguishing the obligation. This rule is petitioner’s case for recovery of the amount of demanded from the other 3 respondents the balance
based on the principle that the genus of a thing can P535,613.00. of each share in the loss based on the computation
never perish. Genus nunquan perit. An obligation Disposition Petition is partly GRANTED. The excluding Travellers Multi-Indemnity in the amount
to pay money is generic; therefore, it is not assailed Decision dated October 11, 2000 and of P30,894.31 (P5,732.79-ZIC: P22,294.62, PBAC:
excused by fortuitous loss of any specific Resolution dated April 11, 2001 of the Court of and P2,866.90, SSS) but was refused, hence, this
property of the debtor. Appeals in CA-G.R. CV No. 61848 are AFFIRMED action.
- Thus, whether fire is a fortuitous event or with the MODIFICATION that the order to pay the
petitioner was negligent are matters immaterial to amount of P535,613.00 to respondent is DELETED ISSUE
this case. What is relevant here is whether it has for lack of factual basis. WON petitioner Tai Tong has insurable interest in the
been established that petitioner has outstanding said policy
accounts with IMC and LSPI. TAI TONG CHUACHE & CO v. INSURANCE
- With respect to IMC, the respondent has COMMISSION and TRAVELLERS MULTI- HELD
adequately established its claim. Petitioner has an INDEMNITY CORPORATION YES
outstanding account with IMC in the amount of 158 SCRA 366 - First, respondent insurance commission based its
P2,119,205.00, check voucher evidencing payment GANCAYCO; February 29, 1988 findings on mere inference. Respondent Insurance
to IMC, subrogation receipt executed by IMC in favor Commission absolved respondent insurance company
of respondent upon receipt of the insurance NATURE from liability on the basis of the certification issued
proceeds. All these documents have been properly Petition for review on certiorari of the decision of the by the then CFI, that in a certain civil action against
identified, presented and marked as exhibits in Insurance Commission the Palomos, Arsenio Lopez Chua stands as the
court. The subrogation receipt, by itself, is sufficient complainant and not Tai Tong Chuache. From said
to establish not only the relationship of respondent FACTS evidence respondent commission inferred that the
as insurer and IMC as the insured, but also the - Complainants Palomo acquired a parcel of land and credit extended by herein petitioner to the Palomos
amount paid to settle the insurance claim. The right a building located in Davao City. They assumed the secured by the insured property must have been
of subrogation accrues simply upon payment by the mortgage of the building in favor of SSS, which paid. Such is a glaring error which this Court cannot
insurance company of the insurance claim. building was insured with respondent SSS Accredited sanction.
Respondent’s action against petitioner is squarely Group of Insurers for P25K. - Second, it has been held in a long line of cases that
sanctioned by Article 2207 of the Civil Code which - On April 19, 1975, Azucena Palomo obtained a when the creditor is in possession of the document of
provides: P100K loan from Tai Tong Chuache Inc. (TTCC) and credit, he need not prove non-payment for it is
Art. 2207. If the plaintiff’s property has been executed a mortgage over the land and the building presumed. The validity of the insurance policy taken
insured, and he has received indemnity from the in favor of Tai Tong Chuache & Co. as security of b petitioner was not assailed by private respondent.
insurance company for the injury or loss arising payment .On April 25, 1975, Arsenio Chua, Moreover, petitioner's claim that the loan extended
out of the wrong or breach of contract complained representative of TTCC insured the latter's interest to the Palomos has not yet been paid was
of, the insurance company shall be subrogated to with Travellers Multi-Indemnity Corporation corroborated by Azucena Palomo who testified that
the rights of the insured against the wrongdoer or (Travellers) for P100K (P70K for bldg and P30K for they are still indebted to herein petitioner. So at the
the person who has violated the contract. x x x the contents thereof) time of the fire, petitioner as mortgagee still had
- Petitioner failed to refute respondent’s evidence. - On June 11, 1975, Pedro Palomo secured a Fire insurable interest therein.
- As to LSPI, respondent failed to present sufficient Insurance Policy, covering the building for P50K with - And third, petitioner's declaration that Arsenio
evidence to prove its cause of action. No evidentiary respondent Zenith Insurance Corporation (ZIC). Lopez Chua acts as the managing partner of the
weight can be given to Exhibit “F Levi Strauss”, a Another Fire Insurance Policy was later procured partnership was corroborated by respondent
letter dated April 23, 1991 from petitioner’s General from respondent Philippine British Assurance insurance company. Thus Chua as the managing
Manager, Stephen S. Gaisano, Jr., since it is not an Company (PBAC), covering the same building for partner of the partnership may execute all acts of
INSURANCE Page
29

administration including the right to sue debtors of of nullity of the insurance contract in question. Mrs. he made a written application on December 26,
the partnership in case of their failure to pay their Perez filed a conterclaim 1932, to the defendant Insular Life Assurance Co.,
obligations when it became due and demandable. Or for the collection of Php150,000 plus damages. Ltd., through its agent, Cristobal Mendoza, for a
at the least, Chua being a partner of petitioner Tai policy of insurance on his life in the sum of P1,000
Tong Chuache & Company is an agent of the ISSUE and he paid to the agent P15 cash as part of the first
partnership. Being an agent, it is understood that he WON there was a consummated contract of premium. It was agreed with the agent that the
acted for and in behalf of the firm. insurance between Perez and BF policy, when and if issued, should be delivered to his
Disposition Appealed decision SET ASIDE and aunt. Felicidad Estrada, with whom Sindayen left the
ANOTHER judgment is rendered order private HELD sum of P26.06 to complete the payment of the first
respondent Travellers to pay petitioner the face NO annual premium of P40.06.
value of Fire Insurance Policy in the amount of - An essential requisite of a valid contract is consent. - On January 1, 1933, Sindayen, who was then
P100K. Costs against said private respondent. Consent must be manifested by the meeting of the twenty-nine years of age, was examined by the
offer and the acceptance upon the thing and the company's doctor who made a favorable report, to
cause which are to constitute the contract. the company. On January 11, 1933, The company
PEREZ v. CA (BF LIFEMAN INSURANCE CORP.) - The offer must be certain and the acceptance accepted the risk and issued policy No. 47710 dated
323 SCRA 613 absolute. When Perez filed the application, it was back to December 1, 1932, and mailed the same to
YNARES-SANTIAGO; January 28, 2000 subject to the acceptance of BF. The perfection was its agent, Cristobal Mendoza, in Camiling, Tarlac, for
also further conditioned upon (1) the issuance of the delivery to the insured. -On January 11, 1933,
NATURE policy, Sindayen was at work in the Bureau of Printing. On
Petition for review on certiorari (2) the payment of the premium, and (3) the January 12, he complained of a severe headache and
delivery to and acceptance by the applicant in good remained at home. On January 15, he called a
FACTS health. physician who found that he was suffering from
- Primitivo Perez has been insured with the BF - The delivery and acceptance by the applicant was a acute nephritis and uremia and on January 19, 1933,
Lifeman Insurance Corporation (BF hereafter) since suspensive condition which was not fulfilled he died.
1980 for Php20,000. inasmuch as the applicant was already dead at the - On January 18, 1933, the agent, in accordance
Sometime in 1987, Rodolfo Lalog (agent of BF) time the policy was issued. The non-fulfillment of the with his agreement with the insured, delivered the
convinced him to apply for additional insurance condition resulted policy to Felicidad Estrada upon her payment of the
coverage of Php50,000. in the non-perfection of the contract. balance of the first year's annual premium. The
Perez accomplished the application form and passed - An application for insurance is merely an offer agent asked Felicidad Estrada if her nephew was in
the required medical examination. He also paid which requires the overt act of the insurer for it to good health and she replied that she believed so
Php2,075 premium) to Lalog. ripen to a contract. Delay in acting on the application because she had no information that he was sick and
- On November 25, 1987, Perez died while riding a does not constitute acceptance even though the he thereupon delivered to her the policy.
banca which capsized during a storm. During this insured has forwarded - On January 20, 1933, the agent learned of the
time his application papers for the additional his first premium with his application. Delay, in this death of Arturo Sindayen and called on Felicidad
insurance coverage was still with the Gumaca, case, does not constitute gross negligence because Estrada and asked her to return the policy. But he
Quezon the application was granted within the normal did not return or offer to return the premium paid.
office of BF. processing time. Felicidad Estrada on his aforesaid statement gave
- Without knowing that Perez died on November 25, Disposition Decision of CA affirmed in so far as it him the policy.
1987, BF approved Perez's application and issued the declared the insurance policy for Php50,000 issued - On February 4, 1933 Insular Life obtained from the
corresponding policy for the Php50,000 on December by BF null and void (no recission because it beneficiary, Sindayen’s wife, her signature to a legal
2, 1987. presupposes a valid contract) document entitled "ACCORD, SATISFACTION AND
- Virginia Perez (wife of the deceased) claimed the RELEASE" whereby in consideration of the sum of
benefits under the insurance policies of the VDA. DE SINDAYEN v. INSULAR P40.06 paid to her by a check of the company, she
deceased, but she was only able to receive 62 Phil 51 "assigns, releases and forever discharges said Isular
Php40,000 under the first insurance policy. BUTTE; September 4, 1935 Life Assurance Co., Ltd., its successors and assigns,
BF refused to pay the proceeds amounting to of all claims, obligation in or indebtedness. The said
Php150,000 under the additional policy coverage of FACTS check for P40.06 was never cashed but returned to
Php50,000 because they maintain that such policy - Arturo Sindayen, up to the time of his death on the company and appears in the record of this case
had not been perfected. January 19, 1933, was employed as a linotype as Exhibit D. Thereupon this action was brought to
- On September 21, 1990, BF filed a complaint operator in the Bureau of Printing at Manila and had enforce payment of the policy.
against Mrs. Perez seeking recission and declaration been such for eleven years prior thereto. While there
INSURANCE Page
30

By the terms of the policy, an annual premium of in the case of MeLaurin vs. Mutual Life Insurance Co. premium and delivered the policy thereafter to the
P40.06 is due on the first day of December of each “It is plain, therefore, that upon the facts it is not insured, the company is now estopped to assert that
year, the first premium already paid by the insured necessarily a case of waiver or of estoppel, but a it never intended that the policy should take effect.
covering the period from December 1, 1932. It is to case where the local agents, in the exercise of the
December 1, 1933. It is to be noted that the policy powers lodged in them, accepted the premium and SEPARATE OPINION
was not issued and the company assumed no actual delivered the policy. That act binds their principal,
risk prior to January 11, 1933.The application which the defendant.” IMPERIAL [dissent]
the insured signed in Camiling, Tarlac, on December - Mendoza was duly licensed by the Insurance - "A local agent of an insurance company, whose
26, 1932, contained among others the following Commissioner to act as the agent of the defendant only power is to solicit applications for insurance,
provisions: insurance company. The well known custom of the and forward them to the company for approval,
“3 That the said policy shall not take effect until the insurance business and the evidence in this case when, if approved to the insured, has no power to
first premium has been paid and the policy has been prove that Mendoza was not regarded by the waive any of the provision of the policy so
delivered to and accepted by me, while I am in good company as a mere conduit or automaton for the delivered."
health.” performance of the physical act of placing the policy - It is clear, therefore, that the delivery of the policy
-Main defense of the company in this case, namely, in the hands of the insured by Mendoza does not bind the defendant, nor is the
that the said policy never took effect because of - Granted that Mendoza's decision that the condition defendant estopped from alleging its defense, for the
paragraph 3 of the application above quoted, for at had been met by the insured and that it was proper simple reason that Mendoza was not an agent with
the time of its delivery by the agent as aforesaid the to make a delivery of the policy to him is just as authority to issue policies or to accept risks in the
insured was not in good health binding on the company as if the decision had been name of his principle.
made by its board of directors. Granted that -There is another ground upon which the majority
ISSUE Mendoza made a mistake of judgement because he opinion is based, namely, that the defendant waived
WON the insurance policy is valid acted on insufficient evidence as to the state of the defense it now invokes, by reason of the delivery
health of the insured. But it is not charged that the of the policy by its invokes, by reason of the delivery
HELD mistake was induced by any misconduct or omission of the policy by its agent. It is admitted that if the
YES of duty of the insured. delivery of the policy was due to fraud, legally there
- There is one line of cases which holds that the - It is the interest not only the applicant but of all could have been no waiver. In view of the facts
stipulation contained in paragraph 3 is in the nature insurance companies as well that there should be established and admitted, there is no doubt, as to
of a condition precedent, that is to say, that there some act which gives the applicant the definite the existence of the fraud. -Estrada, as a
can be no valid delivery to the insured unless he is in assurance that the contract has been consummated. representative of the insured was not only bound to
good health at the time; that this condition This sense of security and of peace of mind that give a truthful information on the state of health of
precedent goes to the very essence of the contract one's defendants are provided for without risk either the insured, but it was her duty to find out it his true
and cannot be waived by the agent making delivery of loss or of litigation is the bedrock of life insurance. state of health in order to give true and correct
of the policy When the policy is issued and delivered, in the information. When she gave Mendoza an incorrect
- On the other hand, a number of American decisions absence of fraud or other grounds for rescission, it is information tending to create the impression that the
hold that an agent to whom a life insurance policy plainly not within the intention of the parties that insured was well when in fact he was seriously ill,
similar to the one here involved was sent with there should be any questions held in abeyance or there is no doubt that she committed fraud and
instructions to deliver it to the insured has authority reserved for future determination that leave the very imparted a deceitful information to the defendant
to bind the company by making such delivery, existence of the contract in suspense and doubt. agent
although the insured was not in good health at the - It is therefore in the public interest, for the public is
time of delivery, on the theory that the delivery of profoundly and generally interested in life insurance, ENRIQUEZ v. SUN LIFE OF CANADA
the policy being the final act to the consummation of as well as in the interest of the insurance companies 41 PHIL 269
the contract, the condition as to the insurer's good themselves by giving certainly and security to their MALCOLM; November 29, 1920
health was waived by the company. policies, that we are constrained to hold, as we, do,
- we are inclined to the view that it is more that the delivery of the policy to the insured by an NATURE
consonant with the well known practice of life agent of the company who is authorized to make Appeal from judgment of trial court denying
insurance companies and the evidence in the present delivery or without delivery is the final act which plaintiff’s (administrator of the estate of the late
case to rest our decision on the proposition that binds the company (and the insured as well) in the Joaquin Ma. Herrer) action to recover from the
Mendoza was authorized by the company to make absence of fraud or other legal ground for rescission defendant life insurance company the sum of pesos
the delivery of the policy when he received the - The company therefore having decided that all the 6,000 paid by the deceased for a life annuity.
payment of the first premium and he was satisfied conditions precedent to the taking effect of the policy
that the insured was in good health. As was well said had been complied with and having accepted the FACTS
INSURANCE Page
31

- On September 24, 1917, Joaquin Herrer made cable the Manila office to that effect, did actually
application to the Sun Life Assurance Company of HELD issue the policy and did, through its agent in Manila,
Canada through its office in Manila for a life annuity. NO actually write the letter of notification and place it in
Two days later he paid the sum of P6,000 to the Ratio The law applicable to the case is found to be the usual channels for transmission to the
manager of the company's Manila office and was the second paragraph of article 1262 of the Civil addressee.
given a receipt. Code providing that an acceptance made by letter - The contract for a life annuity in the case at bar
- The application was immediately forwarded to the shall not bind the person making the offer except was not perfected because it has not been proved
head office of the company at Montreal, Canada. On from the time it came to his knowledge. satisfactorily that the acceptance of the application
November 26, 1917, the head office gave notice of Reasoning ever came to the knowledge of the applicant.
acceptance by cable to Manila. (Whether on the - Until quite recently, all of the provisions concerning Disposition Judgment is reversed, and the plaintiff
same day the cable was received, notice was sent by life insurance in the Philippines were found in the shall have and recover from the defendant the sum
the Manila office of Herrera that the application had Code of Commerce and the Civil Code. After July 1, of P6,000 with legal interest from November 20,
been accepted, is a disputed point, which will be 1915, there was, however, in force the Insurance 1918, until paid, without special finding as to costs in
discussed later.) On December 4, 1917, the policy Act. No. 2427. Chapter IV of this Act concerns life either instance.
was issued at Montreal. On December 18, 1917, and health insurance. The Act expressly repealed
attorney Aurelio A. Torres wrote to the Manila office Title VIII of Book II and Section III of Title III of
of the company stating that Herrer desired to Book III of the code of Commerce. The law of
withdraw his application. The following day the local insurance is consequently now found in the
office replied to Mr. Torres, stating that the policy Insurance Act and the Civil Code. VELASCO and ACOSTA v. APOSTOL and
had been issued, and called attention to the - While, as just noticed, the Insurance Act deals with MAHARLIKA INSURANCE CO., INC.
notification of November 26, 1917. This letter was life insurance, it is silent as to the methods to be 173 SCRA 228
received by Mr. Torres on the morning of December followed in order that there may be a contract of REGALADO.; May 9, 1989
21, 1917. Mr. Herrer died on December 20, 1917. insurance. On the other hand, the Civil Code, in
- The chief clerk of the Manila office of Sun Life article 1802, not only describes a contact of life NATURE
testified that he prepared the letter and handed it to annuity markedly similar to the one we are Petition for review on certiorari
the local manager, Mr. E. E. White, for signature. considering, but in two other articles, gives strong
The local manager, Mr. White, testified to having clues as to the proper disposition of the case. For FACTS
received the cablegram accepting the application of instance, article 16 of the Civil Code provides that - Petitioners were plaintiffs in a civil case of which
Mr. Herrer from the home office on November 26, "In matters which are governed by special laws, any public respondent Hon. Apostol was the judge.
1917. He said that on the same day he signed a deficiency of the latter shall be supplied by the - The case was an offshoot of an incident: plaintiffs
letter notifying Mr. Herrer of this acceptance. The provisions of this Code." On the supposition, were riding in their car, when a taxicab crossed a
witness further said that letters, after being signed, therefore, which is incontestable, that the special law center island in the road and collided with their car.
were sent to the chief clerk and placed on the on the subject of insurance is deficient in enunciating Private respondent Maharlika was eventually
mailing desk for transmission. Mr. Tuason, who was the principles governing acceptance, the subject- impleaded as a defendant in this case, with an
the chief clerk on November 26, 1917, was not matter of the Civil code, if there be any, would be allegation that the taxicab involved was insured
called as a witness. controlling. In the Civil Code is found article 1262 against third party liability for P20,000.00 with
- For the defense, attorney Manuel Torres testified to providing that "Consent is shown by the concurrence private respondent at the time of the accident
having prepared the will of Joaquin Ma. Herrer. That of offer and acceptance with respect to the thing and - Maharlika claimed there was no cause of action
on this occasion, Mr. Herrer mentioned his the consideration which are to constitute the against it because at the time of the accident, the
application for a life annuity, and that he said that contract. An acceptance made by letter shall not bind alleged insurance policy was not in force due to the
the only document relating to the transaction in his the person making the offer except from the time it non-payment of the premium thereon. Also, even if
possession was the provisional receipt. Rafael came to his knowledge. the cab had been insured, the complaint would be
Enriquez, the administrator of the estate, testified - According to the provisional receipt, three things premature since the policy provides that the insurer
that he had gone through the effects of the deceased had to be accomplished by the insurance company would be liable only when the insured becomes
and had found no letter of notification from the before there was a contract: (1) There had to be a legally liable.
insurance company to Mr. Herrer. medical examination of the applicant; (2) there had - Trial court ruled in favor of the plaintiff, holding the
to be approval of the application by the head office defendants liable for repair of the car, medical
ISSUE of the company; and (3) this approval had in some expenses, etc. BUT Maharlike was exonerated on the
WON there exists a contract for life annuity between way to be communicated by the company to the gnd that the policy was not in force.
Herrer and defendant applicant. The further admitted facts are that the - Petitioners elevated this case to this court, faulting
head office in Montreal did accept the application, did the respondent judge for considering the defense of
INSURANCE Page
32

late payment of premium when “the same was deleted the clause "unless there is clear agreement moral damages, and attorney's fees equivalent to
waived at the pre-trial”, hence the evidence of late to grant the insured credit extension of the premium 20% of the total claim. The trial court ruled for
payment should be disregarded supposedly because due" which was then involved in this controversy. petitioners. CA reversed.
the private respondent had admitted that such fact Disposition Fnding no reversible error, the
was not in issue. judgment appealed from is hereby AFFIRMED. ISSUE
- (More pertinent to this class: ) petitioners assert WON a fire insurance policy is valid, binding and
that the private respondent had agreed to grant the enforceable upon mere partial payment of premium
then prospective insured a credit extension of the
premium due. HELD
- This controversy arose under the old insurance law, NO
Act No. 2427. TIBAY v. CA (FORTUNE LIFE & GENERAL Ratio Where the insurer and the insured expressly
- The accident occurred in 1973. The complaint was INSURANCE) stipulated that the policy is not in force until the
filed on July 20, 1974.  both before the 257 SCRA 126 premium has been fully paid the payment of partial
effectivity of Presidential Decree no. 612, the BELLOSILLO; May 24, 1996 premium by the assured in this particular instance
subsequent insurance law which repealed its should not be considered the payment required by
predecessor FACTS the law and the stipulation of the parties. Rather, it
- The former insurance law, which applies to the - On 22 January 1987 Fortune Life and General must be taken in the concept of a deposit to be held
case here, provided: An insurer is entitled to the Insurance Co., Inc. (FORTUNE) issued Fire Insurance in trust by the insurer until such time that the full
payment of premium as soon as the thing insured is Policy No. 136171 in favor of Violeta R. Tibay and/or amount has been tendered and duly receipted for.
exposed to the peril insured against, unless there is Nicolas Roraldo on their two-storey residential Reasoning
clear agreement to grant the insured credit building located at 5855 Zobel Street, Makati City, - As expressly agreed upon in the contract, full
extension of the premium due. No policy issued by together with all their personal effects therein. The payment must be made before the risk occurs for the
an insurance company is valid and binding unless insurance was for P600,000 covering the period from policy to be considered effective and in force. Thus,
and until the premium thereof has been paid. 23 January 1987 to 23 January 1988. On 23 January no vinculum juris whereby the insurer bound itself to
1987, of the total premium of P2,983.50, Violeta indemnify the assured according to law ever resulted
ISSUE Tibay only paid P600 thus leaving a considerable from the fractional payment of premium. The
WON the insurance policy would be valid and binding balance unpaid. insurance contract itself expressly provided that the
notwithstanding the non-payment of the premium - On 8 March 1987 the insured building was policy would be effective only when the premium
completely destroyed by fire. Two days later, Violeta was paid in full. It would have been altogether
HELD Tibay paid the balance of the premium. On the same different were it not so stipulated. Ergo, petitioners
NO day, she filed with FORTUNE a claim on the fire had absolute freedom of choice whether or not to be
Ratio Act No. 2427: an insurance policy would be insurance policy. Her claim was accordingly referred insured by FORTUNE under the terms of its policy
valid and binding notwithstanding the non-payment to its adjuster, Goodwill Adjustment Services, Inc. and they freely opted to adhere thereto.
of the premium if there was a clear agreement to (GASI), which immediately wrote Violeta requesting - Indeed, and far more importantly, the cardinal
grant to the insured credit extension. Such her to furnish it with the necessary documents for polestar in the construction of an insurance contract
agreement may be express or implied. the investigation and processing of her claim. is the intention of the parties as expressed in the
Reasoning Petitioner forthwith complied. On 28 March 1987 she policy. Courts have no other function but to enforce
- Petitioners maintain that in spite of their late signed a nonwaiver agreement with GASI to the the same. The rule that contracts of insurance will be
payment, the policy is binding because there was an effect that any action taken by the companies shall construed in favor of the insured and most strongly
implied agreement to grant a credit extension so as not be, or be claimed to be, an admission of liability. against the insurer should not be permitted to have
to make the policy effective. To them, the - FORTUNE denied the claim of Violeta for violation the effect of making a plain agreement ambiguous
subsequent acceptance of the premium and delivery of Policy Condition No. 2♪ and of Sec. 77 of the and then construe it in favor of the insured. Verily, it
of the policy estops the respondent company from Insurance Code. Efforts to settle the case before the is elemental law that the payment of premium is
asserting that the policy is ineffective. Insurance Commission proved futile. On 3 March requisite to keep the policy of insurance in force. If
The court however sees no proof of any such implied 1988 Violeta and the other petitioners sued the premium is not paid in the manner prescribed in
agreement. The purported nexus between the FORTUNE for damages in the amount of P600,000 the policy as intended by the parties the policy is
delivery of the policy and the grant of credit representing the total coverage of the fire insurance ineffective. Partial payment even when accepted as a
extension is too tenuous to support the conclusion policy plus 12% interest per annum, P100,000 partial payment will not keep the policy alive even
for which petitioners contend. for such fractional part of the year as the part

 Parenthetically mention: in the present law, This policy including any renewal thereof and/or any endorsement payment bears to the whole payment.
Section 77 of the Insurance Code of 1978 has thereon is not in force until the premium has been fully paid to and Disposition Petition is DENIED. Decision of the CA
duly receipted by the Company in the manner provided herein.
INSURANCE Page
33

is AFFIRMED. - A third policy was again issued for the period 1 failure to pay said premiums on or before the
March 1984 to 1 March 1985. For this, petitioner effective dates of said policies rendered them invalid.
made two installment payments, both accepted by Petitioner thus concludes that there cannot be a
AHAC. Thereafter, petitioner refused to pay the perfected contract of insurance upon mere partial
SEPARATE OPINION balance of the premium. AHAC filed an action to payment of the premiums because under Sec. 77 of
recover the unpaid balance of P314,103.05. the Insurance Code, no contract of insurance is valid
VITUG [dissent] - Petitioner explained that it discontinued the and binding unless the premium thereof has been
- The law neither requires, nor measures the payment of premiums because the policy did not paid, notwithstanding any agreement to the
strength of the vinculum juris by, any specific contain a credit clause in its favor and the receipts contrary.
amount of premium payment. It should thus be for the installment payments covering the policy for
enough that payment on the premium, partly or in 1984-85, as well as the two (2) previous policies, ISSUE
full, is made by the insured which the insurer stated the following reservations: WON payment by installment of the premiums due
accepts. In fine, it is either that a juridical tie exists 2. Acceptance of this payment shall not waive on an insurance policy invalidates the contract of
(by such payment) or that it is not extant at all (by any of the company rights to deny liability on any insurance
an absence thereof). Once the juridical relation claim under the policy arising before such
comes into being, the full efficacy, not merely pro payments or after the expiration of the credit HELD
tanto, of the insurance contract naturally follows. clause of the policy; and Ratio Where the risk is entire and the contract is
Verily, not only is there an insurance perfected but 3. Subject to no loss prior to premium payment. indivisible, the insured is not entitled to a refund of
also a partially performed contract. In case of loss, If there be any loss such is not covered. the premiums paid if the insurer was exposed to the
recovery on the basis of the full contract value, less - Petitioner further claimed that the policy was never risk insured for any period, however brief or
the unpaid premium can accordingly be had; binding and valid, and no risk attached to the policy. momentary.
conversely, if no loss occurs, the insurer can demand It then pleaded a counterclaim for P152k for the Reasoning
the payment of the unpaid balance of the premium. premiums already paid for 1984-85, and in its - The obligation to pay premiums when due is
The insured, on the one hand, cannot avoid the answer with amended counterclaim, sought the ordinarily as indivisible obligation to pay the entire
obligation of paying the balance of the premium refund of P924,206.10 representing the premium premium. Here, the parties herein agreed to make
while the insurer, upon the other hand, cannot treat payments for 1982-85. the premiums payable in installments, and there is
the contract as valid only for the purpose of - Trial court dismissed the complaint and the no pretense that the parties never envisioned to
collecting premiums and as invalid for the purpose of counterclaim upon the following findings: (1) make the insurance contract binding between them.
indemnity. payment of the premiums of the three policies were And the insured never informed the insurer that it
made during the term of said policies, hence, it could was terminating the policy because the terms were
MAKATI TUSCANY v. CA ( AMERICAN HOME not be said, inspite of the reservations, that no risk unacceptable.
ASSURANCE CO.) attached under the policies; (2) as regards the - There is nothing in Section 77 which suggests that
215 SCRA 462 unpaid premiums, in view of the reservation in the the parties may not agree to allow payment of the
BELLOSILLO; November 6, 1992 receipts ordinarily issued by AHAC on premium premiums in installment, or to consider the contract
payments the only plausible conclusion is that AHAC as valid and binding upon payment of the first
NATURE has no right to demand their payment after the lapse premium.
Appeal from decision of the CA of the term of said policy on March 1, 1985. - The records clearly show that petitioner and private
Therefore, Tuscany was justified in refusing to pay respondent intended subject insurance policies to be
FACTS the same. binding and effective notwithstanding the staggered
- American Home Assurance Co. (AHAC), - CA modified the decision by ordering Tuscany to payment of the premiums. Acceptance of payments
represented by American International Underwriters pay the balance of the premiums due on the third speaks loudly of the insurer's intention to honor the
(Phils.), Inc., issued in favor of petitioner Makati policy plus legal interest until fully paid, and policies it issued to petitioner.
Tuscany Condominium Corporation an insurance affirming the denial of the counterclaim. - Section 78 of the Insurance Code in effect allows
policy on the latter's building and premises, for the Petitioner’s Claims waiver by the insurer of the condition of prepayment
period 1 March 1982 to1 March 1983. The premium Petitioner argues that where the premiums is not by making an acknowledgment in the insurance
was paid on installments all of which were accepted actually paid in full, the policy would only be policy of receipt of premium as conclusive evidence
by AHAC. effective if there is an acknowledgment in the policy of payment so far as to make the policy binding
- A second policy was issued to renew the first one, of the receipt of premium pursuant to Sec. 78 of the despite the fact that premium is actually unpaid.
this time covering the period 1 March 1983 to 1 Insurance Code. The absence of an express Section 77 merely precludes the parties from
March 1984. This was also pain in installment basis. acknowledgment in the policies of such receipt of the stipulating that the policy is valid even if premiums
corresponding premium payments, and petitioner's are not paid, but does not expressly prohibit an
INSURANCE Page
34

agreement granting credit extension, and such an damage had already been inflicted on him and no
agreement is not contrary to morals, good customs, FACTS amount of rectification could remedy the same.
public order or public policy. - June 29, 1985- 7 months after the issuance of Respondent’s Argument
- At the very least, both parties should be deemed in Santos Areola's Personal Accident Insurance Policy - Prudential argues that where reinstatement, the
estoppel to question the arrangement they have No. PA-20015 (covering a period of one year), equitable relief sought by Areola was granted at an
voluntarily accepted. Prudential unilaterally cancelled the same since opportune moment, i.e. prior to the filing of the
Disposition Judgment affirmed. Costs against company records revealed that Areola failed to pay complaint, Areola is left without a cause of action on
petitioner. his premiums. which to predicate his claim for damages.
o Under the terms of the statement of account - Reinstatement effectively restored Areola to all his
SOUTH SEA SURETY AND INSURANCE v. CA issued by Prudential, Areola was supposed to rights under the policy.
(VALENZUELA HARDWOOD) pay the total amount of P1,609.65 which
244 SCRA 744 included the premium of P1,470.00, ISSUES
VITUG; June 2, 1995 documentary stamp of P110.25 and 2% 1. WON the erroneous act of canceling subject
premium tax of P29.40. insurance policy entitle petitioner-insured to
NATURE o The statement of account stated that it must not payment of damages
Petition for review on certiorari be considered a receipt as an official receipt will 2. WON the subsequent act of reinstating the
be issued upon payment of the account. And if wrongfully cancelled insurance policy obliterate
FACTS payment was made to a representative, the whatever liability for damages Prudential has
- Hardwood entered into agreement with Seven Bros client must demand for a Provisional Receipt and
Shipping, where latter undertook to load the former’s if Official Receipts aren’t received within 7 days, HELD
logs on vessel. Hardwood insured the logs with Prudential should be notified. If payment is 1. YES
South Sea Surety which issued Marine Cargo made to their office, clients should demand for 2. NO
Insurance Policy. The vessel sank Jan 25, 1984. an OR. Reasoning
- Hardwood filed claim with South Sea and Seven - August 3, 1985- Prudential offered to reinstate - Malapit's fraudulent act of misappropriating the
Bros. Trial Court favored Hardwood. CA decided same policy it had previously cancelled and even premiums paid by petitioner-insured is beyond doubt
against South Sea, but absolved Seven Bros. South proposed to extend its lifetime to December 17, directly imputable to Prudential.
Sea filed this instant petition. 1985, upon a finding that the cancellation was - A corporation, such as respondent insurance
erroneous and that the premiums were paid in full by company, acts solely thru its employees. The latter’s
ISSUES Areola but were not remitted by Teofilo M. Malapit, acts are considered as its own for which it can be
WON the insurance contract was already in effect Prudential's branch manager. held to account.
when the vessel sank Petitioners’ Claims - The facts are clear as to the relationship between
- The fraudulent act of in misappropriating Areola’s private respondent insurance company and Malapit.
HELD premium payments is the proximate cause of the His act of receiving the premiums collected is well
YES cancellation of the insurance policy. within the province of his authority as manager.
- It is already in effect because Hardwood has - Areola theorized that Malapit's act of signing and Thus, his receipt of said premiums is receipt by
already paid the insurance premium. even sending the notice of cancellation himself, private respondent insurance company who, by
It delivered the check to Victorio Chua before the notwithstanding his personal knowledge of provision of law, particularly under Article 1910 of
vessel sank, but Victorio Chua was only to deliver petitioner-insured's full payment of premiums, the Civil Code, is bound by the acts of its agent.
the check to South Sea five days after the vessel further reinforces the allegation of bad faith. - Article 1910 thus reads:
sank. - Such fraudulent act committed by Malapit is Art. 1910. The principal must comply with all the
Appellant argues that Chua was not its broker, but it attributable to Prudential. obligations which the agent may have contracted
was found that Chua was authorized by South Sea to - Malapit's actuations are therefore not separate and within the scope of his authority.
receive the premium on its behalf. distinct from that of Prudential’s. It must, therefore, As for any obligation wherein the agent has
bear the consequences of the erroneous cancellation exceeded his power, the principal is not bound
of subject insurance policy caused by the non- except when he ratifies it expressly or tacitly.
AREOLA v. CA (PRUDENTIAL GUARANTEE AND remittance by its own employee of the premiums - Malapit's failure to remit the premiums he received
ASSURANCE, INC.) paid. cannot constitute a defense for private respondent
236 SCRA 643 - Subsequent reinstatement could not possibly insurance company; no exoneration from liability
ROMERO; September 22, 1994 absolve respondent insurance company from liability, could result therefrom.
there being an obvious breach of contract. After all
NATURE CERTIORARI
INSURANCE Page
35

- Prudential’s earlier act of reinstating the insurance breach of contract and no substantial injury or actual policies covering respondent's insured property razed
policy can not obliterate the injury inflicted on damages whatsoever have been or can be shown. by fire, and for attorney's fees.
petitioner-insured. Disposition Petition for review on certiorari is - On October 23, 1992, after its motion to dismiss
- Respondent company should be reminded that a hereby GRANTED. RTC’ s DECISION is REINSTATED. had been denied, petitioner filed an answer to the
contract of insurance creates reciprocal obligations complaint. It alleged that the complaint "fails to
for both insurer and insured. UCPB GENERAL INSURANCE CO., INC. v. state a cause of action"; that petitioner was not
- Reciprocal obligations are those which arise from MASAGANA TELAMART, INC. liable to -respondent for insurance proceeds under
the same cause and in which each party is both a 308 SCRA 259 the policies because at the time of the loss of
debtor and a creditor of the other, such that the PARDO; June 15, 1999 respondent's property due to fire, the policies had
obligation of one is dependent upon the obligation of long expired and were not renewed.
the other. NATURE After due trial, on March 10, 1993, the Regional Trial
- Under the circumstances of instant case, the Petition for review on certiorari of a decision of the Court, Branch 58, Makati, rendered decision, the
relationship as creditor and debtor between the Court of Appeals. dispositive portion of which reads:
parties arose from a common cause: i.e., by reason "WHEREFORE, premises considered, judgment is
of their agreement to enter into a contract of FACTS hereby rendered in favor of the plaintiff and against
insurance under whose terms, Prudential promised - On April 15, 1991, petitioner issued five (5) the defendant, as follows.
to extend protection to Areola against the risk insurance policies covering respondent's various "(1) Authorizing and allowing the plaintiff to
insured for a consideration in the form of premiums property described therein against fire, for the period consign/deposit with this Court the sum of
to be paid by the latter. from May 22, 1991 to May 22, 1992. P225,753.95 (refused by the defendant) as full
- Under the law governing reciprocal obligations, - In March 1992, petitioner evaluated the policies payment of the corresponding premiums for the
particularly the second paragraph of Article 1191, and decided not to renew them upon expiration of replacement-renewal policies for Exhibits A, B, C, D
the injured party, Areola in this case, is given a their terms on May 22, 1992. Petitioner advised and E; "(2) Declaring plaintiff to have fully complied
choice between fulfillment or rescission of the respondent's broker, Zuellig Insurance Brokers, Inc. with its obligation to pay the premium thereby
obligation in case one of the obligors, such as of its intention not to renew the policies. rendering the replacement-renewal policy of Exhibits
respondent insurance company, fails to comply with - On April 6, 1992, petitioner gave written notice to A, B, C, D and E effective and binding for the
what is incumbent upon him. respondent of the non-renewal of the policies at the duration May 22, 1992 until May 22, 1993; and,
- However, said article entitles the injured party to address stated in the policies. ordering defendant to deliver forthwith to plaintiff
payment of damages, regardless of whether he - On June 13, 1992, fire razed respondent's property the said replacement-renewal policies; "(3) Declaring
demands fulfillment or rescission of the obligation. covered by three of the insurance policies petitioner Exhibits A & B, in force from August 22, 1991 up to
- Untenable then is reinstatement insurance issued. August 23, 1992 and August 9, 1991 to August 9,
company's argument, namely, that reinstatement - On July 13, 1992, respondent presented to 1992, respectively; and "(4) Ordering the defendant
being equivalent to fulfillment of its obligation, petitioner's cashier at its head office five (5) to pay plaintiff the sums of. (a) P18,645,000.00
divests petitioner-insured of a rightful claim for manager's checks in the total amount of representing the latter's claim for indemnity under
payment of damages. Such a claim finds no support P225,753.95, representing premium for the renewal Exhibits A, B & C and/or its replacement-renewal
in our laws on obligations and contracts. of the policies from May 22, 1992 to May 22, 1993. policies; (b) 25% of the total amount due as and for
DAMAGES: No notice of loss was filed by respondent under the attorney's fees; (c) P25,000.00 as necessary
- The nature of damages to be awarded, however, policies prior to July 14, 1992. litigation expenses; and, (d) the costs of suit.
would be in the form of nominal damages - On July 14, 1992, respondent filed with petitioner “xxx ”
- Although the erroneous cancellation of the its formal claim for indemnification of the insured - In due time, petitioner appealed to the Court of
insurance policy constituted a breach of contract, property razed by fire. On the same day, petitioner Appeals (CA). The CA promulgated its decision
Prudential within a reasonable time took steps to returned to respondent the five manager's checks affirming that of the Regional Trial Court with the
rectify the wrong committed by reinstating the that it tendered, and at the same time rejected modification that item No. 3 of the dispositive
insurance policy of petitioner. respondent's claim for the reasons (a) that the portion was deleted, and the award of attorney's fees
- Moreover, no actual or substantial damage or policies had expired and were not renewed, and (b) was reduced to 10% of the total amount due.
injury was inflicted on petitioner Areola at the time that the fire occurred on June 13, 1992, before It held that following previous practice, respondent
the insurance policy was cancelled. respondent's tender of premium payment. was allowed a 60- to 90-day credit term for the
- Nominal damages are "recoverable where a legal - On July, 21, 1992, respondent filed with the renewal of its policies, and that the acceptance of
right is technically violated and must be vindicated Regional Trial Court, Branch 58, Makati City, a civil the late premium payment suggested an
against an invasion that has produced no actual complaint against petitioner for recovery, of understanding that payment could be made later.
present loss of any kind, or where there has been a P18.645,000.00, representing the face value of the Hence, this appeal.
INSURANCE Page
36

Co., Inc. v. Court of Appeals; and Tibay v. Court of that Respondent was fully aware of the notice of
ISSUE Appeals. Accordingly, it reversed and set aside the non-renewal. A reading of Section 66 of the
WON the fire insurance policies issued by petitioner decision of the Court of Appeals. Insurance Code readily shows that in order for an
to the respondent covering the period May 22, 1991 - Respondent seasonably filed a motion for the insured to be entitled to a renewal of a non-life
to May 22, 1992, had expired on the latter date or reconsideration of the adverse verdict. It alleges in policy, payment of the premium due on the effective
had been extended or renewed by an implied credit the motion that the SC had made in the decision its date of renewal should first be made. Respondent’s
arrangement though actual payment of premium was own findings of facts, which are not in accord with argument that Section 77 is not a prohibitive
tendered on a later date after the occurrence of the those of the trial court and the Court of Appeals. provision finds no authoritative support.
risk (fire) insured against The courts below correctly found that no notice of - The following facts, as found by the trial court and
non-renewal was made within 45 days before 22 May the Court of Appeals, are indeed duly established:
HELD 1992, or before the expiration date of the fire 1. For years, Petitioner had been issuing fire
NO insurance policies. Thus, the policies in question policies to the Respondent, and these policies
- An insurance policy, other than life, issued were renewed by operation of law and were effective were annually renewed.
originally or on renewal, is not valid and binding until and valid on 30 June 1992 when the fire occurred, 2. Petitioner had been granting Respondent a
actual payment of the premium. Any agreement to since the premiums were paid within the 60- to 90- 60- to 90-day credit term within which to pay
the contrary is void. The parties may not agree day credit term. the premiums on the renewed policies.
expressly or impliedly on the extension of credit or - Respondent likewise disagrees with its ruling that 3. There was no valid notice of non-renewal of
time to pay the premium and consider the policy parties may neither agree expressly or impliedly on the policies in question, as there is no proof at
binding before actual payment. the extension of credit or time to pay the premium all that the notice sent by ordinary mail was
Disposition Judgment reversed and set aside nor consider a policy binding before actual payment. received by Respondent, and the copy thereof
It urges the Court to take judicial notice of the fact allegedly sent to Zuellig was ever transmitted
that despite the express provision of Section 77 of to Respondent.
the Insurance Code, extension of credit terms in 4. The premiums for the policies in question in
premium payment has been the prevalent practice in the aggregate amount of P225,753.95 were
the insurance industry. Most insurance companies, paid by Respondent within the 60- to 90-day
including Petitioner, extend credit terms because credit term and were duly accepted and
Section 77 of the Insurance Code is not a prohibitive received by Petitioner’s cashier.
injunction but is merely designed for the protection
of the parties to an insurance contract. The Code ISSUE
itself, in Section 78, authorizes the validity of a WON Sec. 77 of the Insurance Code of 1978 must be
UCPB GENERAL INSURANCE CO., INC. v. policy notwithstanding non-payment of premiums. strictly applied to Petitioner’s advantage despite its
MASAGANA TELAMART, INC. (EN BANC) - Respondent also asserts that the principle of practice of granting a 60- to 90-day credit term for
356 SCRA 307 estoppel applies to Petitioner. Despite its awareness the payment of premiums
DAVIDE; April 4, 2001 of Section 77 Petitioner persuaded and induced HELD
Respondent to believe that payment of premium on NO
NATURE the 60- to 90-day credit term was perfectly alright; - Section 77 of the Insurance Code of 1978 provides:
Motion for reconsideration of the decision of the in fact it accepted payments within 60 to 90 days SEC. 77. An insurer is entitled to payment of the
Supreme Court. after the due dates. By extending credit and premium as soon as the thing insured is exposed
habitually accepting payments 60 to 90 days from to the peril insured against. Notwithstanding any
FACTS the effective dates of the policies, it has implicitly agreement to the contrary, no policy or contract of
- In its decision of 15 June 1999, the SC defined the agreed to modify the tenor of the insurance policy insurance issued by an insurance company is valid
main issue to be “whether the fire insurance policies and in effect waived the provision therein that it and binding unless and until the premium thereof
issued by petitioner to the respondent covering the would pay only for the loss or damage in case the has been paid, except in the case of a life or an
period from May 22, 1991 to May 22, 1992 had been same occurred after payment of the premium. industrial life policy whenever the grace period
extended or renewed by an implied credit - Petitioner filed an opposition to the Respondent’s provision applies.
arrangement though actual payment of premium was motion for reconsideration. It argues that both the - This Section is a reproduction of Section 77 of P.D.
tendered on a later date and after the occurrence of trial court and the Court of Appeals overlooked the No. 612 (The Insurance Code) promulgated on 18
the (fire) risk insured against.” The Court resolved fact that on 6 April 1992 Petitioner sent by ordinary December 1974. In turn, this Section has its source
this issue in the negative in view of Section 77 of the mail to Respondent a notice of non-renewal and sent in Section 72 of Act No. 2427 otherwise known as
Insurance Code and its decisions in Valenzuela v. by personal delivery a copy thereof to Respondent’s the Insurance Act as amended by R.A. No. 3540,
Court of Appeals; South Sea Surety and Insurance broker, Zuellig. Both courts likewise ignored the fact approved on 21 June 1963, which read:
INSURANCE Page
37

SEC. 72. An insurer is entitled to payment of recovery on the policy would not be permitted reversing the judgment of the Court of First Instance
premium as soon as the thing insured is exposed against Petitioner, which had consistently granted a of Rizal, Branch XII, at Caloocan City, which had
to the peril insured against, unless there is clear 60- to 90-day credit term for the payment of allowed such recovery.
agreement to grant the insured credit extension of premiums despite its full awareness of Section 77.
the premium due. No policy issued by an Estoppel bars it from taking refuge under said FACTS
insurance company is valid and binding unless and Section since Respondent relied in good faith on such - ACME Shoe Rubber and Plastic Corporation (ACME)
until the premium thereof has been paid. practice. Estoppel then is the fifth exception to had been insuring yearly against fire its building,
(Underscoring supplied) Section 77. machines and general merchandise with Domestic
- It can be seen at once that Section 77 does not Disposition Judgment reconsidered and set aside, Insurance Company (INSURER) since 1946. On May
restate the portion of Section 72 expressly that of the Court of Appeals affirmed in toto. 14, 1962, ACME continued to insure its properties
permitting an agreement to extend the period to pay with INSURER in the amount of P200,000 for the
the premium. But there are exceptions to Section SEPARATE OPINION period May 15, 1962 up to May 15, 1963.
77. - On May 14, 1963, INSURER issued Renewal Receipt
The first exception is provided by Section 77 itself, VITUG to cover the period May 15, 1963 to May 15, 1964.
and that is, in case of a life or industrial life policy - An essential characteristic of an insurance is its - On January 8, 1964, ACME paid P3,331.26 as
whenever the grace period provision applies. being synallagmatic, a highly reciprocal contract premium. The INSURER applied the payment as
The second is that covered by Section 78 of the where the rights and obligations of the parties renewal premium for the period of May 15, 1963 to
Insurance Code, which provides: correlate and mutually correspond. May 15, 1964.
SEC. 78. Any acknowledgment in a policy or - By weight of authority, estoppel cannot create a - On May 15, 1964, INSURER issued a Renewal
contract of insurance of the receipt of premium is contract of insurance, neither can it be successfully Receipt for the period of May 15, 1964 to May 15,
conclusive evidence of its payment, so far as to invoked to create a primary liability, nor can it give 1965 (for renewal premium of P3,331.26 yet to be
make the policy binding, notwithstanding any validity to what the law so procribes as a matter of paid) with a stamped note that says that the
stipulation therein that it shall not be binding until public policy. insurance will be deemed valid and binding only
premium is actually paid. when the premium and documentary stamps have
- A third exception was laid down in Makati Tuscany PARDO [dissent] actually been paid in full and duly acknowledged in
Condominium Corporation vs. Court of Appeals, - An assured’s failure to give notice of the fire an official receipt. ACME was given 90 days to pay
wherein we ruled that Section 77 may not apply if immediately upon its occurrence blatantly showed otherwise the policy would automatically become
the parties have agreed to the payment in the fraudulent character of its claims. Respondent is void and ineffective. (ACME should pay short period
installments of the premium and partial payment has required by law and by express terms of the policy to premium for 90 days before the period expires. If
been made at the time of loss. Tuscany has give immediate written notice of loss. This must be they are able to pay the whole amount before the
provided a fourth exception to Section 77, namely, complied with in the utmost good faith. 90-day period, the automatic termination won’t
that the insurer may grant credit extension for the - Assuming arguendo that the 60- to 90-day credit apply anymore).
payment of the premium. This simply means that if has been agreed between the parties, respondent - On May 26, 1964, ACME, through its President,
the insurer has granted the insured a credit term for could not still invoke estoppel to back up its claim. signed a promissory note saying that they promise to
the payment of the premium and loss occurs before Estoppel cannot give validity to an act that is pay the premium and documentary stamps and
the expiration of the term, recovery on the policy prohibited by law or against public policy. The actual agreed to the automatic cancellation penalty for not
should be allowed even though the premium is paid payment of premiums is a condition precedent to the complying.
after the loss but within the credit term. validity of an insurance contract other than life - On October 13, 1964, ACME’s properties were
Moreover, there is nothing in Section 77 which insurance policy. Any agreement to the contrary is completely destroyed by fire. ACME filed insurance
prohibits the parties in an insurance contract to void as against law and public policy. claim but the INSURER disclaimed liability on the
provide a credit term within which to pay the ground that as of the date of loss, the properties
premiums. That agreement is not against the law, ACME SHOE RUBBER & PLASTIC CORP. v. CA burned were not covered by insurance.
morals, good customs, public order or public policy. (DOMESTIC INSURANCE COMPANY OF THE - ACME claims that the January 8, 1964 payment
The agreement binds the parties. Article 1306 of the PHILS.) was for the period 1964-1965 and that INSURER had
Civil Code provides: 134 SCRA 155 no right to apply it to the period 1963-1964 because
ART. 1306. The contracting parties may establish MELENCIO-HERRERA; January 17, 1985. under RA 3540, the policy was void and INSURER
such stipulations clauses, terms and conditions as could have validly disclaimed liability for loss had one
they may deem convenient, provided they are not NATURE occurred then.
contrary to law, morals, good customs, public Petition for Review on Certiorari of the Decision of - TC found INSURER liable for P200k and opined that
order, or public policy. the then Court of Appeals (CA-G. R. No. 58917-R), there was a clear intention on the INSURER's part to
- Finally, it would be unjust and inequitable if denying recovery on an insurance policy, thereby grant ACME a credit extension for the payment of the
INSURANCE Page
38

premium due; and that to allow the INSURER to pay the premium within the 90-day extension "SEC. 72. An insurer is entitled to payment of
apply the premium ACME paid on January 8, 1964. granted, and in accordance with the express terms of premium as soon as the thing insured is exposed
CA reversed TC and dismissed the suit on the ground the Promissory Note that it had signed. to the perils insured against, unless there is clear
that, as of the moment of loss, ACME's properties Disposition The judgment under review is hereby agreement to grant credit extension for the
were not insured and the INSURER could not be held affirmed. Without pronouncement as to costs. premium due. No policy issued by an insurance
liable for any indemnity as a result of the loss. company is valid and binding unless and until the
PEDRO ARCE v. THE CAPITAL INSURANCE & premium thereof has been paid."
ISSUE SURETY CO., INC. - It is obvious from both the Insurance Act, as
WON the premium payment for 1964-1965 was paid 11 SCRA 63 amended, and the stipulation of the parties that time
ABAD SANTOS; September 30, 1982. is of the essence in respect of the payment of the
HELD insurance premium so that if it is not paid the
NO NATURE contract does not take effect unless there is still
- Not having paid the 1964-1965 premium within the Appeal from CFI decision on question of law. another stipulation to the contrary. In the instant
extension granted, and pursuant to R.A. No. 3540, case, the INSURED was given a grace period to pay
the policy was automatically cancelled and there was FACTS the premium but the period having expired with no
no insurance coverage to speak of as of the date of - Arce (INSURED) owned a residential house which payment made, he cannot insist that the COMPANY
the fire on October 13, 1964. was insured with the appellant COMPANY since 1961. is nonetheless obligated to him.
- The pertinent provision of Republic Act No. 3540 In November 1965, the COMPANY sent to the - Prior to the amendment (italicized portion above),
reads: INSURED a Renewal Certificate to cover the period an insurance contract was effective even if the
"Sec. 72. An insurer is entitled to payment of the from December 5, 1965 to December 5,1966, and premium had not been paid so that an insurer was
premium as soon as the thing insured is exposed requested payment of the corresponding premium. obligated to pay indemnity in case of loss and
to the peril insured against, unless there is clear Anticipating that the premium could not be paid on correlatively he had also the right to sue for payment
agreement to grant the insured credit extension of time, the INSURED asked for an extension which was of the premium. But the amendment to Sec. 72 has
the premium due. No policy issued by an granted by the COMPANY. After the lapse of the radically changed the legal regime in that unless the
insurance company is valid and binding unless and requested extension, INSURED still failed to pay the premium is paid there is no insurance.
until the premium thereof has been paid." premium. Thereafter, the house of the INSURED was Disposition The decision of the court a quo is
- RA 3540 was approved on June 20, 2963 and was totally destroyed by fire. Upon INSURED's reversed; the appellee's complaint is dismissed. No
put into effect on Oct 1, 1963. It could not be presentation of claim for indemnity, he was told that special pronouncement as to costs.
applied retroactively to the renewal of the policy for no indemnity was due because the premium was not - Irrelevant facts: The premium costs P38.10. After
the 1963-1964 period because said policy was paid. The INSURED sued the COMPANY for the fire, the COMPANY issued a check for P300 to
renewed on May 14, 1963. (Laws have no retroactive indemnity. Arce as donation. Arce accepted the check, but still
effect unless the contrary is provided.) Therefore, - The trial court held the COMPANY liable to sued the company.
the Jan 8, 1964 payment was properly applied to the indemnify the INSURED on the ground that since the CAPITAL INC. v. PLASTIC ERA CO.
1963-1964 premium. The Trial Court's opinion that COMPANY could have demanded payment of the 65 SCRA 134
there was a clear agreement to grant ACME credit premium, mutuality of obligation required that it MARTIN; July 18, 1975
extension for 1964-1965 is negated by ACME's should be liable on the policy.
Promissory Note binding itself to pay within ninety NATURE
days from the effective date of this policy, 15th May, ISSUE Petition for review of a decision of the CA affirming
1964. The credit extension was granted for 90 days WON the COMPANY can be held liable on its policy the decision of the CFI of Manila
only. (So wala na by August 16, 1964.)
- If ACME was granted credit extensions in the past, HELD FACTS
the promissory note it signed did away with such NO. - On December 17, 1960, petitioner Capital
credit arrangement. Also, before RA 3540, the - The Court commiserates with the INSURED. They Insurance & Surety Co., Inc. delivered to the
Renewal Receipts issued by INSURER did not contain are well aware that many insurance companies have respondent Plastic Era Manufacturing Co., Inc., its
the auto-cancellation after 90 days note. By 1964, fallen into the condemnable practice of collecting open Fire Policy No. 22760 wherein the former
however, the situation had changed by the passage premiums promptly but resort to all kinds of excuses undertook to insure the latter's building, equipments,
of the RA: no policy could be valid and binding to deny or delay payment of just claims. Unhappily raw materials, products and accessories located at
unless and until the premium thereof had been the instant case is one where the insurer has the law Sheridan Street, Mandaluyong, Rizal. The policy
paid. on its side. expressly provides that if the property insured would
- What became automatically cancelled by R.A. No. - Sec. 72 of the Insurance Act, as amended by R.A. be destroyed or damaged by fire after the payment
3540 was the 1964-1965 policy for ACME's failure to No. 3540 reads: of the premiums, at anytime between the 15th day
INSURANCE Page
39

of December 1960 and one o'clock in the afternoon decided to hold the same for thirty-five (35) days Procedural
of the 15th day of December 1961, the insurance before presenting it for payment. Having held the 1. YES
company shall make good all such loss or damage in check for such an unreasonable period of time, - Petitioner invokes Sec 416 of the Insurance Code
an amount not exceeding P100,000.00. When the Capital Insurance was estopped from claiming a which grants it 30 days from notice of the Insurance
policy was delivered, Plastic Era failed to pay the forfeiture of its policy for non-payment even if the Commission within which to appeal by certiorari with
corresponding insurance premium. On January 8, check had been dishonored later. Where the check is the Court. MICO filed its MFR on April 25, 15 days
1961, in partial payment of the insurance premium, held for an unreasonable time before presenting it after the notice; the reglementary period began to
Plastic Era delivered to Capital Insurance, a check for for payment, the insurer may be held estopped from run again after June 13. Since the petition was filed
the amount of P1,000.00 postdated January 16, claiming a forfeiture if the check is dishonored. only on July 2, it was tardy by 4 days. Alternatively it
1961. However, Capital Insurance tried to deposit Disposition The decision of the CA is AFFIRMED in invokes Rule 45 of the Rules of Court for certiorari
the check only on February 20, 1961 and the same toto. but the petition still exceeds the 15 day limit from
was dishonored by the bank for lack of funds. the June 13 notice.
- Two days after the insurance premium became MALAYAN INSURANCE CO., INC. v. ARNALDO -Respondents, on the other hand, invoke Sec. 39 of
due, at about 4:00 to 5:00 o'clock in the morning, and PINCA B.P. 129 which pegs the period for appeal from
the property insured by Plastic Era was destroyed by 154 SCRA 672 decisions of any court in all cases at 15 days from
fire. In less than a month Plastic Era demanded from CRUZ; October 12, 1987 the notice of the decision appealed from. Since the
Capital Insurance the payment of the sum of MFR was filed only 15 days after receiving notice of
P100,000.00 as indemnity for the loss of the insured FACTS the decision, it was already 18 days late by July 2.
property under Policy No. 22760 but the latter - On June 7, 1981, Malayan Insurance Co. (MICO), So whichever is applied, the petition is still late.
refused for the reason that, among others, Plastic issued fire insurance for the amount of P14,000 on Substantive
Era failed to pay the insurance premium. the property of private respondent, Pinca, effective 2. YES
July 1981-1982. MICO later allegedly cancelled the - A valid cancellation requires the following
ISSUES policy for non-payment of the premium and sent a conditions based on Sections 64-65 of the Code:
1. WON a contract of insurance has been duly notice to Pinca. On Dec. 24 Adora, an agent of MICO, prior notice which must be based on the occurrence
perfected between petitioner and respondent received Pinca’s payment, which was remitted to of one or more of the grounds mentioned in Sec 64
2. WON the dishonored check constituted payment MICO. On Jan. 18, 1982, Pinca’s property was (in this case, non-payment of premium), after the
completely burned. On Feb. 5, MICO returned Pinca’s effective date of the policy; the notice must be
HELD payment to Adora on the ground that her policy had written and mailed to the address on the policy; it
1. YES been cancelled; the latter refused to accept it. Her must state the ground(s) for cancellation and the
- Tender of draft or check in order to effect payment demand for payment having been rejected by MICO, insurer must furnish details upon the request of the
that would extinguish the debtor's liability should be Pinca went to the Insurance Commission. Public insured.
actually cashed. If the delivery of the check of Plastic respondent Arnaldo, the Insurance Commissioner, - It is undisputed that payment of premium was
Era to Capital Insurance were to be viewed in the sustained Pinca, hence this petition from MICO. made. Petitioner relies heavily on Sec 77 of the
light of the foregoing, no payment of the premium Records show MICO received Arnaldo’s decision on Insurance Code to contest this, the said provision
had been effected. Significantly, Capital Insurance April 10; MICO filed a MFR on April 25 which was requiring payment of premium as soon as the thing
accepted the promise of Plastic Era to pay the denied on June 4; MICO received notice of this denial is exposed to the peril insured against and that the
insurance premium within 30 days from the effective on June 14; instant petition was filed on July 2. policy is invalid without it. However, this is not
date of policy. By so doing, it has implicitly agreed to applicable in the instant case as payment was
modify the tenor of the insurance policy and in ISSUES eventually made. It is to be noted that the premium
effect, waived the provision therein that it would only Procedural invoice was stamped “Payment Received”, indicating
pay for the loss or damage in case the same occurs 1. WON the petition should be dismissed for late an understanding between the parties that payment
after the payment of the premium. Considering that filing could be made later. This is furthered by the fact
the insurance policy is silent as to the mode of Substantive that Adora had earlier told her to call him anytime
payment, Capital Insurance is deemed to have 2. WON there was a valid insurance contract at the she was ready with her payment. The Court also
accepted the promissory note in payment of the time of the loss finds it strange that MICO only sought to return
premium. This rendered the policy immediately 3. WON Adora was authorized to receive such Pinca’s Jan. 15 payment only on Feb. 5, long after
operative on the date it was delivered. payment her house had burned down—this makes petitioner’s
2. YES 4. WON an adjuster is indispensable in the motives highly suspect.
- Although the check was due for payment on valuation of the loss - MICO claims to have sent a notice to Pinca, who
January 16, 1961 and Plastic Era had sufficient funds flatly denied receiving one. Pinca did not have to
to cover it as of January 19, 1961, Capital Insurance HELD prove this since the strict language of Sec 64
INSURANCE Page
40

requires that MICO ensure the cancellation was - Plaintiff issued a number of life insurance policies in 2. WON, in the application of the automatic premium
actually sent to and received by the insured. the Philippines containing stipulations referred to as loan clause of plaintiff-appellant's policies, there is
- MICO also suggests that Pinca knew the policy had NONFORFEITURE CLAUSES5 'payment in money, notes, credits, or any
been cancelled and was paying the premium in order - From January 1, 1942 to December 31, 1946, substitutes for money
to renew the policy. A close study of the transcripts Plaintiff head office at Toronto applied the provisions 3. WON the collection of the alleged deficiency
show, however, that Pinca only meant to renew the of the automatic premium loan clauses upon the premium taxes constitutes double taxation
policy had it been cancelled but not if it was still in nonpayment of the corresponding premiums by the 4. WON the making of premium advances, granting
effect—it was conditional. Payment was thus legally people who subscribed to the insurance. The net for the sake of argument that it amounted to
made on the original transaction and validly received amount of premiums advanced (by the company) or collection of premiums, were done in Toronto,
by Adora, who was not informed of the alleged loaned (to the insured) as payment for the premium Canada
cancellation and thus saw no reason to reject the due totaled P1,069,254.98. 5. WON the fact that plaintiff-appellant was not
payment. - Meer, the Collector of the National Internal doing business in the Philippines during the period
3. YES Revenue assessed the net amount of premium at from January 1, 1942 to September 30, 1945,
- Sec. 306 of the Insurance Code provides that any P17,917.12 pursuant to SEC.255, National Internal inclusive, exempts it from payment of premium
insurance company that delivers a policy to its agent Revenue Code6 taxes corresponding to said period
is deemed to have authorized such agent to receive - Company protested the assessment, but paid the
payment of premium on its behalf. It is a well-known taxes anyway. Then they filed a complaint to recover
principle under the law of agency that payment to an money paid under protest for taxes HELD
authorized agent is equivalent to payment to the - CFI: Dissmiss complaint NOTE (example given by the plaintiff):
principal himself. MICO’s acknowledgement of Adora - PLAINTIFF’s MAIN CONTENTION: when it made "Suppose that 'A', 30 years of age, secures a 20-
as its agent thus defeats its contention that he was premium loans or premium advances by virtue of the year endowment policy for P5,000 from plaintiff-
not authorized to receive payments on its behalf. non-forfeiture clauses, it did not collect premiums appellant Company and pays an annual premium of
4. NO within the meaning of the above sections of the law, P250. 'A' pays the first ten yearly premiums
- In absence of fraud, the amount of the loss may be and therefore it is not amenable to the tax therein amounting to P2,500 and on this amount plaintiff-
determined on the basis of such proof offered by the provided. appellant pays the corresponding taxes under section
insured. Here. The certification of the Integrated 255 of the National Internal Revenue Code. Suppose
National Police as the extent of the loss should ISSUES also that the cash value of said policy after the
suffice. 1. WON premium advances made by plaintiff- payment of the 10th annual premium amounts to
Disposition petition is DENIED appellant under the automatic premium loan clause P1,000." When on the eleventh year the annual
of its policies are premiums collected' by the premium fell due and the insured remitted no money
MANUFACTURERS LIFE INSURANCE CO. v. MEER Company subject to tax within the mouth grace, the insurer treated the
89 PHIL 351 premium then over due as paid from the cash value,
BENGZON, June 29, 1951 5"'8. Automatic Premium Loan.-This Policy shall not lapse for non-payment of any premium after it has been three full the amount being a loan to the policyholder1
years in force, it, at the due date of such premium, the Cash Value of this Policy and of any bonus additions and dividends who could discharge it at any time with interest at 6
NATURE left on accumulation (after deducting any indebtedness to the company and the interest accrued thereon) shall exceed the
per cent. The insurance contract, therefore,
amount of said premium. In which event the company will, without further request, treat the premium then due as paid, and
APPEAL from a judgment of the Court of First the amount of such premium, with interest from its actual due date at six per cent per annum, compounded yearly, and one continued in force for the eleventh year.
Instance of Manila per cent, compounded yearly, for expenses, shall be a first lien on this Policy in the Company's favour in priority to the 1. YES
claim of any assignee or any other person. The accumulated lien may at any time, while the Policy is in force, be paid in

whole or in part.
- Based on the example given by the plaintiff, the
FACTS 'When the premium falls due and is not paid in cash within the month's grace, if the Cash Value of this policy and of any insurer collected the amount of P250 as the annual
(this is a tax case. What’s really important here is bonus additions and dividends left on accumulation (after deducting any accumulated indebtedness) be less than the
premium for the eleventh year on the said policy
premium then due, the Company will, without further requests, continue this insurance in force for a period * * *.
the definition of CASH SURRENDER VALUE). '10. Cash and Paid-Up Insurance Values.-At the end of the third policy year or thereafter, upon the legal surrender of this
when it loaned to “A” the sum of P250. The insurer
- Manufacturers Life Insurance Company is a duly Policy to the Company while there is no default in premium payments or within two months after the due date of the “became a creditor” of the loan, but not of the
organized corporation which has its head office at premium in default, the Company will (1) grant a cash value as specified in Column (A) increased by the cash value of any

bonus additions and dividends left on accumulation, which have been alloted to this Policy, less all indebtedness to the
premium that had already been paid (advanced by
Toronto. It is duly registered and licensed to engage Company on this Polley an the date of ouch surrender, or (2) endorse this Policy as a Non-Participating Paid-up Polley for the insurer). The insurer is entitled to collect interest
in life insurance business in the Philippines, and, the amount as specified In Column (B) of the Table of Guaranteed Values * * *.
on the loan, not on the premium. "A" paid the
maintains a branch office in Manila. It was engaged
'11. Extended Insurance-After the premiums for three or more full years have been paid hereunder in cash, if any

subsequent premium is not paid when due, and there is no indebtedness to the Company on the written request of the
premium for the eleventh year; but in turn he
in such business in the Philippines for more than five insured * * *." became a debtor of the company for the sum of
years before and including the year 1941. But due to 6"SEC. 255. Taxes on insurance premiums.-There shall be collected from every person, company, or corporation (except P250. This debt he could repay either by later
the exigencies of the war It closed the branch office purely cooperative companies or associations) doing insurance business of any sort in the Philippines a tax of one per remitting the money to the insurer or by letting the
at Manila during 1942 up to September 1945. centum of the total premiums collected * * * whether such permiums are paid in money, notes, credits, or any substitute for
cash value compensate for it. The debt may also be
money but premiums refunded within six months after payment on account of rejection of risk or returned for other reason

to person insured shall not be included in the taxable receipts * * *."


INSURANCE Page
41

deducted from the amount of the policy should "A" - Although during those years the appellant was not the reinstatement of the lapsed policy was subject to
die thereafter during the continuance of the policy. open for new business because its branch office was the payment of the remaining premium balance of
- ON ARGUMENT THAT THE ASSETS OF THE INSURER REMAINED THE closed, still it was practically and legally, operating in P65.15.
SAME AFTER THE APPLICATION OF THE AUTOMATIC PREMIUM LOAN this country by collecting premiums on its - May 3, 1951: Severa Andres died of dystocia,
CLAUSE: there was an increase in assets in the form of outstanding policies, incurring the risks and/or contracted pelvis.
CREDIT for the advances made (in the example, the enjoying the benefits consequent thereto, without - May 5, 1951: Plaintiff sent a letter enclosed with a
P250 for the 11th year). having previously taken any steps indicating money order in the amount of P65, for the remaining
- ON ARGUMENT THAT IF THE CREDIT IS PAID OUT OF THE CASH withdrawal in good faith from this field of economic balance due.
SURRENDER VALUE, THERE WERE NO NEW FUNDS ADDED TO THE activity. - May 15, 1951: Defendant sent a letter with official
COMPANY'S ASSETS”: Cash surrender value "as applied Disposition finding no prejudicial error in the receipt of the P165.15 paid by Rufino as well as a
to a life insurance policy, is the amount of money the appealed decision, we hereby affirm it with costs. Certificate of Reinstatement.
company agrees to pay to the holder of the policy if - June 7, 1951: Rufino presented a death claim as
he surrenders it and releases his claims upon it. The survivor-beneficiary of his deceased wife. Payment
more premiums the insured has paid the greater will was denied by the defendant.
be the surrender value; but the surrender value is - April 1952: Rufino filed a complaint in CFI against
always a lesser sum than the total amount of Crown Life for the recovery of the amount of P5,000
premiums paid." (Cyclopedia Law Dictionary 3d. ed. as the face value of a joint 20-year endowment
1077.) The cash value or cash surrender value is insurance policy issued by defendant in favor of
therefore an amount which the insurance company plaintiff and his wife, on Feb. 13, 1950. In its
holds In trust for the insured to be delivered to him ANDRES v. CROWN LIFE INSURANCE answer, Crown Life disclaimed liability and set forth
upon demand. It is therefore a liability of the 102 Phil. 919 the special defense that the aforementioned policy
company to the insured. Now then, when the REYES, J.B.L., Jan.28, 1958 had already lapsed.
company's credit for advances is paid out of the cash - Aug. 5, 1954: CFI rendered a decision absolving
value or cash surrender value, that value and the NATURE the defendant company from any liability on the
company's liability is thereby diminished pro tanto. Appeal from judgment of CFI ground that the policy had lapsed and it was not
2. YES reinstated at the time of the plaintiff’s wife’s death.
- the insurer agreed to consider the premium paid on FACTS Plaintiff later appealed to the CA but the same was
the strength of the automatic loan. The premium - Feb. 13, 1950: For the sum of P5,000, defendant- certified by the CA to the SC for having no question
was therefore paid by means of a "note" or "credit" appellee Crown Life issued an insurance policy in the of fact.
or "other substitute for money" and the tax is due name of plaintiff-appellant Rufino and his wife, with
because section 255 above quoted levies taxes the stipulation that the premiums are to be paid ISSUE
according to the total premiums collected by the semi-annually. WON the insurance policy, which has been in a state
insurer "whether such premiums are paid in money, - The premiums for the 1st and 2nd semester of the 1st of lapse before May 3, 1951, has been validly and
notes, credits or any substitute for money. year, in the amount of P165.15 were paid by Rufino completely reinstated after said date (Was there a
3. NO but the premium for the third semester, in the same perfected contract of reinstatement after the policy
- No constitutional prohibition against double amount, was not paid. lapsed due to non-payment of premiums?)
taxation. - Jan. 6, 1951, Crown Life, through its branch
4. NO secretary, wrote to Mr. and Mrs. Andres advising HELD
- The loans are made to policyholders in the them that their insurance policy lapsed on Dec. 26, NO
Philippines, who in turn pay therewith the premium 1950 and the amount of P165.15 was overdue, Ratio The stipulation in a life insurance policy giving
to the insurer thru the Manila branch. Approval of giving them 60 days from the date of lapse to file an the insured the privilege to reinstate it upon written
appellant's position will enable foreign insurers to application for reinstatement. Crown Life later sent application does not give the insured absolute right
evade the tax by contriving to require that premium another letter telling the spouses Andres that their to such reinstatement by the mere filing of an
payments shall be made at their head offices. What insurance policy was no longer in force. application. The Company has the right to deny the
is important, the law does not contemplate - Feb. 1951: Plaintiff and his wife executed a reinstatement if it is not satisfied as to the
premiums collected in the Philippines. It is enough Statement of Health and application for insurability of the insured and if the latter does not
that the insurer is doing insurance business in the reinstatement of the aforesaid policy. pay all overdue premiums and all other indebtedness
Philippines, irrespective of the place of its - Feb. 20, 1951: Plaintiff wrote a letter to the to the Company. After the death of the insured the
organization or establishment. defendant, enclosed with a money order for P100. insurance Company cannot be compelled to entertain
5. NO Upon acceptance, defendant advised Rufino that its an application for reinstatement of the policy
main office had approved the application and that
INSURANCE Page
42

because the conditions precedent to reinstatement reinstatement. Plaintiff-Appellant’s failure to remit - Then on December 27, 1978, Philamgen
can no longer be determined and satisfied. the balance before the death of his wife operated to terminated the General Agency Agreement of
Reasoning deprive him of any right to waive the policy and Valenzuela.
- The stipulations of facts render it undisputable that recover the face value thereof. - Lower court: the termination of Valenzuela as
the original policy lapsed for non-payment of Disposition Judgment appealed from is affirmed. General Agent was improper because the record will
premiums on Dec. 26, 1950, upon expiration of the show the principal cause of the termination of the
31-day grace period. VALENZUELA v. CA (PHILIPPINE AMERICAN plaintiff as General Agent of defendant Philamgen
- As found by the lower court, the conditions set GENERAL INSURANCE COMPANY, INC.) was his refusal to share his Delta commission.
forth in the policy for reinstatement as provided in 191 SCRA 1 - CA: In any event the principal's power to revoke an
the contract itself are the following: (A) application GUTIERREZ; October 19, 1990 agency at will is so pervasive, that the Supreme
shall be made within 3 years from the date of lapse; Court has consistently held that termination may be
(B) there should be a production of evidence of the NATURE effected even if the principal acts in bad faith,
good health of the insured; (C) if the rate of Petition for review of the decision of theca. subject only to the principal's liability for damages.
premium depends upon the age of the Beneficiary, (CA ordered Valenzuela to pay Philamgen the
there should likewise be a production of evidence of FACTS amount of One Million Nine Hundred Thirty-Two
his or her good health; (D) there should be - Petitioner Arturo P. Valenzuela is a General Agent Thousand Five Hundred Thirty-Two and 17/100
presented such other evidence of insurability at the of private respondent Philippine American General Pesos (P1,932,532.17) with legal interest)
date of application for reinstatement; (E) there Insurance Company, Inc. (Philamgen for short) since
should be no change which has taken place in such 1965. As such, he was authorized to solicit and sell ISSUES
good health and insurability subsequent to the date in behalf of Philamgen all kinds of non-life insurance, 1. WON whether or not Philamgen and/or its officers
of such application and before the policy is and in consideration of services rendered was can be held liable for damages due to the
reinstated; and (F) all overdue premiums and other entitled to receive the full agent's commission of termination of the General Agency Agreement it
indebtedness in respect of the policy, together with 32.5% from Philamgen under the scheduled entered into with the petitioners
interest at 6%, compounded annually, should first be commission rates. 2. WON petitioners are liable to Philamgen for the
paid. - From 1973 to 1975, Valenzuela solicited marine unpaid and uncollected premiums
- The plaintiff did not comply with the last condition; insurance from one of his clients, the Delta Motors,
for he only paid P100 before his wife’s death; and Inc. (Division of Electronics Airconditioning and HELD
despite the Company’s reminders, he only remitted Refrigeration) in the amount of P4.4 Million from 1. YES
the balance of P65.15 two days after his wife died. which he was entitled to a commission of 32%. - If a principal acts in bad faith and with abuse of
On the face of such facts, the Company had the right However, Valenzuela did not receive his full right in terminating the agency, then he is liable in
to treat the contract as lapsed and refuse payment commission which amounted to P1.6 Million from the damages.
of the policy. P4.4 Million insurance coverage of the Delta Motors. - There is an exception to the principle that an
- Rufino contends that the condition regarding During the period 1976 to 1978, premium payments agency is revocable at will and that is when the
payment of the premium was waived by the amounting to P1,946,886.00 were paid directly to agency has been given not only for the interest of
insurance Company through its letters, wherein it Philamgen and Valenzuela's commission to which he the principal but for the interest of third persons or
made statements such as: “If you are unable to pay is entitled amounted to P632,737.00. for the mutual interest of the principal and the
the full amount immediately, send as large amount - In 1977, Philamgen started to become interested in agent. In these cases, it is evident that the agency
as possible and advise us how soon you expect to be and expressed its intent to share in the commission ceases to be freely revocable by the sole will of the
able to pay the balance; we will work out an due Valenzuela on a fifty-fifty basis. Valenzuela principal
adjustment most beneficial to you.” The Court found refused. (PROCEDURAL: Where the findings of the Court of
the statements to be too vague and indefinite to - Because of the refusal of Valenzuela, Philamgen Appeals and the trial court are contrary to each
indicate an intention on the insurer’s part to waive and its officers took drastic action against other, this Court may scrutinize the evidence on
the full payment as prerequisite to the reinstatement Valenzuela. They: (a) reversed the commission due record
of the lapsed policy. The Court reiterated the rule him by not crediting in his account the commission - After a painstaking review of the entire records of
that a waiver must be clear and positive, the intent earned from the Delta Motors, Inc. insurance ; (b) the case and the findings of facts of both the court a
to waive shown clearly and convincingly. On the placed agency transactions on a cash-and-carry quo and respondent appellate court, the Court
other hand, It found subsequent letters sent by basis; (c) threatened the cancellation of policies affirmed the trial court’s findings.)
defendant indicating that they insisted on full issued by his agency; and (d) started to leak out - The principal cause of the termination of
payment of the premium before the policy was news that Valenzuela has a substantial account with Valenzuela as General Agent of Philamgen arose
reinstated and that defendant did not consider Philamgen. All of these acts resulted in the decline of from his refusal to share his Delta commission. The
partial payment as sufficient consideration for the his business as insurance agent. records sustain the conclusions of the trial court on
INSURANCE Page
43

the apparent bad faith of the private respondents in - This is buttressed by Section 776 of the - The above-mentioned life is to be assured in
terminating the General Agency Agreement of Insurance Code (Presidential Decree No. 612, accordance with the terms and conditions contained
petitioners. promulgated on December 18, 1974), which now or inserted by the Company in the policy which may
- It is also evident from the records that the agency provides that no contract of Insurance by an be granted by it in this particular case for four
involving petitioner and private respondent is one insurance company is valid and binding unless and months only from the date of the application,
"coupled with an interest," and, therefore, should not until the premium thereof has been paid, provided that the Company shall confirm this
be freely revocable at the unilateral will of the latter. notwithstanding any agreement to the contrary." agreement by issuing a policy on said application
- The private respondents by the simple expedient of Disposition Petition is GRANTED. CA decision SET when the same shall be submitted to the Head Office
terminating the General Agency Agreement ASIDE. The decision of the TC REINSTATED with the in Montreal. Should the Company not issue such a
appropriated the entire insurance business of MODIFICATIONS. And that the contractual policy, then this agreement shall be null and void ab
Valenzuela. With the termination of the General relationship between Arturo P. Valenzuela and initio, and the Company shall be held not to have
Agency Agreement, Valenzuela would no longer be Philippine American General Insurance Company been on the risk at all, but in such case the amount
entitled to commission on the renewal of insurance shall be deemed terminated upon the satisfaction of herein acknowledged shall be returned.
policies of clients sourced from his agency. Worse, the judgment as modified.
despite the termination of the agency, Philamgen ISSUE
continued to hold Valenzuela jointly and severally CHAPTER V – THE POLICY, PARTIES THERETO, WON the contract of insurance between Luis Lim and
liable with the insured for unpaid premiums. Under & RIGHTS THEREON Sun Life Assurance Company of Canada was
these circumstances, it is clear that Valenzuela had perfected
an interest in the continuation of the agency when it DE LIM v. SUN LIFE ASSURANCE COMPANY OF
was unceremoniously terminated not only because of CANADA HELD
the commissions he should continue to receive from 41 PHIL 263 NO.
the insurance business he has solicited and procured MALCOLM; November 29, 1920 - The document it is to be a provisional policy "for
but also for the fact that by the very acts of the four months only from the date of this application."
respondents, he was made liable to Philamgen in the NATURE Immediately following the words fixing the four
event the insured fail to pay the premiums due. They Appeal from an order of the CFI of Zamboanga months period comes the word "provided" which has
are estopped by their own positive averments and sustaining a demurrer to plaintiff's complaint upon the meaning of "if." Otherwise stated, the policy for
claims for damages. the ground that it fails to state a cause of action. four months is expressly made subject to the
- "The principal may not defeat the agent's right to affirmative condition that the company shall confirm
indemnification by a termination of the contract of FACTS this agreement by issuing a policy on said application
agency (Erskine v. Chevrolet Motors Co. 185 NC 479, - On July 6, 1917, Luis Lim of Zamboanga made when the same shall be submitted to the head office
117 SE 706, 32 ALR 196). application to the Sun Life Assurance Company of in Montreal. To re-enforce the same there follows the
- For the pivotal factor rendering Philamgen and the Canada for a policy of insurance on his life in the negative condition - "Should the company not issue
other private respondents liable in damages is that sum of P5,000. In his application Lim designated his such a policy, then this agreement shall be null and
the termination by them of the General Agency wife, Pilar de Lim, the plaintiff herein, as the void ab initio, and the company shall be held not to
Agreement was tainted with bad faith. This is in beneficiary. The first premium of P433 was paid by have been on the risks." Certainly language could
accordance with the precepts in Human Relations Lim, and upon such payment the company issued hardly be used which would more clearly stipulate
enshrined in our Civil Code. what was called a ''provisional policy." Luis Lim died that the agreement should not go into effect until the
2. NO. The respondent court erred in holding on August 23, 1917, after the issuance of the home office of the company should confirm it by
Valenzuela liable. There was no factual and legal provisional policy but before approval of the issuing a policy. As we read and understand the so-
basis for the award. Under Section 77 of the application by the home office of the insurance called provisional policy, it amounts to nothing but
Insurance Code, the remedy for the non-payment company. Pilar de Lim brought an action to recover an acknowledgment on behalf of the company, that
of premiums is to put an end to and render the from the Sun Life sum of P5,000, the amount named it has received from the person named therein the
insurance policy not binding - "Sec. 77 . . . in the provisional policy. sum of money agreed upon as the first year's
[N]otwithstanding any agreement to the contrary, no - The "provisional policy" reads: "Received (subject premium upon a policy to be issued upon the
policy or contract of insurance is valid and binding to the following stipulations and agreements) the application, if the application is accepted by the
unless and until the premiums thereof have been sum of P433, being the amount of the first year's company.
paid except in the case of a life or industrial life premium for a Life Assurance Policy on the life of Mr. - It is of course a primary rule that a contract of
policy whenever the grace period provision applies Luis D. Lim of Zamboanga for P5,000, for which an insurance, like other contracts, must be assented to
(P.D. 612, as amended otherwise known as the application dated the 6th day of July, 1917, has been by both parties either in person or by their agents.
Insurance Code of 1974) made to the Sun Life Assurance Company of Canada. So long as an application for insurance has not been
either accepted or rejected, it is merely an offer or
INSURANCE Page
44

proposal to make a contract. The contract, to be constituted concealment that justified the denial of if there is any, shall then be paid to the
binding from the date of the application must have the claim. beneficiary/ies designated by the debtor.” When DBP
been a completed contract, one that leaves nothing - Herein respondent Medarda Leuterio, widow, filed a submitted the insurance claim against Grepalife, the
to be done, nothing to be completed, nothing to be complaint with RTC against Grepalife for "Specific latter denied payment thereof, interposing the
passed upon, or determined, before it shall take Performance with Damages." Dr. Mejia, who issued defense of concealment committed by the insured.
effect. There can be no contract of insurance unless the death certificate, testified that Dr. Leuterio Thereafter, DBP collected the debt from the
the minds of the parties have met in agreement. Our complained of headaches presumably due to high mortgagor and took the necessary action of
view is, that a contract of insurance was not here blood pressure. The inference was not conclusive foreclosure on the residential lot of private
consummated by the parties. because Dr. Leuterio was not autopsied, hence, respondent.
- The trial court committed no error in sustaining the other causes were not ruled out. [b] Since a policy of insurance upon life or health
demurrer and dismissing the case. It is to be noted, - RTC ruled in favor of respondent widow and against may pass by transfer, will or succession to any
however that counsel for appellee admits the liability Grepalife. CA sustained the RTC decision. Hence, the person, whether he has an insurable interest or not,
of the company for the return of the first premium to present petition. and such person may recover it whatever the insured
the estate of the deceased. might have recovered, the widow of the decedent Dr.
Leuterio may file the suit against the insurer,
GREAT PACIFIC LIFE v. CA (LEUTERIO) Grepalife.
316 SCRA 677 ISSUES 2. NO
QUISUMBING; October 13, 1999 1. WON CA erred in holding petitioner liable to DBP Ratio The fraudulent intent on the part of the
as beneficiary in a group life insurance contract from insured must be established to entitle the insurer to
NATURE a complaint filed by the widow of the rescind the contract. Misrepresentation as a defense
Petition for Review of CA decision decedent/mortgagor of the insurer to avoid liability is an affirmative
2. WON CA erred in not finding that Dr. Leuterio defense and the duty to establish such defense by
FACTS concealed that he had hypertension, which would satisfactory and convincing evidence rests upon the
- A contract of group life insurance was executed vitiate the insurance contract insurer. In the case at bar, the petitioner failed to
between petitioner Great Pacific Life Assurance 3. WON CA erred in holding Grepalife liable for clearly and satisfactorily establish its defense, and is
Corporation (hereinafter Grepalife) and Development P86,200.00 without proof of the actual outstanding therefore liable to pay the proceeds of the insurance.
Bank of the Philippines (hereinafter DBP). Grepalife mortgage payable by the mortgagor to DBP Reasoning
agreed to insure the lives of eligible housing loan [a] The insured, Dr. Leuterio, had answered in his
mortgagors of DBP. HELD insurance application that he was in good health and
- In Nov. 1983, Dr. Wilfredo Leuterio, a physician 1. NO that he had not consulted a doctor or any of the
and a housing debtor of DBP applied for membership Ratio Insured, being the person with whom the enumerated ailments, including hypertension; when
in the group life insurance plan. In an application contract was made, is primarily the proper person to he died the attending physician had certified in the
form, Dr. Leuterio answered Qs concerning his health bring suit. Subject to some exceptions, insured may death certificate that the former died of cerebral
condition as follows: thus sue, although the policy is taken wholly or in hemorrhage, probably secondary to hypertension.
Q: Have you ever had, or consulted, a physician for part for the benefit of another person named or From this report, petitioner Grepalife refused to pay
a heart condition, high blood pressure, cancer, unnamed, and although it is expressly made payable the insurance claim. It alleged that the insured had
diabetes, lung, kidney or stomach disorder or any to another as his interest may appear or otherwise. concealed the fact that he had hypertension.
other physical impairment? No. Although a policy issued to a mortgagor is taken out [b] Contrary to Grepalife’s allegations, there was no
Q: Are you now, to the best of your knowledge, in for the benefit of the mortgagee and is made sufficient proof that the insured had suffered from
good health? Yes. payable to him, yet the mortgagor may sue thereon hypertension. Aside from the statement of the
- Grepalife issued an insurance coverage of Dr. in his own name, especially where the mortgagee's insured's widow who was not even sure if the
Leuterio, to the extent of his DBP mortgage interest is less than the full amount recoverable medicines taken by Dr. Leuterio were for
indebtedness of P86,200.00. In Aug. 1984, Dr. under the policy. (See Sec. 8, Insurance Code) hypertension, the appellant had not proven nor
Leuterio died due to "massive cerebral hemorrhage." Reasoning produced any witness who could attest to Dr.
DBP submitted a death claim to Grepalife. Grepalife [a] The insured private respondent did not cede to Leuterio's medical history.
denied the claim because Dr. Leuterio was not the mortgagee all his rights or interests in the [c] Grepalife had failed to establish that there was
physically healthy when he applied for an insurance. insurance, the policy stating that: “In the event of concealment made by the insured, hence, it cannot
Grepalife insisted that Dr. Leuterio did not disclose the debtor's death before his indebtedness with the refuse payment of the claim.
he had been suffering from hypertension, which Creditor (DBP) shall have been fully paid, an amount 3. NO
caused his death. Allegedly, such non-disclosure to pay the outstanding indebtedness shall first be - Considering the supervening event that DBP
paid to the creditor and the balance of sum assured, foreclosed in 1995 their residential lot, in satisfaction
INSURANCE Page
45

of mortgagor's outstanding loan, the insurance intended to be exported were lost during loading Reasoning
proceeds shall inure to the benefit of the heirs of the operations in the Diapitan Bay due to bad weather. a. the cover note in question is subject to the terms
deceased person or his beneficiaries. Equity dictates - April 4, 1963 - The plaintiff informed the defendant and conditions of the marine policies
that DBP should not unjustly enrich itself at the about the loss of 'approximately 32 pieces of logs' b. Nature of the Cover Note: The fact that no
expense of another. Hence, it cannot collect the during loading through a letter. separate premium was paid on the Cover Note
insurance proceeds, after it already foreclosed on the - The plaintiff subsequently submitted a 'Claim before the loss insured against occurred, does not
mortgage. The proceeds now rightly belong to Dr. Statement' demanding payment of the loss under militate against the validity of petitioner's
Leuterio's heirs represented by his widow, herein the second marine cargo policy. contention, for no such premium could have been
private respondent. - July 17, 1963 - the defendant requested the First paid, since by the nature of the Cover Note, it did
- The Court ruled this issue based on the clear Philippine Adjustment Corporation to inspect the loss not contain, as all Cover Notes do not contain
provisions of the policy. The mortgagor paid the and assess the damage. particulars of the shipment that would serve as basis
premium according to the coverage of his insurance, - August 23, 1963 - the adjuster reported that 'the for the computation of the premiums. As a logical
which states that: "The policy states that upon loss of 30 pieces of logs is not covered by the two consequence, no separate premiums are intended or
receipt of due proof of the Debtor's death during the policies inasmuch as said policies covered the actual required to be paid on a Cover Note.
terms of this insurance, a death benefit in the number of logs loaded on board. But it is covered by c. The petitioner paid in full all the premiums as
amount of P86,200.00 shall be paid… In the event of Cover Note. called for by the statement issued by private
the debtor's death before his indebtedness with the - On January 13, 1964 - the defendant wrote the respondent after the issuance of the two regular
creditor shall have been fully paid, an amount to pay plaintiff denying the latter's claim, on the ground marine insurance policies, thereby leaving no
the outstanding indebtedness shall first be paid to that defendant's investigation revealed that the account unpaid by petitioner due on the insurance
the Creditor and the balance of the Sum Assured, if entire shipment of logs covered by the two marines coverage, which must be deemed to include the
there is any shall then be paid to the beneficiary/ies policies were received in good order at their point of Cover Note. If the Note is to be treated as a separate
designated by the debtor." From this, it is clear that destination. It was further stated that the said loss policy instead of integrating it to the regular policies
Grepalife is liable and that Dr. Leuterio’s heirs must may not be considered as covered under cover note subsequently issued, the purpose and function of the
get the proceeds. because the said note had become 'null and void by Cover Note would be set at naught or rendered
Disposition Petition DENIED. CA Decision AFFIRMED virtue of the issuance of two marine policies. meaningless, for it is in a real sense a contract, not a
with modification. - The CFI of Manila ruled in favour of the petitioner. mere application for insurance which is a mere offer.
- The Court of Appeals reversed the decision of the Had all the logs been lost during the loading
PACIFIC TIMBER EXPORT CORPORATION v. CA CFI. operations, but after the issuance of the Cover Note,
(WORKMEN’S INSURANCE CO) liability on the note would have already arisen even
112 SCRA 199 ISSUES before payment of premium. This is how the cover
DE CASTRO; February 25, 1982 1. WON the cover note is null and void for lack of note as a "binder" should legally operate; otherwise,
valuable consideration because no separate it would serve no practical purpose in the realm of
FACTS premiums are collected by private respondent on all commerce, and is supported by the doctrine that
- March 19, 1963 - the plaintiff secured temporary its cover notes where a policy is delivered without requiring
insurance from the defendant for its exportation of 2. WON the court of appeals erred in holding that payment of the premium, the presumption is that a
1,250,000 board feet of Philippine Lauan and Apitong private respondent was released from liability under credit was intended and policy is valid.
logs to be shipped from the Diapitan Bay, Quezon to the cover note due to unreasonable delay in giving 2. NO
Okinawa and Tokyo, Japan. The defendant issued on notice of loss because the court disregarded the - The private respondent company never raised this
said date Cover Note No. 1010, insuring the said proven fact that private respondent did not promptly ground in the proceedings. It must be because it did
cargo of the plaintiff "Subject to the Terms and and specifically object to the claim on the ground of not find any delay, as this Court fails to find a real
Conditions of the WORKMEN'S INSURANCE delay in giving notice of loss and, consequently, and substantial sign thereof. But even on the
COMPANY, INC. printed Marine Policy form as filed objections on that ground are waived under section assumption that there was delay, this Court is
with and approved by the Office of the Insurance 84 of the insurance act satisfied and convinced that as expressly provided by
Commissioner. law, waiver can successfully be raised against private
- April 2, 1963 - The two (2) regular marine cargo HELD respondent. Thus Section 84 of the Insurance Act
policies were issued by the defendant in favor of the 1. NO provides:
plaintiff. The total cargo insured under the two Ratio Cover note is issued with a consideration "Section 84. - Delay in the presentation to an
marine policies accordingly consisted of 1,395 logs, when, by express stipulation, the cover note is made insurer of notice or proof of loss is waived if
or the equivalent of 1,195,498 bd. ft. subject to the terms and conditions of the marine caused by any act of his or if he omits to take
- After the issuance of cover note but before the policies, and the payment of premiums is one of the objection promptly and specifically upon that
issuance of the two marine policies some of the logs terms of the policies. ground."
INSURANCE Page
46

- From what has been said, We find duly only that the total indemnity shall not exceed the applying the open policy clause as expressly agreed
substantiated petitioner's assignments of error. face value of the policy. upon by the parties in their contract, we hold that
Disposition The appealed decision is set aside and - The petitioner argues that since at the time of the the private respondent is entitled to the payment of
the decision of the Court of First Instance is fire the building insured was worth P5,800,000.00, indemnity under the said contract in the total
reinstated in toto with the affirmance of this Court. the private respondent should be considered its own amount of P508,867.00.
insurer for the difference between that amount and - The refusal of its vice-president to receive the
DEVELOPMENT INSURANCE v. IAC (PHIL the face value of the policy and should share pro rata private respondent's complaint, as reported in the
UNION REALTY DEVELOPMENT CORP) in the loss sustained. Accordingly, the private sheriff's return, was the first indication of the
143 SCRA 62 respondent is entitled to an indemnity of only petitioner's intention to prolong this case and
CRUZ; July 16, 1986 P67,629.31, the rest of the loss to be shouldered by postpone the discharge of its obligation to the
it alone. The petitioner cites Condition 17 of the private respondent under this agreement. That
FACTS policy, which provides: intention was revealed further in its subsequent acts
- A fire occurred in the building of the private "If the property hereby insured shall, at the ---- or inaction ---- which indeed enabled it to avoid
respondent and it sued for recovery of damages from breaking out of any fire, be collectively of greater payment for more than five years from the filing of
the petitioner on the basis of an insurance contract value than the sum insured thereon then the the claim against it in 1980.
between them. The petitioner allegedly failed to insured shall be considered as being his own Disposition The appealed decision is affirmed in
answer on time and was declared in default by TC. A insurer for the difference, and shall bear a ratable full, with costs against the petitioner.
judgment of default was rendered on the strength of proportion of the loss accordingly. Every item, if
the evidence submitted ex parte by the private more than one, of the policy shall be separately HARDING v. COMMERCIAL UNION ASSURANCE
respondent, which was allowed full recovery of its subject to this condition." 38 PHIL 464
claimed damages. - However, there is no evidence on record that the FISHER; August 10, 1918
- On learning of this decision, the petitioner moved building was worth P5,800,000.00 at the time of the
to lift the order of default, invoking excusable loss. On the contrary, the building was insured at FACTS
neglect, and to vacate the judgment by default. Its P2,500,000.00, and this must be considered, by - Mrs. Harding was the owner of a Studebaker
motion was denied. agreement, the actual value of the property insured automobile; in consideration of the payment to the
- On appeal, IAC affirmed the TC decision in toto. on the day the fire occurred. This valuation becomes defendant of the premium of P150, by said plaintiff,
even more believable if it is remembered that at the Mrs. Henry E. Harding, with the consent of her
ISSUE time the building was burned it was still under husband, the defendant by its duly authorized agent,
1. WON default of petitioner is based on excusable construction and not yet completed. Smith, Bell & Company (limited), made its policy of
neglect - The Court notes that the policy in this case is an insurance in writing upon said automobile was set
2. What is the amount of indemnity due to the open policy and is subject to the express condition forth in said policy to be P3,000 that the value of
private respondent under its insurance contract? that: said automobile was set forth in said policy to be
WON CFI was correct in interpreting the contract "Open Policy. P3,000; that on March 24, 1916, said automobile
This is an open policy as defined in Sec57 of the was totally destroyed by fire; that the loss thereby to
HELD Insurance Act. In the event of loss, whether total plaintiffs was the sum of P3,000.
1. NO or partial, it is understood that the amount of the - The defendant’s version is that by request of Mrs.
- Summons was served through its vice-president. loss shall be subject to appraisal and the liability of Harding, it issued the policy of insurance on an
There were even several extensions to the original the company, if established, shall be limited to the automobile alleged by the said plaintiff to be her
period to answer. As a consequence, the TC, on actual loss, subject to the applicable terms, property. It was made by means of a proposal in
motion of the private respondent filed declared the conditions, warranties and clauses of this Policy, writing signed and delivered by said plaintiff to the
petitioner in default. This was done almost one and in no case shall exceed the amount of the defendant, guaranteeing the truth of the statements
month later. Even so, the petitioner made no move policy." contained therein which said proposal is referred to
at all for two months thereafter. It was only more - As defined in the aforestated provision, which is in the said policy of insurance made a part thereof;
than one month after the judgment of default was now Sec60 of the Insurance Code, "an open policy is that certain of the statements and representations
rendered by the TC that it filed a motion to lift the one in which the value of the thing insured is not contained in said proposal and warranted by said
order of default and vacate the judgment by default. agreed upon but is left to be ascertained in case of plaintiff to be true, to wit: (a) the price paid by the
- There is a pattern of inexcusable neglect. loss.". proposer for the said automobile; (b) the value of
2. The policy is an open policy which means that the - The actual loss has been ascertained in this case said automobile at the time of the execution and
actual loss, as determined, will represent the total and the Court will respect such factual determination delivery of the said proposal and (c) the ownership
indemnity due the insured from the insurer except in the absence of proof that it was arrived at of said automobile, were false and known to be false
arbitrarily. There is no such showing. Hence, by the said plaintiff at the time of signing and
INSURANCE Page
47

delivering the said proposal and were made for the the car or P3,000 whichever is the greater against upon the circumstances of the parties, as to which
purpose of misleading and deceiving the defendant, any claim at common law made by any person (not nothing is disclosed by the record.
and inducing the defendant, relying upon the being a person in the said motor car nor in the - We are of the opinion that it would be unfair to
warranties, statements, and representations insured's service) for loss of life or for accidental hold the policy void simply because the outlay
contained in the said proposal and believing the bodily injury or damage to property caused by the represented by the automobile was made by the
same to be true, issued the said policy of insurance. said motor car including law costs payable in plaintiff's husband and not by his wife, to whom he
- The evidence shows that Hermanos, the Manila connection with such claim when incurred with the had given the automobile. It cannot be assumed that
agents for the Studebaker automobile, sold the consent of the company." defendant should not have issued the policy unless it
automobile to Canson for P3,200 (testimony of Mr. - On March 24, 1916, the said automobile was totally were strictly true that the price representing the cost
Diehl); who sold the said automobile to Henry destroyed by fire, and that the iron and steel of the machine had been paid by the insured and by
Harding for the sum of P1,500. Harding sold the said portions of said automobile which did not burn were no other person ? that it would no event insure an
automobile to J. Brannigan for the sum of P2,000 taken into the possession of the defendant by and automobile acquired by gift, inheritance, exchange,
who sold the said automobile Henry Harding for the through its agent Smith, Bell & Company (limited), or any other title not requiring the owner to make a
sum of P2,800; Henry Harding gave the said and sold by it for a small sum, which had never been specific cash outlay for its acquisition.
automobile to his wife as a present; that said tendered to the plaintiff prior to the trial of this case, 2. NO
automobile was repaired and repainted at the Luneta but in open court during the trial the sum of P10 as - It has not been shown by the evidence that the
Garage at a cost of some P900; that while the said the proceeds of such sale was tendered to plaintiff statement was false; on the contrary we believe that
automobile was at the Luneta Garage; the latter and refused. it shows that the automobile had in fact cost more
solicited of Mrs. Harding the insurance of said - Trial judge decided that there was no proof of fraud than the amount mentioned. The court below found,
automobile by the Company; that a proposal was on the part of plaintiff in her statement of the value and the evidence shows, that the automobile was
filled out by the said agent and signed by the plaintiff of the automobile, or with respect to its ownership; bought by plaintiff's husband a few weeks before the
Mrs. Henry E. Harding, and in said proposal under that she had an insurable interest therein; and that issuance of the policy in question for the sum of
the heading "Price paid by proposer," is the amount defendant, having agreed to the estimated value, P2,800, and that between that time and the issuance
of "3,500" and under another heading "Present P3,000, and having insured the automobile for that of the policy some P900 was spent upon it in repairs
value" is the amount of "3,000". amount, upon the basis of which the premium was and repainting.
- After the said proposal was made a representative paid, is bound by it and must pay the loss in - The witness Server, an expert automobile
of the Manila agent of defendant went to the Luneta accordance with the stipulated insured value. mechanic, testified that the automobile was
Garage and examined said automobile and Mr. practically as good as new at the time the insurance
Server, the General Manager of the Luneta Garage, ISSUE was effected. The form of proposal upon which the
an experienced automobile mechanic, testified that 1. WON Mrs. Harding was not the owner of the policy was issued does not call for a statement
at the time this automobile was insured it was worth automobile at the time of the issuance of the policy, regarding the value of the automobile at the time of
about P3,000, and the defendant, by and through its and, therefore, had no insurable interest in it its acquisition by the applicant for the insurance, but
said agent Smith, Bell & Company (limited), 2. WON the statement regarding the cost of the merely a statement of its cost. The amount stated
thereafter issued a policy of insurance upon proposal automobile was a warranty, that the statement was was less than the actual outlay which the automobile
in which policy the said automobile was described as false, and that, therefore, the policy never attached represented to Mr. Harding, including repairs, when
of the "present value" of P3,000 and the said to the risk the insurance policy was issued.
defendant charged the said plaintiff Mrs. Henry E. - The court below found and the evidence shows,
Harding as premium on said policy the sum of P150, HELD without dispute, that the proposal upon which the
or 5 per cent of the then estimated value of P3,000. 1. NO policy in question was issued was made out by
- The "Schedule" in said policy of insurance describes - Article 1334 of the Civil Code which provides that defendant's agent by whom the insurance was
the automobile here in question, and provides in part "All gifts between spouses during the marriage shall solicited, and that appellee simply signed the same.
of follows: be void. Moderate gifts which the spouses bestow on It also appears that an examiner employed by the
"That during the period above set forth and during each other on festive days of the family are not defendant made an inspection of the automobile
any period for which the company may agree to included in this rule." before the acceptance of the risk, and that the sum
renew this policy the company will subject to the - Even assuming that defendant might have invoked after this examination. The trial court found that Mrs.
exception and conditions contained herein or article 1334 as a defense, the burden would be upon Harding, in fixing the value of the automobile at
endorsed hereon indemnify the insured against loss it to show that the gift in question does not fall P3,000, acted upon information given her by her
of or damage to any motor car described in the within the exception therein established. We cannot husband and by Mr. Server, the manager of the
schedule hereto (including accessories) by whatever say, as a matter of law, that the gift of an Luneta Garage. She merely repeated the information
cause such loss or damage may be occasioned and automobile by a husband to his wife is not a which had been given her by her husband, and at
will further indemnify the insured up to the value of moderate one. Whether it is or is not would depend the same time disclosed to defendant's agent the
INSURANCE Page
48

source of her information. There is no evidence to Steamship Mutual to secure a license because it was protection and indemnity, war risks, and defense
sustain the contention that this communication was not engaged in the insurance business. It explained costs.
made in bad faith. We do not think that the facts that Steamship Mutual was a Protection and - A P & I Club is “a form of insurance against third
stated in the proposal can be held as a warranty of Indemnity Club (P & I Club). Likewise, Pioneer need party liability, where the third party is anyone other
the insured, even if it should have been shown that not obtain another license as insurance agent and/or than the P & I Club and the members.” By definition
they were incorrect in the absence of proof of willful a broker for Steamship Mutual because Steamship then, Steamship Mutual as a P & I Club is a mutual
misstatement. Under such circumstance, the Mutual was not engaged in the insurance business. insurance association engaged in the marine
proposal is to be regarded as the act of the insurer Moreover, Pioneer was already licensed, hence, a insurance business.
and not of the insured. separate license solely as agent/broker of Steamship - The records reveal Steamship Mutual is doing
Disposition Plaintiff was the owner of the Mutual was already superfluous. business in the country albeit without the requisite
automobile in question and had an insurable interest - The Court of Appeals affirmed the decision of the certificate of authority mandated by Section 187 of
therein; that there was no fraud on her part in Insurance Commissioner. In its decision, the the Insurance Code. It maintains a resident agent in
procuring the insurance; that the valuation of the appellate court distinguished between P & I Clubs the Philippines to solicit insurance and to collect
automobile, for the purposes of the insurance, is vis-à-vis conventional insurance. The appellate court payments in its behalf. We note that Steamship
binding upon the defendant corporation, and that the also held that Pioneer merely acted as a collection Mutual even renewed its P & I Club cover until it was
judgment of the court below is, therefore, correct agent of Steamship Mutual. cancelled due to non-payment of the calls. Thus, to
and must be affirmed, with interest, the costs of this continue doing business here, Steamship Mutual or
appeal to be paid by the appellant. ISSUES through its agent Pioneer, must secure a license
1. WON Steamship Mutual, a P & I Club, is engaged from the Insurance Commission.
WHITE GOLD MARINE SERVICES v. PIONEER in the insurance business in the Philippines - Since a contract of insurance involves public
INSURANCE 2. WON Pioneer needs a license as an insurance interest, regulation by the State is necessary. Thus,
464 SCRA 448 agent/broker for Steamship Mutual no insurer or insurance company is allowed to
QUISUMBING; July 28, 2005 engage in the insurance business without a license or
HELD a certificate of authority from the Insurance
NATURE 1. YES Commission.
This petition for review assails the Decision of the - The test to determine if a contract is an insurance 2. YES
Court of Appeals, affirming the Decision of the contract or not, depends on the nature of the - SEC. 299 . . .
Insurance Commission. Both decisions held that promise, the act required to be performed, and the - No person shall act as an insurance agent or as an
there was no violation of the Insurance Code and the exact nature of the agreement in the light of the insurance broker in the solicitation or procurement of
respondents do not need license as insurer and occurrence, contingency, or circumstances under applications for insurance, or receive for services in
insurance agent/broker. which the performance becomes requisite. It is not obtaining insurance, any commission or other
by what it is called. Basically, an insurance contract compensation from any insurance company doing
FACTS is a contract of indemnity. In it, one undertakes for business in the Philippines or any agent thereof,
- White Gold procured a protection and indemnity a consideration to indemnify another against loss, without first procuring a license so to act from the
coverage for its vessels from Steamship Mutual damage or liability arising from an unknown or Commissioner, which must be renewed annually on
through Pioneer Insurance. Subsequently, White contingent event. the first day of January, or within six months
Gold was issued a Certificate of Entry and - In particular, a marine insurance undertakes to thereafter.
Acceptance. Pioneer also issued receipts evidencing indemnify the assured against marine losses, such as Disposition The petition is PARTIALLY GRANTED.
payments for the coverage. When White Gold failed the losses incident to a marine adventure. Section 99 The Decision dated July 30, 2002 of the Court of
to fully pay its accounts, Steamship Mutual refused of the Insurance Code enumerates the coverage of Appeals affirming the Decision dated May 3, 2000 of
to renew the coverage. marine insurance. the Insurance Commission is hereby REVERSED AND
- Steamship Mutual thereafter filed a case against - Relatedly, a mutual insurance company is a SET ASIDE. The Steamship Mutual Underwriting
White Gold for collection of sum of money to recover cooperative enterprise where the members are both Association (Bermuda) Ltd., and Pioneer Insurance
the latter’s unpaid balance. White Gold on the other the insurer and insured. In it, the members all and Surety Corporation are ORDERED to obtain
hand, filed a complaint before the Insurance contribute, by a system of premiums or licenses and to secure proper authorizations to do
Commission claiming that Steamship Mutual violated assessments, to the creation of a fund from which all business as insurer and insurance agent,
Sections 186 and 187 of the Insurance Code, while losses and liabilities are paid, and where the profits respectively. The petitioner’s prayer for the
Pioneer violated Sections 299, 300 and 301 in are divided among themselves, in proportion to their revocation of Pioneer’s Certificate of Authority and
relation to Sections 302 and 303, thereof. interest. Additionally, mutual insurance associations, removal of its directors and officers, is DENIED.
- The Insurance Commission dismissed the or clubs, provide three types of coverage, namely,
complaint. It said that there was no need for PANDIMAN v. MARINE MANNING MNGT CORP.
INSURANCE Page
49

460 SCRA 418 there was no showing that Pndiman in fact - Filipinas Cia refused to pay the claim on the ground
GARCIA; June 21, 2005 negotiated the insurance contract between Sun that the policy in favor of the respondent had ceased
Richie Five and the insurer OMMIAL. Even, if to be in force on the date the United States declared
NATURE Pandiman were an agent, payment for claims arising war against Germany, the respondent Corporation
Petition for certiorari to review CA decision from peril insured against, to which the insurer is (though organized under and by virtue of the laws of
liable, is definitely not one of the liabilities of an the Philippines) being controlled by the German
FACTS insurance agent. Thus, there is no legal basis subjects and the Filipinas Cia being a company under
- Benito Singhid was hired as chief cook on board the whatsoever for holding petitioner solidarily liable with American jurisdiction when said policy was issued on
vessel MV Sun Richie Five for a term of one year by insurer OMMIAL for the widow’s claim for death October 1, 1941. Filipinas Cia, however, in
Fullwin Maritime Limited through its Philippine agent, benefits. Also, Pandiman is not a party to the pursuance of the order of the Director of Bureau of
Marine Manning and Management Corporation. While insurance contract and hence under Article 1311 of Financing, Philippine Executive Commission, dated
the said vessel was on its way to Shanghai from Ho the Civil Code, it is not liable for the obligation April 9, 1943, paid to the Huenefeld Co the sum of
Chih Minh City, Benito suffered a heart attack and arising out of the insurance contract. P92,650 on April 19, 1943.
subsequently died on June 24, 1997. 2. NO - August 6, 1946 – action filed in CFI Manila to
- Apparently, the vessel and the crew were insured - Fullwin, as Benito’s principal employer is liable recover from the Huenefeld Co the sum of P92,650
with Ocean Marine Mutual Insurance Association under the employment contract. Marine is also above mentioned. The theory of the Filipinas Cia is
Limited (OMMIAL), a Protective and Indemnity Club bound by its undertaking pursuant to the Rules and that the insured merchandise were burned up after
of which Sun Richie Five Bulkers S.A. is a member. Regulations Governing Overseas Employment that “it the policy issued in 1941 in favor of Huenefeld Co
Pandiman Philippines, the petitioner, is the local shall assume joint and solidary liability with the has ceased to be effective because of the outbreak of
correspondent of OMMIAL. employer for all the claims and liabilities which may the war between the United States and Germany on
- Benito’s widow, Rosita, filed a claim for death arise in connection with the implementation of the December 10, 1941, and that the payment made by
benefits with Marine which referred her to Pandiman. contract, including but not limited to the payment of the Filipinas Cia to Huenefeld Co during the Japanese
After her submission of the required documentation, wages, heath and disability compensation and military occupation was under pressure.
Pandiman recommended payment of the death repatriation”. In other words, both Fullwin and - CFI: dismissed the action without pronouncement
benefits amounting to $79,000. However, payment Marine should be held liable for whatever death as to costs.
has not been made. benefits the widow of Benito may be entitled to. - CA: CFI judgment affirmed, with costs. The case is
- Rosita filed a complaint with the Labor Arbiter Disposition The petition is granted and the CA now before us on appeal by certiorari from the
naming Marine, Pandiman, OMMIAL, and Fullwin as decision is reversed and set aside. decision of the Court of Appeals.
respondents. The Arbiter ordered all the
respondents, except Pandiman, to jointly and FILIPINAS COMPANIA DE SEGUROS V ISSUE
severally pay the widow the death benefits plus legal CHRISTERN, HUENEFELD AND CO INC WON the policy in question became null and void
fees. The NLRC, on appeal by Marine, limited the 89 PHIL 54 upon the declaration of war between United States
liable parties to Pandiman and OMMIAL but PARAS; May 25, 1951 and Germany
maintained the money award. The CA sustained the
decision of the NLRC. Hence this appeal. FACTS HELD
- October 1, 1941 - Christern Huenefeld, & Co., Inc., YES
after payment of corresponding premium, obtained Ratio The Philippine Insurance Law (Act No. 2427,
ISSUE from the Filipinas Cia. de Seguros a fire policy in the as amended,) in section 8, provides that "anyone
1. WON Pandiman may be held liable for the death sum of P1000,000, covering merchandise contained except a public enemy may be insured." It stands to
benefits in No. 711 Roman Street, Binondo Manila. reason that an insurance policy ceases to be
2. WON Marine and its foreign principal, Fullwin, - February 27, 1942 or during the Japanese military allowable as soon as an insured becomes a public
should be absolved from the death claim liabilities occupation - building and insured merchandise were enemy.
burned. In due time the Huenefeld Co submitted to > Effect of war, generally. - All intercourse
HELD the Filipinas Cia its claim under the policy. The between citizens of belligerent powers which is
1. NO salvage goods were sold at public auction and, after inconsistent with a state of war is prohibited by
- Pandiman is not an insurance agent as defined by deducting their value, the total loss suffered by the the law of nations. Such prohibition includes all
Section 3007 of the Insurance Code. In this case, respondent was fixed at P92,650. negotiations, commerce, or trading with the
enemy; all acts which will increase, or tend to
7
Section 300. Any person who for compensation solicits or obtains on increase, its income or resources; all acts of
behalf of any insurance company transmits for a person other than insurance shall be an insurance agent within the intent of this section voluntary submission to it; or receiving its
himself an application for a policy or contract of insurance to or from and shall thereby become liable to all the duties, requirements, protection; also all acts concerning the
such company or offers or assumes to act in the negotiating of such liabilities, and penalties to which an insurance agent is subject.
INSURANCE Page
50

transmission of money or goods; and all contracts been adopted. In "Enemy Corporation" by Martin FACTS
relating thereto are thereby nullified. It further Domke, a paper presented to the Second - Buenaventura Ebrado obtained a whole-life
prohibits insurance upon trade with or by the International Conference of the Legal Profession held insurance policy from Insular, for P5,882.00 with a
enemy, upon the life or lives of aliens engaged in at the Hague (Netherlands) in August. 1948 also rider for accidental death benefits for the same
service with the enemy; this for the reason that discussed this dilemma amount. He designated Carponia Ebrado as the
the subjects of one country cannot be permitted to > In Clark vs. Uebersee Finanz Korporation, A. G., revocable beneficiary, referring to her as the wife.
lend their assistance to protect by insurance the dealing with a Swiss corporation allegedly - Afterwards, he died as a result of an accident when
commerce or property of belligerent, alien controlled by German interest, the Court: "The he was hit by a falling branch of a tree. Carponia
subjects, or to do anything detrimental too their property of all foreign interest was placed within filed a claim for the proceeds as the designated
country's interest. The purpose of war is to cripple the reach of the vesting power (of the Alien beneficiary in the policy, although she admits that
the power and exhaust the resources of the Property Custodian) not to appropriate friendly or she and Buenaventura were merely living as
enemy, and it is inconsistent that one country neutral assets but to reach enemy interest which husband and wife without the benefit of marriage.
should destroy its enemy's property and repay in masqueraded under those innocent fronts. . . . The The legal wife, Pascuala Vda De Ebrado, also filed
insurance the value of what has been so power of seizure and vesting was extended to all her claim as the widow of the deceased.
destroyed, or that it should in such manner property of any foreign country or national so that - Insular then filed an interpleader in court (CFI
increase the resources of the enemy, or render it no innocent appearing device could become a Rizal) to determine to whom the proceeds should be
aid, and the commencement of war determines, Trojan horse." paid. CFI declared that Carponia was disqualified
for like reasons, all trading intercourse with the - The respondent having become an enemy from becoming beneficiary of the insured and
enemy, which prior thereto may have been lawful. corporation on December 10, 1941, the insurance directing the Insular to pay the proceeds to the
All individuals therefore, who compose the policy issued in its favor on October 1, 1941, by the estate of Buenaventura.
belligerent powers, exist, as to each other, in a petitioner (a Philippine corporation) had ceased to be
state of utter exclusion, and are public enemies. (6 valid and enforcible, and since the insured goods ISSUE
Couch, Cyc. of Ins. Law, pp. 5352-5353.) were burned after December 10, 1941, and during 1. WON a common-law wife named as beneficiary in
> In the case of an ordinary fire policy, which the war, the respondent was not entitled to any the insurance policy of a legally married man claim
grants insurance only from year, or for some other indemnity under said policy from the petitioner. the proceeds of the same
specified term it is plain that when the parties However, elementary rules of justice (in the absence
become alien enemies, the contractual tie is of specific provision in the Insurance Law) require HELD
broken and the contractual rights of the parties, that the premium paid by the respondent for the 1. NO
so far as not vested. lost. (Vance, the Law on period covered by its policy from December 11, Ratio The prohibition that husband and wife cannot
Insurance, Sec. 44, p. 112.) 1941, should be returned by the petitioner. donate to each other applies to common-law
Reasoning Disposition the appealed decision is hereby relationships. As the appointment of a beneficiary in
- The Court of Appeals overruled the contention of reversed and the respondent corporation is ordered insurance may be considered a donation, one cannot
the petitioner that the respondent corporation to pay to the petitioner the sum of P77,208.33, name as beneficiary his common-law wife.
became an enemy when the United States declared Philippine currency, less the amount of the premium, Reasoning
war against Germany, relying on English and in Philippine currency, that should be returned by the - It is quite unfortunate that the Insurance Code
American cases which held that a corporation is a petitioner for the unexpired term of the policy in does not contain any specific provision grossly
citizen of the country or state by and under the laws question, beginning December 11, 1941. resolutory of the prime question at hand.
of which it was created or organized. It rejected the - Rather, general rules of civil law should be applied
theory that nationality of private corporation is to resolve the issue. Art.2011, CC states: “The
determined by the character or citizenship of its contract of insurance is governed by special laws.
controlling stockholders. Matters not expressly provided for in such special
- There is no question that majority of the laws shall be regulated by this Code.” Thus, when
stockholders of the respondent corporation were not otherwise specifically provided for by the
German subjects. Therefore, Huenefeld Co became Insurance Law, the contract of life insurance is
an enemy corporation upon the outbreak of the war INSULAR LIFE ASSURANCE CO. v. EBRADO governed by the general rules of the civil law
between the United States and Germany. The 80 SCRA 181 regulating contracts.
English and American cases relied upon by the Court MARTIN; October 28, 1977 - Also, Art.2012 “any person who is forbidden from
of Appeals have lost their force in view of the latest receiving any donation under Article 739 cannot be
decision of the Supreme Court of the United States NATURE named beneficiary of a life insurance policy by the
in Clark vs. Uebersee Finanz Korporation, decided on Appeal from judgment of RTC. person who cannot make a donation to him.”
December 8, 1947, in which the controls test has
INSURANCE Page
51

Common-law spouses are, definitely, barred from marriages, the first with herein respondent Rosario ISSUE
receiving donations from each other. Diaz, solemnized in the parish church of San Nicolas WON GSIS was correct in awarding half of the
- Art.739, CC: The following donations shall be void: de Tolentino, Surigao, Surigao, on July 15, 1937, out retirement benefit of the deceased to Rosario Diaz,
1. Those made between persons who were guilty of of which marriage were born two children, namely, the first wife, notwithstanding the fact that the
adultery or concubinage at the time of donation; Jose Consuegra, Jr. and Pedro Consuegra, but both petitioners were named as beneficiaries of the life
- In essence, a life insurance policy is no different predeceased their father; and the second, which was insurance
from a civil donation insofar as the beneficiary is contracted in good faith while the first marriage was
concerned. Both are founded upon the same subsisting, with herein petitioner Basilia Berdin, on HELD
consideration: liberality. A beneficiary is like a May 1, 1957 in the same parish and municipality, out YES
donee, because from the premiums of the policy of which marriage were born seven children, namely, - The GSIS offers two separate and distinct systems
which the insured pays out of liberality, the Juliana, Pacita, Maria Lourdes, Jose, Rodrigo, Lenida of benefits to its members, one is the life insurance
beneficiary will receive the proceeds or profits of said and Luz, all surnamed Consuegra. and the other is the retirement insurance. These two
insurance. As a consequence, the proscription in - Being a member of the Government Service distinct systems of benefits are paid out from two
Art.739 CC should equally operate in life insurance Insurance System (GSIS, for short) when Consuegra distinct and separate funds that are maintained by
contracts. The mandate of Art.2012 cannot be laid died on September 26, 1965, the proceeds of his life the GSIS. Thus, it doesn’t necessarily mean that the
aside: any person who cannot receive a donation insurance under policy No. 601801 were paid by the beneficiaries in the life insurance are also the
cannot be named as beneficiary in the life insurance GSIS to petitioner Basilia Berdin and her children beneficiaries in the retirement insurance.
policy of the person who cannot make the donation. who were the beneficiaries named in the policy. - Consuegra started in the government service
- Policy considerations and dictates of morality - However, Consuegra did not designate any sometime during the early part of 1943, or before
rightly justify the institution of a barrier between beneficiary who would receive the retirement 1943. In 1943 Com. Act 186 was not yet amended,
common-law spouses in regard to property relations insurance benefits due to him. Respondent Rosario and the only benefits then provided for in said Com.
since such relationship ultimately encroaches upon Diaz, the widow by the first marriage, filed a claim Act 186 were those that proceed from a life
the nuptial and filial rights of the legitimate family. with the GSIS asking that the retirement insurance insurance. Upon entering the government service
There is every reason to hold that the bar in benefits be paid to her as the only legal heir of Consuegra became a compulsory member of the
donations between legitimate spouses and those Consuegra, considering that the deceased did not GSIS, being automatically insured on his life,
between illegitimate ones should be enforced in life designate any beneficiary with respect to his pursuant to the provisions of Com. Act 186 which
insurance policies since the same are based on retirement insurance benefits. Petitioner Basilia was in force at the time. During 1943 the operation
similar consideration. Berdin and her children, likewise, filed a similar claim of the Government Service Insurance System was
- So long as marriage remains the threshold of with the GSIS, asserting that being the beneficiaries suspended because of the war, and the operation
family laws, reason and morality dictate that the named in the life insurance policy of Consuegra, they was resumed sometime in 1946. When Consuegra
impediments imposed upon married couple should are the only ones entitled to receive the retirement designated his beneficiaries in his life insurance he
likewise be imposed upon extra-marital relationship. insurance benefits due the deceased Consuegra. could not have intended those beneficiaries of his life
If legitimate relationship is circumscribed by these Resolving the conflicting claims, the GSIS ruled that insurance as also the beneficiaries of his retirement
legal disabilities, with more reason should an illicit the legal heirs of the late Jose Consuegra were insurance because the provisions on retirement
relationship be restricted by these disabilities. Rosario Diaz, his widow by his first marriage who is insurance under the GSIS came about only when
Disposition Decision AFFIRMED. entitled to one-half, or 8/16, of the retirement Com. Act 186 was amended by Rep. Act 660 on June
CONSUEGRA v. GSIS insurance benefits, on the one hand; and Basilia 16, 1951. Hence, it cannot be said that because
37 SCRA 315 Berdin, his widow by the second marriage and their herein appellants were designated beneficiaries in
ZALDIVAR; January 30, 1971 seven children, on the other hand, who are entitled Consuegra's life insurance they automatically
to the remaining one-half, or 8/16, each of them to became the beneficiaries also of his retirement
NATURE receive an equal share of 1/16. insurance.
Appeal from the decision of the Court of First - Dissatisfied with the foregoing ruling and - The provisions of subsection (b) of Section 11 of
Instance of Surigao del Norte awarding the 8/16 part apportionment made by the GSIS, Basilia Berdin and Commonwealth Act 186, as amended by Rep. Act
of the proceeds of the deceased Consuegra’s her children filed on October 10, 1966 a petition for 660, clearly indicate that there is need for the
retirement benefits to Rosario Diaz. mandamus with preliminary injunction in the Court employee to file an application for retirement
of First Instance of Surigao. insurance benefits when he becomes a member of
FACTS - The CFI of Surigao ruled in favor of respondent the GSIS, and he should state in his application the
- The late Jose Consuegra, at the time of his death, Rosario Diaz and upheld the ruling of GSIS in all beneficiary of his retirement insurance. Hence, the
was employed as a shop foreman of the office of the aspect. Thus, Basilia Berdin and her children beneficiary named in the life insurance does not
District Engineer in the province of Surigao del appealed said decision to the Supreme Court. automatically become the beneficiary in the
Norte. In his lifetime, Consuegra contracted two retirement insurance unless the same beneficiary in
INSURANCE Page
52

the life insurance is so designated in the application he had a minor daughter, Elizabeth Davac. Both filed - The disqualification mentioned in Article 739 is not
for retirement insurance. their claims for death benefit with the SSS. applicable to herein appellee Candelaria Davac
- In the case of the proceeds of a life insurance, the - Social Security Commission issued the resolution because she was not guilty of concubinage, there
same are paid to whoever is named the beneficiary declaring respondent Candelaria Davac as the person being no proof that she had knowledge of the
in the life insurance policy. As in the case of a life entitled to receive the death benefits payable for the previous marriage of her husband Petronilo.
insurance provided for in the Insurance Act, the death of Petronilo Davac. ART. 2012. Any person who is forbidden from
beneficiary in a life insurance under the GSIS may receiving any donation under Article 739 cannot be
not necessarily be an heir of the insured. The ISSUES named beneficiary of a life insurance policy by the
insured in a life insurance may designate any 1. WON the Social Security Commission Candelaria person who cannot make any donation to him
person as beneficiary unless disqualified to be Davac is entitled to receive the death benefits according to said article.
so under the provisions of the Civil Code. And in 2. WON a beneficiary under the Social Security ART. 739. The following donations shall be void:
the absence of any beneficiary named in the life System partakes of the nature of a beneficiary in a (1) Those made between persons who were guilty
insurance policy, the proceeds of the insurance will life insurance policy and, therefore the designation of adultery or concubinage at the time of the
go to the estate of the insured. made in the person DAVAC as bigamous wife is null donation; (the court did not decide whether this
- On the other hand, the beneficiary of the and void, because it contravenes the provisions of partakes the nature of a life insurance policy)
retirement insurance can only claim the proceeds of the Civil Code 3. NO
the retirement insurance if the employee dies before 3. WON the benefits accruing from membership with - The benefit receivable under the Act is in the
retirement. If the employee failed or overlooked to SSS forms part of the conjugal property thus the nature of a special privilege or an arrangement
state the beneficiary of his retirement insurance, the resolution deprives the lawful wife of her share in the secured by the law pursuant to the policy of the
retirement benefits will accrue to his estate and will conjugal property as well as of her own and her State to provide social security to the workingmen.
be given to his legal heirs in accordance with law, as child's legitime in the inheritance The amounts that may thus be received cannot be
in the case of a life insurance if no beneficiary is considered as property earned by the member
named in the insurance policy. HELD during his lifetime. His contribution to the fund
Disposition Petition Denied. It is Our view, 1. YES constitutes only an insignificant portion thereof.
therefore, that the respondent GSIS had correctly - Section 13, RA1161 provides that the beneficiary Then, the benefits are specifically declared not
acted when it ruled that the proceeds of the "as recorded" by the employee's employer is the one transferable, and exempted from tax, legal
retirement insurance of the late Jose Consuegra entitled to the death benefits. processes, and lien. Furthermore, in the settlement
should be divided equally between his first living wife - Section 13, Republic Act No. 1161, as amended by of claims thereunder, the procedure to be observed
Rosario Diaz, on the one hand, and his second wife Republic Act No. 1792, in force at the time of is governed not by the general provisions of law, but
Basilia Berdin and his children by her. Petronilo Davac's death provides: Upon the covered by rules and regulations promulgated by the
employee's death or total and permanent disability Commission. Thus, if the money is payable to the
SSS v. DAVAC under such conditions as the Commission may estate of a deceased member, it is the Commission,
17 SCRA 863 define, before becoming eligible for retirement and if not the probate or regular court that determines the
BARRERA: July 30, 1966 either such death or disability is not compensable person or persons to whom it is payable.
under the Workmen's Compensation Act, he or. in - They are disbursed from a public special fund
NATURE case of his death, his beneficiaries, as recorded by created by Congress.The sources of this special fund
APPEAL from a resolution Of the Social Security his employer shall be entitled to the following are the covered employee's contribution (equal to 2-
Commission. benefit: 1/2 per cent of the employee's monthly
- In Tecson vs. Social Security System. Section 13 compensation) ; the employer's 'Contribution
FACTS was construed:"it may be true that the purpose of (equivalent to 3-1/2 per cent of the monthly
- Petronilo Davac, became a member of the Social the coverage under the Social Security System is compensation of the covered employee) ;and the
Security System (SSS for short) on September 1, protection of the employee as well as of his family, Government contribution which consists in yearly
1957. In the Member's Record he designated but this purpose or intention of the law cannot be appropriation of public funds to assure the
respondent, Candelaria Davac as his beneficiary and enforced to the extent of contradicting the very maintenance of an adequate working balance of the
indicated his relationship to her as that of "wife". provisions of said law contained in Section 13, funds of the System. Additionally, Section 21 of the
- He died on April 5, 1959. It appears that the thereof” Social Security Actprovides that the benefits
deceased contracted two marriages, the first, with - When the provisions of a law are clear and explicit, prescribed in this Act shall not be diminished and the
Lourdes Tuplano on August 29, 1946, who bore him the courts can do nothing but apply its clear and Government of the Republic of the Philippines
a child, Romeo Davac, and the second, with explicit provisions (Velasco vs. Lopez) accepts general responsibility for the solvency of the
Candelaria Davac on January 18, 1949, with whom 2. NO System.
INSURANCE Page
53

- The benefits under the Social Security Act are not counterclaim that the redemption of the real estate an insurance policy belong exclusively to the
intended by the lawmaking body to form part of the sold by his father was made in the name of the beneficiary and not to the estate of the person
estate of the covered –members. plaintiffs and himself instead of in his name alone whose life was insured, and that such proceeds are
- Social Security Act is not a law of succession. without his knowledge or consent. Andres contends the separate and individual property of the
Disposition Resolution of the Social Security that it was not his intention to use the proceeds of beneficiary, and not of the heirs of the person whose
Commission appealed is affirmed the insurance policy for the benefit of any person but life was insured, is the doctrine in America. We
himself, he alleging that he was and is the sole believe that the same doctrine obtains in these
FRANCISCO DEL VAL v. ANDRES DEL VAL owner thereof and that it is his individual property. Islands by virtue of section 428 of the Code of
29 PHIL 534 He, therefore, asks that he be declared the owner of Commerce, which reads:
MORELAND; February 16, 1915 the real estate redeemed by the payment of the "The amounts which the underwriter must deliver
P18,365.20, the owner of the remaining P21,634.80, to the person insured, in fulfillment of the
NATURE the balance of the insurance policy, and that the contract, shall be the property of the latter, even
Appeal from a judgment of the Court of First plaintiffs account for the use and occupation of the against the claims of the legitimate heirs or
Instance of the city of Manila dismissing the premises so redeemed since the date of the creditors of any kind whatsoever of the person
complaint with costs. redemption. who effected the insurance in favor of the former."
- The trial court refused to give relief to either party 2. NO
FACTS and dismissed the action. In this appeal, it is claimed - The contract of life insurance is a special contract
- Plaintiffs and defendant are brothers and sisters; by the attorney for the plaintiffs that insurance and the destination of the proceeds thereof is
that they are the only heirs at law and next of kin of provisions in the Code of Commerce are determined by special laws which deal exclusively
Gregorio Nacianceno del Val, who died in Manila on subordinated to the provisions of the Civil Code as with that subject. The Civil Code has no provisions
August 4, 1910, intestate found in article 1035. This article reads: which relate directly and specifically to life-insurance
- During the lifetime of the deceased he took out "An heir by force of law surviving with others of the contracts or to the destination of life insurance
insurance on his life for the sum of P40,000 and same character to a succession must bring into the proceeds. That subject is regulated exclusively by
made it payable to the defendant ANDRES DEL VAL hereditary estate the property or securities he may the Code of Commerce which provides for the terms
as sole beneficiary. After his death the defendant have received from the deceased during the life of of the contract, the relations of the parties and the
collected the face of the policy. From said policy he the same, by way of dowry, gift, or for any good destination of the proceeds of the policy.
paid the sum of P18,365.20 to redeem certain real consideration, in order to compute it in fixing the - Assuming that the proceeds of the life-insurance
estate which the decedent had sold to third persons legal portions and in the account of the division." policy being the exclusive property of the defendant
with a right to repurchase. - Counsel also claims that the proceeds of the and he having used a portion thereof in the
- The redemption of said premises was made by the insurance policy were a donation or gift made by the repurchase of the real estate sold by the decedent
attorney of defendant ANDRES in the name of the father during his lifetime to the defendant and that, prior to his death with right to repurchase, and such
plaintiffs and the defendant as heirs of the deceased as such, its ultimate destination is determined by repurchase having been made and the conveyance
vendor. It further appears from the pleadings that those provisions of the Civil Code which relate to taken in the names of all of the heirs instead of the
the defendant, on the death of the deceased, took donations, especially article 819. This article provides defendant alone, plaintiffs claim that the property
possession of most of his personal property, which that "gifts made to children which are not belongs to the heirs in common and not to the
he still has in his possession, and that he has also betterments shall be considered as part of their legal defendant alone.
the balance on said insurance policy amounting to portion." - The Court rejected this contention unless the fact
P21,634.80. appear or be shown that the defendant acted as he
- Plaintiffs contend that the amount of the insurance ISSUES did with the intention that the other heirs should
policy belonged to the estate of the deceased and 1. WON the insurance belongs to the defendant and enjoy with him the ownership of the estate ---- in
not to the defendant personally; that, therefore, they not to the decedent’s estate other words, that he proposed, in effect, to make a
are entitled to a partition not only of the real and 2. WON the Civil code provisions on succession gift of the real estate to the other heirs. If it is
personal property, but also of the P40,000 life prevail over any other law with respect to the established by the evidence that was his intention
insurance. The complaint prays a partition of all the insurance and that the real estate was delivered to the
property, both real and personal, left by the plaintiffs with that understanding, then it is probable
deceased; that the defendant account for HELD that their contention is correct and that they are
P21,634.80, and that the sum be divided equally 1. YES entitled to share equally with the defendant therein.
among the plaintiffs and defendant along with the - The SC agreed with the finding of the trial court If, however, it appears from the evidence in the case
other property of deceased. that the proceeds of the life-insurance policy belong that the conveyances were taken in the name of the
- The defendant denies the material allegations of exclusively to the defendant as his individual and plaintiffs without his knowledge or consent, or that it
the complaint and sets up as special defense and separate property, we agree. That the proceeds of was not his intention to make a gift to them of the
INSURANCE Page
54

real estate, then it belongs to him. If the facts are as in her stead his present wife, Adela Garcia de Gercio, - Insurance Act- there is likewise no provision
stated, he has two remedies. The one is to compel as the beneficiary of the policy. Gercio requested the either permitting or prohibiting the insured to change
the plaintiffs to reconvey to him and the other is to insurance company to eliminate Andrea Zialcita as the beneficiary.
let the title stand with them and to recover from beneficiary. This, the insurance company has refused 2. NO
them the sum he paid on their behalf. and still refuses to do. Ratio The wife has an insurable interest in the life of
- For the complete and proper determination of the her husband. The beneficiary has an absolute vested
questions at issue in this case, the Court was of the ISSUES interest in the policy from the date of its issuance
opinion that the cause should be returned to the trial 1. (Preliminary) WON the provisions of the Code of and delivery. So when a policy of life insurance is
court with instructions to permit the parties to frame Commerce and the Civil Code shall be in force in taken out by the husband in which the wife is named
such issues as will permit the settlement of all the 1910, or the provisions of the Insurance Act now in as beneficiary, she has a subsisting interest in the
questions involved and to introduce such evidence as force, or the general principles of law, guide the policy. And this applies to a policy to which there are
may be necessary for the full determination of the court in its decision attached the incidents of a loan value, cash
issues framed. Upon such issues and evidence taken 2. WON the insured, the husband, has the power to surrender value, an automatic extension by
thereunder the court will decide the questions change the beneficiary, the former wife, and to name premiums paid, and to an endowment policy, as well
involved according to the evidence, subordinating his instead his actual wife, where the insured and the as to an ordinary life insurance policy. If the husband
conclusions of law to the rules laid down in this beneficiary have been divorced and where the policy wishes to retain to himself the control and ownership
opinion. REMANDED. of insurance does not expressly reserve to the of the policy he may so provide in the policy. But if
insured the right to change the beneficiary the policy contains no provision authorizing a change
GERCIO v. SUN LIFE ASSURANCE OF CANADA of beneficiary without the beneficiary's consent, the
48 PHIL 53 HELD insured cannot make such change. Accordingly, it is
MALCOLM; September 28, 1925 1. Whether the case be considered in the light of the held that a life insurance policy of a husband made
Code of Commerce, the Civil Code, or the Insurance payable to the wife as beneficiary, is the separate
NATURE Act, the deficiencies in the law will have to be property of the beneficiary and beyond the control of
Mandamus to compel Sun Life Assurance Co. of supplemented by the general principles prevailing on the husband.
Canada to change the beneficiary in the policy issued the subject. To that end, we have gathered the rules - Unlike the statutes of a few jurisdictions, there is
by the defendant company on the life of the plaintiff which follow from the best considered American no provision in the Philippine Law permitting the
Hilario Gercio authorities. In adopting these rules, we do so with beneficiary in a policy for the benefit of the wife of
the purpose of having the Philippine Law of the husband to be changed after a divorce. It must
FACTS Insurance conform as nearly as possible to the follow, therefore, in the absence of a statute to the
- On January 29, 1910, the Sun Life Assurance Co. of modern Law of Insurance as found in the United contrary, that if a policy is taken out upon a
Canada issued an insurance policy on the life of States proper. husband's life the wife is named as beneficiary
Hilario Gercio. The policy was what is known as a 20- - Court’s first duty is to determine what law should therein, a subsequent divorce does not destroy her
year endowment policy. By its terms, the insurance be applied to the facts. The insurance policy was rights under the policy.
company agreed to insure the life of Hilario Gercio taken out in 1910, that the Insurance Act. No. 2427, Reasoning
for the sum of P2,000, to be paid him on February 1, became effective in 1914, and that the effort to - Yore vs. Booth
1930, or if the insured should die before said date, change the beneficiary was made in 1922. “. . . It seems to be the settled doctrine, with but
then to his wife, Mrs. Andrea Zialcita, should she - Code of Commerce- there can be found in it no slight dissent in the courts of this country, that a
survive him; otherwise to the executors, provision either permitting or prohibiting the insured person who procures a policy upon his own life,
administrators, or assigns of the insured. The policy to change the beneficiary. payable to a designated beneficiary, although he
did not include any provision reserving to the insured - Civil Code- it would be most difficult, if indeed it is pays the premiums himself, and keeps the policy
the right to change the beneficiary. practicable, to test a life insurance policy by its in his exclusive possession, has no power to
- On the date the policy was issued, Andrea Zialcita provisions. In the case of Del Val vs. Del Val, it change the beneficiary, unless the policy itself, or
was the lawful wife of Hilario Gercio. Towards the declined to consider the proceeds of the insurance the charter of the insurance company, so provides.
end of the year 1919, she was convicted of the crime policy as a donation or gift, saying "the contract of In policy, although he has parted with nothing,
of adultery. On September 4, 1920, a decree of life insurance is a special contract and the and is simply the object of another's bounty, has
divorce was issued in civil case no. 17955, which had destination of the proceeds thereof is determined by acquired a vested and irrevocable interest in the
the effect of completely dissolving their bonds of special laws which deal exclusively with that subject. policy, which he may keep alive for his own benefit
matrimony The Civil Code has no provisions which relate directly by paying the premiums or assessments if the
- On March 4, 1922, Hilario Gercio formally notified and specifically to life-insurance contracts or to the person who effected the insurance fails or refuses
the Sun Life that he had revoked his donation in destination of life-insurance proceeds. . . ." to do so.”
favor of Andrea Zialcita, and that he had designated
INSURANCE Page
55

- Connecticut Mutual Life Insurance Company may have years thereafter secured a divorce from FACTS
vs Schaefer her husband, and he was thereafter again married - In 1968, Private Respondent Rodolfo Dimayuga
“We do not hesitate to say, however, that a policy to one who sustained the relation of wife to him at procured an ordinary life insurance policy from the
taken out in good faith and valid at its inception, is the time of his death. petitioner company and designated his wife and
not avoided by the cessation of the insurable The rights of a beneficiary in an ordinary life children as irrevocable beneficiaries. On Feb. 22,
interest, unless such be the necessary effect of the insurance policy become vested upon the issuance 1980, Dimayuga filed with the CFI a petition to
provisions of the policy itself.. . . .In our judgment of the policy, and can thereafter, during the life of amend the designation of the beneficiaries in his life
of life policy, originally valid, does not cease to be the beneficiary, be defeated only as provided by policy from irrevocable to revocable. Petitioner filed
so by the cessation of the assured party's interest the terms of the policy.” an Urgent Motion to reset hearing as well as its
in the life insured.” - On the admitted facts and the authorities comment and/or Opposition to the respondent’s
- Central National Bank of Washington City vs. supporting the nearly universally accepted principles petition.
Hume of insurance, we are irresistibly led to the conclusion - Respondent Judge denied petitioner’s Urgent
“It is indeed the general rule that a policy, and the that the question at issue must be answered in the Motion, thus allowing private respondent to adduce
money to become due under it, belong, the negative evidence, the consequence of which was the
moment it is issued, to the person or persons Disposition The judgment appealed from will be issuance of the questioned Order granting the
named in it as the beneficiary or beneficiaries, and reversed and the complaint ordered dismissed as to petition. Petitioner then filed a MFR which was also
that there is no power in the person procuring the the appellant. denied hence this petition.
insurance, by any act of his, by deed or by will, to
transfer to any other person the interest of the SEPARATE OPINION ISSUE
person named.” 1. WON the designation of the irrevocable
- In re Dreuil & Co. JOHNSON [concur] beneficiaries could be changed or amended without
“In so far as the law of Louisiana is concerned, it - I agree with the majority of the court, that the the consent of all the irrevocable beneficiaries
may also be considered settled that where a policy judgment of the lower court should be revoked, but 2. WON the irrevocable beneficiaries herein, one of
is of the semitontine variety, as in this case, the for a different reason. The purpose of the petition is whom is already deceased while the others are all
beneficiary has a vested right in the policy, of to have declared the rights of certain persons in an minors could validly give consent to the change or
which she cannot be deprived without her insurance policy which is not yet due and payable. It amendment in the designation of the irrevocable
consent” may never become due and payable. The premiums beneficiaries
- Wallace vs Mutual Benefit Life Insurance Co. may not be paid, thereby rendering the contract of
“As soon as the policy was issued Mrs. Wallace insurance of non effect, and many other things may HELD
acquired a vested interest therein, of which she occur, before the policy becomes due, which would 1. NO
could not be deprived without her consent, except render it non effective. The plaintiff and the other - Based on the provision of their contract and the law
under the terms of the contract with the insurance parties who are claiming an interest in said policy applicable, it is only with the consent of all the
company. No right to change the beneficiary was should wait until there is something due them under beneficiaries that any change or amendment in the
reserved. Her interest in the policy was her the same. For the courts to declare now who are the policy concerning the irrevocable beneficiaries may
individual property, subject to be divested only by persons entitled to receive the amounts due, if they be legally and validly effected. Both the law and the
her death, the lapse of time, or by the failure of ever become due and payable, is impossible, for the Policy do not provide for any other exception.
the insured to pay the premiums. She could keep reason that nothing may ever become payable under Reasoning
the policy alive by paying the premiums, if the the contract of insurance, and for many reasons such - Since the policy was procured in 1968, the
insured did not do so. It was contingent upon persons may never have a right to receive anything applicable law in this case is the Insurance Act and
these events, but it was free from the control of when the policy does become due and payable. In under that law, the beneficiary designated in a life
her husband. He had no interest in her property in my judgment, the action is premature and should insurance contract cannot be changed without the
this policy, contingent or otherwise. Her interest have been dismissed. consent of the beneficiary because he has a vested
was free from any claim on the part of the insured interest in the policy.
or his creditors. He could deprive her of her PHIL. AMERICAN LIFE INSURANCE v. PINEDA - The Beneficiary Designation Indorsement in the
interest absolutely in but one way, by living more 175 SCRA 416 policy in the name of Dimayuga states that the
than twenty years.” PARAS; July 19, 1989 designation of the beneficiaries is irrevocable: “no
- Filley vs. Illinois Life Insurance Company right or privilege under the Policy may be exercised,
“The benefit accruing from a policy of life NATURE or agreement made with the Company to any
insurance upon the life of a married man, payable Petition for review on certiorari the orders of CFI change in or amendment to the Policy, without the
upon his death to his wife, naming her, is payable Judge Pineda consent of the said beneficiary/beneficiaries.”
to the surviving beneficiary named, although she
INSURANCE Page
56

- Contracts which are the private laws of the - July 10, 1919, Dy Poco died, and on August 21, assignee all of the estate of the insolvent debtor not
contracting parties should be fulfilled according to 1919, Tan Sit, was duly appointed as the exempt by law from execution."
the literal sense of their stipulations, if their terms administratrix of his intestate estate. - the Insolvency Law is in great part a copy of the
are clear and leave no room for doubt as to the - By the terms of the policy it was provided that after Insolvency Act of California, enacted in 1895, though
intention of the contracting parties, for contracts are the payment of three full premiums, the assured it contains a few provisions from the American
obligatory, no matter in what form they may be, could surrender the policy to the company for a Bankruptcy Law of 1898
whenever the essential requisites for their validity "cash surrender value," indicated in an annexed - Under each of said laws the assignee acquires all
are present. table; but inasmuch as no more than two premiums the real and personal property, estate, and effects of
- Finally, the fact that the contract of insurance does had been paid upon the policy now in question up to the debtor, not exempt by law from execution, with
not contain a contingency when the change in the the time of the death of the assured, this provision all deeds, books and papers relating thereto; and
designation of beneficiaries could be validly effected had not become effective; and it does not appear while this language is broad, it nevertheless lacks
means that it was never within the contemplation of that the company would in accordance with its own the comprehensiveness of section 70 (a) of the
the parties. usage or otherwise have made any concession to the American Bankruptcy Law of 1898 in at least two
2. NO assured in the event he had desired, before his particulars; for under subsection 3 of section 70 (a)
- The parent-insured cannot exercise rights and/or death, to surrender the policy. It must therefore be of the last mentioned law, the trustee in bankruptcy
privileges pertaining to the insurance contract, for accepted that this policy had no cash surrender acquires the right to exercise any powers which the
otherwise, the vested rights of the irrevocable value, at the time of the assured's death, either by insolvent might have exercised for his own benefit,
beneficiaries would be rendered inconsequential. The contract or by convention practice of the company in and under subsection 5 the trustee acquires any
alleged acquiescence of the 6 children beneficiaries such cases. property of the insolvent which the latter could by
cannot be considered an effective ratification to the - Both Ingersoll, as assignee, and Tan Sit, as any means have assigned to another. The Insolvency
change of the beneficiaries from irrevocable to administratix of Dy Poco's estate, asserted claims to Law here in force, in common with the predecessor
revocable. They were minors at the time, and could the proceeds of the policy. The lower court found laws above-mentioned, contains nothing similar to
not validly give consent. Neither could they act that Ingersoll had a better right and ordered Sun Life these provisions.
through their father-insured since their interests are to pay the insurance proceeds to him. On the applicability of the Insolvency Law
quite divergent from one another. - Sec 32 of the Insolvency Law among other things,
Disposition questioned Orders of respondent judge ISSUE declares that the assignment to be made by the
are nullified and set aside. WON Ingersoll, as assignee, has a right to the clerk of the court "shall operate to vest in the
proceeds of the insurance assignee all of the estate of the insolvent debtor not
SUN LIFE ASSURANCE v. INGERSOLL exempt by law from execution." Moreover, by section
41 PHIL 331 HELD 24, the court is required, upon making an order
STREET; November 8, 1921 NO adjudicating any person insolvent, to stay any civil
On the Philippine Insolvency Law (Act No. 1956) proceedings pending against him; and it is declared
NATURE - The property and interests of the insolvent which in section 60 that no creditor whose debt is provable
Action of interpleader become vested in the assignee of the insolvent are under the Act shall be allowed, after the
specified in section 32 of the Insolvency Law which commencement of proceedings in insolvency, to
FACTS reads as follows: prosecute to final judgment any action therefor
- April 16, 1918, Sun Life Assurance Company of "SEC. 32. As soon as an assignee is elected against the debtor. In connection with the foregoing
Canada (Sun Life), in consideration of the payment or appointed and qualified, the clerk of the court may be mentioned subsections 1 and 2 of section 36,
of a stipulated annual premium during the period of shall, by an instrument under his hand and seal of as well as the opening words of section 33, to the
the policy, or until the premiums had been the court, assign and convey to the assignee all the effect that the assignee shall have the right and
completely paid for twenty years, issued a policy of real and personal property, estate, and effects of the power to recover and to take into his possession, all
insurance on the life of Dy Poco for US$12,500, debtor with all his deeds, books, and papers relating of the estate, assets, and claims belonging to the
payable to the said assured or his assigns on the thereto, and such assignment shall relate back to the insolvent, except such as are exempt by law from
21st day of February, 1938, and if he should die commencement of the proceedings in insolvency, execution.
before that date then to his legal representatives. and shall relate back to the acts upon which the - These provisions clearly evince an intention to vest
- June 23, 1919, the assured, Dy Poco, was adjudication was founded, and by operation of law in the assignee, for the benefit of all the creditors of
adjudged an involuntary insolvent by the CFI Manila, shall vest the title to all such property, estate, and the insolvent, such elements of property and
and Frank B. Ingersoll was appointed assignee of his effects in the assignee, although the same is then property right as could be reached and subjected by
estate. attached on mesne process, as the property of the process of law by any single creditor suing alone.
debtor. Such assignment shall operate to vest in the And this is exactly as it should be: for it cannot be
supposed that the Legislature would suppress the
INSURANCE Page
57

right of action of every individual creditor upon the discharge of the bankrupt, have no insurable interest to the extent that such insurance contains assets
adjudication of insolvency, and at the same time in the life of the bankrupt. which can be realized upon as of the date when the
allow the insolvent debtor to retain anything subject - surrender value of a policy "arises from the fact petition of insolvency is filed. The explanation is to
to the payment of his debts in a normal state of that the fixed annual premiums is much in excess of be found in the consideration that the destruction of
solvency. the annual risk during the earlier years of the policy, a contract of life insurance is not only highly
- "leviable assets" and "assets in insolvency" are an excess made necessary in order to balance the prejudicial to the insured and those dependent upon
practically coextensive terms. Hence, in determining deficiency of the same premium to meet the annual him, but is inimical to the interests of society.
what elements of value constitute assets in risk during the latter years of the policy. This excess Insurance is a species of property that should be
insolvency, SC is at liberty to consider what in the premium paid over the annual cost of conserved and not dissipated. As is well known, life
elements of value are subject to be taken upon insurance, with accumulations of interest, constitutes insurance is increasingly difficult to obtain with
execution, and vice versa. the surrender value. Though this excess of premiums advancing years, and even when procurable after the
On whether a policy of insurance having no cash paid is legally the sole property of the company, still age of fifty, the cost is then so great as to be
surrender value, but payable to insured or his legal in practical effect, though not in law, it is moneys of practically prohibitive to many. Insolvency is a
representative, is property that may be taken upon the assured deposited with the company in advance disaster likely to overtake men in mature life; and
execution against him. to make up the deficiency in later premiums to cover one who has gone through the process of bankruptcy
- Philippine laws declare no exemption with respect the annual cost of insurance, instead of being usually finds himself in his declining years with the
to insurance policies; and this species of property is retained by the assured and paid by him to the accumulated savings of years swept away and
not enumerated, in section 48 of the Insolvency Law, company in the shape of greatly-increased earning power diminished. The courts are therefore
among items from the ownership of which the premiums, when the risk is greatest. It is the 'net practically unanimous in refusing to permit the
assignee is excluded. Moreover, all life insurance reserve' required by law to be kept by the company assignee in insolvency to wrest from the insolvent a
policies are declared by law to be assignable, for the benefit of the assured, and to be maintained policy of insurance which contains in it no present
regardless of whether the assignee has an insurable to the credit of the policy. So long as the policy realizable assets.
interest in the life of the insured or not (Insurance remains in force the company has not practically any On the applicability of the Insolvency Law
Act No. 2427, sec. 166). beneficial interest in it, except as its custodian, with - Sec 32 of the Insolvency Law among other things,
- SC has held that insurance policies having a the obligation to maintain it unimpaired and suitably declares that the assignment to be made by the
present cash surrender value are subject to be taken invested for the benefit of the insured. This is the clerk of the court "shall operate to vest in the
upon execution. (Misut Garcia vs. West Coast San practical, though not the legal, relation of the assignee all of the estate of the insolvent debtor not
Francisco Life Ins. Co.) company to this fund. "Upon the surrender of the exempt by law from execution." Moreover, by section
- a policy devoid of a cash surrender value cannot be policy before the death of the assured, the company, 24, the court is required, upon making an order
either "leviable assets" or "assets in insolvency." to be relieved from all responsibility for the increased adjudicating any person insolvent, to stay any civil
- the assignee in insolvency acquired no beneficial risk, which is represented by this accumulating proceedings pending against him; and it is declared
interest in the policy of insurance in question; that reserve, could well afford to surrender a considerable in section 60 that no creditor whose debt is provable
its proceeds are not liable for any of the debts part of it to the assured, or his representative. A under the Act shall be allowed, after the
provable against the insolvent in the pending return of a part in some form or other is now Usually commencement of proceedings in insolvency, to
proceedings, and that said proceeds should therefore made." (In re McKinney) prosecute to final judgment any action therefor
be delivered to his administratrix. - the stipulation providing for a cash surrender value against the debtor. In connection with the foregoing
On applicable US case is a comparatively recent innovation in life insurance. may be mentioned subsections 1 and 2 of section 36,
- In re McKinney: no beneficial interest in the policy Formerly the contracts provided — as they still as well as the opening words of section 33, to the
had ever passed to the assignee over and beyond commonly do in the policies issued by fraternal effect that the assignee shall have the right and
what constituted the surrender value, and that the organizations and benefit societies — for the power to recover and to take into his possession, all
legal title to the policy was vested in the assignee payment of a premium sufficient to keep the of the estate, assets, and claims belonging to the
merely in order to make the surrender value- estimated risk covered; and in case of a lapse the insolvent, except such as are exempt by law from
available to him. The assignee should surrender the policy-holder received nothing. Furthermore, the execution.
policy upon the payment to him of said value, as he practice is common among insurance companies - These provisions clearly evince an intention to vest
was in fact directed to do. The assignee in even now to concede nothing in the character of in the assignee, for the benefit of all the creditors of
bankruptcy had no right to keep the estate unsettled cash surrender value, until three full premiums have the insolvent, such elements of property and
for an indefinite period, for the mere purpose of been paid, as in this case. property right as could be reached and subjected by
speculating upon the chances of the bankrupt's - CONLUSION (from this case and other English and process of law by any single creditor suing alone.
death. As regards everything beyond the surrender American cases cited following the same opinion): And this is exactly as it should be: for it cannot be
value, the assignee in bankruptcy would, after the the assignee acquires no beneficial interest in supposed that the Legislature would suppress the
insurance effected on the life of the insolvent, except right of action of every individual creditor upon the
INSURANCE Page
58

adjudication of insolvency, and at the same time - He died on Dec 1963 of cancer of the liver with therefore, that when on May 12, 1962 Kwong Nam
allow the insolvent debtor to retain anything subject metastasis. All premiums had been paid at the time answered `No' to the question whether any life
to the payment of his debts in a normal state of of his death. insurance company ever refused his application for
solvency. - Ng presented a claim for payment of the face value reinstatement of a lapsed policy he did not
- "leviable assets" and "assets in insolvency" are of the policy. Appellant (Asian Crusader) denied the misrepresent any fact.
practically coextensive terms. Hence, in determining claim on the ground that the answers given by the 2) Assuming that the aforesaid answer given by the
what elements of value constitute assets in insured to the questions appearing in his application insured is false, Sec. 278 of the Insurance Law
insolvency, SC is at liberty to consider what for life insurance were untrue. nevertheless requires that fraudulent intent on the
elements of value are subject to be taken upon -Appellant: the insured was guilty of part of the insured be established to entitle the
execution, and vice versa. misrepresentation when insurer to rescind the contract. And as correctly
On whether a policy of insurance having no cash 1) he answered "No" to the question (in the observed by the lower court, "misrepresentation as a
surrender value, but payable to the insured or his application) of "Has any life insurance company ever defense of the insurer to avoid liability is an
legal representative, is property that may be taken refused your application for insurance or for `affirmative’ defense. The duty to establish such a
upon execution against him. reinstatement of a lapsed policy or offered you a defense by satisfactory and convincing evidence
- Philippine laws declare no exemption with respect policy different from that applied for?" when in fact, rests upon the defendant. The evidence before the
to insurance policies; and this species of property is Insular Life denied his application for reinstatement Court does not clearly and satisfactorily establish
not enumerated, in section 48 of the Insolvency Law, of his lapsed life insurance policy that defense."
among items from the ownership of which the 2) he gave the appellant's medical examiner false -Kwong Nam had informed the appellant's medical
assignee is excluded. Moreover, all life insurance and misleading information as to his ailment and examiner that the tumor for which he was operated
policies are declared by law to be assignable, previous operation when he said he was on was ''associated with ulcer of the stomach." In
regardless of whether the assignee has an insurable “operated on for a Tumor [mayoma] of the the absence of evidence that the insured had
interest in the life of the insured or not (Insurance stomach… associated with ulcer of stomach. Tumor sufficient medical knowledge as to enable him to
Act No. 2427, sec. 166). taken out was hard and of a hen's egg size. distinguish between "peptic ulcer" and "a tumor", his
- SC has held that insurance policies having a Operation was two years ago in Chinese General statement that said tumor was "associated with ulcer
present cash surrender value are subject to be taken Hospital by Dr. Yap. Claims he is completely of the stomach" should be construed as an
upon execution. (Misut Garcia vs. West Coast San recovered.” Medical report show that insured was expression made in good faith of his belief as to the
Francisco Life Ins. Co., 41 Phil., 258.) operated on for "peptic ulcer", involving the excision nature of his ailment and operation. Indeed, such
- a policy devoid of a cash surrender value cannot be of a portion of the stomach, not tumor. statement must be presumed to have been made by
either "leviable assets" or "assets in insolvency." him without knowledge of its incorrectness and
- the assignee in insolvency acquired no beneficial ISSUE without any deliberate intent on his part to mislead
interest in the policy of insurance in question; that WON there was concealment (Was appellant, the appellant.
its proceeds are not liable for any of the debts because of insured's aforesaid representation, misled 3) Waiver:
provable against the insolvent in the pending or deceived into entering the contract or in accepting While it may be conceded that, from the viewpoint of
proceedings, and that said proceeds should therefore the risk at the rate of premium agreed upon?) a medical expert, the information communicated was
be delivered to his administratrix. imperfect, the same was nevertheless sufficient to
Disposition Judgment reversed. Sun Life is directed HELD have induced appellant to make further inquiries
to pay the proceeds of the policy to Tan Sit. NO about the ailment and operation of the insured.
-"concealment exists where the assured had Section 32 of Insurance Law [Act No. 2427]
CHAPTER VI – RESCISSION OF INSURANCE knowledge of a fact material to the risk, and provides:
CONTRACTS: CONCEALMENT, honesty, good faith, and fair dealing requires that he “The right to information of material facts may be
MISREPRESENTATION, & BREACH OF should communicate it to the assurer, but he waived either by the terms of insurance or by
WARRANTIES designedly and intentionally withholds the same." neglect to make inquiries as to such facts where they
- It has also been held "that the concealment must, are distinctly implied in other facts of which
NG v. ASIAN CRUSADER LIFE ASSURANCE CORP in the absence of inquiries, be not only material, but information is communicated.”
122 SCRA 461 fraudulent, or the fact must have been intentionally It has been held that where, "upon the face of the
ESCOLIN; May 30, 1983 withheld." application, a question appears to be not answered
Reasoning
FACTS 1) The evidence shows that the Insular Life 8
"Sec. 27. Such party to a contract of insurance must communicate
- On May 12, 1962, Kwong Nam applied for a 20- Assurance Co., Ltd. approved Kwong Nam's request to the other, in good faith, all facts within his knowledge which are
year endowment insurance on his life for the sum of for reinstatement and amendment of his lapsed material to the contract, and which the other has not the means of
P20,000, with his wife, Ng Gan Zee, as beneficiary. insurance policy on April 24, 1962…. It results, ascertaining, and as to which he makes no warranty."
INSURANCE Page
59

at all or to be imperfectly answered, and the insurers > Thelma argues that the non-disclosure of Jaime generate a right to rescind on the part of the injured
issue a policy without any further inquiry, they waive did not amount to fraud. party to "intentional concealments."
the imperfection of the answer and render the > She also argues that the CA erred in not holding - In the case at bar, the nature of the facts not
omission to answer more fully immaterial. that the issue in the case agreed upon between the conveyed to the insurer was such that the failure to
Disposition the judgment appealed from is hereby parties before the Insurance Commission is whether communicate must have been intentional rather than
affirmed, with costs against appellant or not Jaime 'intentionally' made material merely inadvertent.
concealment in stating his state of health; > Jaime could not have been unaware that his
CANILANG v. CA (GREAT PACIFIC LIFE Respondents’ Comments: heart beat would at times rise to high and
ASSURANCE CORP.) > The CA reversed the Insurance Commissioner’s alarming levels and that he had consulted a doctor
223 SCRA 443 decision, holding that the use of the word twice two months before applying for non-medical
FELICIANO; June 17, 1993 'intentionally" by the Insurance Commissioner in insurance.
defining and resolving the issue agreed upon by the > The last medical consultation took place just the
NATURE parties at pre-trial before the Insurance day before the insurance application was filed.
Petition for review on certiorari of the decision of the Commissioner was not supported by the evidence 2. YES
Court of Appeals and that the issue agreed upon by the parties had Ratio Materiality relates rather to the "probable and
been whether Jaime made a material concealment as reasonable influence of the facts" upon the party to
FACTS to the state of his health at the time of the filing of whom the communication should have been made, in
- June 18, 1982 – Jaime Canilang was diagnosed by insurance application, justifying the denial of the assessing the risk involved in making or omitting to
Dr. Claudio to have sinus tachycardia. He was claim. make further inquiries and in accepting the
directed by the doctor to take a tranquilizer > It also found that the failure of Jaime to disclose application for insurance; that "probable and
(Trazepam) and a beta-blocker drug (Aptin). previous medical consultation and treatment reasonable influence of the farts" concealed must, of
- August 3, 1982 – Jaime consulted Dr. Claudio again constituted material information which should have course, be determined objectively, by the judge
and was diagnosed to have acute bronchitis. been communicated to Great Pacific to enable the ultimately.
- August 4, 1982 – Jaime applied for a nonmedical latter to make proper inquiries. Reasoning
insurance policy with Great Pacific Life Assurance - The information which Jaime failed to disclose was
Company. He named his wife Thelma as his ISSUES material to the ability of Great Pacific to estimate the
beneficiary. He was issue the policy with a face 1. WON Jaime intentionally withheld information probable risk he presented as a subject of life
value of P19,700 effective August 9, 1982. from Great Pacific insurance.
- August 5, 1983 – Jaime died of congestive heart 2. WON the information withheld would have been - Had Canilang disclosed his visits to his doctor, the
failure, anemia and chronic anemia. Thelma filed material to Great Pacific’s decision to grant Jaime the diagnosis made and the medicines prescribed by
her claim but the insurance company refused to insurance policy such doctor, in the insurance application, it may be
grant it on the ground that Jaime had concealed reasonably assumed that Great Pacific would have
information. HELD made further inquiries and would have probably
- Thelma filed a complaint against Great Pacific to 1. YES refused to issue a non-medical insurance policy or,
recover the insurance proceeds. She testified that Ratio Section 27 of the Insurance Code of 1978 is at the very least, required a higher premium for the
she was not aware of her husband’s ailments and properly read as referring to "any concealment same coverage.
that she thought he had died from a kidney disorder. without regard to whether such concealment is - As held in the case of Saturnino vs. Philippine-
- Great Pacific presented as witness Dr. Quismorio intentional or unintentional. The restoration in 1985 American Life Insurance, “the waiver of medical
who testified that Jaime’s insurance application was by B.P. Blg. 874 of the phrase "whether intentional examination in a non-medical insurance contract
the basis of his medical declaration and she or unintentional" merely underscored the fact that all renders even more material the information inquired
explained that an applicant was required to undergo throughout (from 1914 to 1985), the statute did not of the applicant concerning previous condition of
medical examination only if the applicant had require proof that concealment must be "intentional" health and diseases suffered, for such information
disclosed that he had previously been consulted with in order to authorize rescission by the injured party. necessarily constitutes an important factor which the
a doctor and had been hospitalized. Reasoning insurer takes into consideration in deciding whether
- The Insurance Commissioner ordered Great Pacific - Art. 27 of the 1978 Insurance Code reads that “a to issue the policy or not.”
to pay Thelma the insurance proceeds, including concealment entitles the injured party to rescind a Disposition the Petition for Review is DENIED for
attorney’s fees, holding that Jaime’s illness was not contract of insurance,” which does not include the lack of merit and the Decision of the Court of
that serious as to Great Pacific’s decision to insure words “whether intentional or unintentional” from Appeals dated 16 October 1989 in C.A.-G.R. SP No.
him and that there was no concealment on the part the previous statutes. The Insurance Commissioner 08696 is hereby AFFIRMED.
of Jaime with regard to his illness. relied on this deletion in arguing that the statute
Petitioners’ Claim: intended to limit the kinds of concealment which
INSURANCE Page
60

YU PANG CHENG v. CA ISSUE party has not the means of ascertaining (Section
105 PHIL 930 WON the insured is guilty of concealment of some 27), and the materiality is to be determined not by
BAUTISTA ANGELO; May 29, 1959 facts material to the risk insured against which has the event but solely by the probable and reasonable
the effect of avoiding the policy as found by influence of the facts upon the party to whom the
FACTS respondent court. communication is due (Section 30).
- September 5, 1950: Yu Pang Eng submitted parts - Argente vs. West Coast Life Insurance Co.: "One
II and III of his application for insurance consisting HELD ground for the rescission of a contract of insurance
of the medical declaration made by him to the - It should be noted that the insured's confinement under the Insurance Act is 'a concealment', which in
medical examiner of defendant and the medical in the Chinese General Hospital took place from section 25 is defined 'A neglect to communicate that
examiner's report January 29, 1950 to February 11, 1950, whereas his which a party knows and ought to communicate.'
- September 7: he submitted part I of his application application for insurance wherein he stated his Appellant argues that the concealment was
which is the declaration made by him to an agent of answers to the questions propounded to him by the immaterial and insufficient to avoid the policy. We
defendant examining physician of defendant was submitted to cannot agree. In an action on a life insurance policy
- September 8: defendant issued to the insured defendant on September 5, 1950. where the evidence conclusively shows that the
Policy No. 812858 - It is apparent that when the insured gave his answers to questions concerning diseases were
- December 27, 1950: the insured entered St. answers regarding his previous ailment, particularly untrue, the truth or falsity of the answers become
Luke's Hospital for medical treatment but he died on with regard to "Gastritis, Ulcer of the Stomach or any the determining factor. If the policy was procured by
February 27, 1951. disease of that organ" and "Vertigo, Dizziness, fraudulent representations, the contract of insurance
- According to the death certificate, he died of Fainting-spells or Unconsciousness", he concealed apparently set forth therein was never legally
"infiltrating medullary carcinoma, Grade 4, advanced the ailment of which he was treated in the Chinese existent. It can fairly be assumed that had the true
cardiac and of lesser curvature, stomach metastases General Hospital which precisely has direct facts been disclosed by the assured, the insurance
spleen." connection with the subject of the questions would never have been granted."
- Plaintiff, brother and beneficiary of the insured, propounded. Disposition Decision affirmed.
demanded from defendant the payment of the - The negative answers given by the insured
proceeds of the insurance policy and when the regarding his previous ailment, or his concealment of
demand was refused, he brought the present action. the fact that he was hospitalized and treated for
- The insured, in his application for insurance, sometime of peptic ulcer and had suffered from
particularly in his declarations to the examining "dizziness, anemia, abdominal pains and tarry
physician, stated the following in answering the stools", deprived defendant of the opportunity to
questions propounded to him: make the necessary inquiry as to the nature of his GREAT PACIFIC LIFE v. CA (supra p.34)
14. Have you ever had any of the following diseases past illness so that it may form its estimate relative
or symtoms? Each question must be read and to the approval of his application. PACIFIC BANKING CORP v. CA (ORIENTAL
answered "Yes" or "No.". - Had defendant been given such opportunity, ASSURANCE CORPORATION)
"Gastritis, Ulcer of the Stomach or any disease of considering the previous illness of the insured as 168 SCRA 1
that organ? No. disclosed by the records of the Chinese General PARAS; November 28, 1988
"Vertigo, Dizziness, Fainting-spells or Hospital, defendant would probably had never
Unconsciouness? No. consented to the issuance of the policy in question. NATURE
"Cancer, Tumors or Ulcers of any kind? No. In fact, according to the death certificate, the Petition for review on certiorari of the CA decision,
- 15. Have you ever consulted any physician riot insured died of "infiltrating medullary carcinoma, which set aside the decision of CFI Manila, which had
included in any of the above answers? Give names Grade, 4, advanced cardiac and of lesser curvature, in turn granted the complaint for a sum of money in
and address or physicians list ailments or accidents stomach metastases spleen", which may have a civil case filed by Pacific Banking against Oriental
and date. No." direct connection with his previous illness. Assurance.
- It appears that the insured entered the Chinese - Our Insurance Law provides that "A neglect to
General Hospital for medical treatment on January communicate that which a party knows and ought to FACTS
29, 1950 having stayed there up to February 11, communicate, is called concealment" (Section 25, - October 21,1963: an open Fire Policy was issued to
1950. Act No. 2427). Whether intentional or unintentional, the Paramount Shirt Manufacturing Co. (insured), by
- An X-ray picture of his stomach was taken and the the concealment entitles the insurer to rescind the which Oriental Assurance Corporation bound itself to
diagnosis made of him by his doctors showed that contract of insurance (Section 26). indemnify the insured for any loss or damage, not
his illness was "peptic ulcer, bleeding." - Our law even requires the insured to communicate exceeding P61,000.00, caused by fire to its property
to the insurer all facts within his knowledge which consisting of stocks, materials and supplies usual to
are material to the contract and which the other a shirt factory, including furniture, fixtures,
INSURANCE Page
61

machinery and equipment while contained in the > presented evidence that insured has undeclared guilty of a false declaration; a clear
ground, second and third floors of the building co-insurances with the following: P30,000.00 with misrepresentation and a vital one because where the
situated at number 256 Jaboneros St., San Nicolas, Wellington Insurance; P25,000. 00 with Empire insured had been asked to reveal but did not, that
Manila, for a period of one year commencing from Surety and P250,000.00 with Asian Surety; was deception. Otherwise stated, had the insurer
that date to October 21, 1964. undertaken by insured Paramount on the same known that there were many co-insurances, it could
- Insured was at the time of the issuance of the property covered by its policy with Oriental have hesitated or plainly desisted from entering into
policy and is up to this time, a debtor of Pacific Assurance whereas the only co-insurances declared such contract. Hence, the insured was guilty of clear
Banking in the amount of not less P800,000.00 and in the subject policy are those of P30,000.00 with fraud.
the goods described in the policy were held in trust Malayan, P50,000.00 with South Sea, and - Pacific Banking's contention that the allegation of
by the insured for the Pacific Banking under thrust P25.000.00 with Victory fraud is but a mere inference or suspicion is
receipts. - NOTE: the defense of fraud and/or violation of non- untenable. Concrete evidence of fraud or false
- Said policy was duly endorsed to Pacific Banking as declaration of co-insurances was not pleaded in the declaration by the insured was furnished by the
mortgagee/trustor of the properties insured, with the answer, also not pleaded in the Motion to Dismiss. Pacific Banking itself when the facts alleged in the
knowledge and consent of Oriental Assurance to the - CFI denied Oriental Assurance's motion on the policy under clauses "Co-Insurances Declared" and
effect that "loss if any under this policy is payable to ground that since the defense was raised for the first "Other Insurance Clause" are materially different
the Pacific Banking Corporation". time, it must be deemed to have waived the from the actual number of co-insurances taken over
- While the aforesaid policy was in full force and requirement of proof of loss. Case was submitted for the subject property. Consequently, the whole
effect, a fire broke out on the subject premises decision. But upon MR, Oriental Asurance was foundation of the contract fails, the risk does not
destroying the goods contained in its ground and allowed to present additional evidence, "in order to attach and the policy never becomes a contract
second floors. Counsel for the Pacific Banking sent a prove that 'insured has committed a violation of between the parties. Representations of facts are the
letter of demand to Oriental Assurance for indemnity condition No. 3 of the policy in relation to the other foundation of the contract and if the foundation does
due to the loss of property by fire. Oriental Insurance Clause.' " CFI eventually adjudged not exist, the superstructure does not arise.
Assurance informed counsel that it was not yet ready Oriental Assurance liable to the Pacific Banking under Falsehood in such representations is not shown to
to accede to the latter's demand as the former is the said contract of insurance. vary or add to the contract, or to terminate a
awaiting the final report of the insurance adjuster, - Court of Appeals reversed. Pacific Banking's MR contract which has once been made, but to show
H.H. Bayne Adjustment Company. denied. that no contract has ever existed (Tolentino). A void
- Said insurance adjuster notified counsel for the or inexistent contract is one which has no force and
Pacific Banking that the insured under the policy had ISSUES effect from the very beginning, as if it had never
not filed any claim with it, nor submitted proof of 1. WON insured is guilty of fraud been entered into, and which cannot be validated
loss which is a clear violation of Policy Condition 2. WON mortgagee/assignee can still claim from the either by time or by ratification.
No.11, and for which reason, determination of the insurance - As the insurance policy against fire expressly
liability of Oriental Assurance could not be had. required that notice should be given by the insured
Pacific Banking's counsel replied asking the insurance HELD of other insurance upon the same property, the total
adjuster to verify from the records of the Bureau of 1. YES absence of such notice nullifies the policy.
Customs the entries of merchandise taken into the - The crux of the controversy centers on two points: - Argument that notice of co-insurances may be
customs bonded warehouse razed by fire as a (a) unrevealed co-insurances which violated policy made orally is preposterous and negates policy
reliable proof of loss. conditions No. 3; and (b) failure of the insured to file condition No. 20 which requires every notice and
- For failure of the insurance company to pay the the required proof of loss prior to court action. other communications to the insurer to be written or
loss as demanded, Pacific Banking field before CFI an - Policy Condition No. 3 explicitly provides: “The printed.
action for a sum of money against the Oriental Insured shall give notice to the Company of any 2. NO
Assurance, in the principal sum of P61,000.00 issued insurance already effected, or which may - Subject mortgage clause pecifically provides: “Loss,
in favor of Paramount Shirt Manufacturing Co. subsequently be effected, covering any of the if any, under this policy, shall be payable to the
Oriental Assurance defenses property hereby insured, and unless such notice be PACIFIC BANKING CORPORATION Manila
(a) lack of formal claim by insured over the loss and given and the particulars of such insurance or mortgagee/trustor as its interest may appear, it
(b) premature filing of the suit as neither plaintiff nor insurances be stated in or endorsed on this Policy by being hereby understood and agreed that this
insured had submitted any proof of loss on the basis or on behalf of the Company before the occurrence insurance as to the interest of the mortgagee/trustor
of which defendant would determine its liability and of any loss or damage, all benefit under this policy only herein, shall not be invalidated by any act or
the amount thereof, either to the Oriental Assurance shall be forfeited.” neglect except fraud or misrepresentation, or arson
or its adjuster H.H. Bayne Adjustment Co. - It is not disputed that the insured failed to reveal of the mortgagor or owner/trustee of the property
Pacific Banking before the loss three other insurances. By reason of insured; provided, that in case the mortgagor or
said unrevealed insurances, the insured had been owner/ trustee neglects or refuses to pay any
INSURANCE Page
62

premium, the mortgagee/ trustor shall, on demand pertinent documents were necessary to prove and policy is a condition precedent to the light of
pay the same.” estimate its loss. Instead, Pacific Banking shifted recovery.
- The paragraph clearly states the exceptions to the upon Oriental Assurance the burden of fishing out - It appearing that insured has violated or failed to
general rule that insurance as to the interest of the the necessary information to ascertain the particular perform the conditions under No. 3 and 11 of the
mortgagee, cannot be invalidated; namely: fraud, or account of the articles destroyed by fire as well as contract, and such violation or want of performance
misrepresentation or arson. the amount of loss. has not been waived by the insurer, the insured
- Concealment of the aforecited co-insurances can - Oriental Assurance and its adjuster notified Pacific cannot recover, much less the herein Pacific Banking.
easily be fraud, or in the very least, Banking that insured had not yet filed a written claim Courts are not permitted to make contracts for the
misrepresentation. It is but fair and just that where nor submitted the supporting documents in parties; the function and duty of the courts is simply
the insured who is primarily entitled to receive the compliance with the requirements set forth in the to enforce and carry out the contracts actually made.
proceeds of the policy has by its fraud and/or policy. Despite the notice, the latter remained Disposition Petition dismissed. CA affirmed.
misrepresentation, forfeited said right, with more unheedful. Since the required claim by insured,
reason Pacific Banking which is merely claiming as together with the preliminary submittal of relevant SUNLIFE ASSURANCE COMPANY v. CA (SPS.
indorsee of said insured, cannot be entitled to such documents had not been complied with, it follows BACANI)
proceeds. that Oriental Assurance could not be deemed to have 245 SCRA 268
- The fact of fraud was tried by express or at least finally rejected Pacific Banking's claim and therefore QUIASON; June 22, 1995
implied consent of the parties. Pacific Banking did the latter's cause of action had not yet arisen.
not only object to the introduction of evidence but on Compliance with condition No. 11 is a requirement NATURE
the contrary, presented the very evidence that sine qua non to the right to maintain an action as A petition for review on certiorari.
proved its existence. prior thereto no violation of Pacific Banking's right
- Be that as it may, SC has ample authority to give can be attributable to Oriental Assurance. As before FACTS
beyond the pleadings where in the interest of justice such final rejection, there was no real necessity for - April 15, 1986: Robert John B. Bacani procured a
and the promotion of public policy, there is a need to bringing suit. Pacific Banking should have life insurance contract for himself from SUNLIFE
make its own finding to support its conclusion. endeavored to file the formal claim and procure all (petitioner) valued at P100K. The designated
Otherwise stated, the Court can consider a fact the documents, papers, inventory needed by Oriental beneficiary was his mother, Bernarda Bacani
which surfaced only after trial proper. Assurance or its adjuster to ascertain the amount of (respondent).
- Generally, the cause of action on the policy accrues loss and after compliance await the final rejection of - June 26, 1987: the insured died in a plane crash.
when the loss occurs, but when the policy provides its claim. Indeed, the law does not encourage Bernarda Bacani filed a claim with Sunlife, seeking
that no action shall be brought unless the claim is unnecessary litigation. the benefits of the insurance policy taken by her son.
first presented extrajudicially in the manner provided - Pacific Banking prematurely filed the civil case and Petitioner conducted an investigation and its findings
in the policy, the cause of action will accrue from the dismissal thereof was warranted under the prompted it to reject the claim on the ground that
time the insurer finally rejects the claim for circumstances. While it is a cardinal principle of the insured did not disclose facts material to the
payment. insurance law that a policy or contract of insurance is issuance of the policy. The insured gave false
- In the case at bar, policy condition No. 11 to be construed liberally in favor of the insured and statements in the application when he answered in
specifically provides that the insured shall on the strictly as against the insurer company yet, contracts the negative to the question “have you ever had or
happening of any loss or damage give notice to the of insurance, like other contracts, are to be sought advice for urine, kidney, bladder
company and shall within fifteen (15) days after such construed according to the sense and meaning of the disorder?”
loss or damage deliver to the Oriental Assurance (a) terms which the parties themselves have used. If - Sunlife discovered that two weeks prior to the
a claim in writing giving particular account as to the such terms are clear and unambiguous, they must issuance, insured was diagnosed with renal failure,
articles or goods destroyed and the amount of the be taken and understood in their plain, ordinary and was confined, and underwent tests.
loss or damage and (b) particulars of all other popular sense. - November 17, 1988: Bacani and her husband filed
insurances, if any. Likewise, insured was required "at - Contracts of insurance are contracts of indemnity for specific performance against Sunlife. RTC granted
his own expense to produce, procure and give to the upon the terms and conditions specified in the policy. the plea on the ground that that the facts concealed
company all such further particulars, plans, The parties have a right to impose such reasonable by the insured were made in good faith and under
specifications, books, vouchers, invoices, duplicates conditions at the time of the making of the contract the belief that they need not be disclosed, and that
or copies thereof, documents, proofs and information as they may deem wise and necessary. The the disclosure was not material since the policy was
with respect to the claim". agreement has the force of law between the parties. non-medical.
- Evidence adduced shows that 24 days after the The terms of the policy constitute the measure of the - Sunlife appealed to the CA, but the latter denied
fire, Pacific Banking merely wrote letters to Oriental insurer's liability, and in order to recover, the insured the appeal on the ground that the cause of death
Assurance to serve as a notice of loss, thereafter, must show himself within those terms. The was unrelated to the facts concealed by the insured.
the former did not furnish the latter whatever compliance of the insured with the terms of the
INSURANCE Page
63

Petitioner’s Claim sufficient that his non-disclosure misled the insurer cases the contract is void, even though, despite this
> The insured did not disclose facts relevant to the in forming his estimates of the risks of the proposed nullity, no crime was committed. There may not
issuance of the policy, thus rescission of the contract insurance policy or in making inquiries have been estafa in the case at bar, but it was
may be invoked by the insurance company. Disposition Petition is granted and the decision of conclusively demonstrated by the trial that deceit
Respondents’ Comments CA is reversed and set aside. entered into the insurance contract, fulfillment
> The actual cause of death was not relevant to the whereof is claimed, and therefore the conclusions
concealed information, and the policy was entered EGUARAS v. GREAT EASTERN reached by the court in the judgment it rendered in
into by the insured in good faith. 33 PHIL. 263 the criminal proceedings for estafa do not affect this
TORRES.; January 24, 1916 suit, nor can they produce in the present suit the
ISSUE force of res adjudicata.
WON the concealment renders the insurance policy NATURE Reasoning
rescissible Appeal filed through bill of exceptions from the - It is proven that the signatures on the insurance
judgment of the CFI applications reading "Dominado Albay" are false and
HELD forged; that the person who presented himself to Dr.
YES FACTS Vidal to be examined was not the real Dominador
Ratio The terms of the contract are clear. The - Francisca Eguaras filed a written complaint in court, Albay, but Castor Garcia who was positively
insured is specifically required to disclose to the alleging as a cause of action that her son-in-law identified by Dr. Vidal; that at the time of the
insurer matters relating to his health. Dominador Albay had applied in writing to the application for insurance and the issuance of the
Reasoning defendant insurance company to insure his life for policy which is the subject matter of this suit the real
SEC. 26 (IC) the sum of P5,000, naming as the beneficiary in case Dominador Albay was informed of all those
A neglect to communicate that which a party of his death the plaintiff Francisca Eguaras; that machinations, wherefore it is plain that the insurance
knows and ought to communicate, is called a after compliance with the requisites and the contract between the defendant and Dominador
concealment. investigation carried on by the defendant company, Albay is null and void because it is false, fraudulent
SEC. 31 (IC) it accepted the application for insurance and issued and illegal.
Materiality is to be determined not by the event, the policy; that, said policy being in force, the Disposition The judgment appealed from is
but solely by the probable and reasonable influence insured died, and despite the fact that the reversed and the defendant absolved from the
of the facts upon the party to whom communication beneficiary submitted satisfactory proofs of his death complaint without special finding as to the costs.
is due, in forming his estimate of the disadvantages and that the defendant company investigated the
of the proposed contract or in making his inquiries event, still it refused and continues to refuse to pay QUA CHEE GAN v. LAW UNION AND ROCK
- The information which the insured failed to disclose to the plaintiff the value of the policy. 98 PHIL 85
was material and relevant to the approval and the - Defendant set forth in special defense that the REYES; December 17, 1955
issuance of the insurance policy. The matters insurance policy issued in the name of Dominador
concealed would have definitely affected petitioner's Albay had been obtained through fraud and deceit FACTS
action on his application, either by approving it with known and consented to by the interested parties - Qua Chee Gan insured 4 of his bodegas with Law
the corresponding adjustment for a higher premium and is therefore completely illegal, void, and Union & Rock Insurance Co in 1937. These bodegas
or rejecting the same. ineffective. were used for the storage of stocks of copra and of
- Good faith is no defense in concealment. It appears - A criminal case for frustrated estafa was filed by hemp, baled and loose.
that such concealment was deliberate on the part of defendant against Ponciano Remigio, Castor Garcia - Fire of undetermined origin that broke out in the
the insured. and Francisca Eguaras. They were acquitted, and early morning of July 21, 1940, and lasted almost
- The waiver of a medical examination [in a non- claim that the judgment produces the effect of res one week, gutted and completely destroyed Bodegas
medical insurance contract] renders even more judicata in the present suit. Nos. 1, 2 and 4, with the merchandise stored
material the information required of the applicant therein.
concerning previous condition of health and diseases ISSUE - Qua Chee Gan informed the insurance company of
suffered, for such information necessarily constitutes WON the life insurance obtained by Dominador Albay the fire. Fire adjusters of the company conducted an
an important factor which the insurer takes into was issued through fraud and deceit extensive investigation. Qua Chee Gan submitted the
consideration in deciding whether to issue the policy corresponding fire claims, totaling P398,562.81 (but
or not. HELD reduced to the full amount of the insurance,
- Anent the finding that the facts concealed had no YES P370,000), the Insurance Company resisted
bearing to the cause of death of the insured, it is Ratio In a contract where one of the contracting payment, claiming violation of warranties and
well settled that the insured need not die of the parties may have given his consent through error, conditions, filing of fraudulent claims, and that the
disease he had failed to disclose to the insurer. It is violence, intimidation, or deceit, and in any of such
INSURANCE Page
64

fire had been deliberately caused by the insured or of the contract, the ambiguity must be held strictly - As to the defense that the burned bodegas could
by other persons in connivance with him. against the insurer and liberally in favor of the not possibly have contained the quantities of copra
- Qua Chee Gan, his brother and his employees were insured, especially to avoid a forfeiture and hemp stated in the fire claims, the insurer's case
tried for arson, where counsel of the insurance - Another point that is in favor of the insured is that rests almost exclusively on the estimates, inferences
company acted as a private prosecutor. They were the gasoline kept in Bodega No. 2 was only incidental and conclusions of its adjuster investigator,
acquitted. to his business, being no more than a customary 2 Alexander D. Stewart, who examined the premises
- This civil suit was then instituted to claim against day's supply for the five or six motor vehicles used during and after the fire. His testimony, however,
the insurance company. The CFI ruled in favor of for transporting of the stored merchandise). "It is was based on inferences from the photographs and
Qua Chee Gan and ordered Law Union Rock Co. to well settled that the keeping of inflammable oils on traces found after the fire, and must yield to the
pay. the premises though prohibited by the policy does contradictory testimony of engineer Andres Bolinas,
not void it if such keeping is incidental to the and specially of the then Chief of the Loan
ISSUES business." (Bachrach vs. British American Ass. Co., Department of the National Bank's Legaspi branch,
1. WON there was a breach of the fire hydrant 17 Phil. 555, 560) Porfirio Barrios, and of Bank Appraiser Loreto
warranty 3. NO Samson, who actually saw the contents of the
2. WON the insured violated the Hemp warranty - The charge that the insured failed or refused to bodegas shortly before the fire, while inspecting
3. WON Qua Chee Gan is guilty of overvaluation submit to the examiners of the insurer the books, them for the mortgagee Bank
4. WON Qua Chee Gan caused the fire vouchers, etc. demanded by them was found 6. NO
5. WON there was an error in the amount of copra unsubstantiated by the trial Court, and no reason - Appellant insurance company also contends that
and hemp lost has been shown to alter this finding. the claims filed by the insured contained false and
6. WON the claims contained false and fraudulent - In view of the discrepancy in the valuations fraudulent statements that avoided the insurance
statements between the insured and the adjuster Stewart for the policy. But the trial Court found that the
insurer, the Court referred the controversy to a discrepancies were a result of the insured's
HELD government auditor, Apolonio Ramos; but the latter erroneous interpretation of the provisions of the
1. NO reached a different result from the other two. Not insurance policies and claim forms, caused by his
- It is argued that he should have 11 fire hydrants in only that, but Ramos reported two different imperfect knowledge of English, and that the
the compound, but he only had 2. We are in valuations that could be reached according to the misstatements were innocently made and without
agreement with the trial Court that the appellant is methods employed. Clearly then, the charge of intent to defraud. The trial court’s ruling must be
barred by waiver (or rather estoppel) to claim fraudulent overvaluation cannot be seriously upheld.
violation of the so-called fire hydrants warranty, for entertained. - For example, the occurrence of previous fires in the
the reason that knowing fully all that the number of 4. NO premises insured in 1939, altho omitted in the
hydrants demanded therein never existed from the - This defense is predicted on the assumption that claims, Exhibits EE and FF, were nevertheless
very beginning, the appellant nevertheless issued the insured was in financial difficulties and set the revealed by the insured in his claims Exhibits Q (filed
the policies in question subject to such warranty, and fire to defraud the insurance company, presumably simultaneously with them), KK and WW. Considering
received the corresponding premiums. in order to pay off the Philippine National Bank, to that all these claims were submitted to the smae
2. NO which most of the insured hemp and copra was agent, and that this same agent had paid the loss
- The insurance company avers that the insured pledged. This defense is fatally undermined by the caused by the 1939 fire, we find no error in the trial
violated the hemp warranty when it admitted that it established fact that, notwithstanding the insurer's Court's acceptance of the insured's explanation that
had 36 cans of gasoline in the building. It is well to refusal to pay the value of the policies the extensive the omission in Exhibits EE and FF was due to
note that gasoline is not specifically mentioned resources of the insured enabled him to pay off the inadvertance, for the insured could hardly expect
among the prohibited articles listed in the so-called National Bank in a short time; and if he was able to under such circumstances, that the 1939 would pass
"hemp warranty." The cause relied upon by the do so, no motive appears for attempt to defraud the unnoticed by the insurance agents. Similarly, the 20
insurer speaks of "oils (animal and/or vegetable insurer. While the acquittal of the insured in the per cent overclaim on 70 per cent of the hemo stock,
and/or mineral and/or their liquid products having a arson case is not res judicata on the present civil was explained by the insured as caused by his belief
flash point below 300o Fahrenheit", and is decidedly action, the insurer's evidence, to judge from the that he was entitled to include in the claim his
ambiguous and uncertain; for in ordinary parlance, decision in the criminal case, is practically identical in expected profit on the 70 per cent of the hemp,
"Oils" mean "lubricants" and not gasoline or both cases and must lead to the same result, since because the same was already contracted for and
kerosene. And how many insured, it may well be the proof to establish the defense of connivance at sold to other parties before the fire occurred.
wondered, are in a position to understand or the fire in order to defraud the insurer "cannot be Compared with other cases of over-valuation
determine "flash point below 003o Fahrenheit. Here, materially less convincing than that required in order recorded in our judicial annals, the 20 per cent
again, by reason of the exclusive control of the to convict the insured of the crime of arson. excess in the case of the insured is not by itself
insurance company over the terms and phraseology 5. NO sufficient to establish fraudulent intent. Certainly,
INSURANCE Page
65

the insured's overclaim of 20 per cent in the case at company's physician, each of them was required to WON the contract of insurance may be rescinded
bar, duly explained by him to the Court a quo, file a certificate of health before the policy was
appears puny by comparison (compared to other delivered to them.
cases cited by the court), and can not be regarded - On November 18, 1925, Vicenta de Ocampo died of
as "more than misstatement, more than cerebral apoplexy. Thereafter Bernardo Argente HELD
inadvertence of mistake, more than a mere error in presented a claim. Following investigation conducted YES
opinion, more than a slight exaggeration" that would by the Manager of the Manila office of the insurance - Bernardo Argente and his wife applications’ were
entitle the insurer to avoid the policy. It is well to company, it was apparently disclosed that the false with respect to their state of health during the
note that the overcharge of 20 per cent was claimed answers given by the insured in their medical period of five years preceding the date of such
only on a part (70 per cent) of the hemp stock; had examinations with regard to their health and applications and that they knew the representations
the insured acted with fraudulent intent, nothing previous illnesses and medical attendance were made by them in their applications were false. The
prevented him from increasing the value of all of his untrue. West Coast Life Insurance Co. refused to pay question arises as to the state of the law in relation
copra, hemp and buildings in the same proportion. the claim of Bernardo Argente, and wrote him to the thereto.
This also applies to the alleged fraudulent claim for effect that the claim was rejected because the - One ground for the rescission of a contract of
burned empty sacks, that was likewise explained to insurance was obtained through fraud and insurance under the Insurance Act is "a
our satisfaction and that of the trial Court. The rule misrepresentation. concealment," which in section 25 is defined as "A
is that to avoid a policy, the false swearing must be - It is admitted that it appears in the Medical neglect to communicate that which a party knows
willful and with intent to defraud which was not the Examiner's Report that Bernardo Argente gave false and ought to communicate." In an action on a life
cause. Of course, the lack of fraudulent intent would responses. As well as with the Medical Examiner's insurance policy where the evidence conclusively
not authorize the collection of the expected profit Report that Vicenta de Ocampo. It is, however, not shows that the answers to questions concerning
under the terms of the polices, and the trial Court disputed that Vicenta de Ocampo was taken by a diseases were untrue, the truth or falsity of the
correctly deducted the same from its award. patrolman, at the request of her husband, Bernardo answers become the determining factor. If the policy
Disposition Decision affirmed Argente, on May 19, 1924, to the Meisic police was procured by fraudulent representations, the
station, and from there was transferred to the San contract of insurance apparently set forth therein
ARGENTE v. WEST COAST LIFE Lazaro Hospital. In San Lazaro Hospital, her case was never legally existent. It can fairly be assumed
51 PHIL 725 was diagnosed by the admitting physician as that had the true facts been disclosed by the
MALCOLM; March 19, 1928 "alcoholism," but later Doctor Domingo made a assured, the insurance would never have been
diagnosis of probable "manic-depressive psychosis," granted.
FACTS and still, later in Mary Chiles Hospital, made a final - In Joyce, The Law of Insurance, second edition,
- This is an action upon a joint life insurance policy diagnosis of "phycho-neurosis." volume 3, Chapter LV, is found the following:
for P15,000 issued by the West Coast Life Insurance - Bernardo Argente, while readily conceding most of "The basis of the rule vitiating the contract in
Co., on May 15, 1925, in favor of Bernardo Argente, the facts herein narrated, yet alleges that both he cases of concealment is that it misleads or
and his wife, Vicenta de Ocampo, the latter having and his wife revealed to the company's physician, deceives the insurer into accepting the risk, or
died on November 18, 1925. Fraud in obtaining the Doctor Sta. Ana, all the facts concerning their accepting it at the rate of premium agreed upon;
policy was pleaded by way of special defense. On the previous illnesses and medical attendance, but that The insurer, relying upon the belief that the
issue thus suggested, the court adopted the theory Doctor Sta. Ana, presumably acting in collusion with assured will disclose every material fact within his
of the defendant, and held the insurance policy null the insurance agent, Jose Geronimo del Rosario, actual or presumed knowledge, is misled into a
and void, with the result that the complaint was failed to record them in the medical reports. The belief that the circumstance withheld does not
dismissed, with costs. evidence on these points consists of the testimony of exist, and he is thereby induced to estimate the
-Bernardo Argente signed an application for joint the plaintiff and his subordinate clerk, Apolonio risk upon a false basis that it does not exist. The
insurance with his wife in the sum of P2,000. The Espiritu, on the one hand, and of the testimony of principal question, therefore, must be, Was the
wife, Vicenta de Ocampo, signed a like application Doctor Sta. Ana and Jose Geronimo del Rosario on assurer misled or deceived into entering a contract
for the same policy. the other. This was rejected by the Trial Court. Trial obligation or in fixing the premium of insurance by
- Bernardo Argente and his wife was examined by judge found with the insurance company with regard a withholding of material information or facts
Dr. Cesareo Sta. Ana, a medical examiner for the to the question of fact. SC agrees. There appears no within the assured's knowledge or presumed
West Coast Life Insurance Co. which did not show motive whatever on the part of Doctor Sta. Ana to knowledge?
previous and existing health problems. falsify the Medical Examiner's Reports and thereby "It therefore follows that the assurer in assuming a
- A temporary policy for P15,000 was issued to not only jeopardize his career as a physician, but risk is entitled to know every material fact of which
Bernardo Argente and his wife as of May 15, 1925. also gravely implicate himself criminally. the assured has exclusive or peculiar knowledge,
In view of the fact that more than thirty days had as well as all material facts which directly tend to
elapsed since the applicants were examined by the ISSUE increase the hazard or risk which are known by the
INSURANCE Page
66

assured, or which ought to be or are presumed to case, if construed as it is frequently cited, would representations are false, the insurer's tender of the
be known by him. And a concealment of such facts be opposed to the above conclusion; but, on the premium and notice that the policy is canceled,
vitiates the policy. 'It does not seem to be contrary, it sustains it, for the reason that before the commencement of suit thereon, operate
necessary . . . that the . . . suppression of the symptoms of consumption had so far developed to rescind the contract of insurance. (Rankin vs.
truth should have been willful.' If it were but an themselves within a few months prior to effecting Amazon Insurance Co. [1891], 89 Cal., 203.)
inadvertent omission, yet if it were material to the the insurance as to induce a reasonable belief that Disposition Judgment affirmed, with the costs of
risk and such as the plaintiff should have known to the applicant had that fatal disease, and we should this instance against the appellant.
be so, it would render the policy void. But it is held further construe this case as establishing the rule
that if untrue or false answers are given in that such a matter cannot rest alone upon the GREAT PACIFIC LIFE v. CA (NGO HING)
response to inquiries and they relate to material assured's belief irrespective of what is a 89 SCRA 543
facts the policy is avoided without regard to the reasonable belief, but that it ought to be judged by DE CASTRO, J; April 30, 1979
knowledge or fraud of assured, although under the the criterion whether the belief is one fairly
statute statements are representations which must warranted by the circumstances. A case in NATURE
be fraudulent to avoid the policy. So under certain Indiana, however, holds that if the assured has Petition for certiorari
codes the important inquiries are whether the some affection or ailment of one or more of the
concealment was willful and related to a matter organs inquired about so well defined and marked FACTS
material to the risk. as to materially derange for a time the functions of - On March 14, 1957, private respondent Ngo Hing
xxx xxx xxx such organ, as in the case of Bright's disease, the filed an application with the Great Pacific Life
"If the assured has exclusive knowledge of policy will be avoided by a nondisclosure, Assurance Co. (Pacific Life) for a 20 year endowment
material facts, he should fully and fairly disclose irrespective of the fact whether the assured knew policy of P50k on the life of his 1 year old daughter,
the same, whether he believes them material or of such ailment or not. . . ." Helen. Ngo Hing supplied the essetntial data which
not. But notwithstanding this general rule it will - Lastly, appellant contends that even if the petitioner Mondragon, branch manager of the Pacific
not infrequently happen, especially in life risks, insurance company had a right to rescind the Life in Cebu, wrote on the corresponding form in his
that the assured may have a knowledge actual or contract, such right cannot now be enforced in view own handwriting, later typing the data on an
presumed of material facts, and yet entertain an of the provisions of section 47 of the Insurance Act application form signed by Ngo Hing. The latter paid
honest belief that they are not material. . . . The providing "Whenever a right to rescind a contract of the P1077.75 annual premium but retained P1,317
determination of the point whether there has or insurance is given to the insurer by any provision of as commission as he was also a duly authorized
has not been a material concealment must rest this chapter, such right must be exercised previous agent of Pacific Life. The binding deposit receipt was
largely in all cases upon the form of the questions to the commencement of an action on the contract." then issued to Ngo Hing; Mondragon handwrote his
propounded and the exact terms of the contract. This section was derived from section 2583 of the strong recommendation for the approval of the
Thus, where in addition to specifically named California Civil Code, but in contrast thereto, makes application on the back of the form.
diseases the insured was asked whether he had use of the imperative "must" instead of the - On April 30, Mondragon received a letter from
had any sickness within ten years, to which he permissive "may." Nevertheless, there are two Pacific Life which stated that the 20 year endowment
answered 'No,' and it was proven that within that answers to the problem as propounded. The first is plan was not available for minors below 7, but that
period he had had a slight attack of pharyngitis, it that the California law as construed by the code Pacific Life could consider the same under the
was held a question properly for the jury whether examiners, at whose recommendation it was Juvenile Triple Action Plan, advising that if the offer
such an inflammation of the throat was a 'sickness' adopted, conceded that "A failure to exercise the was acceptable, the Juvenile Non-Medical Declaration
within the intent of the inquiry, and the court right (of rescission), cannot, of course, prejudice any be sent to the company.
remarked on the appeal decision that if it could be defense to the action which the concealment may -Mondragon allegedly failed to inform Ngo Hing of
held as a matter of law that the policy was thereby furnish." (Codes of California Annotated; Tan Chay the non-acceptance of the insurance plan, instead
avoided, then it was a mere device on the part of Heng vs. West Coast Life Insurance Company writing Pacific Life again, recommending the
insurance companies to obtain money without [1927], p. 80, ante.) The second answer is that the approval of the endowment plan to children since
rendering themselves liable under the policy. . . . insurance company more than one month previous customers had been asking for such coverage since
" . . . The question should be left to the jury to the commencement of the present action wrote 1954.
whether the assured truly represented the state of the plaintiff and informed him that the insurance -On May 28, 1957, Helen died of influenza. Ngo Hing
his health so as not to mislead or deceive the contract was void because it had been procured sought the payment of the proceeds of the
insurer; and if he did not deal in good faith with through fraudulent representations, and offered to insurance, but having failed to do so, filed an action
the insurer in that matter, then the inquiry should refund to the plaintiff the premium which the latter for recovery with the CFI of Cebu. The Court ordered
be made, Did he know the state of his health so as had paid upon the return of the policy for Pacific Life to pay P50k with 6% interest, hence this
to be able to furnish a proper answer to such cancellation. As held in California as to a fire petition.
questions as are propounded? A Massachusetts insurance policy, where any of the material
INSURANCE Page
67

ISSUE same date, she paid the sum of P20.00 representing considering its prominence thereon and its
WON the binding deposit receipt constituted a the premium for which she was issued the materiality to the coverage applied for, the
temporary contract of the life insurance in question corresponding receipt signed by an authorized agent respondent insurance corporation received her
of the respondent insurance corporation. Upon the payment of premium and issued the corresponding
HELD filing of said application and the payment of the certificate of insurance without question. The
NO premium on the policy applied for, the respondent accident which resulted in the death of the insured, a
- The binding deposit receipt is merely a provisional insurance corporation issued to Carmen O. Lapuz its risk covered by the policy, occurred on May 31, 1969
contract and only upon compliance with the ff Certificate of Insurance. The policy was to be or FORTY-FIVE (45) DAYS after the insurance
conditions: (1) that the company be satisfied that effective for a period of 90 days. coverage was applied for. There was sufficient time
the applicant was insurable on standard rates (2) - On May 31, 1969 or during the effectivity of the for the private respondent to process the application
that if the company does not accept the application Insurance, Carmen O. Lapuz died in a vehicular and to notice that the applicant was over 60 years of
and offers a different policy, the insurance contract accident. age and thereby cancel the policy on that ground if it
shall not be binding until the applicant accepts the - On June 7, 1969, petitioner Regina L. Edillon, a was minded to do so. If the private respondent failed
new policy (3) that if the applicant is not found to be sister of the insured and who was the named to act, it is either because it was willing to waive
insurable on standard rates and the application is beneficiary in the policy, filed her claim for the such disqualification; or, through the negligence or
disapproved, the insurance shall not be in force at proceeds of the insurance, submitting all the incompetence of its employees for which it has only
any time and the premium be returned to the necessary papers and other requisites with the itself to blame, it simply overlooked such fact. Under
applicant. private respondent. Her claim having been denied, the circumstances, the insurance corporation is
-This implies the receipt is merely an Regina L. Edillon instituted this action in the Court of already deemed in estoppel. Its inaction to revoke
acknowledgement, on behalf of the company, that First Instance of Rizal. the policy despite a departure from the exclusionary
the Cebu branch of Pacific Life had received the - In resisting the claim of the petitioner, the condition contained in the said policy constituted a
premium and had accepted the application subject to respondent insurance corporation relies on a waiver of such condition.
processing by the insurance company, which will provision contained in the Certificate of Insurance, Disposition Judgment appealed from is REVERSED
approve or reject it depending on whether the excluding its liability to pay claims under the policy and SET ASIDE and respondent insurance
applicant is insurable on standard rates. As such, the in behalf of "persons who are under the age of corporation is ordered to pay to the petitioner the
receipt was never in force—it does not insure sixteen (16) years of age or over the age of sixty proceeds of Insurance
outright. No liability attaches until the principal (60) years ..." It is pointed out that the insured
approves the risk and a receipt is given by the being over sixty (60) years of age when she applied HARDING v. COMMERCIAL UNION (supra p.36)
agent; because private respondent failed to accept for the insurance coverage, the policy was null and
Pacific Life’s offer for the Juvenile Triple Action plan, void, and no risk on the part of the respondent TAN v. CA (PHILIPPINE AMERICAN LIFE
there was no meeting of the minds and thus no insurance corporation had arisen therefrom. INSURANCE COMPANY)
contract. Also, being an authorized agent of Pacific - RTC dismissed the complaint. 174 SCRA 403
Life, Ngo Hing must have known the company did GUTIERREZ; June 29, 1989
not offer the insurance applied for and merely took a ISSUE
chance on Mondragon’s recommendation. WON the acceptance by the private respondent NATURE
Disposition the decision appealed from is set aside, insurance corporation of the premium and the Review on certiorari of the decision of the Court of
absolving Pacific Life from their civil liabilities issuance of the corresponding certificate of insurance Appeals affirming the decision of the Insurance
should be deemed a waiver of the exclusionary Commissioner
EDILLON v. MANILA BANKERS LIFE condition of overage stated in the said certificate of
117 SCRA 187 insurance FACTS
VASQUEZ; September 30, 1982 - On September 23,1973, Tan Lee Siong, father of
HELD herein petitioners, applied for life insurance in the
NATURE YES amount of P 80,000.00 with respondent company.
Appeal from a decision of the CFI - The age of the insured Carmen 0. Lapuz was not Said application was approved and was issued
concealed to the insurance company. Her application effective November 6, 1973
FACTS for insurance coverage which was on a printed form - On April 26,1975, Tan Lee Siong died of hepatoma
- Sometime in April 1969, Carmen O, Lapuz applied furnished by private respondent and which contained (Exhibit B). Petitioners then filed with respondent
with respondent insurance corporation for insurance very few items of information clearly indicated her company their claim for the proceeds of the life
coverage against accident and injuries. In the age of the time of filing the same to be almost 65 insurance policy
application form which was dated April 15, 1969, she years of age. Despite such information which could -respondent company denied petitioners' claim and
gave the date of her birth as July 11, 1904. On the hardly be overlooked in the application form, rescinded the policy by reason of the alleged
INSURANCE Page
68

misrepresentation and concealment of material facts ab initio by reason of the insured's fraudulent merchant, and that the plaintiff Tan Chai Heng,
made by the deceased Tan Lee Siong in his concealment or misrepresentation. the beneficiary, was his nephew, whereas in
application for insurance. The premiums paid on the - The petitioners contend that there could have been truth and in fact and as the plaintiff and his said
policy were thereupon refunded no concealment or misrepresentation by their late coconspirators well knew, the said Tan Ceang
- Petitioners filed on November 27, 1975, a father because Tan Lee Siong did not have to buy was not single but was married and had several
complaint against the former with the Office of the insurance. He was only pressured by insistent children; and was not a merchant but a mere
Insurance Commissioner. Commissioner denied salesmen to do so employee of Tan Quina from whom he received
petition. CA affirmed Commissioners decision -The legislative answer to the arguments posed by only a meager salary, and that plaintiff was not
the petitioners is the "incontestability clause" added a nephew of the said Tan Ceang.
ISSUE by the second paragraph of Section 48. The insurer 2. Tan Ceang was seriously ill, suffering
WON according to Sec. 48 of the Insurance Code, has two years from the date of issuance of the from pulmonary tuberculosis of about three
insurance company is barred from rescinding insurance contract or of its last reinstatement within years' duration, which illness was incurable and
contract which to contest the policy, whether or not, the was well known to the plaintiff and his said
insured still lives within such period. After two years, coconspirators.
HELD the defenses of concealment or misrepresentation, 3. Locsin, in his capacity as medical
- Section 48. Whenever a right to rescind a contract no matter how patent or well founded, no longer lie examiner for the defendant, prepared and
of insurance is given to the insurer by any provision falsified the necessary medical certificate, in
of this chapter, such right must be exercised TAN CHAY HENG v. WEST COAST LIFE which it was made to appear, among other
previous to the commencement of an action on the INSURANCE things, that Tan Ceang had never used
contract. 51 PHIL 80 morphine, cocaine or any other drug; that he
After a policy of life insurance made payable on the JOHNS; November 21, 1927 was then in good health and had never
death of the insured shall have been in force during consulted any physician; that he had never spit
the lifetime of the insured for a period of two years FACTS blood; and that there was no sign of either
from the date of its issue or of its last reinstatement, - Plaintiff alleges that defendant accepted and present or past disease of his lungs; whereas in
the insurer cannot prove that the policy is void ab approved a life insurance policy of for the sum of truth and in fact, plaintiff and coconspirators
initio or is rescindable by reason of the fraudulent P10,000 in which the plaintiff was the sole well knew, Tan Ceang was addicted to
concealment or misrepresentation of the insured or beneficiary; that the policy was issued upon the morphine, cocaine, and opium and had been
his agent. payment by the said Tan Ceang of the first year's convicted and imprisoned therefor, and for
- According to the petitioners, the Insurance Law premium amounting to P936; that in and by its about three year prior thereto had been
was amended and the second paragraph of Section terms, the defendant agreed to pay the plaintiff as suffering from pulmonary tuberculosis.
48 added to prevent the insurance company from beneficiary the amount of the policy upon the receipt 4. Plaintiff caused a confidential report to
exercising a right to rescind after the death of the of the proofs of the death of the insured while the the defendant insurance company to be signed
insured policy was in force; that without any premium being by one V. Sy Yock Kian, who was an employee
- The so-called "incontestability clause" precludes the due or unpaid, Tan Ceang died on May 10, 1925; of Go Chulian, in which it was falsely
insurer from raising the defenses of false that in June, 1925, plaintiff submitted the proofs of represented that Tan Ceang was worth about
representations or concealment of material facts the death of Tan Ceang with a claim for the payment P40,000, had an annual income of from eight to
insofar as health and previous diseases are of the policy which the defendant refused to pay, for ten thousand pesos net, had the appearance of
concerned if the insurance has been in force for at which he prays for a corresponding judgment, with good health, and never had tuberculosis.
least two years during the insured's lifetime. The legal interest from the date of the policy, and costs. 5. After said application for insurance,
phrase "during the lifetime" found in Section 48 - Defendant alleges that the insurance policy on the medical certificate and confidential report had
simply means that the policy is no longer considered life of Tan Ceang, upon which plaintiff's action is been prepared and falsified, plaintiff and
in force after the insured has died. The key phrase in based, was obtained by the plaintiff in confabulation coconspirators caused the same to be forwarded
the second paragraph of Section 48 is "for a period with one Go Chulian, of Bacolod, Negros Occidental; to the defendant at its office in Manila, the
of two years." Francisco Sanchez of the same place; and Dr. V. S. medical certificate thru the said Dr. V. S. Locsin
- The policy was issued on November 6,1973 and the Locsin, of La Carlota, Negros Occidental, thru fraud as medical examiner, and said application for
insured died on April 26,1975. The policy was thus in and deceit perpetrated against this defendant in the insurance and confidential report thru the said
force for a period of only one year and five months. following manner, to wit: Francisco Sanchez in his capacity as one of the
Considering that the insured died before the two- 1. Go, Sanchez and Locsin, caused Tan agents of the defendant insurance company in
year period had lapsed, respondent company is not, Caeng to sign an application for insurance with the Province of Occidental Negros; that the
therefore, barred from proving that the policy is void the defendant in the sum of P10,000, in which it defendant, believing that the representations
was said that Tan Ceang was single and was a made in said document were true, and relying
INSURANCE Page
69

thereon, provisionally accepted the said the City of Manila, in criminal case No. 31425 of that rescinded. If all of the material matters set forth and
application for insurance on the life of Tan court, of the crime of falsification of private alleged in the defendant's special plea are true, there
Ceang in the sum of P10,000 and issued a documents in connection with an fraudulent was no valid contract of insurance, for the simple
temporary policy pending the final approval or insurance, similar to the present, committed against reason that the minds of the parties never met and
disapproval of said application by defendant's this defendant in the month of September, 1924; never agreed upon the terms and conditions of the
home-office in San Francisco, California, where that in the same case the said Francisco Sanchez contract. We are clearly of the opinion that, if such
in case of approval a permanent policy was to was one of the coaccused of the said Go Chulian but matters are known to exist by a preponderance of
be issued; that such permanent policy was was discharged from the complaint, because he the evidence, they would constitute a valid defense
never delivered to the plaintiff because offered himself and was utilized as a state's witness; to plaintiff's cause of action. Upon the question as to
defendant discovered the fraud before its that there is another civil action now pending against whether or not they or are not true, we do not at
delivery. Go Chulian and Sanchez in the Court of First this time have or express any opinion, but we are
6. That the first agreed annual premium Instance of Manila (civil case No. 28680), in which clear that section 47 does not apply to the
on the insurance in question of P936.50 not the present defendant is the plaintiff, for the allegations made in the answer, and that the trial
having been paid within 60 days after medical recovery of the amounts of two insurance policies court erred in sustaining the demurrer.
examination of the applicant as required by the aggregating P19,000, fraudulently obtained by the Disposition The judgment of the lower court is
regulations of the defendant insurance company, said Go Chulian and Sanchez. reversed and the case is remanded for such other
plaintiff and coconspirators caused Tan Ceang to - To this, plaintiff filed a demurrer which was and further proceedings as are not inconsistent with
sign a health certificate for reinstatement; that granted. this opinion, with costs against the plaintiff.
the said temporary policy was delivered by
defendant to the insured on April 10, 1925, in ISSUE PIONEER INSURANCE AND SURETY
the belief that said statements and WON defense is barred by Art. 47 CORPORATION v. YAP
representations were true and in reliance 61 SCRA 426
thereon. HELD FERNANDEZ; December 19, 1974
7. 2 ½ months after the supposed medical NO
examination above referred to, and exactly 1 Ratio The word "rescind" has a well defined legal NATURE
month after the date of the health certificate for meaning, and as applied to contracts, it presupposes Appeal by certiorari from CA decision affirming a CFI
reinstatement above set forth, Tan Ceang died the existence of a contract to rescind. decision which declared plaintiff Yap entitled to
in Valladolid, Occidental Negros, of pulmonary Reasoning recover from defendant Pioneer Insurance and
tuberculosis, the same illness from which - Plaintiff vigorously contends that section 47 of the Surety Corp, the full amount of the damage inquired
suffering at the time it is supposed he was Insurance Act should be applied, and that when so in Policy No. 4219
examined by Dr. V. S. Locsin, but that the applied, defendant is barred and estopped to plead
plaintiff coconspirators, pursuant to their and set forth the matters alleged in its special FACTS
conspiracy, caused the said Dr. V. S. Locsin to defense. That section is as follows: - Yap owned a store in a 2 storey building, where
state falsely in the certificate of death that the Whenever a right to rescind a contract of she sold shopping bags and footwear. Her son-in-law
said Tan Ceang had died of cerebral insurance is given to the insurer by any provision was in charge of the store
hemorrhage. of this chapter, such right must be exercised - April 19, 1962- Yap took out Fire Insurance Policy
- Defendant also alleges that plaintiff was, like V. Sy previous to the commencement of an action on the No. 4216 from Pioneer with a face value of P25,000
Yock Kian, an employee of Go Chulian; that the contract. covering her stocks, office furniture, fixtures, etc.
latter was the ringleader of a gang of malefactors, - It will be noted that defendant does not seek to - among the conditions set forth:
who, during, and for some years previous to the have the alleged insurance contract rescinded. It The Insured shall give notice to the
dates above mentioned, were engaged in the illicit denies that it ever made any contract of insurance Company of any insurance or insurances already
enterprise of procuring fraudulent life insurances on the life of Tan Ceang or that any such a contract effected, or which may subsequently be
from the present defendant, similar to the one in ever existed, and that is the question which it seeks effected, covering any of the property hereby
question, and which enterprise was capitalized by to have litigated by its special defense. In the very insured, and unless such notice be given and
him by furnishing the funds with which to pay the nature of things, if the defendant never made or the particulars of such insurance or insurances
premium on said fraudulent insurance; that the said entered into the contract in question, there is no be stated in, or endorsed on this Policy by or on
Go Chulian was the one who furnished the money contract to rescind, and, hence, section 47 upon behalf of the Company before the occurrence of
with which to pay the first and only annual premium which the lower based its decision in sustaining the any loss or damage, all benefits under this
on the insurance here in question, amounting to demurrer does not apply. As stated, an action to Policy shall be forfeited. (emphasis supplied)
P936.50; that the said Go Chulian, on August 28, rescind a contract is founded upon and presupposes It is understood that, except as may be
1926, was convicted by the Court of First Instance of the existence of the contract which is sought to be stated on the face of this policy there is no other
INSURANCE Page
70

insurance on the property hereby covered and in which a fire would be profitable to the insured. WON petitioners violated the “Other Insurance
no other insurance is allowed except by the According to Justice Story: "The insured has no right Clause” of the insurance policies
consent of the Company endorsed hereon. Any to complain, for he assents to comply with all the
false declaration or breach or this condition will stipulation on his side, in order to entitle himself to HELD
render this policy null and void. the benefit of the contract, which, upon reason or YES
- At the time of insurance of Policy 4219(April 19, principle, he has no right to ask the court to - Petitioners admit that the respective insurance
1962), an insurance policy for P20,000 issued by the dispense with the performance of his own part of the policies issued by private respondents did not state
Great American Insurance Company covering the agreement, and yet to bind the other party to or endorse thereon the other insurance coverage
same properties was noted on said policy as co- obligations, which, but for those stipulation would obtained or subsequently effected on the same
insurance. not have been entered into." stocks in trade for the loss of which compensation is
- August 29, 1962 : parties executed an Disposition the appealed judgment of the Court of claimed by petitioners. It is further admitted by
endorsement on Policy 4219 stating: Appeals is reversed and set aside, and the petitioner petitioners that Equitable's policy stated "nil" in the
It is hereby declared and agreed that the co- absolved from all liability under the policy. space thereon requiring indication of any co-
insurance existing at present under this policy is insurance although there were 3 policies subsisting
as follows: P20,000.00 � Northwest Ins., and not NEW LIFE ENTERPRISES v. CA on the same stocks in trade at the time of the loss,
as originally stated. (emphasis supplied) 207 SCRA 609 namely, that of Western in the amount of
Except as varied by this endorsement, all other REGALADO; March 31, 1992 P350,000.00 and two 2 policies of Reliance in the
terms and conditions remain unchanged. total amount of P1,000,000.00.
- September 26, 1962: Yap took out another fire FACTS - The coverage by other insurance or co-insurance
insurance policy for P20,000 covering the same - Julian Sy and Jose Sy Bang are partners engaged effected or subsequently arranged by petitioners
properties, from Federal Insurance Company. This in the business of selling construction materials were neither stated nor endorsed in the policies of
policy was procured without notice to and the under the business name “New Life Enterprises.” the 3 private respondents, warranting forfeiture of all
written consent of Pioneer, and was therefore not Julian Sy insured against fire the stocks in trade of benefits thereunder if we are to follow the express
noted as a co-insurance in Policy 4219. New Life Enterprises with Western Guaranty stipulation in Policy Condition No. 3.
- December 19, 1962: Fire burned Yap’s store Corporation, Reliance Surety and Insurance Co. Inc., - The terms of the contract are clear and
and Equitable Insurance Corporation in the unambiguous. The insured is specifically required to
ISSUE aggregate amount of P1,550,000.00. When the disclose to the insurer any other insurance and its
WON petitioner should be absolved from liability on building where New Life Enterprises was located, particulars which he may have effected on the same
Fire insurance Policy No. 4219 on account of any along with the stocks in trade therein, were gutted subject matter. The knowledge of such insurance by
violation by respondent Yap of the co-insurance by fire, petitioners filed an insurance claim against the insurer's agents, even assuming the acquisition
clause therein the three companies. The insurance companies all thereof by the former, is not the "notice" that would
denied Julian Sy’s claim on the ground of “breach of stop the insurers from denying the claim. Besides,
HELD policy condition,” (i.e., the “other insurance” clause the so-called theory of imputed knowledge, that is,
YES which required New Life Enterprises to inform each knowledge of the agent is knowledge of the principal,
- The petitioner should be absolved. of the insurance companies in case the former aside from being of dubious applicability here has
Reasoning insures with another company the same property likewise been roundly refuted by respondent court
- There was a violation by Yap of the co-insurance already insured by each of the insurance whose factual findings we find acceptable. The mere
clause contained in Policy No. 4219 which resulted in companies). fact that Yap Kam Chuan was an agent for both
the avoidance of the petitioner’s liability. - Because of the denial of their claims for payment Reliance and Equitable does not justify the allegation
- By the plain terms of the policy, other insurance by the 3 insurance companies, petitioners filed that the two are sister companies. Availment of the
without the consent of petitioner would ipso facto separate civil actions against the former before the services of the same agents and adjusters by
avoid the contract. It required no affirmative act of Regional Trial Court of Lucena City, which cases were different companies is a common practice in the
election on the part of the company to make consolidated for trial. The trial court ruled in favor of insurance business and such facts do not warrant the
operative the clause avoiding the contract, wherever petitioner. However, the Court of Appeals reversed speculative conclusion of the trial court.
the specified conditions should occur. Its obligations the trial court’s decision, found petitioner to have - Considering the terms of the policy which required
ceased, unless, being informed of the fact, it violated Clauses 3 and 27 of the separate insurance the insured to declare other insurances, the
consented to the additional insurance. policies issued by the 3 companies, and exonerated statement in question must be deemed to be a
- The obvious purpose of the aforesaid requirement the insurance companies from liability. statement (warranty) binding on both insurer and
in the policy is to prevent over-insurance and thus insured, that there were no other insurance on the
avert the perpetration of fraud. The public, as well as ISSUE property. The annotation then, must be deemed to
the insurer, is interested in preventing the situation be a warranty that the property was not insured by
INSURANCE Page
71

any other policy. Violation thereof entitled the


insurer to rescind. HELD
- The obvious purpose of the aforesaid requirement 1. YES.
in the policy is to prevent over-insurance and thus YOUNG v. MIDLAND TEXTILE INSURANCE CO. Reasoning It is admitted by both parties that the
avert the perpetration of fraud. The public, as well as 30 PHIL 617 fireworks are hazardous goods. The defendant
the insurer, is interested in preventing the situation JOHNSON; March 31, 1915 alleged that they were "stored." The plaintiff
in which a fire would be profitable to the insured. contends that under all the facts and circumstances
The insured has no right to complain, for he assents FACTS of the case, they were not “stored” in said building,
to comply with all the stipulations on his side, in - K.S. Young had a candy and fruit store on the and that the placing of them in the building was not
order to entitle himself to the benefit of the contract, Escolta, Manila, and occupied a building at 321 Calle a violation of the terms of the contract.
which, upon reason or principle, he has no right to Claveria, as a residence and bodega. The Midland - Whether a particular article is "stored" or not must,
ask the court to dispense with the performance of his Textile Insurance Co. in consideration of the in some degree, depend upon the intention of the
own part of the agreement, and yet to bind the other payment of a premium of P60, entered into a parties. Nearly all of the cases cited by the lower
party to obligations, which, but for those contract of insurance with Young by the terms of court are cases where the article was being put to
stipulations, would not nave been entered into. which the company, upon certain conditions, some reasonable and actual use, which might easily
- It is not disputed that the insured failed to reveal promised to pay Young the sum of P3,000 in case have been permitted by the terms of the policy, and
before the loss three other insurances. By reason of said residence and bodega and contents should be within the intention of the parties, and excepted
said unrevealed insurances, the insured had been destroyed by fire. from the operation of the warranty, like the present.
guilty of a false declaration; a clear - One of the conditions of the contract is: "Warranty - (1) Where merchants have had or kept the
misrepresentation and a vital one because where the B – It is hereby declared and agreed that during the "hazardous" articles in small quantities, and for
insured had been asked to reveal but did not, that pendency of this policy no hazardous goods be actual daily use, for sale, such as gasoline,
was deception. Otherwise stated, had the insurer stored or kept for sale, and no hazardous trade or gunpowder, etc.; (2) Where such articles have been
known that there were many co-insurances, it could process be carried on, in the building to which this brought on the premises for actual use thereon, and
have hesitated or plainly desisted from entering into insurance applies, or in any building connected in small quantities, such as oil, paints, etc; and (3)
such contract. Hence, the insured was guilty of clear therewith." Where such articles or goods were used for lighting
fraud. - Young placed in the residence and bodega three purposes, and in small quantities.
- As the insurance policy against fire expressly boxes filled with fireworks. Said residence and - In the present case no claim is made that the
required that notice should be given by the insured bodega and the contents thereof were partially "hazardous goods" were placed in the bodega for
of other insurance upon the same property, the total destroyed by fire. present or daily use. It is admitted that they were
absence of such notice nullifies the policy. - The fireworks had been given to Young by the placed in the bodega "for future use," or for future
- Additionally, insofar as the liability of respondent former owner of the Luneta Candy Store. He consumption, or for safe keeping. It seems clear to
Reliance is concerned, it is not denied that the intended to use them in the celebration of the us that the "hazardous goods" in question were
complaint for recovery was filed in court by Chinese New Year. However, the authorities of the "stored" in the bodega, as that word is generally
petitioners only on January 31, 1984, or after more city of Manila had prohibited the use of fireworks on defined. That being true, suppose the defendant had
than one (1) year had elapsed from petitioners' said occasion, so Young then placed them in the made an examination of the premises, even in the
receipt of the insurers' letter of denial on November bodega where they remained from the 4th or 5th of absence of a fire, and had found the "hazardous
29, 1982. February, 1913 until after the fire of March 18, 1913. goods" there, would it not have been justified in
- The condition contained in an insurance policy that - Both parties agree that the fireworks come within declaring the policy null and of no effect by reason of
claims must be presented within one year after the phrase "hazardous goods," mentioned in a violation of its terms? If it might, then may it not
rejection is not merely a procedural requirement but "Warranty B" of the policy; that the fireworks were repudiate its liability, even after the fire? If the
an important matter essential to a prompt found in a part of the building not destroyed by the "warranty" is a term of the contract, will not its
settlement of claims against insurance companies as fire and that they in no way contributed to the fire, violation cause a breach and justify noncompliance
it demands that insurance suits be brought by the or to the loss that resulted. or repudiation?
insured while the evidence as to the origin and cause - The lower court rendered a judgment in favor of - Contracts of insurance are contracts of indemnity,
of destruction have not yet disappeared. Young for the sum of P2,708.78, and costs. upon the terms and conditions specified therein.
Parties have a right to impose such reasonable
QUA CHEE GAN v. LAW UNION (supra p.48) ISSUE conditions at the time of the making of the contract
1. WON the placing of the fireworks in the building as they deem wise and necessary. The rate of
insured, they being "hazardous goods," was a premium is measured by the character of the risk
violation of the terms of the contract of insurance assumed. The insurer, for a comparatively small
and especially of "Warranty B." consideration, undertakes to guarantee the insured
INSURANCE Page
72

against loss or damage, upon the terms and NATURE - Sept 30, 1948: Nava called the attention of the
conditions agreed upon, and upon no other. When Petition for review of a decision of the Court of insurance companies to the SC decision (Haw Pia v.
the insurer is called upon to pay, in case of loss, he Appeals China Banking Corporation) establishing and
may justly insist upon a fulfillment of the terms of recognizing the relationship of debtor and creditor
the contract. If the insured, cannot bring himself FACTS with respect to payments in fiat currency made
within the terms and conditions of the contract, he is - Before the war, Nava entered into a contract of during the Japanese occupation on pre-war
not entitled to recover for any loss suffered. The insurance with Insular Life Assurance Co., Ltd. (face obligations.
terms of the contract constitute the measure of the value of P5k), and 17 separate contracts of life - Companies still refused saying that the SC decision
insurer's liability. If the contract has been insurance with Filipinas Life Assurance Co. (total face was not applicable to transactions undertaken during
terminated, by a violation of its terms on the part of value of P90k). Each and everyone of the 18 policies Japanese occupation when they relate to life
the insured, there can be no recovery. Compliance issued by defendants to plaintiff contains a loan insurance policies.
with the terms of the contract is a condition clause of the following tenor: - Feb 4, 1949: Nava was again refused even if the
precedent to the right of recovery. Policy loans. After three full years' total amount of the cash surrender values of the 18
- Young argues that since the "storing" of the premiums have been paid upon this Policy, if no policies reached the sum of P9,468.29.
fireworks on the premises did not contribute in any premium payment is in default, the Company, - Feb 10, 1949: Nava brought case to the CFI Manila
way to the damage occasioned by the fire, he should subject to its then existing rules, will advance on praying for the rescission of the abovementioned 18
be permitted to recover. That argument, however, is proper assignment and delivery of this Policy and policies and for the refund to him of all the premiums
beside the question, if the "storing" was a violation on the sole security thereof a sum equal to, or at so far paid by him to defendants in the amount of
of the terms of the contract. The violation of the the option of the owner less than, the cash value P31,633.80, plus 6% interest thereon as damages
terms of the contract, by virtue of the provisions of specified in the Schedule of Policy Values, less any - Nov 28, 1951: companies passed a resolution
the policy itself, terminated, at the election of either existing indebtedness on or secured by this Policy which was approved by the Insurance Commissioner,
party, the contractual relations. and any unpaid balance of the premium for the giving full credit to all premium payments made by
- Young paid a premium based upon the risk at the current policy-year; provided interest at six per their policyholders in fiat currency during the
time the policy was issued. Certainly, the placing of centum per annum on the whole amount of the Japanese occupation on account of pre-war policies
the firecrackers in the building insured increased the loan is paid in advance to the end of the current for which reason they filed an amended answer
risk. Young had not paid a premium based upon the policy-year. At the end of the current policy-year offering to pay plaintiff the amount of P9,468.29
increased risk, neither had the defendant issued a interest at the same rate for one year in advance which represents the aggregate cash surrender
policy upon the theory of a different risk. He was will be due and payable, and annually thereafter, values of all the policies in question as of February
enjoying, if his contention may be allowed, the and if not so paid will be added to the principal 10, 1949, but apparently this offer was refused.
benefits of an insurance policy upon one risk, and bear the same rate of interest. Failure to - CFI: (1) rescinded the insurance contracts; (2)
whereas, as a matter of fact, it was issued upon an repay any such loan or interest shall not avoid this ordered defendant Filipinas Life Assurance Co. to pay
entirely different risk. The defendant had neither Policy unless the total indebtedness shall equal or plaintiff the amount of P32,072.60; and (3) ordered
been paid nor had issued a policy to cover the exceed the full amount of the loan value available defendant Insular Life Assurance Co., Ltd. to pay
increased risk. An increase of risk which is hereunder. plaintiff the amount of P2,574.00
substantial and which is continued for a considerable Any indebtedness on this Policy shall first - CA affirmed.
period of time, is a direct and certain injury to the be deducted from any money payable or in any
insurer, and changes the basis upon which the settlement under this Policy. ISSUES
contract of insurance rests. - Nava had so far paid to Insular a total of P2,574; 1. WON CA erred in ruling that as a consequence of
Disposition Decision of the lower court is and to Filipinas Life, a total of P32,072.60. the decision in the Haw Pia case petitioners violated
REVERSED. - April 28, 1948: Nava applied to the companies for the loan clause contained in the insurance policies
a P5k loan in line with the loan clause, but they thereby entitling respondent to their rescission
TAN v. CA (supra p.51) refused to grant it because certain regulations issued 2. WON CA erred in ruling that by virtue of Article
by the Insurance Commissioner required the 1295 of the old Civil Code petitioners should refund
AREOLA v. CA (supra p.26) insurance companies to withhold the payments on to defendant all the premiums paid on his insurance
premiums made during the Japanese occupation policies as a consequence of their rescission
TAN CHAY v. WEST COAST (supra p.51) because the same shall be subject to future 3. WON CA erred in not ruling that, even if
adjustments " as soon as debtor-creditor relationship respondent is entitled to the rescission of said
FILIPINAS LIFE ASSURANCE v. NAVA is established" and because of such process of insurance policies, he can only recover their cash
17 SCRA 210 "withholding" plaintiff was not entitled to borrow any surrender value at the time the complaint was filed
BAUTISTA ANGELO; May 20, 1966 amount until such adjustment has been made.
HELD
INSURANCE Page
73

1. NO. warranty, or other material provision of a policy, on - It cannot be denied that petitioners had in turn
- Even assuming the validity of the Insurance the part of either party thereto, entitles the other to already derived material benefits from the use of
Commissioner’s regulations, the fact however is that rescind." premiums paid to them by respondent before, during
such requirement has already lost its legal effect and - "The general rule is that a breach of the agreement and after the last war from which they must have
value when our Supreme Court rendered its decision to make the loan does not entitle the insured to realized huge profits, and in this light alone
in the Haw Pia case wherein it was declared, among rescind the contract," is not controlling in this petitioners cannot claim prejudice or unfairness if
others, that all payments made in fiat currency jurisdiction. Firstly, it was not shown that the they are ordered to refund the premiums paid by
during the Japanese occupation in relation with any insurance laws in the states where said ruling respondents.
contractual obligation executed before the war were prevails contain a provision identical to Section 69 of 3. NO.
valid to all intents and purposes, and yet petitioners our Insurance Law we quoted above, and secondly, - Issue is corollary to preceding issue. No need to
apparently did not give any importance to such the rule cited by Vance is not a rule uniformly refute.
decision for in their opinion it does not have any followed by all states in the US, for on this matter Disposition Decision appealed from is AFFIRMED.
application to transactions which have any relation to there is a marked divergence of opinion. Costs against petitioners
payment of premiums on life insurance policies. 2. NO
- It cannot be denied that a life insurance policy - Considering that our Insurance Law does not CHAPTER VII. RISKS AND COVERAGES
involves a contractual obligation wherein the insured contain an express provision as to what the court
becomes duty bound to pay the premiums agreed should do in cases of rescission of an insurance VDA. DE BATACLAN v. MEDINA
upon, lest he runs the risk of having his insurance policy under Section 69, the provision that should 102 PHIL 181
policy lapse if he fails to pay such premiums. apply is that embodied in Article 1225 of the old Civil MONTEMAYOR; October 22, 1957
- The fact that if the insured had paid in full the Code, as postulated in Article 16 of the same Code,
premiums corresponding to the first 3 years of the which provides that on matters which are not FACTS
life of his policy he cannot be considered delinquent governed by special laws the provisions of said Code - Juan Bataclan rode Bus 30 of Medina Transport,
that would cause the lapse of his policy if the same shall supplement its deficiency. And said Article 1295 driven by Saylon, shortly after midnight. The bus
contains an automatic premium payment clause provides: was running very fast. One of the front tires burst.
cannot divest such policy of its contractual nature, ART. 1295. Rescission makes necessary the return Bus fell into canal and turned turtle. 4 passengers
for the result of such failure would only be for him to of the things which were the subject-matter of the couldn’t get out, including Bataclan. Gasoline began
pay later the premium plus the corresponding contract, with their fruits, and of the price paid, to leak from the overturned bus. 10 men came to
interest depending upon the condition of the policy. with interest thereon. ...xxx help. 1 carried a torch and when he approached the
But certainly it does not cease to be a contractual - Said the petitioners: "Recovery of the full amount bus, fire started, killing the trapped passengers.
liability insofar as the payment of that premium is of the premium after the insurer has sustained for - TC opined that proximate cause of Bataclan’s
concerned for whether he likes it or not that sometime the risk of the insurance and the insured death was not the overturning of bus but the fire. At
premium has to be paid lest he allows the lapse of has enjoyed the benefit of protection is obviously the time fire started, Bataclan, though injured, was
his policy. Consequently, the payment of premiums unjust and is so recognized by the better still alive and damages were awarded, not for his
on the life insurance policies made by Nava before authorities." The ruling above quoted merely death, but for physical injuries suffered.
and during the war up to the time he applied for the represents the minority rule in the US, the majority
loan in question with petitioners should be rule being that the insured can recover all premiums ISSUE/S
considered likewise as valid payments upon the paid, in some cases with interest in case of wrongful WON the proximate cause is the overturning of the
theory that such insurance policies are in the nature cancellation, repudiation, termination or rescission of bus or the fire
of a contractual obligation within the meaning of the the contract of life insurance.
civil law. In effect, therefore, those payments were - Contention that because respondent cannot restore HELD
made by a debtor to a creditor within the meaning of to petitioners the "value of the benefit of protection" - The proximate cause is the overturning of the bus.
the requirement of the regulations of the Insurance which he might have received under the 18 life - Ordinarily, when a bus overturns and pins down
Commissioner and as such they can offer no excuse insurance policies in question he is not entitled to passenger, merely causing him injuries. If through
to petitioners for refusing to grant the loan as rescind them under the provision of Article 1295 of some event, unexpected and extraordinary, the bus
contemplated in the loan clause embodied in the the old Civil Code, is untenable because said article is set on fire, and passenger is burned to death, one
policies in question. only contemplates a transaction whether material might contend that the proximate cause was the fire
- It is clear from the foregoing that the petitioners things are involved, and do not refer to intangible and not the overturning of the vehicle.
violated the loan clause embodied in each of the 18 ones which cannot be the subject of restoration, for - But here, the proximate cause of Bataclan’s death
life insurance policies issued to respondent to rescind to interpret it otherwise would be to defeat the law is the overturning of the bus, this for the reason that
all said policies under Section 69 of the Insurance itself with the result that rescission can never be had when the vehicle turned not only on its side but
Act, which provides: "The violation of a material under Section 69 of our Insurance Law.
INSURANCE Page
74

completely on its back, leaking of gasoline from the their way home after attending the celebration of the Ratio The generally accepted rule is that, death or
tank was not unnatural or unexpected. "Maskarra Annual Festival." injury does not result from accident or accidental
- The coming of the men with the torch was in - Thereafter, Julia Surposa and the other means within the terms of an accident-policy if it is
response to the call for help, made only not by the beneficiaries of said insurance policy filed a written the natural result of the insured's voluntary act,
passengers but even the driver and conductor, and notice of claim with the FINMAN Corp which denied unaccompanied by anything unforeseen except the
because it was very dark, about 2:30 am, rescuers said claim contending that murder and assault are death or injury. There is no accident when a
had to carry a light with them. Coming as they did not within the scope of the coverage of the insurance deliberate act is performed unless some additional,
from rural area where lanterns and flashlights were policy. unexpected, independent, and unforeseen happening
not available, they had to use a torch. What was - Feb. 24, 1989: Surposa filed a complaint with the occurs which produces or brings about the result of
more natural than that said rescuers should Insurance Commission which subsequently ordered injury or death. In other words, where the death or
innocently approach the overturned vehicle to extend FINMAN to pay Surposa the proceeds of the policy injury is not the natural or probable result of the
aid. with interest. insured's voluntary act, or if something unforeseen
- The coming of the men with the torch was to be - CA affirmed said decision. occurs in the doing of the act which produces the
expected, and was a natural sequence of the injury, the resulting death is within the protection of
overturning of the bus, the trapping of some of its ISSUE the policies insuring against death or injury from
passengers and the call for outside help. WON CA committed GAD in applying the principle of accident.
- The burning of bus can also in part be attributed to "expresso unius exclusio alterius" in a personal - The personal accident insurance policy involved
negligence of carrier, through its driver and accident insurance policy (since death resulting from herein specifically enumerated only 10 circumstances
conductor. They, or at least the driver, should have murder and/or assault are impliedly excluded in said wherein no liability attaches to FINMAN for any
known that in the position in which the overturned insurance policy considering that the cause of death injury, disability or loss suffered by the insured as a
bus was, gasoline could and must have leaked from of the insured was not accidental but rather a result of any of the stimulated causes.
the gasoline tank and soaked the area in and around deliberate and intentional act of the assailant in -The principle of " expresso unius exclusio alterius"
the bus. Gasoline can be smelt and detected even killing the former as indicated by the location of the the mention of one thing implies the exclusion of
from a distance, and yet neither the driver nor the lone stab wound on the insured) [TF they cannot be another thing is therefore applicable in the instant
conductor would appear to have cautioned or taken made to indemnify the Surposa heirs] case since murder and assault, not having been
steps to warn rescuers not to bring the lighted torch expressly included in the enumeration of the
too near the bus. HELD circumstances that would negate liability in said
NO insurance policy: the failure of the FINMAN to
FINMAN GENERAL ASSURANCE CORPORATION - The record is barren of any circumstance showing include death resulting from murder or assault
v. CA (SURPOSA) how the stab wound was inflicted. While the act may among the prohibited risks leads inevitably to the
213 SCRA 493 not exempt the unknown perpetrator from criminal conclusion that it did not intend to limit or exempt
NOCON; September 2, 1992 liability, the fact remains that the happening was a itself from liability for such death.
pure accident on the part of the victim. The insured - A1377 NCC: The interpretation of obscure words
NATURE died from an event that took place without his or stipulations in a contract shall not favor the party
Certiorari foresight or expectation, an event that proceeded who caused the obscurity.
from an unusual effect of a known cause and, - NPC vs. CA [1986]~ It is well settled that
FACTS therefore, not expected. contracts of insurance are to be construed liberally in
- Oct. 22, 1986: Carlie Surposa was insured with Reasoning favor of the insured and strictly against the insurer.
Finman General Assurance Corporation under Finman - De la Cruz vs. Capital Insurance & Surety Co., Inc Thus ambiguity in the words of an insurance contract
General Teachers Protection Plan Master Policy No. (1966)~ The terms "accident" and "accidental" as should be interpreted in favor of its beneficiary.
2005 and Individual Policy No. 08924 with his used in insurance contracts have not acquired any Disposition DENIED for lack of merit.
parents, spouses Julia and Carlos Surposa, and technical meaning, and are construed by the courts
brothers Christopher, Charles, Chester and Clifton, in their ordinary and common acceptation. Thus, the CALANOC v. CA
all surnamed, Surposa, as beneficiaries. terms have been taken to mean that which happen 98 PHIL 79
- While said insurance policy was in full force and by chance or fortuitously, without intention and BAUTISTA; December 16, 1955
effect, the insured, Carlie Surposa, died on October design, and which is unexpected, unusual, and
18, 1988 as a result of a stab wound inflicted by one unforeseen. An accident is an event that takes place FACTS
of the 3 unidentified men without provocation and without one's foresight or expectation an event that - Basilio was a watchman of the Manila Auto Supply
warning on the part of the former as he and his proceeds from an unknown cause, or is an unusual located at the corner of Avenida Rizal and Zurbaran.
cousin, Winston Surposa, were waiting for a ride on effect of a known cause and, therefore, not He secured a life insurance policy from the Philippine
expected. American Life Insurance Company in the amount of
INSURANCE Page
75

P2,000 to which was attached a supplementary a matter that affects the security of the The clause, however, expressly provided that it
contract covering death by accident. On January 25, neighborhood. No doubt there was some risk coming would not apply where death resulted from an injury
1951, he died of a gunshot wound on the occasion of to him in pursuing that errand, but that risk always "intentionally inflicted by a third party."
a robbery committed in the house of Atty. Ojeda at existed it being inherent in the position he was - One night, a band of robbers entered their house.
the corner of Oroquieta and Zurbaran streets. holding. He cannot therefore be blamed solely for Juan went out of his room and he was met with 9
Calanoc, the widow, was paid the sum of P2,000, doing what he believed was in keeping with his duty knife stabs. He died. The robbers were convicted of
face value of the policy, but when she demanded the as a watchman and as a citizen. And he cannot be robbery with homicide.
payment of the additional sum of P2,000 considered as making an arrest as an officer of the - The family was claiming the additional P5k from
representing the value of the supplemental policy, law, as contended, simply because he went with the Insular, under the Accidental Death Benefit clause.
the company refused alleging, as main defense, that traffic policeman, for certainly he did not go there for Insular refused on the ground that the death
the deceased died because he was murdered by a that purpose nor was he asked to do so by the resulted from injuries intentionally inflicted by 3rd
person who took part in the commission of the policeman. parties and was therefore not covered. Biagtans filed
robbery and while making an arrest as an officer of - Much less can it be pretended that Basilio died in against Insular. CFI ruled in favor of Biagtans.
the law which contingencies were expressly excluded the course of an assault or murder considering the
in the contract and have the effect of exempting the very nature of these crimes. In the first place, there ISSUE
company from liability. is no proof that the death of Basilio is the result of WON the injuries were intentionally inflicted
- It is contended in behalf of the company that either crime for the record is barren of any
Basilio was killed which "making an arrest as an circumstance showing how the fatal shot was fired. HELD
officer of the law" or as a result of an "assault or Perhaps this may be clarified in the criminal case YES
murder" committed in the place and therefore his now pending in court as regards the incident but - Whether the robbers had the intent to kill or
death was caused by one of the risks excluded by before that is done anything that might be said on merely to scare the victim or to ward off any defense
the supplementary contract which exempts the the point would be a mere conjecture. Nor can it be he might offer, it cannot be denied that the act itself
company from liability. This contention was upheld said that the killing was intentional for there is the of inflicting the injuries was intentional.
by the Court of Appeals. Hence, this petition. possibility that the malefactor had fired the shot - The exception in the accidental benefit clause
merely to scare away the people around for his own invoked by the appellant does not speak of the
ISSUE protection and not necessarily to kill or hit the purpose — whether homicidal or not — of a third
WON the death of the victim comes within the victim. In any event, while the act may not exempt party in causing the injuries, but only of the fact that
purview of the exception clause of the the triggerman from liability for the damage done, such injuries have been "intentionally" inflicted —
supplementary policy and, hence, exempts the the fact remains that the happening was a pure this obviously to distinguish them from injuries
company from liability accident on the part of the victim. The victim could which, although received at the hands of a third
have been either the policeman or Atty. Ojeda for it party, are purely accidental.
HELD cannot be pretended that the malefactor aimed at - Examples of unintentional:
NO the deceased precisely because he wanted to take >> A gun which discharges while being cleaned and
- Basilio was a watchman of the Manila Auto Supply his life. kills a bystander;
which was a block away from the house of Atty. Disposition Decision set aside >> a hunter who shoots at his prey and hits a
Ojeda where something suspicious was happening person instead;
which caused the latter to ask for help. While at first BIAGTAN v. THE INSULAR LIFE ASSURANCE >> an athlete in a competitive game involving
he declined the invitation of Atty. Ojeda to go with COMPANY, LTD. physical effort who collides with an opponent and
him to his residence to inquire into what was going 44 SCRA 58 fatally injures him as a result.
on because he was not a regular policeman, he later MAKALINTAL; March 29, 1972 - In Calanoc vs. CA: Where a shot was fired and it
agreed to come along when prompted by the traffic turned out afterwards that the watchman was hit in
policeman, and upon approaching the gate of the NATURE the abdomen, the wound causing his death, the
residence he was shot and died. The circumstance Appeal from decision of CFI Pangasinan. Court held that it could not be said that the killing
that he was a mere watchman and had no duty to was intentional for there was the possibility that the
heed the call of Atty. Ojeda should not be taken as a FACTS malefactor had fired the shot to scare the people
capricious desire on his part to expose his life to - Juan Biagtan was insured with Insular for P5k and around for his own protection and not necessarily to
danger considering the fact that the place he was in a supplementary contract “Accidental Death Benefit” kill or hit the victim. A similar possibility is clearly
duty-bound to guard was only a block away. In clause for another P5k if "the death of the Insured ruled out by the facts in this case. For while a single
volunteering to extend help under the situation, he resulted directly from bodily injury effected solely shot fired from a distance, and by a person who was
might have thought, rightly or wrongly, that to know through external and violent means sustained in an not even seen aiming at the victim, could indeed
the truth was in the interest of his employer it being accident . . . and independently of all other causes." have been fired without intent to kill or injure, nine
INSURANCE Page
76

wounds inflicted with bladed weapons at close range negligence or intent of a third party which is usually been adopted by the courts is that an
cannot conceivably be considered as innocent insofar unforeseen and unexpected by the insured. All the accident is an event that takes place without one's
as such intent is concerned. associated words and concepts in the policy plainly foresight or expectation — an event that proceeds
- In Hucthcraft's Ex'r vs. Travelers' Ins. Co. (US exclude the accidental death from the coverage of from an unknown cause, or is an unusual effect of
case): where the insured was waylaid and the policy only where the injuries are self-inflicted or a known case, and therefore not expected.
assassinated for the purpose of robbery, the court attended by some proscribed act of the insured or - An accident is an event which happens without any
rendered judgment for the insurance company and are incurred in some expressly excluded calamity human agency or, if happening through human
held that while the assassination of the insured was such as riot, war or atomic explosion. agency, an event which, under the circumstances, is
as to him an unforeseen event and therefore - The untenability of insurer's claim that the unusual to and not expected by the person to whom
accidental, "the clause of the proviso that excludes insured's death fell within the exception is further it happens. It has also been defined as an injury
the (insurer's) liability, in case death or injury is heightened by the stipulated fact that two other which happens by reason of some violence or
intentionally inflicted by any other person, applies to insurance companies which likewise covered the casualty to the insured without his design, consent,
this case." insured for much larger sums under similar or voluntary co-operation.
Disposition CFI decision reversed. accidental death benefit clauses promptly paid the
benefits thereof to plaintiffs beneficiaries. ISSUE
SEPARATE OPINION WON what happened was an accident
SUN INSURANCE v. CA (LIM)
TEEHANKEE [dissent] 211 SCRA 554 HELD
- Calanoc v. CA is controlling in this case because CRUZ; July 17, 1992 YES
the insurance company wasn’t able to prove that the - The petitioner, however, cites one of the four
killing was intentional. (Burden of proof is with the FACTS exceptions provided for in the insurance contract and
insurance company) - The petitioner issued Personal Accident Policy to contends that the private petitioner's claim is barred
- Insurance, being contracts of adhesion, must be Felix Lim, Jr. with a face value of P200,000.00. Two by such provision. It is there stated:
construed strictly against insurance company in months later, he was dead with a bullet wound in his Exceptions —The company shall not be liable in
cases of ambiguity. head. As beneficiary, his wife Nerissa Lim sought respect of.
- The supplementary contract enumerated payment on the policy but her claim was rejected. 1. Bodily injury.
exceptions. The only exception which is not The petitioner agreed that there was no suicide. It xxx xxx xxx
susceptible of classification is that provided in argued, however, that there was no accident either. b. consequent upon.
paragraph 5(e), the very exception herein involved, Pilar Nalagon, Lim's secretary, was the only i) The insured persons attempting to commit
which would also except injuries "inflicted eyewitness to his death. According to Nalagon, Lim suicide or wilfully exposing himself to needless peril
intentionally by a third party, either with or without was in a happy mood (but not drunk) and was except in an attempt to save human life.
provocation on the part of the insured, and whether playing with his handgun, from which he had - To repeat, the parties agree that Lim did not
or not the attack or the defense by the third party previously removed the magazine. As she watched commit suicide. Nevertheless, the petitioner
was caused by a violation of the law by the insured." the television, he stood in front of her and pointed contends that the insured willfully exposed himself to
- This ambiguous clause conflicts with all the other the gun at her. She pushed it aside and said it might needless peril and thus removed himself from the
four exceptions in the same paragraph 5 particularly be loaded. He assured her it was not and then coverage of the insurance policy. That posture is
that immediately preceding it in item (d) which pointed it to his temple. The next moment there was arguable. But what is not is that, as the secretary
excepts injuries received where the insured has an explosion and Lim slumped to the floor. He was testified, Lim had removed the magazine from the
violated the law or provoked the injury, while this dead before he fell. gun and believed it was no longer dangerous. He
clause, construed as the insurance company now - The term "accident" has been defined as follows: expressed assured her that the gun was not loaded.
claims, would seemingly except also all other The words "accident" and "accidental" have never It is submitted that Lim did not willfully expose
injuries, intentionally inflicted by a third party, acquired any technical signification in law, and himself to needless peril when he pointed the gun to
regardless of any violation of law or provocation by when used in an insurance contract are to be his temple because the fact is that he thought it was
the insured, and defeat the very purpose of the construed and considered according to the not unsafe to do so. The act was precisely intended
policy of giving the insured double indemnity in case ordinary understanding and common usage and to assure Nalagon that the gun was indeed harmless.
of accidental death by "external and violent means" speech of people generally. In substance, the Disposition CA Affirmed
— in the very language of the policy.' courts are practically agreed that the words
- It is obvious from the very classification of the "accident" and "accidental" mean that which DE LA CRUZ v. CAPITAL INSURANCE
exceptions and applying the rule of noscitus a sociis, happens by change or fortuitously, without 17 SCRA 554
that the double-indemnity policy covers the insured intention or design, and which is unexpected, BARRERA; June 30, 1966
against accidental death, whether caused by fault, unusual, and unforeseen. The definition that has
INSURANCE Page
77

NATURE natural result of the insured's voluntary act, - Eastern Shipping Lines, Inc. loaded on board SS
Appeal from the decision of the CFI of Pangasinan unaccompanied by anything unforeseen except the Eastern Explorer in Kobe, Japan, the following
death or injury. There is no accident when a shipment for carriage to Manila and Cebu, freight
FACTS deliberate act is performed unless some additional, pre-paid and in good order and condition: (a) 2
- Eduardo de la Cruz, employed in the Itogon-Suyoc unexpected, independent, and unforeseen happening boxes internal combustion engine parts, consigned to
Mines, Inc., was the holder of an accident insurance occurs which produces or brings about the result of William Lines, Inc.; (b) 10 metric tons (334 bags)
policy underwritten by the Capital Insurance & injury or death. In other words, where the death or ammonium chloride, consigned to Orca's Company;
Surety Co., Inc., for the period beginning November injury is not the natural or probable result of the (c) 200 bags Glue 300, consigned to Pan Oriental
13, 1956 to November 12, 1957. insured's voluntary act, or if something unforeseen Match Company; and (d) garments, consigned to
- On January 1, 1957, the Itogon-Suyoc Mines, Inc. occurs in the doing of the act which produces the Ding Velayo. All consignations were made by virtue
sponsored a boxing contest wherein the insured injury, the resulting death is within the protection of of a Bill of Lading.
Eduardo de la Cruz participated. policies insuring against death or injury from - While the vessel was off Okinawa, a small flame
- In the course of his bout, Eduardo slipped and was accident. was detected on the acetylene cylinder located in the
hit by his opponent on the left part of the back of the - In the present case, while the participation of the accommodation area near the engine room. As the
head, causing Eduardo to fall, with his head hitting insured in the boxing contest is voluntary, the injury crew was trying to extinguish the fire, the cylinder
the rope of the ring. was sustained when he slid, giving occasion to the suddenly exploded, thus causing death and severe
- He was brought to the Baguio General Hospital, but infliction by his opponent of the blow that threw him injuries to the crew and instantly setting fire to the
he died as a result of hemorrhage, intracranial, left. to the ropes of the ring. whole vessel.
- Simon de la Cruz, the father and named beneficiary - The fact that boxing is attended with some risks of - SS Eastern Explorer was then found to be a
of the insured, filed a claim with the insurance external injuries does not make any injuries received constructive total loss and its voyage was declared
company for payment of the indemnity, but it was in the course of the game not accidental. In boxing abandoned.
denied. as in other equally physically rigorous sports, such - After the fire was extinguished, the cargoes which
- He instituted the action in the CFI of Pangasinan as basketball or baseball, death is not ordinarily were saved were loaded to another vessel for
for specific performance. anticipated to result. If, therefore, it ever does, the delivery to their original ports of destination. ESLI
- Defendant insurer set up the defense that the injury or death can only be accidental or produced by charged the consignees several amounts
death of the insured, caused by his participation in a some unforeseen happening or event as what corresponding to additional freight and salvage
boxing contest, was not accidental and, therefore, occurred in this case. charges.
not covered by insurance - Furthermore, the policy involved herein specifically - The charges were all paid by Philippine Home
- The court rendered the decision in favor of the excluded from its coverage: Assurance Corporation (PHAC) under protest for and
plaintiff, hence, the present appeal. (e) Death or disablement consequent upon the in behalf of the consignees. PHAC, as subrogee of
Insured engaging in football, hunting, pigsticking, the consignees, then filed a complaint before the
ISSUE steeplechasing, polo-playing, racing of any kind, RTC of Manila, against ESLI to recover the sum paid
WON the death of the insured was not accidental mountaineering, or motorcycling. under protest on the ground that the same were
and, therefore, not covered by insurance - Death or disablement resulting from engagement in actually damages directly brought about by the fault,
HELD boxing contests was not declared outside of the negligence, illegal act and/or breach of contract of
NO protection of the insurance contract. Failure of the ESLI.
- The terms "accident" and "accidental", as used in defendant insurance company to include death - ESLI contended that it exercised the diligence
insurance contracts, have not acquired any technical resulting from a boxing match or other sports among required by law in the handling, custody and carriage
meaning, and are construed by the courts in their the prohibitive risks leads inevitably to the of the shipment; that the fire was caused by an
ordinary and common acceptation. Thus, the terms conclusion that it did not intend to limit or exempt unforeseen event; that the additional freight charges
have been taken to mean that which happen by itself from liability for such death. are due and demandable pursuant to the Bill of
chance or fortuitously, without intention and design, Disposition The decision appealed from is affirmed Lading; and that salvage charges are properly
and which is unexpected, unusual, and unforeseen. collectible under Act No. 2616, known as the Salvage
An accident is an event that takes place without FORTUNE INSURANCE v. CA (supra p.7) Law.
one's foresight or expectation, an event that - RTC: dismissed PHAC's complaint and ruled in
proceeds from an unknown cause, or is an unusual PHIL HOME ASSURANCE CORP v. CA (EASTERN favor of ESLI.
effect of a known cause and, therefore, not SHIPPING) - The burning of the vessel was not the fault or
expected. 257 SCRA 468 negligence of defendant but a natural disaster or
- The generally accepted rule is that, death or injury KAPUNAN; June 20, 1996 calamity. Salvage operations conducted by Fukuda
does not result from accident or accidental means Salvage Company was perfectly a legal operation
within the terms of an accident-policy if it is the NATURE and charges made on the goods recovered were
INSURANCE Page
78

legitimate charges. Section 19 of Act No. 2616, the disaster. On the contrary, there is strong evidence FACTS
Salvage Law is applicable. With respect to the indicating that the acetylene cylinder caught fire - Philamlife, a domestic life insurance corp., and
additional freight charged by defendant from the because of the fault and negligence of respondent American International Reinsurance Company
consignees of the goods, the same are also validly ESLI, its captain and its crew: (Airco), a corporation organized under the laws of
demandable. (1) The acetylene cylinder which was fully loaded the Republic of Panama, entered into a
- The burning of "EASTERN EXPLORER" while off should not have been stored near the engine room REINSURANCE TREATY wherein Philamlife agrees to
Okinawa rendered it physically impossible for where the heat generated therefrom could cause the reinsure with Airco on January 1950. Philamlife
defendant to comply with its obligation of delivering acetylene cylinder to explode by reason of agreed to pay premiums for all reinsurances on an
the goods to their port of destination pursuant to the spontaneous combustion. ESLI should have easily annual premium basis.
contract of carriage. Under Article 1266 of the Civil foreseen that the acetylene cylinder, containing - In July 16, 1959, the Margin Law was approved
Code, the physical impossibility of the prestation highly inflammable material, was in a real danger of and became effective, which exempts certain
extinguished defendant's obligation. exploding. “obligations from payment of margin fees,
- Note: The goods subject of the present (2) ESLI should have known that by storing the particularly contractual obligations calling for
controversy were neither lost nor damaged in transit acetylene cylinder in the accommodation area payment of foreign exchange issued, approved and
by the fire that razed the carrier. In fact, these were supposed to be reserved for passengers, it outstanding as of the date this Act takes place”.
all delivered to the consignees, even if the unnecessarily exposed its passengers to grave - Central Bank of the Philippines collected
transshipment took longer than necessary. What is danger and injury. P268,747.48 as foreign exchange margin on
at issue therefore is not whether or not the carrier is (3) The fact that the acetylene cylinder was checked, Philamlife remittances to Airco purportedly totalling
liable for the loss, damage, or deterioration of the tested and examined and subsequently certified as $610,998.63 and made subsequent to July 16, 1959.
goods transported by them but who, among the having complied with the safety measures and Philamlife filed a claim for refund on the ground that
carrier, consignee or insurer of the goods, is liable standards by qualified experts before it was loaded the reinsurance premiums remitted were paid in
for the additional charges or expenses incurred by in the vessel only shows to a great extent that pursuant to the January 1950 reinsurance treaty,
the owner of the ship in the salvage operations and negligence was present in the handling of the and therefore exempted.
in the transshipment of the goods via a different acetylene cylinder after it was loaded and while it - Monetary Board exempted Philamlife from payment
carrier. In absolving respondent carrier of any was on board the ship. of margin fee. However, Auditor of CB refused to
liability, CA sustained the trial court's finding that the - From the foregoing premises, it indubitably follows pass in audit Philamlife’s claim for refund. Philamlife
fire that gutted the ship was a natural disaster or that the cargo consignees cannot be made liable to sought reconsideration but was denied, saying
calamity. respondent carrier for additional freight and salvage reinsurance treaty NOT EXEMPTED.
charges.
ISSUE Disposition Judgment appealed from is REVERSED ISSUES
WON the burning of the SS Eastern Explorer and SET ASIDE. Respondent Eastern Shipping Lines, 1. WON the premia remitted were in pursuance of
rendering it a constructive total loss was a natural Inc. is ORDERED to return to petitioner Philippine the reinsurance treaty between Philamlife and Airco
disaster or calamity Home Assurance Corporation the amount it paid of January 1959, a contract antedating the Margin
under protest in behalf of the consignees herein. Law, and therefore, Philamlife exempted from paying
HELD margin fee
NO 2. WON Margin Law impairs the obligation of
Ratio In our jurisprudence, fire may not be contract
considered a natural disaster or calamity since it 3. WON reinsurance contracts abroad would be made
almost always arises from some act of man or by impractical by the imposition of the 25% margin fee
human means. It cannot be an act of God unless
caused by lightning or a natural disaster or casualty HELD
not attributable to human agency. 1. NO
Reasoning PHILIPPINE AMERICAN LIFE INSURANCE - For an exemption to come into play, there must be
- There was no showing, and none was alleged by COMPANY v. THE AUDITOR GENERAL a reinsurance policy or, as in the reinsurance treaty
the parties, that the fire was caused by a natural 22 SCRA 135 provided, a "reinsurance cession" which may be
SANCHEZ, JANUARY 18, 1968 automatic or facultative.
9
Section 1. When in case of shipwreck, the vessel or its cargo shall Ratio A reinsurance policy is thus a contract of
be beyond the control of the crew, or shall have been abandoned by NATURE indemnity one insurer makes with another to protect
them, and picked up and conveyed to a safe place by other persons,
the latter shall be entitled to a reward for the salvage.
PETITION FOR REVIEW of a ruling of the Auditor the first insurer from a risk it has already assumed. .
Those who, not being included in the above paragraph, assist in saving General. . . In contradistinction, a reinsurance treaty is
a vessel or its cargo from shipwreck, shall be entitled to like reward. merely an agreement between two insurance
INSURANCE Page
79

companies whereby one agrees to cede and the with an additional instrument for effectively coping received by ASIAN on Sep. 25, 1961, did not elicit
other to accept reinsurance business pursuant to with the problem and achieving domestic and any reply from ASIAN.
provisions specified in the treaty. The practice of international stability of our currency; to reduce the - On Dec. 7, 1961 FIELDMEN'S sent another letter to
issuing policies by insurance companies includes, excessive demand-for foreign exchange. ASIAN expressing regrets at alleged violations
among other things, the issuance of reinsurance - implementation of Margin Law in accordance with committed by the latter with respect to the various
policies on standard risks and also on substandard police power agreements between them and reiterated its position
risks under special arrangements. The lumping of the 3. NO that it would consider itself "no longer at risk for any
different agreements under a contract has resulted Reasoning reinsurance and/or cession" given by ASIAN which
in the term known to the insurance world as - First, there is no concrete evidence that such might be in force on Dec. 31, 1961. Not having
'treaties.' Such a treaty is, in fact, an agreement imposition of the 25% margin fee is unreasonable, received any formal reply from ASIAN, FIELDMEN'S
between insurance companies to cover the different Second, if really continuance of the existing sent a new a letter on Feb. 17, 1962 reminding
situations described. Reinsurance treaties and reinsurance treaty becomes unbearable, that ASIAN of the cancellation of all the reinsurance
reinsurance policies are not synonymous. Treaties contract itself provides that petitioner may treaties and cessions as of Dec. 31, 1961 and
are contracts for insurance; reinsurance potestatively write finis thereto on ninety days' requested ASIAN to submit its final accounting of all
policies or cessions are contracts of insurance. written notice. Petitioner is not forced to continue cessions made to the former for the preceding
Reasoning its reinsurance treaty indefinitely with Airco. months when the reinsurance agreements were in
- Even if reinsurance treaty preceded the Margin Law Disposition For the reasons given, the petition for force.
by over nine years, nothing in the treaty obligates review is hereby denied, and the ruling of the - Meanwhile one of the risks reinsured with
Philamlife to remit to Airco a fixed, certain, and Auditor General of October 24, 1961 denying refund FIELDMEN'S issued in favor of the GSIS became a
obligatory sum by way of reinsurance premiums. The is hereby affirmed. liability when the insured property was burned on
reinsurance treaty per se cannot give rise to a Costs against petitioner. So ordered. February 16, 1962. Since the policy was issued on
contractual obligation for the payment of foreign July 1, 1961, it was supposed to expire on July 1,
exchange. Philamlife’s obligation to remit FIELDMEN'S INSURANCE CO INC v. ASIAN 1962. 2 The next day, Feb. 17, ASIAN immediately
reinsurance premiums becomes fixed and definite SURETY & INSURANCE CO INC notified FIELDMEN'S of said fire loss.
upon the execution of the reinsurance cession. 34 SCRA 36 - FIELDMEN'S, relying on the sufficiency of its notice
It is only after a reinsurance cession is made that MAKALINTAL; July 31, 1970 of termination dated September 19, 1961 and
payment of reinsurance premium may be exacted, as obviously bent on avoiding its liability under the
it is only after Philamlife seeks to remit that FACTS reinsurance agreements with ASIAN, filed a petition
reinsurance premium that the obligation to pay the - On various dates between April 11, 1960 and Jan. for declaratory relief with the CFI of Manila to seek a
margin fee arises. 9, 1961 the Asian Surety & Insurance Company, declaration that all the reinsurance contracts entered
2. NO Inc. and the Fieldmen's Insurance Company, Inc. into between them had terminated as of December
Ratio. Existing laws form part of the contract "as the entered into 7 reinsurance agreements under which 31, 1961 and to obtain an order directing ASIAN to
measure of the obligation to perform them by the the former, as the ceding company undertook to render final accounting of the transactions between
one party and the right acquired by the other. If the cede to the latter, as the reinsuring company, a them with respect to said reinsurance treaties as of
obligation does not inhere and subsist in the specified portion of the amount of insurance the cut-off date.
contract itself, propio vigore, but in the law underwritten by ASIAN upon payment to - In its answer below ASIAN denied having received
applicable to the contract. FIELDMEN'S of a proportionate share of the gross FIELDMEN'S letter dated Sep 19, 1961, and argued
rate of the premium applicable with respect to each that even assuming it did, FIELDMEN'S could not
cession after deducting a commission. Said have terminated the reinsurance treaties as of Dec
agreements were to take effect from certain specific 31, 1961 because the letter was merely an
Reasoning dates and were to be in force until cancelled by expression of FIELDMEN'S desire to cancel the
- . When petitioner entered into the reinsurance either party upon previous notice of at least 3 treaties and not a formal notice of cancellation as
treaty of January 1, 1950 with Airco, it did so with months by registered mail to the other party, the contemplated in their reinsurance agreements. By
the understanding that the municipal laws of the cancellation to take effect as of Dec. 31 of the year way of special defense Asian contended that even if
Philippines at the time said treaty was executed, in which the notice was given. the Sep. 19 letter were considered sufficient notice
became an unwritten condition thereof. Such - On Sep. 19, 1961 FIELDMEN'S, by means of of cancellation — thereby rendering the reinsurance
municipal laws constitute part of the obligation of registered mail, served notice to ASIAN of the agreements terminated as of December 31, 1961 —
contract. former's desire to be relieved from all participation in the liability of FIELDMEN'S with respect to policies or
-Rationale of Margin Law: to reduce the excessive its various agreements with the latter effective Dec. cessions issued under two of the said agreements
demand on and prevent further decline of our 31, 1961. This communication, although admittedly prior to their cancellation continued to have full force
international reserves; to provide the Central Bank
INSURANCE Page
80

and effect until the stated expiry dates of such it ipso facto the termination of all reinsurance arbitrators. The Court denied the motion and
policies or cessions. cessions thereunder. Such cessions continued to be required defendant to answer.
- On Dec. 4, 1962, the trial court declared 6 of the 7 in force until their respective dates of expiration. - Defendant filed its answer, alleging that the nature
reinsurance agreements in question cancelled as of Since it was under one of said agreements that the of the agreement is “self-liquidating between the
Dec 31, 1961. At the same time, it upheld ASIAN'S reinsurance cession corresponding to the GSIS policy parties”, the reinsurer becoming the reinsured and
position that all cessions of reinsurance made by it to had been made, FIELDMEN'S cannot avoid liability vice versa; and that said agreement has not yet
FIELDMEN'S prior to the cancellation of the which arose by reason of the burning of the insured been abrogated so the liability of either to the other
reinsurance treaties continued in full force and effect property. is not yet known. Defendant prayed that the
until expiry dates and ordered FIELDMEN'S to make - With respect to the other 4 agreements, it would complaint be dismissed and plaintiff filed a motion
an accounting of its business transactions with seem that the petition for declaratory relief is moot, for judgment on the pleadings which the court
ASIAN within 30 days. and that no useful purpose would be served by denied.
- On appeal to the CA, the decision of the trial court defining the respective rights and obligations of the - Instead of going into a formal hearing, the parties
was substantially affirmed, with the slight parties thereunder. The said agreements have been submitted their case for decision stipulating the ff
modification that the order for accounting was cancelled, and it does not appear that any claim by facts: defendant admits the allegations of the
eliminated, without prejudice to the filing of a proper or liability in favor of the insured has actually arisen complaint and that plaintiff admits that the issues of
action between the parties for that purpose. under any of the reinsurance cessions made prior to the complaint were not submitted to a Board of
such cancellation. Future conflicts of the same nature Arbitrators as provided in par VIII of the complaint,
ISSUE as those which have motivated the present action but instead referred it to the Insurance
WON the cancellation as of Dec. 31, 1961 of the can of course be obviated by using more precise and Commissioner. The CFI rendered judgment in favor
reinsurance treaties had the effect of terminating definite terminology in the reinsurance agreements of plaintiff. Hence this appeal.
also the liability of FIELDMEN'S as reinsurer with which the parties may enter into henceforth.
respect to policies or cessions issued prior to the ISSUES
termination of the principal reinsurance contracts or EQUITABLE INSURANCE v. RURAL INSURANCE 1. WON Equitable had no cause of action as the
treaties 4 SCRA 343 matter was not referred to the decision of arbitrators
BARRERA; January 31, 1962 2. WON in a facultative obligation the right to choose
HELD an alternative remedy lies only with the debtor (here
NO to the 2 reinsurance contracts FACTS the defendant) under Art 1206
- Of the 6 reinsurance contracts, 2 contain - Plaintiff Equitable Insurance file a complaint with
provisions, which clearly and expressly recognize the the CFI of Manila against defendant Rural Insurance HELD
continuing effectivity of policies ceded under them alleging, as first cause of action, that they entered 1. NO
for reinsurance notwithstanding the cancellation of into a reciprocal facultative reinsurance agreement, - The requirement of submission for decision to 2
the contracts themselves. The said treaties provide wherein they agreed to cede to each other. Pursuant arbitrators or an umpire the matter of losses by fire
"that in the event of termination of this to said agreement, plaintiff reinsured for P2k with or the liability of the parties thereto under Art VIII of
Agreement . . ., the liability of the Fieldmen's under defendant the stock covered by fire insurance Policy the agreement arises only if the same is disputed by
current cessions shall continue in full force and effect No. 5880 issued by plaintiff which was later burned; one of the parties. In the instant case, there is no
until their natural expiry . . .;" and the 4th the share of the loss of defendant as per insurance dispute between the parties; in the stipulation of
paragraph of Article VI of the Personal Accident agreement was computed at P2,024 for which facts defendant admitted that plaintiff had paid its
Reinsurance Treaty states: plaintiff sent to defendant a statement of account for liability and defendant likewise admitted that it
"4. On the termination of this Agreement from payment by the latter. Despite repeated demands ignored plaintiff’s demands for reimbursement for
any cause whatever, the liability of the by plaintiff, defendant refused to pay. defendant’s failure to pay its share as reinsurer. As
REINSURER (Fieldmen's) under any current - On the second cause of action, plaintiff reinsured held in Maligad v United Assurance Co., if in the
cession including any amounts due to be ceded for P2k with defendant stock covered by fire course of the settlement of a loss, the action of the
under the terms of this Agreement and which are insurance Policy No. 6062 which also burned. Again, company or its agents amounts to refusal to pay, the
not cancelled in the ordinary course of business defendant refused to pay its share of the loss of company will be deemed to have waived the
shall continue in full force until their expiry unless P1,334 hence said complaint. condition precedent with reference to arbitration and
the COMPANY (Asian) shall, prior to the thirty-first - Defendant filed a motion to dismiss on the ground a suit upon the policy will lie.
December next following such notice, elect to that it states no cause of action, as pursuant to Art 2. NO
withdraw the existing cessions . . ." VIII of the Reinsurance Agreement between the - There is no connection between Art 1206 NCC and
- Thus, insofar as the two reinsurance agreements parties, before a court action can be brought, the the agreement of this action. The term “facultative”
are concerned, there is clearly no merit in parties agreed to submit all disputes to a board of is used in reinsurance contracts, and it is so used in
FIELDMEN'S claim that their cancellation carried with this particular case, merely to define the right of the
INSURANCE Page
81

reinsurer to accept or not to accept participation in - With regard the balance unpaid, Wellington Anti-Carnapping Unit of the Philippine Constabulary
the risk insured. But once the share is accepted, as it contends that Artex should have been directed to report said incident and thereafter, went to the
was in the case at bar, the obligation is absolute and against the reinsurers to cover the liability and not nearest police substation to make a police report
the liability assumed thereunder can be discharged against Wellington. regarding said incident.
by only one way—the payment of the share of the - On Nov10, 1982, Evelyn Lim reported said incident
losses. ISSUE to the LTO in compliance with the insurance
Disposition judgment appealed from the TC is WON the insured (Artex) has a cause of action requirement. She also filed a complaint with the
affirmed against the reinsurer Headquarters. Constabulary Highway Patrol Group.
- On Nov11, 1982, private respondent filed a claim
ARTEX DEVELOPMENT CO INC v. WELLINGTON HELD for loss with the petitioner Perla but said claim was
INSURANCE CO INC NO denied on Nov18, 1982 on the ground that Evelyn
51 SCRA 352 - Unless there is a specific grant in, or assignment Lim, who was using the vehicle before it was
TEEHANKEE; June 27, 1973 of, the reinsurance contract in favor of the insured or carnapped, was in possession of an expired driver's
a manifest intention of the contracting parties to the license at the time of the loss of said vehicle which is
FACTS reinsurance contract to grant such benefit or favor to in violation of the authorized driver clause of the
- Wellington Insurance Co. Inc. insured for the insured, the insured, not being privy to the insurance policy, which states, to wit:
P24,346,509.00 the buildings, stocks and machinery reinsurance contract, has no cause of action against "AUTHORIZED DRIVER:
of plaintiff Artex Development Co. Inc. against loss the reinsurer. It is expressly provided in Section 91 Any of the following: (a) The Insured (b) Any
or damage by fire or lighting upon payment of the the Insurance Act 1 that "(T)he original insured has person driving on the Insured's order, or with his
plaintiff of the corresponding premiums; that said no interest in a contract of insurance." permission. Provided that the person driving is
properties were insured for an additional sum of permitted, in accordance with the licensing or
P883,034.00; that defendant insured plaintiff against PERLA COMPANIA DE SEGUROS v. CA(LIM) other laws or regulations, to drive the Scheduled
business interruption (use and occupancy) for 208 SCRA 487 Vehicle, or has been permitted and is not
P5,200,000.00; Wellington entered into a contract NOCON; May 7, 1992 disqualified by order of a Court of Law or by
of reinsurance with Alexander and Alexander, Inc. of reason of any enactment or regulation in that
New York. USA. NATURE behalf."
- The buildings, stocks and machineries of plaintiffs Petition for certiorari by Perla Compania de Seguros - On Nov17, 1982, private respondents requested
spinning department were burned. and FOC Credit Corporation seeking to annul and set from petitioner FCP for a suspension of payment on
- Notice of the loss and damage was given the aside CA decision revering the RTC decision for the monthly amortization agreed upon due to the
defendant; that as per report of the adjusters, the replevin and damages. loss of the vehicle and, since the carnapped vehicle
total property loss of the plaintiff was the sum of was insured with petitioner Perla, said insurance
P10,106,554.40 and the total business interruption FACTS company should be made to pay the remaining
loss was P3,000,000.00; - Private respondents spouses Herminio and Evelyn balance of the promissory note and the chattel
- That defendant has paid to the plaintiff the sum of Lim executed a promissory note in favor of mortgage contract.
P6,481,870.07 of the property loss suffered by Supercars, Inc. in the sum of P77,940.00, payable in - Perla, however, denied private respondents' claim.
plaintiff and P1,864,134.08 on its business monthly installments according to the schedule of Consequently, petitioner FCP demanded that private
interruption loss, leaving a balance of P3,624,683.43 payment indicated in said note, and secured by a respondents pay the whole balance of the
and P1,748,460.00, respectively. chattel mortgage over a brand new red Ford Laser, promissory note or to return the vehicle but the
- The counsel for Artex filed a Manifestation saying which is registered under the name of private latter refused.
that in view of the Deeds of Discharge and Collateral respondent Herminio Lim and insured with the - On July25, 1983, petitioner FCP filed a complaint
Agreement, the only remaining liability subject of petitioner Perla Compania de Seguros, Inc. (Perla for against private respondents, who in turn filed an
litigation shall be the proportion of the loss reinsured brevity) for comprehensive coverage. amended third party complaint against petitioner
with or through Alexander and Alexander, Inc. of - On the same date, Supercars, Inc., with notice to Perla on Dec8, 1983. After trial on the merits, TC
New York, USA, namely, P397,813.00. private respondents spouses, assigned to petitioner ordered sps Lim to pay jointly and severally, plaintiff
- The document recited further that Artex FCP Credit Corporation (FCP for brevity) its rights, the sum of P55,055.93 plus interest thereon at the
acknowledges receipt of the sum of P3.6M paid by title and interest on said promissory note and chattel rate of 24% per annum from July 2, 1983 until fully
the insurer in full and final settlement of all or any mortgage as shown by the Deed of Assignment. paid; as well as the cost of suit. It also ordered the
claims of Artex against its insurer. It discharges its - At around 2:30pm Nov9, 1982, said vehicle was dismissal of the Third party complaint against Third-
insurer from all actions, proceedings, claims, carnapped while parked at the back of Broadway Party Defendant.
demands, costs and expenses in respect thereof. Centrum. Evelyn Lim, who was driving said car - Upon appeal, CA reversed said decision
before it was carnapped, immediately called up the
INSURANCE Page
82

- After petitioners' separate MFRs were denied by - There is no causal connection between the mortgage contract. If the claim on the insurance
CA, petitioners filed these separate petitions for possession of a valid driver's license and the loss of a policy had been approved by petitioner Perla, it
review on certiorari. vehicle. To rule otherwise would render car insurance would have paid the proceeds thereof directly to
ISSUE practically a sham since an insurance company can petitioner FCP, and this would have had the effect of
1. WON there was grave abuse of discretion on the easily escape liability by citing restrictions which are extinguishing private respondents' obligation to
part of the appellate court in holding that private not applicable or germane to the claim, thereby petitioner FCP. Therefore, private respondents were
respondents did not violate the insurance contract reducing indemnity to a shadow. justified in asking petitioner FCP to demand the
because the authorized driver clause is not 2. The court agrees with FCP that Lim spouses are unpaid installments from petitioner Perla.
applicable to the "Theft" clause of said Contract not relieved of their obligation to pay the former the - Because petitioner Perla had unreasonably denied
2. WON the loss of the collateral exempted the installments due on the promissory note on account their valid claim, private respondents should not be
debtor from his admitted obligations under the of the loss of the automobile. The chattel mortgage made to pay the interest, liquidated damages and
promissory note particularly the payment of interest, constituted over the automobile is merely an attorney's fees as stipulated in the promissory note.
litigation expenses and attorney's fees accessory contract to the promissory note. Being the As mentioned above, the contract of indemnity was
principal contract, the promissory note is unaffected procured to insure the return of the money loaned
HELD by whatever befalls the subject matter of the from petitioner FCP, and the unjustified refusal of
1. NO accessory contract. petitioner Perla to recognize the valid claim of the
- The comprehensive insurance policy issued by - The unpaid balance on the promissory note should private respondents should not in any way prejudice
petitioner Perla undertook to indemnify the private be paid, and not just the installments due and the latter.
respondents against loss or damages to the car (a) payable before the automobile was carnapped, as - Private respondents can not be said to have unduly
by accidental collision or overturning, or collision or erronously held by the CA enriched themselves at the expense of FCP since
overturning consequent upon mechanical breakdown - However, this does not mean that private they will be required to pay the latter the unpaid
or consequent upon wear and tear; (b) by fire, respondents are bound to pay the interest, litigation balance of its obligation under the promissory note.
external explosion, self-ignition or lightning or expenses and attorney's fees stipulated in the - In view of the foregoing discussion, We hold that
burglary, housebreaking or theft; and (c) by promissory note. Because of the peculiar relationship the Court of Appeals did not err in requiring
malicious act. between the three contracts in this case, i. e., the petitioner Perla to indemnify private respondents for
- Where a car is unlawfully and wrongfully taken promissory note, the chattel mortgage contract and the loss of their insured vehicle. However, the latter
without the owner's consent or knowledge, such the insurance policy, the Court is compelled to should be ordered to pay petitioner FCP the amount
taking constitutes theft, and, therefore, it is the construe all three contracts as intimately interrelated of P55,055.93, representing the unpaid installments
"THEFT" clause, and not the "AUTHORIZED DRIVER" to each other, despite the fact that at first glance from December 30, 1982 up to July 1, 1983, as
clause, that should apply.The risk against accident is there is no relationship whatsoever between the shown in the statement of account prepared by
distinct from the risk against theft. The 'authorized parties thereto. petitioner FCP, 18 plus legal interest from July 2,
driver clause' in a typical insurance policy as in - Under the promissory note, Lim spouses are 1983 until fully paid.
contemplation or anticipation of accident in the legal obliged to pay Supercars, Inc. the amount stated - As to the award of moral damages, exemplary
sense in which it should be understood, and not in therein in accordance with the schedule provided for. damages and attorney's fees, private respondents
contemplation or anticipation of an event such as To secure said promissory note, private respondents are legally entitled to the same since Perla had acted
theft. The distinction often seized upon by insurance constituted a chattel mortgage in favor of Supercars, in bad faith by unreasonably refusing to honor the
companies in resisting claims from their assureds Inc. over the automobile the former purchased from insurance claim of the private respondents. Besides,
between death occurring as a result of accident and the latter. The chattel mortgage, in turn, required awards for moral and exemplary damages, as well as
death occurring as a result of intent may apply to the private respondents to insure the automobile and to attorney's fees are left to the sound discretion of the
case at bar. make the proceeds thereof payable to Supercars, Court. Such discretion, if well exercised, will not be
- If the insured vehicle had figured in an accident at Inc. The promissory note and chattel mortgage were disturbed on appeal.
the time she drove it with an expired license, then, assigned by Supercars, Inc. to petitioner FCP, with Disposition the assailed decision of the CA is
appellee Perla Compania could properly resist the knowledge of private respondents. Private hereby MODIFIED to require private respondents to
appellants' claim for indemnification for the loss or respondents were able to secure an insurance policy pay petitioner FCP the amount of P55,055.93, with
destruction of the vehicle resulting from the from petitioner Perla, and the same was made legal interest from July 2, 1983 until fully paid. The
accident. But in the present case, the loss of the specifically payable to petitioner FCP. decision appealed from is hereby affirmed as to all
insured vehicle did not result from an accident where - From the abovementioned provision that upon the other respects. No pronouncement as to costs.
intent was involved; the loss in the present case was loss of the insured vehicle, the insurance company
caused by theft, the commission of which was Perla undertakes to pay directly to the mortgagor or SHAFER v. JUDGE
attended by intent." to their assignee, FCP, the outstanding balance of 167 SCRA 386
the mortgage at the time of said loss under the PADILLA; November 14, 1988
INSURANCE Page
83

YES directly against liability, the insurer's liability accrues


NATURE - There is no need on the part of the insured to wait immediately upon the occurrence of the injury or
Petition for review on certiorari for the decision of the trial court finding him guilty of event upon which the liability depends, and does not
reckless imprudence. The occurrence of the injury to depend on the recovery of judgment by the injured
FACTS the third party immediately gave rise to the liability party against the insured.
- Sherman Shafer obtained a private car policy over of the insurer under its policy. Respondent - The injured for whom the contract of insurance is
his Ford Laser from Makati Insurance Company, Inc., insurance company's contention that the third party intended can sue directly the insurer. The general
for third party liability. During the effectivity of the complaint involves extraneous matter which will only purpose of statutes enabling an injured person to
policy, an information for reckless imprudence clutter, complicate and delay the criminal case is proceed directly against the insurer is to protect
resulting in damage to property and serious physical without merit. The civil aspect of the offense injured persons against the insolvency of the insured
injuries was filed against shafer. The information charged, i.e., serious physical injuries allegedly who causes such injury, and to give such injured
said that on or about the 17th day of May 1985, in suffered by Jovencio Poblete, Sr., was impliedly person a certain beneficial interest in the proceeds of
the City of Olongapo. Shafer hit and bumped a instituted with the criminal case. Petitioner may thus the policy, and statutes are to be liberally construed
Volkswagen car owned and driven by Felino llano y raise all defenses available to him insofar as the so that their intended purpose may be accomplished.
Legaspi, thereby causing damage in the total amount criminal and civil aspects of the case are concerned. It has even been held that such a provision creates a
of P12,345.00 and as a result thereof one Jovencio The claim of petitioner for payment of indemnity to contractual relation which inures to the benefit of
Poblete, Sr. who was on board of the said the injured third party, under the insurance policy, any and every person who may be negligently
Volkswagen car sustained physical injuries which for the alleged bodily injuries caused to said third injured by the named insured as if such injured
injuries causing deformity on the face. The owner of party, arose from the offense charged in the criminal person were specifically named in the policy.
the damaged Volkswagen car filed a separate civil case, from which the injured (Jovencio Poblete, Sr.) - In the event that the injured fails or refuses to
action against petitioner for damages, while Jovencio has sought to recover civil damages. Hence, such include the insurer as party defendant in his claim
Poblete, Sr., who was a passenger in the Volkswagen claim of petitioner against the insurance company for indemnity against the insured, the latter is not
car, did not reserve his right to file a separate civil cannot be regarded as not related to the criminal prevented by law to avail of the procedural rules
action for damages. Instead, in the course of the action. intended to avoid multiplicity of suits. Not even a "no
trial in the criminal case, Poblete, Sr. testified on his - A third party complaint is a device allowed by the action" clause under the policy-which requires that a
claim for damages for the serious physical injuries rules of procedure by which the defendant can bring final judgment be first obtained against the insured
which he claimed to have sustained as a result of the into the original suit a party against whom he will and that only thereafter can the person insured
accident. have a claim for indemnity or remuneration as a recover on the policy can prevail over the Rules of
- The court issued an order dismissing the third party result of a liability established against him in the Court provisions aimed at avoiding multiplicity of
complaint on the ground that it was premature, original suit. 13 Third party complaints are allowed to suits.
based on the premise that unless the accused minimize the number of lawsuits and avoid the Disposition instant petition is GRANTED. The
(herein petitioner) is found guilty and sentenced to necessity of bringing two (2) or more actions questioned order dated 24 April 1987 is SET ASIDE
pay the offended party (Poblete Sr.) indemnity or involving the same subject matter. They are and a new one entered admitting petitioner's third
damages, the third party complaint is without cause predicated on the need for expediency and the party complaint against the private respondent
of action. The court further stated that the better avoidance of unnecessary lawsuits. If it appears Makati Insurance Company, Inc.
procedure is for the accused (petitioner) to wait for probable that a second action will result if the
the outcome of the criminal aspect of the case to plaintiff prevails, and that this result can be avoided VDA DE MAGLANA v. CONSOLACION
determine whether or not the accused, also the third by allowing the third party complaint to remain, then 212 SCRA 268
party plaintiff, has a cause of action against the third the motion to dismiss the third party complaint ROMERO; August 6, 1992
party defendant for the enforcement of its third party should be denied.
liability (TPL) under the insurance contract. 6 - Compulsory Motor Vehicle Liability Insurance (third NATURE
Petitioner moved for reconsideration of said order, party liability, or TPL) is primarily intended to Petition for certiorari
but the motion was denied; hence, this petition. provide compensation for the death or bodily injuries
suffered by innocent third parties or passengers as a FACTS
ISSUE result of a negligent operation and use of motor - Lope Maglana was an employee of the Bureau of
WON the court a quo erred in dismissing petitioner's vehicles. The victims and/or their dependents are Customs whose work station was at Lasa, here in
third party complaint on the ground that petitioner assured of immediate financial assistance, regardless Davao City. One day, when he was on his way to his
had no cause of action yet against the insurance of the financial capacity of motor vehicle owners. work, he met an accident that resulted in his death.
company - The liability of the insurance company under the He died on the spot.
Compulsory Motor Vehicle Liability Insurance is for - The PUJ jeep that bumped the deceased was driven
HELD loss or damage. Where an insurance policy insures by Pepito Into, operated and owned by defendant
INSURANCE Page
84

Destrajo. From the investigation conducted by the liability of the insurer is secondary only up to the respondents Sio Choy and San Leon Rice Mill, Inc.
traffic investigator, the PUJ jeep was overtaking extent of the insurance coverage." For if petitioner-insurer were solidarily liable with
another passenger jeep that was going towards the - Petitioners filed a second motion for said, two (2) respondents by reason of the
city poblacion. While overtaking, the PUJ jeep of reconsideration reiterating that the liability of the indemnity contract against third party liability under
defendant Destrajo running abreast with the insurer is direct, primary and solidary with the which an insurer can be directly sued by a third party
overtaken jeep, bumped the motorcycle driven by jeepney operator because the petitioners became this will result in a violation of the principles
the deceased. The point of impact was on the lane of direct beneficiaries under the provision of the policy underlying solidary obligation and insurance
the motorcycle and the deceased was thrown from which, in effect, is a stipulation pour autrui. This contracts.
the road and met his untimely death. motion was likewise denied for lack of merit. - While in solidary obligations, the creditor may
- Heirs of Lope Maglana, Sr. filed an action for enforce the entire obligation against one of the
damages and attorney's fees against operator ISSUE solidary debtors, in an insurance contract, the
Patricio Destrajo and the Afisco Insurance WON AFISCO can be held directly liable insurer undertakes for a consideration to indemnify
Corporation (AFISCO). An information for homicide the insured against loss, damage or liability arising
thru reckless imprudence was also filed against HELD from an unknown or contingent event.
Pepito Into. YES - Similarly, petitioners herein cannot validly claim
- During the pendency of the civil case, Into was - As this Court ruled in Shafer vs. Judge, RTC of that AFISCO, whose liability under the insurance
sentenced to suffer an indeterminate penalty, with Olongapo City, Br. 75, "[w]here an insurance policy policy is also P20,000.00, can be held solidarily liable
all the accessory penalties provided by law, and to insures directly against liability, the insurer's liability with Destrajo for the total amount of P53,901.70 in
indemnify the heirs of Lope Maglana, Sr. in the accrues immediately upon the occurrence of the accordance with the decision of the lower court.
amount of twelve thousand pesos with subsidiary injury or even upon which the liability depends, and Since under both the law and the insurance policy,
imprisonment in case of insolvency, plus five does not depend on the recovery of judgment by the AFISCO's liability is only up to P20,000.00, the
thousand pesos in the concept of moral and injured party against the insured." second paragraph of the dispositive portion of the
exemplary damages with costs. No appeal was - The underlying reason behind the third party decision in question may have unwittingly sown
interposed by accused who later applied for liability (TPL) of the Compulsory Motor Vehicle confusion among the petitioners and their counsel.
probation. Liability Insurance is "to protect injured persons What should have been clearly stressed as to leave
- The lower court rendered a decision finding that against the insolvency of the insured who causes no room for doubt was the liability of AFISCO under
Destrajo had not exercised sufficient diligence as the such injury, and to give such injured person a certain the explicit terms of the insurance contract.
operator of the jeepney ordering him to pay plaintiffs beneficial interest in the proceeds of the policy . . ." Disposition present petition is hereby GRANTED.
the sum for loss of income; funeral and burial Since petitioners had received from AFISCO the sum The award of P28,800.00 representing loss of income
expenses of the deceased; moral damages, and of P5,000.00 under the no-fault clause, AFISCO's is INCREASED to P192,000.00 and the death
attorney's fees and costs of suit. The defendant liability is now limited to P15,000.00. indemnity of P12,000.00 to P50,000.00.
insurance company is ordered to reimburse - However, we cannot agree that AFISCO is likewise
defendant Destrajo whatever amounts the latter solidarily liable with Destrajo. In Malayan Insurance
shall have paid only up to the extent of its insurance Co., Inc. v. Court of Appeals, this Court had the
coverage. opportunity to resolve the issue as to the nature of
- Petitioners filed a motion for the reconsideration of the liability of the insurer and the insured vis-a-vis
the second paragraph of the decision contending that the third party injured in an accident. We
AFISCO should not merely be held secondarily liable categorically ruled thus: While it is true that where FAR EASTERN SURETY v. MISA
because the Insurance Code provides that the the insurance contract provides for indemnity against 25 SCRA 663
insurer's liability is "direct and primary and/or jointly liability to third persons, such third persons can REYES; October 26, 1968
and severally with the operator of the vehicle, directly sue the insurer, however, the direct liability
although only up to the extent of the insurance of the insurer under indemnity contracts against NATURE
coverage." Hence, they argued that the P20,000.00 third party liability does not mean that the insurer Appeal by petition for review from a CA judgment
coverage of the insurance policy issued by AFISCO, can be held solidarily liable with the insured and/or
should have been awarded in their favor. the other parties found at fault. The liability of the FACTS
- AFISCO argued that since the Insurance Code insurer is based on contract; that of the insured is - Socorro Dancel Vda.de Misa and Araceli Pinto hired
does not expressly provide for a solidary obligation, based on tort. In the case at bar, petitioner as a taxi cab operated by La Mallorca on September 3,
the presumption is that the obligation is joint. insurer of Sio Choy, is liable to respondent Vallejos 1957. The taxi they were riding in collided with a
- The lower court denied the motion for (the injured third party), but it cannot, as incorrectly gravel and sand truck resulting to injuries to both
reconsideration ruling that since the insurance held by the trial court, be made "solidarily" liable Misa and Pinto.
contract "is in the nature of suretyship, then the with the two principal tortfeasors, namely
INSURANCE Page
85

- The two passengers instituted a suit for damages - In the case of the insurance company, the SC ruled amended her complaint shortly thereafter to include
against La Mallorca who, while denying that it neither authorized nor consented to the Diman & Co. as alternative party plaintiff.
responsibility, instituted a third party complaint representations made by the taxi company to its - Empire's basic defense to the suit was anchored
against Far Eastern Surety to recoup from the latter passengers. As such, the liability of the said on the explicit requirement in the policy limiting the
any award for damages that might be recovered by insurance company based on its insurance contract operation of the insured vehicle to the "authorized
the passengers. is limited to the recovery by the insured of all sums, driver" therein defined, namely, (a) the insured, or
- It would appear from the case that a sticker was cost and expenses which the insured shall become (b) any person driving on the insured order or with
placed in all the taxis of La Mallorca stating that legally liable. The insurance company therefore his permission, provided that-
passengers of the taxis were insured against cannot be held liable for the award. ... that the person driving is permited in
accidents. This was done to entice the public into - The taxi company is adjudged to be the sole party accordance with the licensing or other laws or
patronizing La Mallorca. responsible for the award. regulations to drive the Motor vehicle or has been
- The trial court awarded to Misa and Pinto actual, Disposition The decision of the CA is modified by so permitted and is not disqualified by order of the
moral and exemplary damages, and attorney’s fees eliminating the award against Far Eastern. Court of Law of by reason of any enactment or
payable by La Mallorca and sentenced Far Eastern to regulation in that behalf from driving such Motor
pay La Mallorca P10,000. on its third party liability PEZA v. ALIKPALA Vehicle.-
insurance. 160 SCRA 31 - driver Perfecto Amar, only having a temporary
- On appeal, the CA, while holding that the collision NARVASA; April 15, 1988 operator's permit (TVR) [already expired] his driver’s
was due to the fault of the driver of the gravel and license having earlier been confiscated by an agent
sand truck, found the taxi company liable for NATURE of the Land Transportation Commission for an
damages to the passengers on the strength of its Motion praying that Judge Alikpala be declared guilty alleged violation of Land Transportation and Traffic
representation contained in the sticker above noted of contempt of court for having decided the case on Rules, was not permitted by law and was in truth
that the passengers were insured against accidents. the merits despite the pendency in this Court of the disqualified to operate any motor vehicle; Peza
In so ruling, the CA overruled the defense of the certiorari action instituted by the plaintiffs attempted to neutralize that fact by(1) the issuance
insurance company to the effect that it was of the TVR by the LTC officer to Amar; in proof of the
responsible only if the insured, La Mallorca, was FACTS proposition that there was no reason for confiscation
involved in accidents caused by, or arising out of, the - vehicular accident with 2 children running across of Amar's license (2) Amar's license had not expired,
use of the motor vehicle. A motion for the path of a Chevrolet "Carry-All", belonging to a but had been renewed.
reconsideration was filed in and dismissed by the CA. partnership known as Diman & Company driven by - Judge Alikpala did not admit such evidence
its driver, Perfecto Amar, as it was passing a
ISSUE national highway at barrio Makiling Calamba, ISSUES
WON Far Eastern Surety is liable to the insured on its Laguna. They were killed. It was insured with the 1. WON Judge Alikapala committed grave abuse of
insurance policy Empire Insurance Co., Inc. under a so-called discretion in not admitting evidence
'comprehensive coverage" policy, loss by theft 2. WON confiscation of license and expiration of TVR
HELD excluded. The policy was in force at the time of the of the driver would serve as bar for Peza in
NO accident. recovering from Empire
- The award for damages made to the passengers - Placida Peza, the managing partner of Diman & Co.
was exclusively predicated on the representation filed a claim with Empire, for payment of HELD
made by La Mallorca that its passengers were compensation to the family of the 2 children who 1. NO
insured against accidents and not because it was at died as a result of the accident. Empire refused to - Even positing error in the Judge's analysis of the
fault in causing the accident. pay on the ground that the driver had no authority to evidence attempted to be introduced and his
Reasoning operate the vehicle, a fact which it expressly rejection thereof, it is clear that it was at most an
- In this case, the findings of the CA and the trial excepted from liability under the policy. What Peza error of judgment, not such an error as may be
court that the causative factor of the mishap was the did was to negotiate directly with the deceased branded a grave abuse of discretion, i.e., such
negligence of the gravel and truck driver would have children father for an out-of-court settlement. The capricious and whimsical exercise of judgment as is
been sufficient to relieve the taxi company of any father agreed to accept P6,200.00 in fun settlement equivalent to lack of jurisdiction, against which the
liability arising from the accident. However, in view of the liability of the vehicles owner and driver, and writ of certiorari will lie. In any event, the
of the sticker in all of its taxicabs, La Mallorca has Peza paid him this sum. established principle is "that ruling of the trial court
insured its passengers against accidents, whether it - Peza thereafter sued Empire to recover this sum of on procedural questions and on admissibility of
was at fault or not. In other words, La Mallorca P6,200.00 as actual damages, as well as P20,000.00 evidence during the course of the trial are
accepted the responsibility for damages or injuries to as moral damages, P10,000.00 as exemplary interlocutory in nature and may not be the subject of
passengers even if it had no fault at all. damages, and P10,000.00 as attorney's fees. She separate appeal or review on certiorari, but are to be
INSURANCE Page
86

assigned as errors and reviewed in the appeal Sec. 378: Any claim for death or injury to any 1. A claim may be made against one motor vehicle
properly taken from the decision rendered by the passenger or 3rd party pursuant to the provisions of only.
trial court on the merits of the case. this chapter shall be paid without the necessity of 2. If the victim is an occupant of a vehicle, the claim
- In the meantime, Judge Alikpala rendered proving fault or negligence of any kind. Provided, shall lie against the insurer of the vehicle in which he
judgment on the merits, since the case was then That for purposes of this section is riding, mounting or dismounting from.
already ripe for adjudication. The judgment ordered (i) The indemnity in respect of any one person shall 3. In any other case (i.e. if the victim is not an
dismissal of the case for failure on the part of the not exceed P5,000; occupant of a vehicle), the claim shall lie against the
plaintiff to prove their cause of action against (ii) The following proofs of loss, when submitted insurer of the directly offending vehicle.
Empire. Notice of the judgment was served on the under oath, shall be sufficient evidence to 4. In all cases, the right of the party paying the
parties in due course. substantiate the claim: claim to recover against the owner of the vehicle
2. YES (a) Police report of accident, and responsible for the accident shall be maintained.
- It would seem fairly obvious that whether the LTC (b) Death certificate and evidence sufficient to -That the vehicle ridden might not be the one that
agent was correct or not in his opinion that driver establish, the proper payee, or caused the accident is of no moment since the law
Amar had violated some traffic regulation warranting (c) Medical report and evidence of medical or itself provides that the party paying the claim under
confiscation of his license and issuance of a TVR in hospital disbursement in respect of which refund is Sec. 378 may recover against the owner of the
lieu thereof, this would not alter the undisputed fact claimed; vehicle responsible for the accident. This is precisely
that Amar's licence had indeed been confiscated and (iii) Claim may be made against one motor vehicle the essence of “no fault indemnity” insurance which
a TVR issued to him, and the TVR had already only. In the case of an occupant of a vehicle, claim was introduced to and made part of our laws in order
expired at the time that the vehicle being operated shall lie against the insurer of the vehicle in which to provide victims of vehicular accidents or their
by him killed two children by accident. Neither would the occupant is riding, mounting or dismounting heirs immediate compensation, although in a limited
proof of the renewal of Amar's license change the from. In any other case, claim shall lie against the amount, pending final determination of who is
fact that it had really been earlier confiscated by the insurer of the directly offending vehicle. In all cases, responsible for the accident and liable for the
LTC agent. the right of the party paying the claim to recover victims' injuries or death. In turn, the “no fault
Disposition petition is DISMISSED for lack of merit against the owner of the vehicle responsible for the indemnity” provision is part and parcel of the Code
accident shall be maintained. provisions on compulsory motor vehicle liability
Perla denied its liability under the above provision insurance and should be read together with the
and said that the insurer of the vehicle that the requirement for compulsory passenger and/or 3rd
respondents were riding (Malayan Insurance in this party liability insurance (Sec. 377) which was
case) should be liable. Its 2 MFRs denied, Perla filed mandated in order to ensure ready compensation for
this action victims of vehicular accidents.
-Irrespective of whether or not fault or negligence
ISSUE lies with the driver of the Superlines bus, as
PERLA COMPANIA DE SEGUROS v. ANCHETA WON Perla is the insurer liable to indemnify under respondents were not occupants of the bus, they
164 SCRA 144 Sec. 378 cannot claim the “no fault indemnity” provided in
CORTES; August 8, 1988 Sec. 378 from Perla. The claim should be made
HELD against the insurer of the vehicle they were riding.
NATURE NO Disposition Petition GRANTED. Orders of CFI
Petition for certiorari and prohibition with prelim Ratio The law is very clear – the claim shall lie ordering Perla to pay respondents immediately
injunction to review orders of CFI Camarines Norte against the insurer of the vehicle in which the P5000 ANNULLED and SET ASIDE
“occupant xxx is riding,” and no other. The claimant
FACTS is not free to choose from which insurer he will claim WESTERN GUARANTY CORPORATION v. CA
- Perla was the insurer of a Superlines bus which the “no fault indemnity,” as the law, by using the (RODRIGUEZ & DE DIOS TRANS)
figured in a collision with a III Scout (it’s a kind of word “shall”, makes it mandatory that the claim be 185 SCRA 652
vehicle). Injured passengers of the latter (and made against the insurer of the vehicle in which the FELICIANO; July 20, 1990
respondents in this case) filed a complaint for occupant is riding, mounting or dismounting from.
damages against Superlines, the bus driver, and Reasoning NATURE
Perla (as insurer of the bus). CFI Judge Ancheta - the rules on claims under the “no fault indemnity” Petition for review the decision of CA affirming in
ordered that Perla should pay the respondents provision, where proof of fault or negligence is not toto the damages awarded to private respondent by
immediately the P5000 under the “no fault clause” as necessary for payment of any claim for death or the trial court.
provided in Sec. 378. injury to a passenger or to a 3rd party, are
established: FACTS
INSURANCE Page
87

- Respondent Priscilla E. Rodriguez was struck by a which may be awarded under the Master Policy
De Dios passenger bus owned by respondent De Dios ISSUE Western has issued.
Transportation Co., Inc. Priscilla was thrown to the WON petitioner can be held liable for loss of - The reading urged by Western of the Schedule of
ground, hitting her forehead. She was treated at the earnings, moral damages and attorney's fees Indemnities comes too close to working fraud upon
Protacio Emergency Hospital and later on both the insured and the third party beneficiary of
hospitalized at the San Juan De Dios Hospital. Her HELD Section 1, quoted above. For Western's reading
face was permanently disfigured, causing her serious YES would drastically and without warning limit the
anxiety and moral distress. Respondent bus - The Schedule of Indemnities does not purport to otherwise unlimited and comprehensive scope of
company was insured with petitioner Western restrict the kinds of damages that may be awarded liability assumed by the insurer Western under
Guaranty Corporation ("Western") under its Master against Western once liability has arisen. It was Section 1: "all sums necessary to discharge liability
Policy which provided, among other things, for merely meant to set limits to the amounts the of the insured in respect of [bodily injury to a third
protection against third party liability, the relevant movant would be liable for in cases of claims for party]". This result- which is not essentially different
section reading as follows: death, bodily injuries of, professional services and from taking away with the left hand what had been
Section 1. Liability to the Public ? Company will, hospital charges, for services rendered to traffic given with the right hand we must avoid as obviously
subject to the Limits of Liability, pay all sums accident victims,' and not necessarily exclude claims repugnant to public policy. If what Western now
necessary to discharge liability of the insured in against the insurance policy for other kinds of urges is what Western intended to achieve by its
respect of ? damages, such as those in question. Schedule of Indemnities, it was incumbent upon
(a) death of or bodily injury to or damage to - It will be seen that the above quoted Schedule of Western to use language far more specific and
property of any passenger as defined herein. Indemnities establishes monetary limits which precise than that used in fact by Western, so that
(b) death of or bodily injury or damage to property Western may invoke in case of occurrence of the the insured, and potential purchasers of its Master
of any THIRD PARTY as defined herein in any particular kinds of physical injury there listed. Policy, and the Office of the Insurance
accident caused by or arising out of the use of the - It must be stressed, however, that the Schedule of Commissioner, may be properly informed and act
Schedule Vehicle, provided that the liability shall Indemnities does not purport to limit, or to accordingly.
have first been determined. In no case, however, enumerate exhaustively, the species of bodily injury - Moreover, an insurance contract is a contract of
shall the Company's total payment under both occurrence of which generate liability for petitioner adhesion. The rule is well entrenched in our
Section I and Section 11 combined exceed the Western. A car accident may, for instance, result in jurisprudence that the terms of such contract are to
Limits of Liability set forth herein. With respect to injury to internal organs of a passenger or third be construed strictly against the party which
death of or bodily injury to any third party or party, without any accompanying amputation or loss prepared the contract, which in this case happens to
passenger, the company's payment per victim in of an external member (e.g., a foot or an arm or an be petitioner Western.
any one accident shall not exceed the limits eye). But such internal injuries are surely covered by Disposition Petition denied.
indicated in the Schedule of indemnities provided Section I of the Master Policy, since they certainly
for in this policy excluding the cost of additional constitute bodily injuries. SUMMIT GUARANTY & INSURANCE COMPANY v.
medicines, and such other burial and funeral - The Schedule of Indemnities does not purport to ARNALDO
expenses that might have been incurred. restrict the kinds of damages that may be awarded 158 SCRA 332
- Respondent Priscilla Rodriguez filed a complaint for against Western once liability has arisen. Section 1, GANCAYCO; February 29, 1988
damages before the Regional Trial Court of Makati quoted above, does refer to certain "Limits of
against De Dios Transportation Co. and Walter A. Liability" which in the case of the third party liability NATURE
Saga Respondent De Dios Transportation Co., in section of the Master Policy, is apparently PETITION to review the order of the Insurance
turn, filed a third-party complaint against its P50,000.00 per person per accident. Within this Commissioner.
insurance carrier, petitioner Western. On 6 August over-all quantitative limit, all kinds of damages
1985, the trial court rendered a decision in favor of allowable by law" actual or compensatory FACTS
respondent Priscilla E. Rodriguez, awarding moral damages"; "moral damages'; "nominal damages"; - On Nov. 26, 1976, a Ford Pick-up truck owned by
damages, lossof earning and attorney's fees among "temperate or moderate damages"; "liquidated Marcos Olasco was bumped by a cargo truck owned
others. damages"; and "exemplary damages" ? may be by Floralde.
- On appeal, the Court of Appeals affirmed in toto awarded by a competent court against the insurer FGU Insurance Corporation (FG U) by reason of
the decision of the trial court. once liability is shown to have arisen, and the Motor Vehicle Insurance Policy No. IC-VF-07185 paid
- Petitioner contends that it cannot be held liable for essential requisites or conditions for grant of each Olaso the sum of P2,817.50 as its share in the repair
loss of earnings, moral damages and attorney's fees species of damages are present. It appears to us cost of the said Ford Pick-up. Having been
because these items are not among those included in self-evident that the Schedule of Indemnities was subrogated to the rights and causes of action of
the Schedule of Indemnities set forth in the not intended to be an enumeration, much less a Olaso in the said amount FGU formally demanded
insurance policy. closed enumeration, of the specific kinds of damages payment of said amount from Floralde and
INSURANCE Page
88

attempted to verify Floralde's insurance carrier but assumes it to be, then the phrase 'in proper cases' but petitioner failed to respond to the same.
failed to do so. In 1978 FGU was able to ascertain would not have been inserted. Nevertheless the complaint was filed even before a
the identity of Floralde's insurance carrier to be the - in Aisporna. vs. Court of Appeals: denial of the claim was made by petitioner. For all
Summit Guaranty and Insurance Company, Inc. 'Legislative intent must be ascertained from a legal purposes, the one-year prescriptive period
(Summit) and thus requested the insurance consideration of the statute as a whole. The provided for in Section 384 of the Insurance Code
commissioner for a conference with Summit and particular words, clauses and phrases should not has not begun to run.The cause of action arises only
demanded from Summit through counsel on be studied as detached and isolated expressions, and starts to run upon the denial of the claim by the
February 28, 1978 the payment of the damages but the whole and every part of the statute must insurance company.The court takes note of the
sustained by the car of Olaso but to no avail. be considered in fixing the meaning of any of its dilatory tactics employed by petitioner in this as in
- Hence on May 22, 1978 FGU filed a case in the parts and in order to produce a harmonious whole. the several cases aforecited to avoid payment of its
Insurance Commissioner's Office against Summit for A statute must be so construed as to harmonize liabilities.
recovery of said amount. and give effect to all its provisions whenever
- Summit filed a motion to dismiss on the ground of possible.' VILLACORTA v. THE INSURANCE COMMISSION
prescription under Section 384 of PD No. 612. - Petitioner company is trying to use Section 384 of 100 SCRA 467
Averring that the accident happened on November the Insurance Code as a cloak to hide itself from its TEEHANKEE; October 30, 1980
26, 1976 while the complaint was filed on May 22, liabilities. The facts of these cases evidently reflect
1978 beyond the one-year period from the time of the deliberate efforts of petitioner company to FACTS
the accident provided for by the said provision. prevent the filing of a formal action against it. - JEWEL VILLACORTA was the owner of a Colt
Bearing in mind that if it succeeds in doing so until Lancer, Model 1976, insured with respondent
ISSUE one year lapses from the date of the accident it could company for P35,000.00 - Own Damage; P30,000.00
WON the action must be dismissed on the ground of set up the defense of prescription, petitioner - Theft; and P30,000.00 - Third Party Liability,
prescription under Section 384 of PD No. 612 company made private respondents believe that effective May 16, 1977 to May 16, 1978.
their claims would be settled in order that the latter - On May 9, 1978, the vehicle was brought to the
HELD will not find it necessary to immediately bring suit. In Sunday Machine Works, Inc., for general check-up
NO violation of its duties to adopt and implement and repairs. On May 11, 1978, while it was in the
- The case do not fall within the meaning of proper reasonable standards for the prompt investigation of custody of the Sunday Machine Works, the car was
cases' as contemplated in Section 384 of the claims and to effectuate prompt, fair and equitable allegedly taken by six (6) persons and driven out to
Insurance Code. settlement of claims, and with manifest bad faith, Montalban, Rizal. While travelling along Mabini St.,
petitioner company devised means and ways of Sitio Palyasan, Barrio Burgos, going North at
stalling the settlement proceedings. In G.R. No. L- Montalban, Rizal, the car figured in an accident,
Reasoning 50997, no steps were taken to process the claim and hitting and bumping a gravel and sand truck parked
- Section 384 of PD 612 (Insurance Code) no rejection of said claim was ever made even if at the right side of the road going south. As a
Any person having any claim upon the policy private respondent had already complied with all the consequence, the gravel and sand truck veered to
issued pursuant to this chapter shall, without any requirements. the right side of the pavement going south and the
unnecessary delay, present to the insurance - In G.R. No. L-48758-petitioner company even car veered to the right side of the pavement going
company concerned a written notice of claim provided legal assistance to one of the private north. The driver, Benito Mabasa, and one of the
setting forth the amount of his loss, and/or the respondents in the criminal case filed against him passengers died and the other four sustained
nature, extent and duration of the injuries leading Private respondents to believe that it was physical injuries. The car, as well, suffered extensive
sustained as certified by a duly licensed physician. ready to pay. In the same case, petitioner company damage. Complainant, thereafter, filed a claim for
Notice of claim must be filed within six months admits that it took no final action or adjudication of total loss with the respondent company but claim
from date of the accident, otherwise, the claim the claim. Worse still, in G.R. No. L-48679, was denied. Hence, complainant was compelled to
shall be deemed waived. Action or suit for assurances of payment were constantly given and institute the present action."
recovery of damage due to loss or injury must be petitioner company even said that a check was ready - The comprehensive motor car insurance policy for
brought, in proper cases, with the Commissioner for release. This Court has made the observation P35,000.00 issued by respondent Empire Insurance
or the Courts within one year from date of that some insurance companies have been inventing Company admittedly undertook to indemnify the
accident, otherwise, the claimant's right of action excuses to avoid their just obligations and it is only petitioner-insured against loss or damage to the car
shall prescribe. the State that can give the protection which the (a) by accidental collision or overturning, or collision
- It is very clear that the one-year period is only insuring public needs from possible abuses of the or overturning consequent upon mechanical
required In proper cases. Had the lawmakers insurers. In view of the foregoing, breakdown or consequent upon wear and tear; (b)
intended it to be the way Petitioner Company - It is not denied that an extrajudicial demand for by fire, external explosion, self-ignition or lightning
payment was made by respondent FGU on petitioner
INSURANCE Page
89

or burglary, housebreaking or theft; and (c) by with his permission, such as a friend or member of 96 PHIL 504
malicious act. the family or the employees of a car service or repair REYES; January 31, 1955
- Respondent insurance commission, however, shop must be duly licensed drivers and have no
dismissed petitioner's complaint for recovery of the disqualification to drive a motor vehicle. A car owner FACTS
total loss of the vehicle against private respondent, who entrusts his car to an established car service - The S S "San Antonio", vessel owned and operated
sustaining respondent insurer's contention that the and repair shop necessarily entrusts his car key to by plaintiff, left Manila on October 6, 1949, bound for
accident did not fall within the provisions of the the shop owner and employees who are presumed to Basco, Batanes, vis Aparri, Cagayan, with general
policy either for the Own Damage or Theft coverage, have the insured's permission to drive the car for cargo belonging to different shippers, among them
invoking the policy provision on "Authorized Driver" legitimate purposes of checking or road-testing the the defendant. The vessel reached Aparri, but while
clause, which clause limits the use of the insured car. The mere happenstance that the employee(s) of still in the port, it ran aground at the mouth of the
vehicle to two (2) persons only, namely: the insured the shop owner diverts the use of the car to his own Cagayan river, and, attempts to refloat it under its
himself or any person on his (insured's) permission. illicit or unauthorized purpose in violation of the trust own power having failed, plaintiff had it refloated by
Apparently, the Insurance commission sees the reposed in the shop by the insured car owner does the Luzon Stevedoring Co. at an agreed
unauthorized taking of the vehicle for a joyride as a not mean that the "authorized driver" clause has compensation. Once afloat the vessel returned to
violation of the 'Authorized Driver' clause of the been violated such as to bar recovery, provided that Manila to refuel and then proceeded to Basco, the
policy." such employee is duly qualified to drive under a valid port of destination. There the cargoes were delivered
- Respondent commission likewise upheld private driver's license. to their respective owners or consignees, who, with
respondent's assertion that the car was not stolen - Secondly, and independently of the foregoing the exception of defendant, made a deposit or signed
and therefore not covered by the Theft clause, ruling (since when a car is unlawfully taken, it is the theft a bond to answer for their contribution to the
that "(T)he element of 'taking' in Article 308 of the clause, not the "authorized driver" clause, that average.
Revised Penal Code means that the act of depriving applies), where a car is admittedly as in this case - On the theory that the expenses incurred in
another of the possession and dominion of a unlawfully and wrongfully taken by some people, be floating the vessel constitute general average to
movable thing is coupled . . . with the intention, at they employees of the car shop or not to whom it which both ship and cargo should contribute, plaintiff
the time of the 'taking', of withholding it with the had been entrusted, and taken on a long trip to brought the present action in the CFI of Manila to
character of permanency Montalban without the owner's consent or make defendant pay his contribution, which, as
knowledge, such taking constitutes or partakes of determined by the average adjuster, amounts to
ISSUE the nature of theft as defined in Article 308 of the P841.40.
WON the Insurance commission’s findings are in Revised Penal Code. - Defendant denies liability to his amount, alleging,
accord with law - The Court rejects respondent commission's among other things, that the stranding of the vessel
premise that there must be an intent on the part of was due to the fault, negligence and lack of skill of
HELD the taker of the car "permanently to deprive the its master, that the expenses incurred in putting it
NO insured of his car" and that since the taking here was afloat did not constitute general average, and that
- First, respondent commission's ruling that the for a "joy ride" and "merely temporary in nature," a the liquidation of the average was not made in
person who drove the vehicle in the person of Benito "temporary taking is held not a taking insured accordance with law.
Mabasa, who, according to its own finding, was one against." - The lower court found for plaintiff
of the residents of the Sunday Machine Works, Inc. - The insurer must therefore indemnify the petitioner
to whom the car had been entrusted for general owner for the total loss of the insured car in the sum ISSUE
check-up and repairs was not an "authorized driver" of P35,000.00 under the theft clause of the policy, WON the expenses incurred in floating a vessel so
of petitioner-complainant is too restrictive and subject to the filing of such claim for reimbursement stranded should be considered general average and
contrary to the established principle that insurance or payment as it may have as subrogee against the shared by the cargo owners
contracts, being contracts of adhesion where the Sunday Machine Works, Inc.
only participation of the other party is the signing of HELD
his signature or his "adhesion" thereto, "obviously NO
call for greater strictness and vigilance on the part of Ratio The law on averages is contained in the Code
courts of justice with a view of protecting the weaker of Commerce. Under that law, averages are
party from abuse and imposition, and prevent their classified into simple or particular and general or
becoming traps for the unwary." gross. Generally speaking, simple or particular
- The main purpose of the "authorized driver" clause, averages include all expenses and damages caused
as may be seen from its text, supra, is that a person CHAPTER VIII. MARINE INSURANCE to the vessel or cargo which have not inured to the
other than the insured owner, who drives the car on common benefit (Art. 809), and are, therefore, to be
the insured's order, such as his regular driver, or MAGSAYSAY INC v. AGAN borne only by the owner of the property gave rise to
INSURANCE Page
90

same (Art. 810); while general or gross averages - The final requisite has not been proved, for it does Misfortunes, it shall be lawful for the Assured, his
include "all the damages and expenses which are not appear that the expenses here in question were or their Factors, Servants, or assigns, to sue,
deliberately caused in order to save the vessel, its incurred after following the procedure laid down in labour and travel for, in and about the Defence.
cargo, or both at the same time, from a real and article 813. Safeguard, and recovery of the said Vessel or any
known risk" (Art. 811). Being for the common Disposition Wherefore, the decision appealed from part thereof, without Prejudice to this Insurance;
benefit, gross averages are to be borne by the is reversed. to the Charges whereof the said Company, will
owners of the articles saved (Art. 812). contribute, according to the rate and quantity of
Reasoning JARQUE v. SMITH, BELL & CO. the sum herein assured...”
- the stranding of plaintiff's vessel was due to the 56 PHIL 758 - Attached to the policy over and above the said
sudden shifting of the sandbars at the mouth of the OSTRAND; November11, 1932 clause is a “rider” containing typewritten provisions,
river which the port pilot did not anticipate. The among which appears in capitalized type the
standing may, therefore, be regarded as accidental. NATURE following clause:
- Tolentino, in his commentaries on the Code of Appeal from judgment of the lower court “AGAINST THE ABSOLUTE TOTAL LOSS OF THE
Commerce, gives the following requisites for VESSEL ONLY, AND TO PAY PROPORTIONATE
general average: FACTS SALVAGE CHARGES OF THE DECLARED VALUE.”
First, there must be a common danger. This means, - Plaintiff’s motorboat, “Pandan” was insured on a
that both the ship and the cargo, after has been marine insurance policy with National Union Fire ISSUES
loaded, are subject to the same danger, whether Insurance Company (NUFIC) for P45K. According to 1. WON the lower court erred in disregarding the
during the voyage, or in the port of loading or the provisions of a “rider” attached to the policy, the typewritten clause endorsed upon the policy,
unloading; that the danger arises from the accidents insurance was against the “absolute total loss of the expressly limiting insurer's liability thereunder of the
of the sea, dispositions of the authority, or faults of vessel only.” On Oct. 31, 1928, the ship ran into total loss of the wooden vessel Pandan and to
men, provided that the circumstances producing the very heavy sea and it became necessary to jettison a proportionate salvage charges
peril should be ascertained and imminent or may portion of the cargo. As a result of the jettison, the 2. WON lower court erred in concluding that
rationally be said to be certain and imminent. This NUFIC was assessed P2,610.86 as its contribution to defendant and appellant, NUFIC is liable to
last requirement exclude measures undertaken the general average. contribute to the general average resulting from the
against a distant peril. - The insurance company, insisting that its obligation jettison of a part of said vessel's cargo
Second, that for the common safety part of the did not extend beyond the insurance of the “absolute
vessel or of the cargo or both is sacrificed total loss of the vessel only, and to pay HELD
deliberately. proportionate salvage of the declared value,” refused 1. NO
Third, that from the expenses or damages caused to contribute to the settlement of the gen. ave. The Ratio In case repugnance exists between written
follows the successful saving of the vessel and cargo. present action was thereupon instituted, and after and printed portions of a policy, the written portion
Fourth, that the expenses or damages should have trial the court below rendered judgment in favor of prevails.
been incurred or inflicted after taking proper legal the plaintiff and ordered the defendant to pay the Reasoning
steps and authority. plaintiff P2,610.86 as its part of the indemnity for - Section 291 of the Code of Civil Procedure provides
- With respect to the first requisite, the evidence the gen. ave. brought about by the jettison of cargo. that “when an instrument consists partly of written
does not disclose that the expenses sought to be The insurance company then appealed to the SC. words and partly of a printed form and the two are
recovered from defendant were incurred to save - The insurance contract is printed in the English inconsistent, the former controls the latter.”
vessel and cargo from a common danger...it is the common form of marine policies. One of the clauses
safety of the property, and not of the voyage, which of the document originally read as follows:
constitutes the true foundation of the general “Touching the Adventures and Perils which the 2. NO
average. said NUFIC is content to bear, and to take upon Ratio The liability for contribution in general average
- As to the second requisite, we need only repeat them in this Voyage; they are of the Seas, Men-of- is not based on the express terms of the policy, but
that the expenses in question were not incurred for War, Fire, Pirates, Thieves, Jettison, Letters of rests upon the theory that from the relation of the
the common safety of vessel and cargo, since they, Mart and Countermart, Surprisals, and Takings at parties and for their benefit, a quasi contract is
or at least the cargo, were not in imminent peril. Sea. Arrests, Restraints and Detainments, of all implied by law.
- With respect to the third requisite, the salvage Kings, Princes and People of what Nation, Reasoning
operation, it is true, was a success. But as the Condition or Quality soever; Barratry of the Master - In the absence of positive legislation to the
sacrifice was for the benefit of the vessel to enable it and Marines, and of all other Perils, Losses and contrary, the liability of the defendant insurance
to proceed to destination and not for the purpose of Misfortunes, that have or shall come to the Hurt, company on its policy would, perhaps, be limited to
saving the cargo, the cargo owners are not in law Detriment, or Damage of the said Vessel or any “absolute loss of the vessel only, and to pay
bound to contribute to the expenses. part thereof; and in case of any Loss or proportionate salvage of the declared value.” But the
INSURANCE Page
91

policy was executed in this jurisdiction and of the drain pipes of the ship and concluded that the NO
“warranted to trade within the waters of the loss was not covered by the policy of insurance. The - the words "all other perils, losses, and misfortunes"
Philippine Archipelago only.” Here, Art. 859 of the trial court made the ff findings: are to be interpreted as covering risks which are of
Code of Commerce is still in force: The drain pipe which served as a discharge like kind (ejusdem generis) with the particular risks
“ART. 859. The underwriters of the vessel, of the from the water closet passed down through which are enumerated in the preceding part of the
freight, and of the cargo shall be obliged to pay for the compartment where the rice in question same clause of the contract. ''According to the
the indemnity of the gross average in so far as is was stowed and thence out to sea through the ordinary rules of construction, these words must be
required of each one of these objects wall of the compartment, which was a part of interpreted with reference to the words which
respectively.” the wall of the ship. The joint or elbow where immediately precede them. They were no doubt
- The article is mandatory in its terms, and the the pipe changed its direction was of cast inserted in order to prevent disputes founded on nice
insurers (whether for the vessel or for the freight or iron; and in course of time it had become distinctions. X x x For example, if the expression
for the cargo) are bound to contribute to the corroded and abraded until a longitudinal 'perils of the seas' is given its widest sense the
indemnity of the general average. The provision opening had appeared in the pipe about one general words have little or no effect as applied to
simply places the insurer on the same footing as inch in length. This hole had been in existence that case. If on the other hand that expression is to
other persons who have an interest in the vessel, or before the voyage was begun, and an attempt receive a limited construction, as apparently it did in
the cargo therein, at the time of the occurrence of had been made to repair it by filling with Cullen vs. Butler (5 M. & S., 461), and loss by perils
the general average and who are compelled to cement and bolting over it a strip of iron. The of the seas is to be confined to loss ex marine
contribute (Art. 812, Code of Commerce). effect of loading the boat was to submerge tempestatis discrimine, the general words become
- In the present case it is not disputed that the ship the vent, or orifice, of the pipe until it was most important. X x x" (Thames and Mersey Marine
was in grave peril and that the jettison of part of the about 18 inches or 2 feet below the level of Insurance Co. vs. Hamilton, Fraser & Co.)
cargo was necessary. If the cargo was in peril to the the sea. As a consequence the sea water rose - a loss which, in the ordinary course of events,
extent of call for general average, the ship must also in the pipe. Navigation under these conditions results from the natural and inevitable action of the
have been in great danger, possibly sufficient to resulted in the washing out of the cement- sea, from the ordinary wear and tear of the ship, or
cause its absolute loss. The jettison was therefore as filling from the action of the sea water, thus from the negligent failure of the ship's owner to
much to the benefit of the underwriter as to the permitting the continued flow of the salt water provide the vessel with proper equipment to convey
owner of the cargo. The latter was compelled to into the compartment of rice. the cargo under ordinary conditions, is not a peril of
contribute to the indemnity; why should not the - The court found in effect that the opening above the sea. Such a loss is rather due to what has been
insurer be required to do likewise? If no jettison had described had resulted in course of time from aptly called the "peril of the ship." The insurer
taken place and if the ship by reason thereof had ordinary wear and tear and not from the straining of undertakes to insure against perils of the sea and
foundered, the underwriter's loss would have been the ship in rough weather on that voyage. The court similar perils, not against perils of the ship. There
many times as large as the contribution now also found that the repairs that had been made on must, in order to make the insurer liable, be "some
demanded. the pipe were slovenly and defective and that, by casualty, something which could not be foreseen as
Disposition Appealed judgment is affirmed reason of the condition of this pipe, the ship was not one of the necessary incidents of the adventure. The
properly equipped to receive the rice at the time the purpose of the policy is to secure an indemnity
GO TIACO v. UNION INSURANCE voyage was begun. For this reason the court held against accidents which may happen, not against
40 PHIL 40 that the ship was unseaworthy. events which must happen." (Wilson, Sons & Co. vs.
STREET; September 1, 1919 - The policy purports to insure the cargo from the Owners of Cargo per the Xantho)
following among other risks: "Perils . . . of the seas, - In the present case the entrance of the sea water
FACTS men, of war, fire, enemies, pirates, rovers, into the ship's hold through the defective pipe
- Union Insurance Society of Canton, Ltd., issued a thieves, .jettisons, . . . barratry of the master and already described was not due to any accident which
marine insurance policy upon a cargo of rice mariners, and of all other perils, losses, and happened during the voyage, but to the failure of the
belonging to the Go Tiaoco Brothers, which was misfortunes that have or shall come to the hurt, ship's owner properly to repair a defect of the
transported in the early days of May, 1915, on the detriment, or damage of the said goods and existence of which he was apprised. The loss was
steamship Hondagua from the port of Saigon to merchandise or any part thereof." therefore more analogous to that which directly
Cebu. results from simple unseaworthiness than to that
- On discharging the rice from one of the which results from perils of the sea.
compartments in the after hold, upon arrival at ISSUE - there is no room to doubt the liability of the
Cebu, it was discovered that 1473 sacks had been WON Union Insurance is liable for the loss of the Go shipowner for such a loss as occurred in this case. By
damaged by sea water. The loss was P3,875.25. Tiaco Brothers parity of reasoning the insurer is not liable; for,
- The trial court found that the inflow of the sea generally speaking, the shipowner excepts the perils
water during the voyage was due to a defect in one HELD of the sea from his engagement under the bill of
INSURANCE Page
92

lading, while this is the very peril against which the The insistence of private respondent that rusting is a Palawan to Manila North Harbor. Roque insured the
insurer intends to give protection. As applied to the peril of the sea is erroneous; 3. Rusting is not a logs with Pioneer Insurance for P100,000.
present case it results that the owners of the risk insured against, since a risk to be insured - February 29, 1972 – 811 logs were loaded in
damaged rice must look to the shipowner for redress against should be a casualty or some casualty, Malampaya but en route to Manila, Mable 10 sank.
and not to the insurer. something which could not be foreseen as one of the - March 8,1972 – Roque and Chiong wrote a letter to
The same conclusion must be reached if the question necessary incidents of adventure; 4. A fact capable Manila Bay, demanding payment of P150,000.00 for
be discussed with reference to the seaworthiness of of unquestionable demonstration or of public the loss of the shipment plus P100,000.00 as
the ship. It is universally accepted that in every knowledge needs no evidence. This fact of unrealized profits but the latter ignored the demand.
contract of insurance upon anything which is the unquestionable demonstration or of public knowledge - A letter was also sent to Pioneer, claiming the full
subject of marine insurance, a warranty is implied is that heavy rusting of steel or iron pipes cannot amount of P100,000.00 under the insurance policy
that the ship shall be seaworthy at the time of the occur within a period of a seven (7) day voyage. but Pioneer refused to pay on the ground that its
inception of the voyage. This rule is accepted in our Besides, petitioner had introduced the clear cargo liability depended upon the "Total Loss by Total Loss
own Insurance Law (Act No. 2427, sec. 106). It is receipts or tally sheets indicating that there was no of Vessel only".
also well settled that a ship which is seaworthy for damage on the steel pipes during the voyage. - After hearing, the trial court favored Roque.
the purpose of insurance upon the ship may yet be Pioneer and Manila Bay were ordered to pay Roque
unseaworthy for the purpose of insurance upon the ISSUE P100,000. Pioneer appealed the decision.
cargo (Act No. 2427, sec. 106). WON rusting is a “peril of the sea” - January 30, 1984 – Pioneer was absolved from
Disposition Decision of trial court is affirmed liability after finding that there was a breach of
HELD implied warranty of seaworthiness on the part of the
CATHAY INSURANCE CO. v. CA (REMINGTON YES petitioners and that the loss of the insured cargo was
INDUSTRIAL SALES CORP.) - There is no question that the rusting of steel pipes caused by the "perils of the ship" and not by the
151 SCRA 710 in the course of a voyage is a "peril of the sea" in "perils of the sea". It ruled that the loss is not
PARAS; June 30 1987 view of the toll on the cargo of wind, water, and salt covered by the marine insurance policy.
conditions. At any rate if the insurer cannot be held - It was alleged that Mable 10 was not seaworthy
FACTS accountable therefor, We would fail to observe a and that it developed a leak
- Remington Industrial Sales Corp insured its cardinal rule in the interpretation of contracts, - The IAC found that one of the hatches was left
shipment of seamless steel pipes. It incurred losses namely, that any ambiguity therein should be open, causing water to enter the barge and because
and damages (I gather the steel pipes rusted during construed against the maker/issuer/drafter thereof, the barge was not provided with the necessary cover
the voyage from Japan to the Phils. on board vessel namely, the insurer. Besides the precise purpose of or tarpaulin, the splash of sea waves brought more
SS "Eastern Mariner”) and filed complaint against insuring cargo during a voyage would be rendered water inside the barge.
Cathay Insurance Co seeking collection of the sum of fruitless. - Petitioners contend that the implied warranty of
P868,339.15 Disposition WHEREFORE, this petition is hereby seaworthiness provided for in the Insurance Code
- TC decided for Remington. Cathay filed MR, which DENIED, and the assailed decision of the Court of refers only to the responsibility of the shipowner who
was denied. CA affirmed. Appeals is hereby AFFIRMED. must see to it that his ship is reasonably fit to make
- CA said (among other things): 1. Coverage of in safety the contemplated voyage.
private respondent's loss under the insurance policy ROQUE v. IAC (PIONEER INSURANCE AND - The petitioners state that a mere shipper of cargo,
issued by petitioner is unmistakable; SURETY CORP.) having no control over the ship, has nothing to do
2. Alleged contractual limitations contained in 139 SCRA 596 with its seaworthiness. They argue that a cargo
insurance policies are regarded with extreme caution GUTIERREZ; November 11, 1985 owner has no control over the structure of the ship,
by courts and are to be strictly construed against the its cables, anchors, fuel and provisions, the manner
insurer; obscure phrases and exceptions should not NATURE of loading his cargo and the cargo of other shippers,
be allowed to defeat the very purpose for which the Petition for certiorari to review the decision of the and the hiring of a sufficient number of competent
policy was procured; IAC officers and seamen.
3. Rust is not an inherent vice of the seamless steel ISSUE
pipes without interference of external factors FACTS WON the loss should have been covered by the
- Cathay contend (among other things): 1. private - February 19, 1972 – Common carrier Manila Bay marine insurance policy
respondent has admitted that the questioned Lighterage Corp. entered into a contract with Roque
shipment is not covered by a "square provision of Timber Enterprises and Chiong. The contract stated HELD
the contract," but private respondent claims implied that Manila Bay would carry 422.18 cu. meters of NO
coverage from the phrase "perils of the sea" logs on its vessel Mable 10 from Malampaya Sound, Ratio It is universally accepted that in every
mentioned in the opening sentence of the policy; 2. contract of insurance upon anything which is the
INSURANCE Page
93

subject of marine insurance, a warranty is implied thereto are now so generally accepted as to be
that the ship shall be seaworthy at the time of the FACTS considered definitely settled.
inception of the voyage. In marine insurance, the - This is an action on a policy of marine insurance - The words "all other perils, losses, and
risks insured against are classified as 'perils of the issued by the Union Insurance Society of Canton, misfortunes" are to be interpreted as covering risks
sea,’ which includes such losses that are of Ltd., upon a cargo of rice belonging to the plaintiffs, which are of like kind (ejusdem generis) with the
extraordinary nature, or arise from some Go Tiaoco Brothers, which was transported on the particular risks which are enumerated in the
overwhelming power, which cannot be guarded steamship Hondagua from the port of Saigon to preceding part of the same clause of the contract.
against by the ordinary exertion of human skill and Cebu. - A loss which, in the ordinary course of events,
prudence. - On discharging the rice from one of the results from the natural and inevitable action of the
Reasoning compartments in the after hold, upon arrival at sea, from the ordinary wear and tear of the ship, or
- Based on Sec. 113 and Sec. 99 of the Insurance Cebu, it was discovered that 1,473 sacks had been from the negligent failure of the ship's owner to
Code, the term "cargo" can be the subject of marine damaged by sea water. provide the vessel with proper equipment to convey
insurance and that once it is so made, the implied - The loss so resulting to the owners of rice, after the cargo under ordinary conditions, is not a peril of
warranty of seaworthiness immediately attaches to proper deduction had been made for the portion the sea. Such a loss is rather due to what has been
whoever is insuring the cargo whether he be the saved, was P3,875.25. aptly called the "peril of the ship." The insurer
shipowner or not. - The trial court found that the inflow of the sea undertakes to insure against perils of the sea and
- The fact that the un-seaworthiness of the ship was water during the voyage was due to a defect in one similar perils, not against perils of the ship.
unknown to the insured is immaterial in ordinary of the drain pipes of the ship and concluded that the - As was said by Lord Herschell in Wilson, Sons & Co.
marine insurance and may not be used by him as a loss was not covered by the policy of insurance. vs. Owners of Cargo per the Xantho, there must, in
defense in order to recover on the marine insurance Judgment was accordingly entered in favor of the order to make the insurer liable, be "some casualty,
policy. defendant and the plaintiffs appealed. something which could not be foreseen as one of the
- Since the law provides for an implied warranty of - The court found in effect that the opening above necessary incidents of the adventure. The purpose of
seaworthiness in every contract of ordinary marine described had resulted in course of time from the policy is to secure an indemnity against accidents
insurance, it becomes the obligation of a cargo ordinary wear and tear and not from the straining of which may happen, not against events which must
owner to look for a reliable common carrier which the ship in rough weather on that voyage. The court happen."
keeps its vessels in seaworthy condition. The shipper also found that the repairs that had been made on - In the present case the entrance of the sea water
of cargo my have no control over the vessel but he the pipe were slovenly and defective and that, by into the ship's hold through the defective pipe
has full control in the choice of the common carrier reason of the condition of this pipe, the ship was not already described was not due to any accident which
that will transport his goods. properly equipped to receive the rice at the time the happened during the voyage, but to the failure of the
- In marine cases, the risks insured against are voyage was begun. For this reason the court held ship's owner properly to repair a defect of the
'perils of the sea.’ The term extends only to losses that the ship was unseaworthy. existence of which, he was apprised. The loss was
caused by sea damage, or by the violence of the therefore more analogous to that which directly
elements, and does not embrace all losses ISSUE results from simple unseaworthiness than to that
happening at sea. WON the insurer is liable which results from perils of the sea.
- It is quite unmistakable that the loss of the cargo - It is universally accepted that in every contract of
was due to the perils of the ship rather than the HELD insurance upon anything which is the subject of
perils of the sea. - The question whether the insurer is liable on this marine insurance, a warranty is implied that the ship
- Loss which, in the ordinary course of events, policy for the loss caused in the manner above shall be seaworthy at the time of the inception of the
results from the natural and inevitable action of the stated presents two phases which are in a manner voyage. This rule is accepted in our own Insurance
sea, from the ordinary wear and tear of the ship, or involved with each other. One has reference to the Law (Act No. 2427, see. 106).
from the negligent failure of the ship's owner to meaning of the expression "perils of the seas and all - It is also well settled that a ship which is
provide the vessel with proper equipment to convey other perils, losses, and misfortunes," as used in the seaworthy for the purpose of insurance upon the
the cargo under ordinary conditions, is not a ‘peril of policy; the other has reference to the implied ship may yet be unseaworthy for the purpose of
the sea’ but is called ‘peril of the ship.’ warranty, on the part of the insured, as to the insurance upon the cargo (Act No. 2427, see. 106).
Disposition Decision appealed from is affirmed. seaworthiness of the ship. Disposition Jjudgment affirmed.
- The meaning of the expression "perils * * * of the
LA RAZON v. UNION INSURANCE SOCIETY OF seas * * * and all other perils, losses, and MALAYAN INSURANCE v. CA (supra p.10)
CANTON, LTD. misfortunes," used in describing the risks covered by
40 PHIL 40 policies of marine insurance, has been the subject of FILIPINO MERCHANTS INS. CO. v. CA (supra
STREET; September 1, 1919 frequent discussion; and certain propositions relative p.19)
INSURANCE Page
94

COASTWISE LIGHTERAGE CORP v. CA 2. WON the insurer was subrogated into the rights of of affreightment, was not converted into a private
(PHILIPPINE GENERAL INSURANCE COMPANY) the consignee against the carrier, upon payment by carrier, but remained a common carrier and was still
245 SCRA 796 the insurer of the value of the consignee's goods lost liable as such.
FRANCISCO; July 12, 1995 while on board one of the carrier's vessels - Therefore, the mere proof of delivery of goods in
good order to a carrier and the subsequent arrival of
NATURE HELD the same goods at the place of destination in bad
Petition for review of CA Decision affirming decision 1. NO order makes for a prima facie case against the
of RTC Manila holding that Coastwise is liable to pay - The distinction between the two kinds of charter carrier. The presumption of negligence that attaches
PhilGen Insurance the amount of P700thou plus legal parties (i.e. bareboat or demise and contract of to common carriers, once the goods it transports are
interest thereon, another sum of P100thou as affreightment) is more clearly set out in the case of lost, destroyed or deteriorated, applies to Coastwise.
attorney's fees and the cost of the suit. Puromines, Inc. vs. Court of Appeals, wherein SC This presumption, which is overcome only by proof
ruled: of the exercise of extraordinary diligence, remained
FACTS “Under the demise or bareboat charter of the unrebutted in this case.
- Pag-asa Sales, Inc. entered into a contract to vessel, the charterer will generally be regarded - The damage to the barge which carried the cargo
transport molasses from the province of Negros to as the owner for the voyage or service of molasses was caused by its hitting an unknown
Manila with Coastwise, using the latter's dumb stipulated. The charterer mans the vessel with sunken object as it was heading for Pier 18. The
barges. The barges were towed in tandem by the his own people and becomes the owner pro hac object turned out to be a submerged derelict vessel.
tugboat MT Marica, also owned by Coastwise. Upon vice, subject to liability to others for damages The evidence on record appeared that far from
reaching Manila Bay, while approaching Pier 18, one caused by negligence. To create a demise, the having rendered service with the greatest skill and
of the barges struck an unknown sunken object. The owner of a vessel must completely and utmost foresight, and being free from fault, the
forward buoyancy compartment was damaged, and exclusively relinquish possession, command and carrier was culpably remiss in the observance of its
water gushed in through a hole "two inches wide and navigation thereof to the charterer, anything duties.
twenty-two inches long." short of such a complete transfer is a contract of - Jesus R. Constantino, the patron of the vessel
- As a consequence, the molasses at the cargo tanks affreightment (time or voyage charter party) or "Coastwise 9" admitted that he was not licensed. The
were contaminated and rendered unfit for the use it not a charter party at all. Code of Commerce, which subsidiarily governs
was intended. This prompted consignee Pag-asa On the other hand a contract of affreightment is common carriers (which are primarily governed by
Sales to reject the shipment of molasses as a total one in which the owner of the vessel leases part the provisions of the Civil Code) provides: “Art. 609.
loss. Thereafter, Pag-asa Sales filed a formal claim or all of its space to haul goods for others. It is a Captains, masters, or patrons of vessels must be
with the insurer of its lost cargo (PhilGen) and contract for special service to be rendered by Filipinos, have legal capacity to contract in
against the carrier (Coastwise). Coastwise denied the the owner of the vessel and under such contract accordance with this code, and prove the skill
claim and it was PhilGen which paid Pag-asa Sales the general owner retains the possession, capacity and qualifications necessary to command
the amount of P700k representing the value of the command and navigation of the ship, the and direct the vessel, as established by marine and
damaged cargo of molasses. charterer or freighter merely having use of the navigation laws, ordinances or regulations, and must
- PhilGen then filed an action against Coastwise space in the vessel in return for his payment of not be disqualified according to the same for the
before the RTC Manila, seeking to recover the P700k the charter hire... discharge of the duties of the position.”
which it paid to Pag-asa Sales for the latter's lost An owner who retains possession of the ship - Clearly, Coastwise Lighterage's embarking on a
cargo. PhilGen now claims to be subrogated to all the though the hold is the property of the charterer, voyage with an unlicensed patron violates this rule.
contractual rights and claims which the consignee remains liable as carrier and must answer for It cannot safely claim to have exercised
may have against the carrier, which is presumed to any breach of duty as to the care, loading and extraordinary diligence, by placing a person whose
have violated the contract of carriage. unloading of the cargo.” navigational skills are questionable, at the helm of
- RTC awarded the amount prayed for by PhilGen. CA - Although a charter party may transform a common the vessel which eventually met the fateful accident.
affirmed. Hence, this petition. carrier into a private one, the same however is not It may also logically, follow that a person without
true in a contract of affreightment on account of the license to navigate, lacks not just the skill to do so,
ISSUES aforementioned distinctions between the two. but also the utmost familiarity with the usual and
1. WON Coastwise Lighterage was transformed into a - SC agrees with Coastwise's admission that the safe routes taken by seasoned and legally authorized
private carrier, by virtue of the contract of contract it entered into with the consignee was one ones. Had the patron been licensed, he could be
affreightment which it entered into with the of affreightment. Pag-asa Sales, Inc. only leased presumed to have both the skill and the knowledge
consignee, Pag-asa Sales, Inc. (Corollarily, if it were three of petitioner's vessels, in order to carry cargo that would have prevented the vessel's hitting the
in fact transformed into a private carrier, did it from one point to another, but the possession, sunken derelict ship that lay on their way to Pier 18.
exercise the ordinary diligence to which a private command and navigation of the vessels remained - As a common carrier, Coastwise is liable for breach
carrier is in turn bound?) with Coastwise. As such, Coastwise, by the contract of the contract of carriage, having failed to overcome
INSURANCE Page
95

the presumption of negligence with the loss and - Coca-Cola Bottlers Philippines, Inc., loaded on Thus the loss of the vessel and its entire shipment
destruction of goods it transported, by proof of its board “MV Asilda,” a vessel owned and operated by could only be attributed to either a fortuitous event,
exercise of extraordinary diligence. Felman 7,500 cases of 1-liter Coca-Cola softdrink in which case, no liability should attach unless there
2. YES bottles to be transported from Zamboanga City to was a stipulation to the contrary, or to the
- Coastwise is liable for breach of the contract of Cebu for consignee Coca-Cola Bottlers Philippines, negligence of the captain and his crew, in which
carriage it entered into with Pag-asa Sales, Inc. Inc., Cebu. The shipment was insured with petitioner case, Art. 587 of the Code of Commerce should
However, for the damage sustained by the loss of Philippine American General under Marine Open apply.
the cargo which petitioner-carrier was transporting, Policy. - CA ruled that “MV Asilda” was unseaworthy for
it was not the carrier which paid the value thereof to - The vessel sank in the waters of Zamboanga del being top- heavy as 2,500 cases of Coca-Cola
Pag-asa Sales, Inc. but the latter's insurer, herein Norte bringing down her entire cargo with her softdrink bottles were improperly stowed on deck.
private respondent PhilGen. including the subject 7,500 cases of 1-liter Coca-Cola Nonetheless, the appellate court denied the claim of
- Article 2207 of the Civil Code: If the plaintiffs softdrink bottles. PHILAMGEN on the ground that the assured’s implied
property has been insured, and he has received - The consignee filed a claim with respondent warranty of seaworthiness was not complied with.
indemnity from the insurance company for the injury FELMAN for recovery of damages it sustained as a Perfunctorily, PHILAMGEN was not properly
or loss arising out of the wrong or breach of contract result of the loss of its softdrink bottles that sank subrogated to the rights and interests of the shipper.
complained of, the insurance company shall be with “MV Asilda.” Respondent denied the claim Furthermore, respondent court held that the filing of
subrogated to the rights of the insured against the thus prompting the consignee to file an insurance notice of abandonment had absolved the
wrongdoer or the person who violated the contract.” claim with PHILAMGEN which paid its claim of shipowner/agent from liability under the limited
- This legal provision is founded on the well-settled P755,250.00. liability rule.
principle of subrogation. If the insured property is - Claiming its right of subrogation PHILAMGEN
destroyed or damaged through the fault or sought recourse against respondent FELMAN which ISSUES
negligence of a party other than the assured, then disclaimed any liability for the loss. Consequently, 1. WON “MV Asilda” was seaworthy when it left the
the insurer, upon payment to the assured will be PHILAMGEN sued the shipowner for sum of money port of Zamboanga
subrogated to the rights of the assured to recover and damages. 2. WON the limited liability under Art. 587 of the
from the wrongdoer to the extent that the insurer - PHILAMGEN alleged that the sinking and total loss Code of Commerce should apply
has been obligated to pay. Payment by the insurer of “MV Asilda” and its cargo were due to the vessel’s 3. WON PHILAMGEN was properly subrogated to the
to the assured operated as an equitable assignment unseaworthiness as she was put to sea in an rights and legal actions which the shipper had
to the former of all remedies which the latter may unstable condition. It further alleged that the against FELMAN, the shipowner
have against the third party whose negligence or vessel was improperly manned and that its
wrongful act caused the loss. The right of officers were grossly negligent in failing to take HELD
subrogation is not dependent upon, nor does it grow appropriate measures to proceed to a nearby port or 1. YES
out of, any privity of contract or upon written beach after the vessel started to list. - “MV Asilda” was unseaworthy when it left the port
assignment of claim. It accrues simply upon payment - FELMAN filed a motion to dismiss based on the of Zamboanga. We subscribe to the findings of the
of the insurance claim by the insurer. affirmative defense that no right of subrogation in Elite Adjusters, Inc., and the Court of Appeals that
- Undoubtedly, upon payment by respondent insurer favor of PHILAMGEN was transmitted by the shipper, the proximate cause of the sinking of “MV Asilda”
PhilGen of the amount of P700,000.00 to Pag-asa and that, in any event, FELMAN had abandoned all was its being top-heavy. Contrary to the ship
Sales, Inc., the consignee of the cargo of molasses its rights, interests and ownership over “MV Asilda” captain’s allegations, evidence shows that
totally damaged while being transported by together with her freight and appurtenances for the approximately 2,500 cases of softdrink bottles were
petitioner Coastwise Lighterage, the former was purpose of limiting and extinguishing its liability stowed on deck. Several days after “MV Asilda”
subrogated into all the rights which Pag-asa Sales, under Art. 587 of the Code of Commerce. sank, an estimated 2,500 empty Coca-Cola plastic
Inc. may have had against the carrier, herein - Trial court dismissed the complaint of PHILAMGEN. cases were recovered near the vicinity of the sinking.
petitioner Coastwise Lighterage. On appeal the Court of Appeals set aside the Considering that the ship’s hatches were properly
Disposition Petition denied. CA affrimed. dismissal and remanded the case to the lower court secured, the empty Coca-Cola cases recovered could
for trial on the merits. FELMAN filed a petition for have come only from the vessel’s deck cargo. It is
THE PHILIPPINE AMERICAN GENERAL certiorari with this Court but it was subsequently settled that carrying a deck cargo raises the
INSURANCE COMPANY INC v. CA (FELMAN denied on 13 February 1989. presumption of unseaworthiness unless it can be
SHIPPING LINES) - Trial court rendered judgment in favor of FELMAN. shown that the deck cargo will not interfere with the
273 SCRA 226 It ruled that “MV Asilda” was seaworthy when it left proper management of the ship. However, in this
BELLOSILLO; June 11, 1997 the port of Zamboanga as confirmed by certificates case it was established that “MV Asilda” was not
issued by the Philippine Coast Guard and the designed to carry substantial amount of cargo on
FACTS shipowner’s surveyor attesting to its seaworthiness. deck. The inordinate loading of cargo deck resulted
INSURANCE Page
96

in the decrease of the vessel’s metacentric height by terms in writing in the policy in the clearest - The trial court decided in favor of the defendant,
thus making it unstable. The strong winds and language. And where the policy stipulates that the saying that the policy only covered an actual total
waves encountered by the vessel are but the seaworthiness of the vessel as between the assured loss, not a constructive total loss.
ordinary vicissitudes of a sea voyage and as such and the assurer is admitted, the question of
merely contributed to its already unstable and seaworthiness cannot be raised by the assurer ISSUES
unseaworthy condition. without showing concealment or misrepresentation 1. WON there was an absolute total loss that can be
2. NO by the assured. covered by the policy
- The ship agent is liable for the negligent acts of the - PHILAMGEN’s action against FELMAN is squarely 2. WON the Marine Law of Great Britain applies
captain in the care of goods loaded on the vessel. sanctioned by Art. 2207 of the Civil Code which
This liability however can be limited through provides: HELD
abandonment of the vessel, its equipment and Art. 2207. If the plaintiff’s property has been 1. YES
freightage as provided in Art. 587. Nonetheless, insured, and he has received indemnity from - At the time that the lighter was at the bottom of
there are exceptional circumstances wherein the the insurance company for the injury or loss the bay, it was of no value to the owner, thus there
ship agent could still be held answerable despite the arising out of the wrong or breach of contract was an actual total loss.
abandonment, as where the loss or injury was due to complained of, the insurance company shall - The ship was sunk in July 1, 1918. After several
the fault of the shipowner and the captain. The be subrogated to the rights of the insured futile attempts, it was finally raised on Sept. 20,
international rule is to the effect that the right of against the wrongdoer or the person who has 1918. It is faitr to assume that in its then condition
abandonment of vessels, as a legal limitation of a violated the contract. If the amount paid by much further time would be required to make the
shipowner’s liability, does not apply to cases where the insurance company does not fully cover necessary repairs and install the new machinery
the injury or average was occasioned by the the injury or loss, the aggrieved party shall before it could again be placed in commission.
shipowner’s own fault. be entitled to recover the deficiency from the During that time the owner would be deprived of the
3. YES person causing the loss or injury. use of its vessel or the interest on its investment.
- The doctrine of subrogation has its roots in equity. Disposition Petition is GRANTED. Respondent When those questions are considered the testimony
It is designed to promote and to accomplish justice FELMAN SHIPPING LINES is ordered to pay petitioner is conclusive that the cost of salvage, repair and
and is the mode which equity adopts to compel the PHILIPPINE AMERICAN GENERAL INSURANCE CO., reconstruction was more than the original cost of the
ultimate payment of a debt by one who in justice, INC., Seven Hundred Fifty-five Thousand Two ship at the time the policy was issued. As found by
equity and good conscience ought to pay. Therefore, Hundred and Fifty Pesos (P755,250.00) plus legal the trial court, “t is difficult to see how there could
the payment made by PHILAMGEN to Coca-Cola interest thereon counted from 29 November 1983, have been a more complete loss of the vessel than
Bottlers Philippines, Inc., gave the former the right the date of judicial demand, pursuant to Arts. 2212 that which actually occurred”. Upon the facts shown
to bring an action as subrogee against FELMAN. and 2213 of the Civil Code. here, any other construction would nullify the statute
Having failed to rebut the presumption of fault, the and as applied to the conditions existing in the
liability of FELMAN for the loss of the 7,500 cases of PHILIPPINE MFTG. CO. v. UNION INSURANCE Manila Bay, this kind of policy would be worthless,
1-liter Coca-Cola softdrink bottles is inevitable. SOCIETY OF CANTON and there would not be any consideration for the
- Sec. 113 of the Insurance Code provides that “(i)n 42 PHIL 378 premium.
every marine insurance upon a ship or freight, or JOHNS; November 22, 1921 2. NO
freightage, or upon anything which is the subject of - The defendant argues that the policy contains the
marine insurance, a warranty is implied that the ship FACTS provision that it “shall be of as force and effect as
is seaworthy.” Under Sec. 114, a ship is “seaworthy - The plaintiff’s steel tank lighter was insured by the surest writing or policy of insurance made in
when reasonably fit to perform the service, and to defendant company for absolute total loss. As a London”. However, for such law to apply to our
encounter the ordinary perils of the voyage, result of a typhoon, the lighter sunk in Manila Bay. courts the existence of such law must be proven. It
contemplated by the parties to the policy.” Thus it The plaintiff demanded payment from the defendant cannot apply when such proof is lacking.
becomes the obligation of the cargo owner to look insurance company but the latter refused. The Nevertheless, in the English practice, a ship is a total
for a reliable common carrier which keeps its company asked the plaintiff to salvage the ship, loss when she has sustained such extensive
vessels in seaworthy condition. He may have no which it was able to do so. damages that it would not be reasonably practical to
control over the vessel but he has full control in - With the plaintiff able to raise the lighter, repair her.
the selection of the common carrier that will reconstruct it and placed it in commission, the Disposition Decision reversed
transport his goods. He also has full discretion in the defendant insurance company claims that it was only
choice of assurer that will underwrite a particular liable for a total absolute loss and that there was no
venture. total destruction of the lighter.
- In policies where the law will generally imply a
warranty of seaworthiness, it can only be excluded
INSURANCE Page
97

- Petitioner filed a complaint in the RTC against the demonstrating that a specific provision expressly
insurance company seeking payment of the sum of excludes the loss from coverage.
P33,117.63 as damages plus attorney's fees and - In this case, the damage caused to the cargo has
expenses of litigation. Insurance company denied all not been attributed to any of the exceptions provided
the material allegations of the complaint and raised for nor is there any pretension to this effect. Thus,
CHOA TIEK SENG v. CA (FILIPINO MERCHANTS several special defenses as well as a compulsory the liability of respondent insurance company is
INSURANCE) counterclaim. Insurance company filed a third-party clear.
183 SCRA 223 complaint against respondents Ben Lines and broker. Disposition the decision appealed from is hereby
GANCAYO; March 15, 1990 - RTC dismissed the complaint, the counterclaim and REVERSED AND SET ASIDE and another judgment is
the third-party complaint with costs against the hereby rendered ordering the respondent Filipinas
NATURE petitioner. Appealed in CA but denied. MFR was Merchants Insurance Company, Inc. to pay the sum
Appeal from a decision of the Court of Appeals denied as well. of P33,117.63 as damages to petitioner with legal
interest from the filing of the complaint, plus
FACTS ISSUE attorney's fees and expenses of litigation in the
- Petitioner imported some lactose crystals from WON insurance company should be held liable even amount of P10,000.00 as well as the costs of the
Holland. if the technical meaning in marine insurance of an suit.
- The importation involved fifteen (15) metric tons “insurance against all risk" is applied
packed in 600 6-ply paper bags with polythelene FILIPINO MERCHANTS INS. CO. v. CA (supra
inner bags, each bag at 25 kilos net. The goods were HELD p.19)
loaded at the port at Rotterdam in sea vans on board YES
the vessel "MS Benalder' as the mother vessel, and - In Gloren Inc. vs. Filipinas Cia. de Seguros, 12 it ABOITIZ SHIPPING v. PHILAMGEN INSURANCE
thereafter aboard the feeder vessel "Wesser Broker was held that an all risk insurance policy insures 179 SCRA 357
V-25" of respondent Ben Lines Container, Ltd. (Ben against all causes of conceivable loss or damage, GANCAYCO; October 5, 1989
Lines for short). The goods were insured by the except as otherwise excluded in the policy or due to
respondent Filipino Merchants' Insurance Co., Inc. fraud or intentional misconduct on the part of the NATURE
(insurance company for short) for the sum of insured. It covers all losses during the voyage Petition for review on certiorari
P98,882.35, the equivalent of US$8,765.00 plus whether arising from a marine peril or not, including
50% mark-up or US $13,147.50, against all risks pilferage losses during the war. FACTS
under the terms of the insurance cargo policy. Upon - In the present case, the "all risks" clause of the - Marinduque Mining Industrial Corporation
arrival at the port of Manila, the cargo was policy sued upon reads as follows: (Marinduque) shipped on board SS Arthur Maersk
discharged into the custody of the arrastre operator "5. This insurance is against all risks of loss or from Boston, U.S.A. a shipment of 1 skid carton
respondent E. Razon, Inc. (broker for short), prior to damage to the subject matter insured but shall in parts for valves. The shipment was ordered from
the delivery to petitioner through his broker. Of the no case be deemed to extend to cover loss, Jamesbury, Singapore PTE, LTD., which issued the
600 bags delivered to petitioner, 403 were in bad damage, or expense proximately caused by delay cargo's packing list and Invoice number showing the
order. The surveys showed that the bad order bags or inherent vice or nature of the subject matter contents of the carton. The Philippine Consulate in
suffered spillage and loss later valued at P33,117.63. insured. Claims recoverable hereunder shall be Singapore issued invoice for the shipment showing
Petitioner filed a claim for said loss dated February payable irrespective of percentage." the contents and its total price of $39,419.60 and
16, 1977 against respondent insurance company in - The terms of the policy are so clear and require no the freight and other charges of $2,791.73. When
the amount of P33,117.63 as the insured value of interpretation. The insurance policy covers all loss or the cargo arrived in Manila, it was received and
the loss. damage to the cargo except those caused by delay deposited in the office of Aboitiz Shipping
- Respondent insurance company rejected the claim or inherent vice or nature of the cargo insured. It is Corporation (Aboitiz) for transhipment to Nonoc
alleging that assuming that spillage took place while the duty of the respondent insurance company to Island.
the goods were in transit, petitioner and his agent establish that said loss or damage falls within the - In July 1980, Marinduque, as consignee of the
failed to avert or minimize the loss by failing to exceptions provided for by law, otherwise it is liable cargo, made a report that said cargo was pilfered on
recover spillage from the sea van, thus violating the therefor. July 3, 1980 due to heavy rain at the Aboitiz
terms of the insurance policy sued upon; and that - An "all risks" provision of a marine policy creates a terminal and that of the total value of the cargo of
assuming that the spillage did not occur while the special type of insurance which extends coverage to $42,209.33, only $7,412.00 worth remains of the
cargo was in transit, the said 400 bags were loaded risks not usually contemplated and avoids putting cargo with the recommendation that the claim be
in bad order, and that in any case, the van did not upon the insured the burden of establishing that the made against Aboitiz.
carry any evidence of spillage. loss was due to peril falling within the policy's - The services of the Manila Adjusters and Surveyors
coverage. The insurer can avoid coverage upon Co. (Manila Adjusters) were engaged by the Phil-
INSURANCE Page
98

American General Insurance Co., Inc. (Phil Am) shipment in question was already pilfered. Obviously ISSUE
which came out with the report that the cargo in TC mistook said Marine Risk Note as an insurance WON Oriental Assurance can be held liable under its
question, when inspected, showed that it was policy when it is NOT. It is only an acknowledgment marine insurance policy based on the theory of a
pilfered. A confirmatory report was submitted by the or declaration of the private respondent confirming divisible contract of insurance and, consequently, a
Manila Adjusters. the specific shipment covered by its Marine Open constructive total loss
- On August 11, 1980 Marinduque then filed a claim Policy, the evaluation of the cargo and the
against Aboitiz in the amount of P246,430.80 chargeable premium. HELD
representing the value of the pilfered cargo. On the [c] The contention of the Aboitiz that it could not be NO
same day Marinduque filed a claim for the same liable for the pilferage of the cargo as it was stolen - The terms of the contract constitute the measure of
amount against the Phil-Am on the latter's policy. even before it was loaded on its vessel is untenable. the insurer liability and compliance therewith is a
Phil-Am paid Marinduque the sum of P246,430.80 as Aboitiz received cargo when it arrived in Manila at its condition precedent to the insured's right to recovery
insurer of the cargo. offices, and it was while in its possession and before from the insurer. Whether a contract is entire or
- Phil-Am then filed a complaint in RTC Manila loading it in its vessel that the cargo was pilfered. Its severable is a question of intention to be determined
against Aboitiz for recovery of same amount alleging liability is clear. by the language employed by the parties. The policy
that it has been subrogated to the rights of Disposition Petition DISMISSED. in question shows that the subject matter insured
Marinduque. Complaint dismissed and MFR denied. was the entire shipment of 2,000 cubic meters of
CA reversed. MFR thereof was denied. Hence, this ORIENTAL ASSURANCE v. CA (PANAMA SAW apitong logs. The fact that the logs were loaded on
petition. MILL) two different barges did not make the contract
200 SCRA 459 several and divisible as to the items insured. The
ISSUE MELENCIO-HERRERA; August 9, 1991 logs on the two barges were not separately valued or
WON petitioner Aboitiz was properly held liable to separately insured. Only one premium was paid for
the private respondent Phil-Am by the appellate NATURE the entire shipment, making for only one cause or
court Petition for review on certiorari consideration. The insurance contract must,
therefore, be considered indivisible.
HELD FACTS - More importantly, the insurer's liability was for
YES - Sometime in January 1986, private respondent "total loss only." A total loss may be either actual or
- The questioned shipment is covered by a Panama Sawmill Co., Inc. (Panama) bought, in constructive (Sec. 129, Insurance Code). An actual
continuing open insurance coverage (which took Palawan, 1,208 pieces of apitong logs, with a total total loss is caused by:
effect after Sept. 1, 1975, as contained in Marine volume of 2,000 cubic meters. It hired Transpacific (a) A total destruction of the thing insured;
Open Policy No. 100184) from the time it was loaded Towage, Inc., to transport the logs by sea to Manila (b) The irretrievable loss of the thing by sinking, or
aboard the SS Arthur Maersk in Boston, U.S.A. to the and insured it against loss for P1-M with petitioner by being broken up;
time it was delivered to the possession of petitioner Oriental Assurance Corporation (Oriental Assurance). (c) Any damage to the thing which renders it
at its offices at Pier 4 in Manila until it was pilfered - While the logs were being transported, rough seas valueless to the owner for the purpose for which
when the great majority of the cargo was lost on July and strong winds caused damage to one of the two he held it; or
3, 1980. Hence, petitioner Aboitiz was properly held barges resulting in the loss of 497 pieces of logs out (d) Any other event which effectively deprives the
liable to Phil-Am. of the 598 pieces loaded thereon. owner of the possession, at the port of destination,
Reasoning - Panama demanded payment for the loss but of the thing insured. (Section 130, Insurance
[a] Records of the case show that Phil-Am executed Oriental Assurance refuse on the ground that its Code).
a continuous and open insurance coverage covering contracted liability was for "TOTAL LOSS ONLY." - A constructive total loss is one which gives to a
goods of Marinduque imported into and exported - Unable to convince Oriental Assurance to pay its person insured a right to abandon, under Section
from the Philippines which took effect after Sept. 1, claim, Panama filed a Complaint for Damages against 139 of the Insurance Code. This provision reads:
1975, as contained in Marine Open Policy No. Oriental Assurance before the Regional Trial Court. SECTION 139. A person insured by a contract of
100184. A similar insurance coverage was also - RTC ordered Oriental Assurance to pay Panama marine insurance may abandon the thing insured,
executed by petitioner in favor of Marinduque for all with the view that the insurance contract should be or any particular portion thereof separately valued
its goods shipped or moved within the territorial liberally construed in order to avoid a denial of by the policy, or otherwise separately insured, and
limits of the Philippines also effective after Sept. 1, substantial justice; and that the logs loaded in the recover for a total loss thereof, when the cause of
1975 and contained in Marine Open Policy No. two barges should be treated separately such that the loss is a peril injured against,
100185. the loss sustained by the shipment in one of them (a) If more than three-fourths thereof in value is
[b] TC in dismissing the complaint apparently relied may be considered as "constructive total loss" and actually lost, or would have to be expended to
on Marine Risk Note No. 017545 issued by private correspondingly compensable. CA affirmed in toto. recover it from the peril;
respondent Phil-Am only on July 28, 1980 after the
INSURANCE Page
99

(b) If it is injured to such an extent as to reduce insurance premiums on the shipment was paid by the New Insurance Code, a total loss may either be
its value more than three-fourths; FAO petitioner actual or constructive. In case of total loss in Marine
xxx xxx xxx - On June 26, 1980, FAO was advised of the sinking Insurance, the assured is entitled to recover from
- The requirements for the application of Section 139 of the barge in the China Sea, hence it informed the underwriter the whole amount of his subscription
of the Insurance Code, quoted above, have not been petitioner thereof and, later, formally filed its claim - SEC. 130. An actual total loss is caused by: (c) Any
met. The logs involved, although placed in two under the marine insurance policy. On July 29, 1980, damage to the thing which renders it valueless to the
barges, were not separately valued by the policy, nor FAO was informed by LUSTEVECO of the recovery of owner for the purpose for which he held it; or
separately insured. Resultantly, the logs lost in the the lost shipment, for which reason FAO formally (d) Any other event which effectively deprives the
damaged barge in relation to the total number of filed its claim with LUZTEVECO for compensation of owner of the possession, at the port of destination of
logs loaded on the same barge cannot be made the damage to its cargo the thing insured.
basis for determining constructive total loss. The logs - LUZTEVECO failed and refused to pay. Pan Malayan -as said and proven, the seeds were of fragile
having been insured as one inseparable unit, the likewise failed to pay for the losses and damages nature. And the wetting of said seeds affected the
correct basis for determining the existence of sustained by FAO by reason of its inability to recover state of seeds. Thus rendering them useless for FAO.
constructive total loss is the totality of the shipment the value of the shipment from LUZTEVECO Although there were bags which were recovered,
of logs. Of the entirety of 1,208, pieces of logs, only - Pan Malayan claims that part of the cargo was these were “stained” and not in the same condition it
497 pieces thereof were lost or 41.45% of the entire recovered and thus the claim by FAO was was brought in. in addition to this, FAO did not
shipment. Since the cost of those 497 pieces does unwarranted. This is evidenced by two surveys upon receive any compensation for said recovered bags as
not exceed 75% of the value of all 1,208 pieces of the cargo wherein it was found that only around the same were distributed by LUZVETECO without
logs, the shipment cannot be said to have sustained 78% was lost. authorization of FAO
a constructive total loss under Section 139(a) of the - FAO filed a civil case against both LUZTEVECO and - the complete physical destruction of the subject
Insurance Code. Pan Malayan. Trial court found in favor of FAO and matter is not essential to constitute an actual total
Disposition judgment under review is SET ASIDE ordered both to pay jointly and severally the full loss. Such a loss may exist where the form and
amount of the claim. This was affirmed by CA specie of the thing is destroyed, although the
PAN MALAYAN INSURANCE v. CA (THE FOOD materials of which it consisted still exist (Great
AND AGRICULTURAL ORGANIZATION OF THE ISSUE Western Ins. Co. vs. Fogarty, N.Y., 19 Wall 640, 22
UNITED NATIONS) 1. WON respondent court committed a reversible L. Ed. 216), as where the cargo by the process of
201 SCRA 382 error in holding that the trial court is correct in decomposition or other chemical agency no longer
REGALADO; September 5, 1991 holding that there is a total loss of the shipment remains the same kind of thing as before (Williams
vs. Cole, 16 Me. 207).
FACTS HELD - It is thus clear that FAO suffered actual total loss
- The Food and Agricultural Organization of the 1. NO under Section 130 of the Insurance Code, specifically
United Nations (hereinafter referred to as FAO), - The law classifies loss into either total or partial. under paragraphs (c) and (d) thereof, recompense
ntended and made arrangements to send to Total loss may be actual or absolute, or it may for which it has been denied up to the present
Kampuchea 1,500 metric petitions of IR-36 certified otherwise be constructive or technical. Petitioner -Section 135 of the Insurance Code explicitly
rice seeds to be distributed to the people for seedling submits that respondent court erred in ruling that provides that "(u)pon an actual total loss, a person
purposes there was total loss of the shipment despite the fact insured is entitled to payment without notice of
- LUZTEVECO was to ship the cargo amounting to that only 27,922 bags of rice seeds out of 34,122 abandonment." This is a statutory adoption of a long
US$83,325.92 in respect of one lot of 1,500 metric bags were rendered valueless to FAO and the standing doctrine in maritime insurance law that in
petitions winch is the subject of the present action. shipment sustained only a loss of 78%. - FAO, case of actual total loss, the right of the insured to
The cargo was loaded on board LUZTEVECO Barge however, claims that, for all intents and purposes, it claim the whole insurance is absolute, without need
No. LC-3000 and consisted of 34,122 bags of IR-36 has practically lost its total or entire shipment in this of a notice of abandonment
certified rice seeds purchased by FAO from the case, inclusive of expenses, premium fees, and so
Bureau of Plant Industry for P4,602,270.00 forth, despite the alleged recovery by defendant PHILIPPINE AMERICAN LIFE INSURANCE
- FAO secured insurance coverage in the amount of LUZTEVECO. As found by the court below and COMPANY v. CA (ELIZA PULIDO)
P5,250,000.00 from petitioner, Pan Malayan reproduced with approval by respondent court, FAO 344 SCRA 260
Insurance Corporation "has never been compensated for this total loss or GONZAGA-REYES; November 15, 2000
- On June 16, 1980, FAO gave instructions to damage, a fact which is not denied nor controverted
LUZTEVECO to leave for Vaung Tau, Vietnam to - If there were some cargoes saved, by LUZTEVECO, NATURE
deliver the cargo which, by its nature, could not private respondent abandoned it and the same was This petition for review on certiorari seeks to reverse
withstand delay because of the inherent risks of sold or used for the benefit of LUZTEVECO or Pan the Decision of the Special Second Division of the
termination and/or spoilage. On the same date, the Malayan Corporation. Under Sections 129 and 130 of Court of Appeals
INSURANCE Page
100

since 1988. Piganto’s statement was signed by him, - We cannot likewise give credence to petitioner’s
FACTS and witnessed by his wife, Nenita Piganto. This submission that the inconsistencies in the
- On January 9, 1989, petitioner received from one report was petitioner’s basis for treating the disputed testimonies of the witnesses for plaintiff-private
Florence Pulido an application for life insurance, policy as void since April 1992, even before receipt respondent are in themselves evidence of fraud.
dated December 16, 1988, in the amount of of private respondent’s claim. Such alleged inconsistencies are matters of
P100,000.00 which designated her sister, herein credibility which had been ably passed upon by the
private respondent, as its principal beneficiary. ISSUE lower court.
Because the insurance applied for was non-medical, WON there was fraud (whether the insured, Florence - The absence of fraud, as a factual finding of the
petitioner did not require a medical examination and Pulido, was in fact dead before the application for lower court adopted by the Court of Appeals, entirely
issued a policy on the sole basis of the application on insurance on her life was made) consistent with the evidence on record, will not be
February 11, 1989. On April 1992, petitioner reversed and, hence, is final and conclusive upon
received private respondent’s claim, which declared HELD this Court.
that the insured, Florence Pulido, died of acute NO Disposition The instant petition is DENIED
pneumonia on September 10, 1991. - This the lower courts had effected ruled on, upon a
- Petitioner withheld payment on the ground that the preponderance of the evidence duly received from CHAPTER IX. CLAIMS SETTLEMENT &
policy claimed under was void from the start for both parties. We see no reversible error in the SUBROGATION
having been procured in fraud. It is petitioner’s finding of both respondent court and the trial court in
contention that even before they received private favor of the correctness of the entries in Certificate LONDRES v. NATIONAL LIFE INSURANCE
respondent’s claim for death benefits, their of Death, duly registered with the Local Civil 94 PHIL 627
investigation concerning the subject policy yielded Registrar of Bagulin, La Union, which declared that BAUTISTA ANGELO; March 29, 1954
the information that the insured, Florence Pulido, Florence Pulido died of acute pneumonia on
died in 1988, before the application for insurance on September 10, 1991. Dr. Irineo Gutierrez, the NATURE
her life was made. While this was communicated to Municipal Health Officer of Bagulin, La Union whose Appeal from a decision of the Court of First Instance
private respondent in a letter dated April 29, 1992, signature appeared in the death certificate, testified of Manila ordering defendant to pay to plaintiff the
private respondent had already filed her claim earlier in addition that he ministered to the ailing Florence sum of P3,000, Philippine currency, plus legal
that month. In another letter dated July 27, 1992, Pulido for two days immediately prior to her death. interest thereon from the time of the filing of the
however, petitioner confirmed to private respondent This fact is likewise noted in the death certificate. complaint until its full payment.
receipt of the claim papers and assured her that her - Death certificates, and notes by a municipal health
case was “being given preferential attention and officer prepared in the regular performance of his FACTS
prompt action”. duties, are prima facie evidence of facts therein - On April 14, 1943, the National Life Insurance
- Following the filing by private respondent of her stated. A duly-registered death certificate is Company of the Philippines issued a policy on the life
claim, petitioner caused another investigation considered a public document and the entries found of Jose C. Londres whereby it undertook to pay its
respecting the subject policy. Pursuant to the therein are presumed correct, unless the party who beneficiary upon his death the sum of P3,000. All the
findings of this second investigation, petitioner stood contests its accuracy can produce positive evidence premiums due under the policy were actually paid on
by its initial decision to treat the policy as void and establishing otherwise. Petitioner’s contention that their dates of maturity and the policy was in force
not to honor the claim. On November 9, 1992, the death certificate is suspect because Dr. Gutierrez when the insured died on February 7, 1945.
private respondent enlisted the services of counsel in was not present when Florence Pulido died, and Salvacion V. Londres, as beneficiary, demanded from
reiterating her claim for death benefits. Petitioner knew of Florence’s death only through Ramon the company the payment of the proceeds of the
still refused to make payment and thus, this action. Piganto, does not merit a conclusion of fraud. No policy, and her demand having been refused, she
- Petitioner: the results of its investigations having motive was imputed to Dr. Gutierrez for seeking to instituted the present action against the company in
indicated that the insured was already dead at the perpetuate a falsity in public records. Petitioner was the Court of First Instance of Manila.
time the policy was applied for. It also likewise unable to make out any clear motive as to - Defendant in its answer denied, for lack of
counterclaimed for attorney’s fees. The first report, why Ramon Piganto would purposely lie. Mere sufficient proof, the allegation that the insured died
prepared by one Dr. Benedicto Briones, was dated allegations of fraud could not substitute for the full on February 7, 1945, and set up the following
April 1, 1992, and had attached to it a questionnaire, and convincing evidence that is required to prove it. special defenses: (a) that plaintiff's claim is covered
responded to by one Ramon Piganto, who A failure to do so would leave intact the presumption by the Moratorium Law; (b) that the policy having
represented to be the brother-in-law of the insured of good faith and regularity in the performance of been issued during the Japanese occupation, it is
and the barangay chairman of Cardiz, Bagulin, La public duties, which was the basis of both presumed that its face value should be paid in
Union. To the question “Where does [Florence respondent court and the trial court in finding the Japanese currency, there being no provision in the
Pulido] reside now?”, Piganto had replied that date of Florence Pulido’s death to be as plaintiff- policy from which can be inferred that the parties
Florence Pulido used to live in Cardiz, but was dead private respondent maintained. contemplated payment in any other currency; (c)
INSURANCE Page
101

that the money paid by the insured as premiums, - In the case of Rutter vs. Esteban, 93 Phil., 68, the nothing immoral or unlawful in it." (Gomez vs.
together with the money received from other policy- Moratorium Law was declared invalid and Tabia)
holders, was all deposited by the defendant in the unconstitutional. Disposition Wherefore, the decision appealed from
Philippine National Bank and said deposit was - During those days of liberation, while the people is affirmed, with costs against appellant.
declared without value by Executive Order No. 49 of were rejoicing because of the happy event, the
the President of the Philippines; and (d) that the banks, the insurance companies, and for that matter
policy having been issued under abnormal other commercial and business firms, were still
circumstances, it should be considered in the light of feeling the adverse effects of the sudden fall of
equity which does not permit anyone to enrich values and were uncertain and apprehensive as to
himself at the expense of another. Defendant, the manner the readjustment would be made by the VDA. DE FERNANDEZ v. NATIONAL LIFE
however, as a proof of good faith, offered to pay the new Government. It is for this reason that the INSURANCE CO OF THE PHILS
value of the policy in accordance with the Ballantyne beneficiary, after realizing the truth about the death 105 PHIL 59
scale of values, or the sum of P2,400, Philippine of her husband, and after gathering evidence to ENDENCIA; January 27, 1959
currency. substantiate his death, had difficulty in effecting the
- It appears that the deceased took up the policy collection of her claim from the insurance company NATURE
under consideration on April 15, 1943 for the sum of because at that time it had not yet reopened for Appeal from CFI decision applying the Ballantyne
P3,000. All the premiums due under the policy were business purposes. Although the record does not scale of values upon the proceeds of life insurance
actually paid on their dates of maturity and the disclose the exact date on which the insurance taken and maturing during the Japanese occupation
policy was in force when the insured died on company reopened for this purpose, this Court can but claimed after liberation
February 7, 1945. On said date, the battle of the take judicial notice that it only did so after liberation.
liberation of the City of Manila was still raging. While At that time the legal tender was already the FACTS
the northern part may have been liberated, not so present currency. - National Life Insurance Company (NLIC) insured J.
the southern part, as shown from the very affidavits - As final plea, appellant invokes equity in its favor in Fernandez’s life for P10,000 upon his payment of
submitted by appellee wherein it was stated that on view of the nullification of the deposits made by it P444 from July 15, 1944 to July 14, 1945
the aforesaid date, the insured, Jose Londres, and with the Philippine National Bank of all fiat money - The insured died on November 2, 1944, while the
his two sons were taken by the Japanese soldiers received from its policyholders, which money was policy was in force
from their house at Singalong Street and were declared without value by Executive Order No. 49 of - After more than 7 years, in 1952, Atty de la Torre,
massacred by their captors. It may therefore be said the President of the Philippines. Appellant claims representing the benficiaries of the policy, informed
that the policy became due when the City of Manila that, considering the unexpected circumstances that the company that Fernandez had died in 1944, and
was still under the yoke of the enemy and became developed, the indemnity to be paid by it should be claimed the proceeds of the policy. The company
payable only after liberation which took place on suffered by it under Article 307 of the Code of said that the status of the policies issued during the
March 10, 1945 when President Osmena issued Commerce which provides: "When the deposits are Japanese occupation was still pending consideration
Proclamation No. 6 following the restoration of the of cash, with a specification of the coins constituting before the courts. NLIC said that because the policy
civil government by General Douglas Mac Arthur. them, . . . the increase or reduction which their matured upon the insured’s death in November,
And we say that the policy became payable only value may suffer shall be for the account of the 1944, they should compute the value of their claim
after liberation even if it matured sometime before, depositor." Appellant, by entering into an under the Ballantyne scale of values (which would
because before that eventuality the insurance insurance contract, cannot claim, if it suffers amount only to P500)
company, appellant herein, was not yet in a position loss, that the beneficiary cannot enrich herself - beneficiaries commenced suit, and the lower court
to pay the value of the policy for the simple reason at its expense. This is a risk attendant to any sustained the stand of the company, dismissed the
that it had not yet reopened. wagering contract. One who gambles and loses complaint.
cannot be heard to complain of his loss. To - beneficiaries maintain that the obligation of the
ISSUE appellant, we can only repeat the following company to pay accrued not upon the death of
WON the amount of P3,000 which appellant bound admonition: Fernandez, but only upon the receipt and approval
itself to pay to the insured under the policy upon his "The parties herein gambled and speculated on the by the company, on proof of death of the insured,
death should be paid in accordance with the present date of the termination of the war and the which was in 1954. The policy reads:
currency or should be adjusted under the Ballantyne liberation of the Philippines by the Americans. This National Life Insurance Company of the Philippine
scale of values can be gleaned from the stipulation about hereby agrees to pay at its Home Office, Manila,
redemption, particularly that portion to the effect Ten Thousand Pesos to Juan D. Fernandez
HELD that redemption could be effected not before the (hereinafter called the insured) on the 15th day of
YES, present currency. expiration of one year from June 24, 1944. This July, 1964, if the Insured is living and this Policy is
Reasoning kind of agreement is permitted by law. We find in force, or upon receipt and approved at its Office
INSURANCE Page
102

of due proofs of the title of the claimant and of the interest thereon to which the beneficiaries may be trial court denied EASCO's motion. On appeal, the
prior death of the Insured while this Policy is in entitled to collect in case of unwarranted refusal of Court of Appeals reversed the trial court’s denial of
force to Teresa Duat Vda. De Fernandez, Maria T. the company to pay, and also to enable the insurer EASCO’s motion and ruled that the applicable
and Manuela Fernandez, mother and sisters to verify or check on the fact of death which it may interest is 6% per annum. Hence, this petition.
respectively of the Insured (Hereinafter called the even validly waive. It is the happening of the
Beneficiary) subject to the right of the Insured to suspensive condition of death that renders a life ISSUE
change the beneficiary as stated on the second policy matured, and not ht efiling of proof of death WON the applicable rate of interest is 12% per
page of this Policy. which, as above stated, is merely procedural. The annum
- The above stipulation is apparently based on Sec. insured having died during the Japanese occupation,
91-A of the Insurance Law which provides as follows: the proceeds of his policy should be adjusted HELD
The proceeds of a life insurance policy shall be paid accordingly, for “The rule is already settled that NO
immediately upon maturity of the policy, unless such where a debtor could have paid his obligation at any - Sections 243 and 244 of the Insurance Code apply
proceeds are made payable in installments or a as an time during the Japanese occupation, payment after only when there is an unjustified refusal or
annuity, in which case the installments or annuities liberation must be adjusted in accordance with the withholding of payment on the insured’s claim. In
shall be paid as they become due: Provided, Ballantyne schedule (De Asis vs. Agdamag, among this case, EASCO's refusal to settle the claim to Tio
however, That in case of a policy maturing by the other cases). (Collaboration is defined as the acts Khe Chio was based on some ground which, while
death of the insured, the proceeds thereof shall be of working together in a joint project. not sufficient to free it from liability under its policy,
paid within sixty days after presentation of the claim Disposition Judgment affirmed nevertheless is sufficient to negate any assertion
and filing of the proof of the death of the insured. that in refusing to pay, it acted unjustifiably. Simply
Refused to pay the claim within the time prescribed TIO KHE CHIO v. CA (EASTERN ASSURANCE & put, the said provisions of the Insurance Code are
herein will entitle the beneficiary to collect interest SURETY) not pertinent to the instant case. They apply only
on the proceeds of the policy for the duration of the 202 SCRA 119 when the court finds an unreasonable delay or
delay at the rate of six per centum per annum, FERNAN; September 30, 1991 refusal in the payment of the claims.
unless such failure or refusal to pay is based on the - Circular No. 416 of the Central Bank, which raised
ground that the claim is fraudulent . . . . FACTS the legal rate of interest from 6% to 12% per annum
- Based on the foregoing provision of law and the - Petitioner Tio Khe Chio imported 1,000 bags of refers only to loans or forbearances of money, goods
aforequoted stipulation as well as on the allegation fishmeal valued at $36,000.30 from Agro Impex, or credits and court judgments thereon but not to
that the filing of proof of death by the beneficiaries is S.A. Dallas, Texas, U.S.A. The goods were insured court judgments for damages arising from injury to
a condition precedent of the demandability of the with respondent EASCO and shipped on board the persons and loss of property which does not involve
obligation of the insurer to pay the proceeds, M/V Peskov, a vessel owned by Far Eastern Shipping a loan. Clearly, the applicable law is Article 2209 of
appellants claim that they should be paid P10,000 in Company. When the goods reached Manila, they the Civil Code.
Philippine currency and not under the Ballantyne were found to have been damaged by sea water - And in the light of the fact that the contending
scale of values. which rendered the fishmeal useless. Petitioner filed parties did not allege the rate of interest stipulated in
a claim with EASCO and Far Eastern Shipping. Both the insurance contract, the legal interest was
ISSUE refused to pay. Whereupon, petitioner sued them properly pegged by the Appellate Court at 6% per
WON the policy matured upon the death of the before the then Court of First Instance of Cebu for cent.
insured damages. EASCO, as the insurer, filed a
counterclaim against the petitioner for the recovery CATHAY INSURANCE v. CA(LUGAY)
HELD of the unpaid insurance premiums. 174 SCRA 11
YES - The trial court rendered judgment in favor of GRINO-AQUINO; June 5, 1989
Ratio In life insurance, the policy matures either petitioner. The judgment became final as to EASCO
upon the expiration of the term set forth therein, or but the shipping company appealed to the Court of FACTS
upon his death occuring at any time prior to the Appeals and was absolved from liability by the said - Petitioners are 6 insurance companies that issued
expiration of such stipulated term, in which case, the court. fire insurance policies for the total sum of
proceeds are payable to his beneficiaries within sixty - The trial court, upon motion by petitioner, issued a P4,000,000 to the Cebu Filipina Press owned by
days after their filing of proof of death. writ of execution against EASCO. The sheriff Emilia Chan Lugay. The fire policies described the
Reasoning enforcing the writ reportedly fixed the legal rate of insured property as "stocks of Printing materials,
- The sixty day period fixed by law within which to interest at 12%. Respondent EASCO moved to quash papers and general merchandise usual to the
pay the proceeds after presentation of proof of death the writ alleging that the legal interest to be Assured's trade" stored in a one-storey building of
Is merely procedural in nature, evidently to computed should be 6% per cent per annum in strong materials housing the Cebu Filipina Press
determine the exact amount to be paid and the accordance with Article 2209 of the Civil Code. The located at UNNO Pres. Quirino cor. Don V. Sotto
INSURANCE Page
103

Sts., Mabolo, Cebu City. The co-insurers were December 15, 1982, her cause of action had already the claim within the time fixed in both Sec. 242 and
indicated in each of the policies. All, except one accrued. 243 of the IC.
policy (Paramount's), were renewals of earlier 2. YES - In view of the not insubstantial value of the private
policies issued for the same property. - There is no merit in the petitioners' contention that respondent's claims and the considerable time and
- On December 18, 1981, the Cebu Filipina Press was the proofs of loss were insufficient because Lugay effort expended by them and their counsel in
razed by electrical fire together with all the stocks failed to comply with the adjuster's request for the prosecuting these claims for the past 8 years,
and merchandise stored in the premises. On submission of her bank statements. Condition No. 13 attorney's fees were properly awarded to the private
January 15, 1982, Lugay submitted sworn of the policy does not require the insured to produce respondents.
Statements of Loss and Formal Claims to the her bank statements. Therefore, the insured was not
insurers, through their adjusters. She claimed a total obligated to produce them and the insurers had no 5. YES
loss of P4,595,000. right to ask for them. Condition No. 13 was prepared - An award equivalent to 10% of the proceeds of the
- After nearly 10 months of waiting, she sued to by the insurers themselves, hence, it should be policies would be more reasonable than the 20%
collect on December 15, 1982. The insurance taken most strongly against them. awarded by the trial court and the CA.
companies denied liability, alleging violation of 3. NO Disposition Decision of the CA AFFIRMED with
certain conditions of the policy, misdeclaration, and - Both the trial court and the CA noted that the MODIFICATION.
even arson which was not seriously pressed for, proofs were ample and more than enough for
come the pre-trial, the petitioners offered to pay defendant insurers to do a just assessment NODA v. CRUZ
50% of her claim, but she insisted on full recovery. supporting the 1981 fire claim for an amount 151 SCRA 227
- Trial court rendered judgment in her favor ordering exceeding four million pesos. FERNAN; June 22, 1987
the insurers to pay her a total of P4,000,000 as 4. NO
indemnity, P48,000 representing expenses of the - The award of double interest on the claim is lawful NATURE
plaintiff, a separate amount of 20% of the and justified under Sections 243 and 244 of the Petition to review decision of the Insurance
P4,000,000 representing fees of counsel, interests at Insurance Code which provide: Commissioner
the rate of twice the ceiling being prescribed by the Sec. 243 Refusal or failure to pay the loss or
Monetary Board starting from the time when the damage within the time prescribed herein will FACTS
case was filed, and finally, with costs. CA affirmed. entitle the assured to collect interest on the - In 1977, Noda obtained from Zenith Insurance
proceeds of the policy for the duration of the delay Corporation 2 fire insurance policies: [1] No. F-
ISSUES at the rate of twice the ceiling prescribed by the 03724 with a face value of P30k covering the goods
1. WON the insured's cause of action had already Monetary Board. and stocks in trade in his business establishment at
accrued before she filed her complaint Sec. 244 In case of any litigation for the the market site in Mangagoy, Bislig, Surigao del Sur
2. WON sufficient proofs of loss had been presented enforcement of any policy or contract of insurance, and [2] No. F-03734 with a face value in the
by the insured it shall be the duty of the Commissioner or the aggregate amount of P100k and consisting of Item 1
3. WON the private respondents claim for loss was Court, as the case may be, to make a finding as to for P40k on household furniture, fixtures, fittings and
inflated whether the payment of the claim of the insured other personal effects, and Item 2 for P60k on stocks
4. WON lower court erred in awarding damages to has been unreasonably denied or withheld; and in in trade usual to petitioner's retail business situated
the private respondent in the form of interest the affirmative case, the insurance company shall in a two-storey building at 039 Barreda St.,
equivalent to double the interest ceiling set by the be adjudged to pay damages which shall consist of Mangagoy, Bislig, Surigao del Sur.
Monetary Board attorney's fees and other expenses incurred by the - While both policies were in force, fire destroyed
5. WON attorney's fees awarded were exorbitant insured person by reason of such unreasonable petitioner's insured properties at the market site on
denial or withholding of payment plus interest of September 5, 1977 and at Barreda St. on November
HELD twice the ceiling prescribed by the Monetary Board 9, 1977.
1. YES of the amount of claim due the insured. - When petitioner failed to obtain indemnity on his
- As the fire which destroyed the Cebu Filipina Press - The petitioners' contention that the charging of claims from Zenith, he filed a complaint with the
occurred on December 19, 1981 and the proofs of double interest was improper because no Insurance Commission praying that Zenith be
loss were submitted from January 15, 1982 through unreasonable delay in the processing of the fire claim ordered to pay him P130kj representing the value of
June 21, 1982 in compliance with the adjusters' was proven is refuted by the trial court's explicit the 2 policies insured by respondent with interest at
numerous requests for various documents, payment finding that "there was a delay that was not 12% per annum, plus damages, attorney's fees and
should have been made within 90 days thereafter reasonable in processing the claim and doing other expenses of litigation. ...
(Sec 243), or on or before September 21, 1982. payments". Under Section 244, a prima facie - Zenith interposed that petitioner had no cause of
Hence, when the assured filed her complaint on evidence of unreasonable delay in payment of the action; that Policy No. F-03724 was not in full force
claim is created by the failure of the insurer to pay and effect at the time of the fire because the
INSURANCE Page
104

premium on the policy was not paid; that Zenith's if they are unsatisfactory, they may not set up for DELSAN TRANSPORT, INC. v. CA (AMERICAN
liability under Policy No. F-03734, if any, was limited themselves an arbitrary standard of satisfaction. HOME ASSURANCE)
to P15,472.50 in view of the co-insurance; and that Substantial compliance with the requirements will 369 SCRA 24
petitioner failed to substantiate his claim as to the always be deemed sufficient. DE LEON, JR; November 15, 2001
value of the goods reputedly destroyed by fire. - Zenith introduced in evidence the final report on
- While the case was pending, Zenith settled Policy No. F-03734 submitted by its own adjuster, NATURE
petitioner’s fire loss claim under Item 1 of Policy No. Dela Merced Adjustment Corporation. Respondent A petition for review on certiorari of the decision of
03734 in the amount of P15,472.50. Commissioner however ignored such report, CA.
- Insurance Commissioner allowed petitioner to reasoning that with regard to Item 2 of Policy No. F-
recover under said policy and ordered Zenith to pay 03734 the claim for loss of the stocks in trade was FACTS
him the amount of P20k with legal interest from the not successfully proven in view of petitioner's failure - Caltex entered into a contract of affreightment with
date the complaint was filed, including P1k as to present evidence; that the adjuster's report the petitioner, Delsan Transport Lines, Inc.
attorney's fees but excluding the actual, moral and deserved scant consideration since the allegations (petitioner), for a period of one year whereby the
exemplary damages prayed for. As for petitioner's therein were not substantiated, and that said report said common carrier agreed to transport Caltex’s
claim under Policy No. F-03734, she held that in view did not even make a recommendation for payment. industrial fuel oil from the Batangas-Bataan Refinery
of the payment of P15,472.50 to petitioner, Zenith - A scrutiny of the abovementioned adjuster's report to different parts of the country. Delsan took on
had fully discharged its liability under said policy reveals that together with the formal demand for full board its vessel, MT Maysun, 2,277.314 kiloliters of
which covered furniture, fixtures, fittings and other indemnity, petitioner submitted his income tax industrial fuel oil of Caltex to be delivered to the
personal belongings of petitioner. return for 1978, purchase invoices, certification from Caltex Oil Terminal in Zamboanga City. The
- In allowing recovery under Policy No. F-03734, his suppliers as to his purchases, and other shipment was insured by American Home Assurance
Commissioner placed much weight on the final report supporting papers. The report even took into account Corporation (respondent).
prepared by Dela Merced Adjustment Corporation, an the appraisals of the other adjusters and concluded - August 14, 1986: MT Maysun set sail from
independent fire, marine and casualty adjuster that the total loss sustained by petitioner in his Batangas for Zamboanga City. The vessel sank in
contracted by Zenith to investigate the claims of its household effects and stocks in trade reached the early morning of August 16, 1986 near Panay
various policyholders. Said report concluded that P379,302.12. But after apportioning said amount Gulf in the Visayas taking with it the entire cargo of
"the sound value of P26,666.67 represented the among petitioner's six different insurers [the co- fuel oil.
whole loss and damage" incurred by petitioner, but insurance being known to Zenith], the liability of - Respondent paid Caltex P5,096,635.57
with the application of the three-fourths loss clause, Zenith was placed at P60,592.10. It therefore representing the insured value of the lost cargo.
Zenith's liability was reduced to P20k. recommended that Zenith pay the petitioner the Exercising its right of subrogation under Article 2207
amount of P60, 592.10. of the New Civil Code, the private respondent
ISSUES - Said document was offered as evidence by Zenith demanded of the petitioner the same amount it paid
1. WON Insurance Commissioner erred in denying itself and could very well be considered as an to Caltex. Delsan refused to pay, forcing American
petitioner's demand for P60k under Item 2 of Policy admission of its liability up to the amount home to file a case for collection in the RTC.
No. F-03734 recommended. Being in the nature of an admission - RTC found that the vessel, MT Maysun, was
2. WON Insurance Commissioner erred in not against interest, it is the best evidence which affords seaworthy to undertake the voyage, and that the
awarding in favor of petitioner exemplary damages the greatest certainty of the facts in dispute. incident was caused by an unexpected inclement
for Zenith's unjustified and wanton refusal to pay Respondent Commissioner should not have weather condition or force majeure, thus exempting
petitioner's claim under the said two insurance perfunctorily dismissed that particular evidence as a the common carrier from liability for the loss of its
contracts worthless piece of paper. cargo.
2. NO - CA reversed RTC decision on the basis of evidence
HELD - There is no showing that Zenith, in contesting from PAG-ASA that there were no 20 ft. waves in the
1. YES payment, had acted in a wanton, oppressive or area. CA ruled that the petitioner is liable on its
- To prove the existence of the stocks in trade malevolent manner to warrant the imposition of obligation as common carrier to respondent
covered by Policy No. F-03734, petitioner offered his corrective damages. insurance company as subrogee of Caltex.
testimony and that of his wife as well as Disposition Zenith Insurance Corporation ordered Petitioner’s Claim
documentary exhibits. The foregoing evidence for to pay petitioner Norman R. Noda the sum of > In every marine insurance upon a ship or freight,
petitioner preponderantly showed the presence of P60,592.10 with legal interest from the filing of the or freightage, or upon any thing which is the subject
some P590k worth of goods in his retail store during complaint until full payment, but deducting of marine insurance there is an implied warranty by
the fire of November 9, 1977. therefrom the amount of P15,472.50 which it had the shipper that the ship is seaworthy.10 When
- While the insurer, and the Insurance Commissioner earlier paid to petitioner.
for that matter, have the right to reject proofs of loss 10
Sec. 113 Insurance Code
INSURANCE Page
105

private respondent paid Caltex the value of its lost Ratio Seaworthiness relates to a vessel’s actual Disposition Petition is denied, and the decision of
cargo, the act of the private respondent is equivalent condition. Neither the granting of classification or the CA is affirmed.
to a tacit recognition that the ill-fated vessel was the issuance of certificates establishes
seaworthy. seaworthiness.
Reasoning
Respondent’s Comment - Common carriers are bound to observe
> American Home Assurance is entitled to payment extraordinary diligence in the vigilance over the FINMAN GENERAL ASSURANCE CORP v.
by its right of subrogation. goods and for the safety of passengers transported INOCENCIO
by them, according to all the circumstances of each 179 SCRA 480
ISSUES case. There is no liability if the loss, destruction or FELICIANO; November 15, 1989
1. WON payment made by American Home to Caltex deterioration is by force majeure.
for the insured value of the lost cargo amounted to - The tale of strong winds and big waves by the said FACTS
an admission that the vessel was seaworthy, thus officers of the petitioner however, was effectively - Pan Pacific is a recruitment and employment
precluding any action for recovery against the rebutted and belied by the weather report from PAG- agency. It posted surety bond issued by Finman
petitioner ASA. MT Maysun sank with its entire cargo for the General Assurance and was granted license to
2. WON MT Maysun was seaworthy at the time of the reason that it was not seaworthy. There was no operate by POEA.
voyage (outline topic) squall or bad weather or extremely poor sea - Inocencio, Palero, Cardones, Hernandez filed with
3. WON non-presentation of the marine insurance condition in the vicinity when the said vessel sank. POEA complaints against Pan Pacific for violation of
policy bars the complaint for recovery of sum of - Petitioner may not escape liability by presenting in Labor Code and for refund of placement fees. POEA
money for lack of cause of action evidence certificates that tend to show that at the Administrator motu propio impleaded Finman as
time of dry-docking and inspection by the Philippine surety for Pan Pacific.
HELD Coast Guard MT Maysun, was fit for voyage. These - Pan Pacific moved out and no notice of transfer was
1. NO pieces of evidence do not necessarily take into furnished to POEA as required. POEA considered
Ratio The fact of payment grants American Home account the actual condition of the vessel at the time that constructive service of complaints had been
the subrogatory right which enables it to exercise of the commencement of the voyage. At the time of effected.
legal remedies that would otherwise be available to dry-docking and inspection, the ship may have - Finman denied liability and said that
Caltex as owner of the lost cargo against the appeared fit. The certificates issued, however, do - POEA had no jurisdiction over surety bonds;
petitioner common carrier. not negate the presumption of unseaworthiness jurisdiction is vested in Insurance Commission
Reasoning triggered by an unexplained sinking. - Finman had not violated Labor Code
Art. 2207. (Civil Code) - Authorities are clear that diligence in securing - Complainants have no cause of action against
If the plaintiff’s property has been insured, and he certificates of seaworthiness does not satisfy the Finman
has received indemnity from the insurance vessel owner’s obligation. Also securing the approval - Amounts claimed were paid as deposits and not as
company for the injury or loss arising out of the of the shipper of the cargo, or his surveyor, of the placement fees.
wrong or breach of contract complained of, the condition of the vessel or her stowage does not - POEA Administrator issued Order that respondents
insurance company shall be subrogated to the establish due diligence if the vessel was in fact should pay. Finman appealed to Secretary of Labor.
rights of the insured against the wrongdoer or the unseaworthy, for the cargo owner has no obligation Secretary upheld the POEA order.
person who has violated the contract. If the in relation to seaworthiness.
amount paid by the insurance company does not 3. NO ISSUE
fully cover the injury or loss, the aggrieved party Ratio The presentation in evidence of the marine WON Finman can be held liable for complainants’
shall be entitled to recover the deficiency from the insurance policy is not indispensable in this case claims against Pan Pacific
person causing the loss or injury. before the insurer may recover from the common
- The right of subrogation is designed to promote carrier the insured value of the lost cargo in the HELD
and to accomplish justice and is the mode which exercise of its subrogatory right. The subrogation YES
equity adopts to compel the ultimate payment of a receipt, by itself, is sufficient to establish not only - Under Insurance Code, liability of surety in a
debt by one who in justice and good conscience the relationship of respondent as insurer and Caltex, surety bond is joint and several with the principal
ought to pay. It is not dependent upon, nor does it as the assured shipper of the lost cargo of industrial obligor.
grow out of, any privity of contract or upon written fuel oil, but also the amount paid to settle the - Conditions of a bond specified and required in the
assignment of claim. It accrues simply upon insurance claim. The right of subrogation accrues provisions of a statute providing for submission of
payment by the insurance company of the insurance simply upon payment by the insurance company of the bond, are incorporated into all bonds tendered
claim. the insurance claim. under that statute even though not set out in
2. NO printer’s ink.
INSURANCE Page
106

- POEA held and Secretary of Labor affirmed that *** CARRIER’s defense of prescription is made to - The prescription clause could be harmonized with
Pan Pacific had violated Labor Code, and at least one rest on the following stipulation of the bill of lading: section 61-A of the Insurance Act by taking it to
of the conditions for the grant and continued use of In any event the carrier and the ship shall be mean that the time given the insured for bringing his
the recruitment license. POEA and Secretary of discharged from all liability in respect of loss or suit is twelve months after the cause of action
Labor can require Pan Pacific to refund the damage unless suit is brought within one year accrues.
placement fees and to impose the fine. after the delivery of the goods or the date when - If so, when did the cause of action accrue? Chia
- If Pan Pacific is liable, and if Finman is solidarily the goods should have been delivered. (This Yu’s action did not accrue until his claim was finally
liable with Pan Pacific, then Finman is liable both to stipulation is but a repetition of a provision in the rejected by the insurance company. This is because,
private respondents and to POEA. CA 65 which says that bills of lading covering before such final rejection, there was no real
- Cash and surety bonds are required from shipments from the US to the Phils should be necessity for bringing suit.
recruitment companies as means of ensuring prompt brought w/in one year after the delivery of the - As the policy provides that the insured should file
and effective recourse against such companies when goods or the date when the goods should have his claim, first, with the carrier and then with the
held liable. Public policy will be effectively negated if been delivered to hold the carrier liable.) insurer, he had a right to wait for his claim to be
POEA and the DoLE were held powerless to compel a *** INSURER’s claim of prescription is founded upon finally decided before going to court.
surety company to make good on its solidary the terms of the policy and not upon the bill of - Furthermore, there is nothing in the record to show
undertaking. lading. (But in our jurisdiction, as per A1144, that the claim was rejected in the year 1947, either
prescription is 10 years after action accrues.) by the insurance company in London or its settling
EAGLE STAR INSURANCE CO LTD v. CHIA YU No suit action on this Policy, for the recovery of agents in the Philippines.
96 PHIL 696 any claim, shall be sustainable in any Court of law - For the purpose of this action, Chia Yu's claim was
REYES; March 31, 1955 or equity unless the insured shall have fully considered to have been finally rejected by the
complied with all the terms and conditions of this insurer on April 22, 1948. Having been filed within
NATURE Policy nor unless commenced with twelve (12) twelve months form that date, the action cannot be
Certiorari months next after the happening of the loss . . . deemed to have prescribed even on the supposition
that the period given the insured for bringing suit
FACTS ISSUE under the prescriptive clause of the policy is twelve
- Atkin, Kroll & Co., loaded on the S. S. Roeph WON ATKIN’ s action has prescribed months after the accrual of the cause of action.
Silverlight owned and operated by Leigh Hoegh & - Contractual limitations contained in insurance
Co., A/S, of San Francisco California, 14 bales of HELD policies are regarded with extreme jealousy by
assorted underwear valued at P8,085.23 consigned NO courts and will be strictly construed against the
to Chia Yu in the City of Manila. - Being contrary to the law of the forum, the insurer and should not be permitted to prevent a
- The shipment was insured against all risks by Eagle stipulation in the policy cannot be given effect as it recovery when their just and honest application
Star Ins. Co. of San Francisco, California, under a would reduce the period allowed the insured for would not produce that result. (46 C. J. S. 273.)
policy issued to the shipper and by the latter bringing his action to less than one year (because Disposition Judgment appealed from is REVERSED
assigned to the consignee. the prescription period begins from the “happening with respect to the carrier and its agents but
- The vessel arrived in Manila but of the 14 bales of the loss” and that before any suit could be AFFIRMED with respect to the insurance company
(a.k.a. freights =p) consigned to Chia Yu only 10 sustained the insured shall have to comply with the and its agents.
were delivered to him as the remaining 3 could not terms and conditions of the policy first TF lessening
be found.3 of those delivered were also found the period to less than a year. )
damaged to the extent of 50 per cent. - Insular Government vs. Frank(13 Phil. 236)~
-Chia Yu claimed indemnity for the missing and "matters respecting a remedy, such as the bringing
damaged bales. But the claim was declined, first, by of suit, admissibility of evidence, and statute of
the carrier and afterward by the insurer, whereupon limitations, depend upon the law of the place where ACCFA v. ALPHA INSURANCE
Chia Yu brought the present action against both, the suit is brought" TF any policy clause repugnant 24 SCRA 151
including their respective agents in the Philippines. to this amendment to the Insurance Act cannot be REYES; July 29, 1968
- An action was filed at the CFI after more than 2 given effect in an action in our courts.
years after delivery of the damaged bales and the SEC. 61-A. (Insurance Code) ~ Any condition, FACTS
date when the missing bales should have been stipulation or agreement in any policy of - In order to guarantee the Asingan Farmers'
delivered, the action was resisted by the Atkins and insurance, limiting the time for commencing an Cooperative Marketing Association, Inc. (FACOMA)
Eagle Star principally on the ground of prescription. action thereunder to a period of less than one year against loss on account of "personal dishonesty,
-TC favored Chia Yu and CA affirmed. from the time when the cause of action accrues, is amounting to larceny or estafa of its Secretary-
void. Treasurer, Ladines, the appellee, Alpha Insurance &
INSURANCE Page
107

Surety Company had issued, on 14 February 1958, commencing an action thereon to less than one year Appeal from judgment of the CFI ordering the
its bond, No. P-FID-15-58, for the sum of P5,000 from the time the cause of action accrues defendant Fulton Fire Insurance Co. to pay the
with said Ladines as principal and the appellee as plaintiffs the sum of P10,000.00, with interest, and
solidary surety. On the same date, the Asingan HELD an additional sum of P2,000.00 as attorney's fees,
FACOMA assigned its rights to the appellant, NO and costs.
Agricultural Credit Cooperative and Financing - A fidelity bond is, in effect, in the nature of a
Administration (ACCFA for short), with approval of contract of insurance against loss from misconduct, FACTS
the principal and the surety. and is governed by the same principles of - The stocks of general merchandise in the store of
- During the effectivity of the bond, Ladines interpretation. Consequently, the condition of the the Ang spouses are insured with Fulton. While the
converted and misappropriated, to his personal bond in question, limiting the period for bringing insurance was in force, fire destroyed the goods. The
benefit, some P11,513.22 of the FACOMA funds, of action thereon, is subject to the provisions of Section Angs filed their first claim immediately after the fire.
which P6,307.33 belonged to the ACCFA. Upon 61-A of the Insurance Act (No. 2427), as amended - Their claim was denied on April 6, 1956. They
discovery of the loss, ACCFA immediately notified in by Act 4101 of the pre-Commonwealth Philippine received notice on April 19, 1956.
writing the survey company on 10 October 1958, Legislature, prescribing that: - The Angs brought an action against the agent on
and presented the proof of loss within the period SEC. 61-A: A condition, stipulation or agreement May 11, 1956. The court denied the suit and the mfr
fixed in the bond; but despite repeated demands the in any policy of insurance, limiting the time for on Sept. 3 and 12, 1957.
surety company refused and failed to pay. commencing an action thereunder to a period of - The Angs filed against Fulton on May 26, 1958.
Whereupon, ACCFA filed suit against appellee on 30 less than one year from the time when the cause - There was a clause in the policy:
May 1960. of action accrues is void. 13.If the claim be in any respect fraudulent, or if
- Defendant Alpha Insurance & Surety Co., Inc., - Since a "cause of action" requires, as essential any false declaration is made or used in support
(now appellee) moved to dismiss the complaint for elements, not only a legal right of the plaintiff and a thereof, or if any fraudulent means or devices are
failure to state a cause of action, giving as reason correlative obligation of the defendant but also "an used by the Insured or any one acting on his
that (1) the same was filed more than one year after act or omission of the defendant in violation of said behalf to obtain any benefit under this Policy, or, if
plaintiff made claim for loss, contrary to the eighth legal right," the cause of action does not accrue until the loss or damage be occasioned by the wilful act
condition of the bond, providing as follows: the party obligated refuses, expressly or impliedly, to or with the connivance of the Insured, or, if the
EIGHT LIMITATION OF ACTION: No action, suit or comply with its duty (in this case, to pay the amount claim be made and rejected and an action or suit
proceeding shall be had or maintained upon this of the bond). The year for instituting action in court be not commenced within twelve months after
Bond unless the same be commenced within one must be reckoned, therefore, from the time of such rejection or (in case of arbitration taking
year from the time of making claim for the loss upon appellee's refusal to comply with its bond; it can not place in pursuance of the 18th condition of this
which such action, suit or proceeding, is based, in be counted from the creditor's filing of the claim of Policy) within twelve months after the arbitrator or
accordance with the fourth section hereof. loss, for that does not import that the surety arbitrators or umpire shall have made their award
(2) the complaint failed to show that plaintiff had company will refuse to pay. In so far, therefore, as all benefit under this Policy shall be forfeited."
filed civil or criminal action against Ladines, as condition eight of the bond requires action to be filed
required by conditions 4 and 11 of the bond; and (3) within one year from the filing of the claim for loss, ISSUE
that Ladines was a necessary and indispensable such stipulation contradicts the public policy WON the suit against the agent tolled the
party but had not been joined as such. expressed in Section 61-A of the Philippine Insurance prescription period, such that the filing against
- At first, the Court of First Instance denied Act. Fulton was only 9 months after the claim was
dismissal; but, upon reconsideration, the court - Condition eight of the bond, therefore, is null and rejected
reversed its original stand, and dismissed the void, and the appellant is not bound to comply with
complaint on the ground that the action was filed its provisions. The discouraging of unnecessary HELD
beyond the contractual limitation period. Hence, this litigation must be deemed a rule of public policy, NO
appeal. considering the unrelieved congestion in the courts. - The bringing of the action against the Paramount
As a consequence of the foregoing, action may be Surety & Insurance Company, the agent of the
ISSUE brought within the statutory period of limitation for defendant company, cannot have any legal effect
WON the provision of a fidelity bond that no action written contracts (New Civil Code, Article 1144). except that of notifying the agent of the claim.
shall be had or maintained thereon unless Beyond such notification, the filing of the action can
commenced within one year from the making of a ANG v. FULTON FIRE INSURANCE CO. serve no other purpose. There is no law giving any
claim for the loss upon which the action is based, is 2 SCRA 945 effect to such action upon the principal. Besides,
valid, in view of Section 61-A of the Insurance Act LABRADOR; July 31, 1961 there is no condition in the policy that the action
invalidating stipulations limiting the time for must be filed against the agent, and the Court can
NATURE not by interpretation extend the clear scope of the
INSURANCE Page
108

agreement beyond what is agreed upon by the against the owner and the driver of the Lady Love reimbursement, alleging that San Miguel had already
parties. taxicab and his cause of action against petitioner. paid petitioner P4,500, as evidenced by a cash
- Their contract is the law between the parties, and The former is based on torts and quasi-delicts while voucher and a Release of Claim executed by the
their agreement that an action on a claim denied by the latter is based on contract. General Manager of petitioner.
the insurer must be brought within one year from - Even assuming arguendo that there was such a - ZENITH thus demanded from petitioner
the denial, governs, not the rules on the prescription contract, private respondent's cause of action can reimbursement of the sum of P4,500 paid by San
of actions. not prevail because he failed to file the written Miguel.
Disposition The judgment appealed from is hereby claim mandated by the Insurance Code (before - City Court ordered petitioner to pay respondent
set aside and the case dismissed, with costs against it was amended-action must be brought within P4,500.
plaintiffs-appellees. six months from date of the accident (this is - CFI affirmed the City Court's decision in toto.
what’s applicable here) ; after amendment- - CA affirned CFI, with the modification that
TRAVELLERS INSURANCE & SURETY CORP. v. "action or suit for recovery of damage due to loss or petitioner was to pay the total amount of P5,000 it
CA (MENDOZA) injury must be brought in proper cases, with the had earlier received from ZENITH.
272 SCRA 536 Commissioner or the Courts within one year from Petitioner’s Claims
HERMOSISIMA, JR; May 22, 1997 denial of the claim, otherwise the claimant's right of > It is not bound to pay P4,500, and much more,
action shall prescribe" ). He is deemed, under this P5,000 to ZENITH as the subrogation in the Release
NATURE legal provision, to have waived his rights as against of Claim it executed in favor of respondent was
The petition herein seeks the review and reversal of petitioner-insurer. conditioned on recovery of the total amount of
the decision of respondent Court of Appeals affirming Disposition petition granted damages petitioner had sustained. Since total
in toto the judgment of the Regional Trial Court in an damages were valued by petitioner at P9,486.43 and
action for damages filed by private respondent SUN INSURANCE v. CA (supra p.57) only P5,000 was received by petitioner, MLA
Vicente Mendoza, Jr. as heir of his mother who was MAHOGANY argues that it was entitled to go after
killed in a vehicular accident. COASTWISE v. CA (supra p.70) San Miguel to claim the additional P4,500.
> It cites Art. 220711 and Art. 130412 of the Civil
FACTS CEBU SHIPYARD v. WILLIAM LINES (supra p.3) Code, and claims a preferred right to retain the
-an old lady was hit by a taxicab. The taxicab was amount coming from San Miguel, despite the
later identified and a case was filed against the MANILA MAHOGANY MANUFACTURING CORP v. subrogation in favor of ZENITH.
driver and owner. Later, an amendment was filed to CA (ZENITH INSURANCE CORP) Respondent’s Arguments
include the insurance company. RTC and CA ordered 154 SCRA 652 > There was no qualification to its right of
that the owner, driver as well as the insurance PADILLA; October 12, 1987 subrogation under the Release of Claim executed by
company be held solidarily liable. petitioner, the contents having expressed all intents
NATURE and purposes of the parties.
ISSUE Petition to review CA decision ordering Manila
WON RTC and CA erred Mahogany Manufacturing Corporation to pay Zenith ISSUE
HELD Insurance Corporation P5,000 with 6% annual WON the insurer may recover the sum of P5,000
YES interest, attorney's fees, and costs of suit
- Where the contract provides for indemnity against HELD
liability to third persons, then third persons to whom FACTS YES
the insured is liable can sue the insurer. Where the - From 6 March 1970 to 6 March 1971, MLA Ratio Since the insurer can be subrogated to only
contract is for indemnity against actual loss or MAHOGANY insured its Mercedes Benz 4-door sedan such rights as the insured may have, should the
payment, then third persons cannot proceed against with ZENITH. insured, after receiving payment from the insurer,
the insurer, the contract being solely to reimburse - On 4 May 1970, the insured vehicle was bumped
11
the insured for liability actually discharged by him and damaged by a truck owned by San Miguel Article 2207: “If the plaintiff's property has been insured, and he
thru payment to third persons, said third persons' Corporation. For the damage caused, ZENITH paid has received indemnity from the insurance company for the injury or
loss arising out of the wrong or breach of contract complained of the
recourse being thus limited to the insured alone. But MLA MAHOGANY P5,000 in amicable settlement. MLA insurance company shall be subrogated to the rights of the insured
in the case at bar, there was no contract shown. MAHOGANY's general manager executed a Release of against the wrongdoer or the person who has violated the contract. If
the amount paid by the insurance company does not fully cover the
What then was the basis of the RTC and the CA Claim, subrogating respondent company to all its injury or loss the aggrieved party shall be entitled to recover the
to say that the insurance contract was a third- right to action against San Miguel Corporation. deficiency from the person causing the loss or injury.”
party liability insurance policy? Consequently, - On 11 Dec 1972, ZENITH wrote Insurance 12
Article 1305: “A creditor, to whom partial payment has been made,
the trial court was confused as it did not distinguish Adjusters, Inc. to demand reimbursement from San may exercise his right for the remainder, and he shall be preferred to
between the private respondent's cause of action Miguel. Insurance Adjusters, Inc. refused the person who has been subrogated in his place in virtue of the partial
payment of the same credit.”
INSURANCE Page
109

release the wrongdoer who caused the loss, the - In 1965, Jacob S. Lim was engaged in the airline petition but will no longer be discussed because it is
insurer loses his rights against the latter. But in such business as owner-operator of Southern Air Lines not relevant to the topic)
a case, the insurer will be entitled to recover from (SAL), a single proprietorship. - After trial on the merits, a decision was rendered
the insured whatever it has paid to the latter, unless -On May 17, 1965, at Tokyo, Japan, Japan Domestic holding Lim liable to pay Pioneer but dismissed
the release was made with the consent of the Airlines (JDA) and Lim entered into and executed a Pioneer's complaint against all other
insurer. sales contract for the sale and purchase of two (2) defendants.
Reasoning DC-3A Type aircrafts and one (1) set of necessary - CA modified the trial court's decision in that the
- Although petitioner’s right to file a deficiency claim spare parts for the total agreed price of US plaintiff’s complaint against all the defendants
against San Miguel is with legal basis, without $109,000.00 to be paid in installments. (including Lim) was dismissed.
prejudice to the insurer's right of subrogation, - On May 22, 1965, Pioneer Insurance and Surety
nevertheless, when Manila Mahogany executed Corporation, as surety, executed and issued its ISSUE
another release claim discharging San Miguel from Surety Bond No. 6639 in favor of JDA, in behalf of its WON the petition of Pioneer Insurance and Surety
"all actions, claims, demands and rights of action principal, Lim, for the balance price of the aircrafts Corporation against all defendants was rightly
that now exist or hereafter arising out of or as a and spare parts. dismissed
consequence of the accident" after the insurer had -Border Machinery and Heavy Equipment Company,
paid the proceeds of the policy - the compromise Inc. (Bormaheco), Francisco and Modesto Cervantes HELD
agreement of P5,000 being based on the insurance (Cervanteses) and Constancio Maglana contributed YES
policy - the insurer is entitled to recover from the some funds used in the purchase of the above - Both the TC and CA made the finding that Pioneer
insured the amount of insurance money paid. Since aircrafts and spare parts. They executed two (2) reinsured its risk of liability under the surety bond it
petitioner by its own acts released San Miguel, separate indemnity agreements in favor of Pioneer, had executed in favor of JDA, collected the proceeds
thereby defeating private respondent’s right of one signed by Maglana and the other jointly signed of such reinsurance in the sum of P295,000, and
subrogation, the right of action of petitioner against by Lim for SAL, Bormaheco and the Cervanteses. paid with the said amount the bulk of its alleged
the insurer was also nullified. - On June 10, 1965, Lim doing business under the liability to JDA under the said surety bond. The total
- As held in Phil. Air Lines v. Heald Lumber Co., name and style of SAL executed in favor of Pioneer amount paid by Pioneer to JDA is P299,666.29. Since
under Art. 2207, the real party in interest with as deed of chattel mortgage as security for the Pioneer has collected P295,000.00 from the
regard to the portion of the indemnity paid is the latter's suretyship in favor of the former. It was reinsurers, the uninsured portion of what it paid to
insurer and not the insured. stipulated therein that Lim transfer and convey to JDA is the difference between the two amounts, or
SUBROGATION: The right of subrogation can only the surety the two aircrafts. P3,666.28. This is the amount for which Pioneer may
exist after the insurer has paid the insured, - Lim defaulted on his subsequent installment sue defendants, assuming that the indemnity
otherwise the insured will be deprived of his right to payments prompting JDA to request payments from agreement is still valid and effective. But since the
full indemnity. If the insurance proceeds are not the surety. amount realized from the sale of the mortgaged
sufficient to cover the damages suffered by the - Pioneer paid a total sum of P298,626.12. chattels are P35,000.00 for one of the airplanes and
insured, then he may sue the party responsible for - Pioneer then filed a petition for the extrajudicial P2,050.00 for a spare engine, or a total of
the damage for the remainder. To the extent of the foreclosure of the said chattel mortgage before the P37,050.00, Pioneer is still overpaid by P33,383.72.
amount he has already received from the insurer Sheriff of Davao City. Therefore, Pioneer has no more claim against
enjoys the right of subrogation. - The Cervanteses and Maglana, however, filed a defendants.
Disposition Petition DENIED. Judgment appealed third party claim alleging that they are co-owners of - The payment to the petitioner made by the
from is AFFIRMED with costs against petitioner. the aircrafts, reinsurers was not disputed. Considering this
- On July 19, 1966, Pioneer filed an action for judicial admitted payment, the only question was the effect
PIONEER INSURANCE v. CA (BORDER foreclosure with an application for a writ of of payment made by the reinsurers to the petitioner
MACHINERY & HEAVY EQUIPMENT INC) preliminary attachment against Lim and - In general a reinsurer, on payment of a loss
175 SCRA 668 respondents, the Cervanteses, Bormaheco and acquires the same rights by subrogation as are
GUTIERREZ, JR.; July 28, 1989 Maglana. acquired in similar cases where the original insurer
**Maglana, Bormaheco and the Cervanteses filed pays a loss (Universal Ins. Co. v. Old Time Molasses
NATURE cross-claims against Lim alleging that they were not Co.).
Petitions for review on certiorari of a decision of the privies to the contracts signed by Lim and, by way of - The rules of practice in actions on original
CA counterclaim, sought for damages for being exposed insurance policies are in general applicable to actions
to litigation and for recovery of the sums of money or contracts of reinsurance (Delaware, Ins. Co. v.
FACTS they advanced to Lim for the purchase of the Pennsylvania Fire Ins. Co.).
aircrafts in question. (this constitutes the second - Hence the applicable law is Article 2207 of the new
Civil Code, to wit: Art. 2207. If the plaintiff’s
INSURANCE Page
110

property has been insured, and he has received caused by “accidental collision or overturning, or written assignment of claim. It accrues simply upon
indemnity from the insurance company for the injury collision or overturning consequent upon mechanical payment of the insurance claim by the insurer
or loss arising out of the wrong or breach of contract breakdown or consequent upon wear and tear”. Exceptions
complained of, the insurance company shall be - On 1985, the insured car was sideswept and (1) if the assured by his own act releases the
subrogated to the rights of the insured against the damaged by a car owned by Erlinda Fabie, driven by wrongdoer or third party liable for the loss or
wrongdoer or the person who has violated the an unknown driver who fled the scene. Panmalay, in damage, from liability, the insurer's right of
contract. If the amount paid by the insurance accordance with the policy, defrayed the cost of subrogation is defeated;
company does not fully cover the injury or loss, the repair of the insured car and was subrogated to the (2) where the insurer pays the assured the value of
aggrieved party shall be entitled to recover the rights of Canlubang against the driver and owner of the lost goods without notifying the carrier who has
deficiency from the person causing the loss or injury the pick-up. Panmalay then filed a complaint for in good faith settled the assured's claim for loss, the
- If a property is insured and the owner receives the damages with RTC Makati against Erlinda Fabie and settlement is binding on both the assured and the
indemnity from the insurer, it is provided in said her driver on the grounds of subrogation, with the insurer, and the latter cannot bring an action against
article that the insurer is deemed subrogated to the latter failing and refusing to pay their claim. Fabie the carrier on his right of subrogation;
rights of the insured against the wrongdoer and if filed a Motion for Bill of Particulars. (3) where the insurer pays the assured for a loss
the amount paid by the insurer does not fully cover - RTC: dismissed complaint for lack of cause of which is not a risk covered by the policy, thereby
the loss, then the aggrieved party is the one entitled action (payment by PANMALAY of CANLUBANG's effecting "voluntary payment", the former has no
to recover the deficiency. Evidently, under this legal claim under the "own damage" clause of the right of subrogation against the third party liable for
provision, the real party in interest with regard to insurance policy was an admission by the insurer the loss
the portion of the indemnity paid is the insurer and that the damage was caused by the assured and/or Reasoning
not the insured (. Air Lines, Inc. v. Heald Lumber its representatives) – Panmalay appealed - Both TC and CA are incorrect.
Co., and Manila Mahogany Manufacturing - CA: dismissed appeal, affirmed RTC (applying the ON TC: “Own damage” (not found in the insurance
Corporation v. Court of Appeals) ejusdem generis rule held that Section III-1 of the policy) simply meant that Panmalay had assumed to
- It is clear from the records that Pioneer sued in its policy, which was the basis for settlement of reimburse the cost for repairing the damage to the
own name and not as an attorney-in-fact of the CANLUBANG's claim, did not cover damage arising insured vehicle. It’s different from “Third Party
reinsurer. Accordingly, the appellate court did not from collision or overturning due to the negligence of Liability” coverage (liabilities arising from the death
commit a reversible error in dismissing the third parties as one of the insurable risk) of or bodily injuries suffered by 3rd parties) and from
petitioner's complaint as against the respondents for “Property Damage” coverage (liabilities from damage
the reason that the petitioner was not the real party ISSUE caused by insured vehicle to properties of 3rd parties)
in interest in the complaint and, therefore, has no WON the insurer PANMALAY may institute an action ON CA: the terms of a contract are to be construed
cause of action against the respondents. to recover the amount it had paid its assured in according to the sense and meaning of the terms
Disposition Petitions dismissed. Questioned decision settlement of an insurance claim against private which the parties thereto have used. In the case of
of CA affirmed. respondents as the parties allegedly responsible for property insurance policies, the evident intention of
the damage caused to the insured vehicle, in the contracting parties, i.e., the insurer and the
accordance with A2207, NCC assured, determine the import of the various terms
PAN MALAYAN INSURANCE CORPORATION v. and provisions embodied in the policy. It is only
CA (FABIE, HER UNKNOWN DRIVER) HELD when the terms of the policy are ambiguous,
184 SCRA 54 YES equivocal or uncertain, such that the parties
CORTES, April 3, 1990 Ratio Article 2207 of the Civil Code is founded on themselves disagree about the meaning of particular
the well-settled principle of subrogation. If the provisions, that the courts will intervene. In such an
NATURE insured property is destroyed or damaged through event, the policy will be construed by the courts
PETITION to review the decision of the Court of the fault or negligence of a party other than the liberally in favor of the assured and strictly against
Appeals assured, then the insurer, upon payment to the the insurer
assured, will be subrogated to the rights of the - Both Panmalay and Canlubang had the same
FACTS assured to recover from the wrongdoer to the extent interpretation regarding the coverage of insured risk
- Pan Malayan Insurance Company (Panmalay) that the insurer has been obligated to pay. Payment regarding “accidental collision or overturning…” to
insured the Mitsubishi Colt Lancer car registered in by the insurer to the assured operates as an include damages caused by 3rd party to Canlubang so
the name of Canlubang Automotive Resources equitable assignment to the former of all remedies it was improper for CA to ascribe meaning contrary
Corporation (Canlubang) under its motor vehicle which the latter may have against the third party to the clear intention and understanding of the
insurance policy. Among the provisions of the policy whose negligence or wrongful act caused the loss. parties.
was a “own-damage” clause whereby Panmalay The right of subrogation is not dependent upon, nor - Court on several occasions defined “accident” or
agrees to indemnify Canlubang in cases of damage does it grow out of, any privity of contract or upon “accidental” as taking place “without one’s foresight
INSURANCE Page
111

or expectation, an event that proceeds from an that there was no allegation that it had consented to - The lower court denied plaintiff's motion. They
unknown cause, or is an unusual effect of a known the subrogation and, therefore, Fireman's Fund had filed a second MR, calling the lower court's attention
cause and, therefore, not expected” [Dela Cruz v. no cause of action against it. It also dismissed the to the fact that the issue of subrogation was of no
Capital Insurance & Surety Co.] The concept complaint as to First Quezon City on the ground of moment because Firestone, the subrogor, is a party-
"accident" is not necessarily synonymous with the res judicata. It appears that the same action was plaintiff and could sue directly Jamila in its own right.
concept of "no fault". It may be utilized simply to previously filed in a civil case which was dismissed Without resolving that contention, the lower court
distinguish intentional or malicious acts from because of the failure of the same plaintiffs and their denied plaintiffs' second MR.
negligent or careless acts of man. counsel to appear at the pre-trial.
- damage/loss to insured vehicle due to negligence - Upon an MR, the lower court set aside its order of ISSUE
of 3rd parties not listed as exceptions to coverage in dismissal and sustained plaintiff's contention that WON the complaint of Firestone and Fireman's Fund
the insurance policy there was no res judicata as to First Quezon City states a cause of action against Jamila
- Interpretation given by Panmalay is more in because the civil case was dismissed without
keeping with rationale behind rules on interpretation prejudice. However, due to inadvertence, the lower HELD
of insurance contracts in favor of assured or court did not state in its order why it set aside its YES
beneficiary: indemnity or payment prior order dismissing the complaint with respect to - Fireman's Fund's action against Jamila is squarely
- EVEN if voluntarily indemnified Canlubang, as Jamila. Jamilla had originally moved for the dismissal sanctioned by article 2207. As the insurer, Fireman's
interpreted by TC: the insurer who may have no of the complaint on the ground of lack of cause of Fund is entitled to go after the person or entity that
rights of subrogation due to "voluntary" payment action. Its basis for its contention were: (1) that the violated its contractual commitment to answer for
may never. theless recover from the third party complaint did not allege that Firestone, pursuant to the loss insured against.
responsible for the damage to the insured property the contractual stipulation quoted in the complaint, - The trial court erred in applying to this case the
under Article 1236 of the Civil Code. [Sveriges had investigated the loss and that Jamila was rules on novation. The plaintiffs in alleging in their
Angfartygs Assurans Forening v. Qua Chee Gan] represented in the investigation and (2) that Jamila complaint that Fireman's Fund "became a party in
Disposition the present petition is GRANTED. did not consent to the subrogation of Fireman's Fund interest in this case by virtue of a subrogation right
Petitioner's complaint for damages against private to Firestone's right to get reimbursement from given in its favor by" Firestone, were not relying on
respondents is hereby REINSTATED. Let the case be Jamila and its surety. The lower court in its order of the novation by change of creditors as contemplated
remanded to the lower court for trial on the merits. dismissal had sustained the second ground. in articles 1291 and 1300 to 1303 of the Civil Code
- Jamila in its MR invoked the first ground which but rather on article 2207.
FIREMAN'S FUND INSURANCE COMPANY v. had never been passed upon by the lower court. But - Article 2207 is a restatement of a settled principle
JAMILA & COMPANY, INC. the lower court in its order granting Jamila's motion of American jurisprudence. Subrogation has been
70 SCRA 323 for reconsideration, completely ignored that first referred to as the doctrine of substitution. It is an
AQUINO; April 1976 ground. It reverted to the second ground which was arm of equity that may guide or even force one to
relied upon in its order previous order. The lower pay a debt for which an obligation was incurred but
FACTS court reiterated its order, stating that Fireman's which was in whole or in part paid by another.
- Jamila & Co., Inc. or the Veterans Philippine Fund had no cause of action against Jamila because - Subrogation is founded on principles of justice
Scouts Security Agency contracted to supply security Jamila did not consent to the subrogation. The court and equity, and its operation is governed by
guards to Firestone. Jamila assumed responsibility did not mention Firestone, the co-plaintiff of principles of equity. It rests on the principle that
for the acts of its security guards. The First Quezon Fireman's Fund. substantial justice should be attained regardless of
City Insurance Co., Inc. executed a bond in the sum - Firestone and Fireman's Fund filed an MR on the form, that is, its basis is the doing of complete,
of P20,000 to guarantee Jamila's obligations under ground that Fireman's Fund was suing on the basis essential, and perfect justice between all the parties
that contract. of legal subrogation whereas the lower court without regard to form.
- On May 18, 1963 properties of Firestone valued at erroneously predicated its dismissal order on the - Subrogation is a normal incident of indemnity
P11,925 were lost allegedly due to the acts of its theory that there was no conventional subrogation insurance. Upon payment of the loss, the insurer is
employees who connived with Jamila's security because the debtor's consent was lacking. entitled to be subrogated pro tanto to any right of
guard. Fireman's Fund, as insurer, paid to Firestone - The plaintiffs cited article 2207 of the Civil Code action which the insured may have against the third
the amount of the loss. Fireman's Fund was which provides that "if the plaintiff's property has person whose negligence or wrongful act caused the
subrogated to Firestone's right to get reimbursement been insured, and he has received indemnity from loss. The right of subrogation is of the highest
from Jamila. Jamila and its surety, First Quezon City the insurance company for the injury or loss arising equity. The loss in the first instance is that of the
failed to pay the amount of the loss in spite of out of the wrong or breach of contract complained insured but after reimbursement or compensation, it
repeated demands. of, the insurance company shall be subrogated to the becomes the loss of the insurer.
- Upon defendant's motions, the lower court rights of the insured against the wrongdoer or the - Although many policies including policies in the
dismissed the complaint as to Jamila on the ground person who has violated the contract". standard form, now provide for subrogation, and
INSURANCE Page
112

thus determine the rights of the insurer in this - By virtue of the insurance companies’ payment, character of the goods or defects in their packing; or
respect, the equitable right of subrogation as the they were subrogated to the rights of Republic Flour. an order or act of a competent public authority.
legal effect of payment inures to the insurer without Petitioners filed a complaint against North Front - However, Republic Flour is also found to be guilty
any formal assignment or any express stipulation to Shipping, claiming the loss was exclusively of contributory negligence for not immediately
that effect in the policy. Stated otherwise, when the attributable to the latter’s fault and negligence. staring the unloading operations and for providing no
insurance company pays for the loss, such payment Having surveyed the vessel, it was found that the explanation for the delay. As such, it should share at
operates as an equitable assignment to the insurer barge had cracks in its bodega. The hatches on the least 40% of the loss.
of the property and all remedies which the insured crates of grain were not sealed and the tarpaulins Disposition The decision of the CA is REVERSED
may have for the recovery thereof. That right is not used in covering them were not new, contrary to and SET ASIDE
dependent upon, nor does it grow out of, any privity North Front Shipping’s claims. North Front Shipping
of contract, or upon written assignment of claim, and reiterated that the barge was inspected prior to PHILIPPINE AMERICAN LIFE INSURANCE
payment to the insured makes the insurer an loading and found seaworthy and were issued a COMPANY v. CA (ELIZA PULIDO)
assignee in equity. permit to sail by the Coast Guard. They further 344 SCRA 360
- On the other hand, Firestone is really a nominal averred that the grains were farm wet and not GONZAGA-REYES; November 15, 2000
party in this case. It had already been indemnified properly dried before loading.
for the loss which it had sustained. Obviously, it - The court dismissed the complaint, ruling that the NATURE
joined as a party-plaintiff in order to help Fireman's contract entered into was a charter-party This petition for review on certiorari seeks to reverse
Fund to recover the amount of the loss from Jamila agreement; as such, only ordinary diligence in the the Decision of the Special Second Division of the
and First Quezon City. Firestone had tacitly assigned care of the goods was required of North Front Court of Appeals
to Fireman's Fund its cause of action against Jamila Shipping.
for breach of contract. Sufficient ultimate facts are FACTS
alleged in the complaint to sustain that cause of ISSUE - On January 9, 1989, petitioner received from one
action. WON defendant is required to observe extraordinary Florence Pulido an application for life insurance,
diligence in its vigilance over the goods it transports dated December 16, 1988, in the amount of
TABACALERA v. NORTH FRONT SHIPPING P100,000.00 which designated her sister, herein
272 SCRA 527 HELD private respondent, as its principal beneficiary.
BELLOSILLO; May 16, 1997 YES Because the insurance applied for was non-medical,
- As a corporation engaged in the business of petitioner did not require a medical examination and
FACTS transporting cargo offering its services issued a policy on the sole basis of the application on
- 20,234 sacks of corn grains valued at P3.5M were indiscriminately to the public, it is without a doubt a February 11, 1989. On April 1992, petitioner
shipped on board North Front 777, defendant’s common carrier. As such, it has the burden of received private respondent’s claim, which declared
vessel. The cargo was consigned to Republic Flour proving that it observed extraordinary diligence to that the insured, Florence Pulido, died of acute
Mills Corp. under Bill of Lading No. 001 and insured avoid responsibility for the lost cargo. The clean bill pneumonia on September 10, 1991.
with Tabacalera, Prudential Guarantee & Assurance, of lading it issued disprove the master of the vessel’s - Petitioner withheld payment on the ground that the
and New Zealand Insurance. claim that the grains were farm wet when loaded. If policy claimed under was void from the start for
- Republic Flour was advised of the vessel’s arrival in they were wet, the master of the vessel should have having been procured in fraud. It is petitioner’s
Manila, but did not immediately commence the known that the grains would eventually deteriorate contention that even before they received private
unloading operations. Unloading was sometimes when sealed in hot compartments in hatches of a respondent’s claim for death benefits, their
stopped due to varying weather and sometimes for ship and should have undertaken precautionary investigation concerning the subject policy yielded
no apparent reason. Unloading was only completed measures to avoid this. The arrival of the goods at the information that the insured, Florence Pulido,
20 days after the arrival of the barge; by then, the the place of destination in bad order makes a prima died in 1988, before the application for insurance on
cargo was short 26.333 metric tons and the rest was facie case against the common carrier, which must her life was made. While this was communicated to
already moldy and deteriorating. prove its non-liability. private respondent in a letter, private respondent
- Analyses showed that the deterioration was caused - While petitioners presented evidence of the vessel’s had already filed her claim earlier that month. In
by moisture content from salt water, which could be bad shape and a laboratory analysis revealing that another letter, however, petitioner confirmed to
arrested by drying. However, Republic Flour rejected the grains were contaminated with salt water, private respondent receipt of the claim papers and
the entire cargo and demanded that defendant North defendants failed to rebut said arguments or even assured her that her case was “being given
Front Shipping pay the damages suffered by it. The endeavor to establish that the loss, destruction or preferential attention and prompt action”.
demands were unheeded and the insurance deterioration was due to a fortuitous event; an - Petitioner caused another investigation respecting
companies were obliged to pay Republic Flour act/omission of the owner of the goods; the the subject policy. Pursuant to the findings of this
P2,189,433 second investigation, petitioner stood by its initial
INSURANCE Page
113

decision to treat the policy as void and not to honor public duties, which was the basis of both - As subrogee of the rights of' the shipper and/or
the claim. On November 9, 1992, private respondent court and the trial court in finding the consignee, the insurer, St. Paul Fire & Marine
respondent enlisted the services of counsel in date of Florence Pulido’s death to be as plaintiff- Insurance Co., instituted an action against the
reiterating her claim for death benefitsPetitioner still private respondent maintained. defendants for the recovery of said amount of
refused to make payment and thus, this action. - We cannot likewise give credence to petitioner’s $1,134.46, plus costs.
submission that the inconsistencies in the - The defendants resisted the action. However, for
ISSUE testimonies of the witnesses for plaintiff-private the purpose only of avoiding litigation without
WON lower court erred in holding that there was no respondent are in themselves evidence of fraud. admitting liability to the consignee, the defendants
fraud Such alleged inconsistencies are matters of offered to settle the latter’s claim in full by paying
credibility which had been ably passed upon by the the C.I.F. value of the damaged cargo, but this offer
HELD lower court. was declined by the plaintiff.
- The records bear out that since the onset of this Disposition the instant petition is DENIED - The LC rendered judgment ordering the
case, the main issue has always been whether there defendants to pay the plaintiff the sum of P300.00.
was fraud in the obtainment of the disputed policy, ST.PAUL FIRE & MARINE INSURANCE CO v. The plaintiff filed a MFR contending that it should
or put differently, whether the insured, Florence MACONDRAY & CO INC recover the amount of $1,134.46 or its equivalent in
Pulido, was in fact dead before the application for 70 SCRA 122 pesos at the rate of P3.90, instead of P2.00, but this
insurance on her life was made. This the lower ANTONIO; March 25, 1976 was denied. Hence, this appeal.
courts had effected ruled on, upon a preponderance
of the evidence duly received from both parties. We FACTS ISSUES
see no reversible error in the finding of both - Winthrop Products, Inc. shipped aboard the SS 1. WON in case of loss or damage, the liability of the
respondent court and the trial court in favor of the "Tai Ping", owned and operated by Wilhelm carrier to the consignee is limited to the C.I.F. value
correctness of the entries in Certificate of Death, Wilhelmsen, 218 cartons and drums of drugs and of the goods which were lost or damaged
duly registered with the Local Civil Registrar of medicine, with the freight prepaid, which were 2. WON the insurer who has paid the claim in dollars
Bagulin, La Union, which declared that Florence consigned to Winthrop-Stearns, Inc. Barber to the consignee should be reimbursed in its peso
Pulido died of acute pneumonia on September 10, Steamship Lines, Inc., agent of Wilhelm Wilhelmsen equivalent on the date of discharge of the cargo or
1991. Dr. Irineo Gutierrez, the Municipal Health issued Bill of Lading No. 34, in the name of Winthrop on the date of the decision
Officer of Bagulin, La Union whose signature Products, Inc. as shipper, with arrival notice in-
appeared in the death certificate, testified in addition Manila to consignee Winthrop-Stearns, Inc. The HELD
that he ministered to the ailing Florence Pulido for shipment was insured by the shipper against loss 1. YES
two days immediately prior to her death. This fact is and/or damage with the St. Paul Fire & Marine Ratio The purpose of the bill of lading is to provide
likewise noted in the death certificate. Insurance Company. for the rights and liabilities of the parties in reference
- Death certificates, and notes by a municipal health - The SS "Tai Ping" arrived at the Port of Manila and to the contract to carry. The stipulation in the bill of
officer prepared in the regular performance of his discharged its aforesaid shipment into the custody of lading limiting the common carrier's liability to the
duties, are prima facie evidence of facts therein Manila Port Service, the arrastre contractor for the value of the goods appearing in the bill, unless the
stated. A duly-registered death certificate is Port of Manila. The said shipment was discharged shipper or owner declares a greater value, is valid
considered a public document and the entries found complete and in good order with the exception of and binding. This limitation of the carrier's liability is
therein are presumed correct, unless the party who one (1) drum and several cartons which were in bad sanctioned by the freedom of the contracting parties
contests its accuracy can produce positive evidence order condition. Because consignee failed to receive to establish such stipulations, clauses, terms, or
establishing otherwise. Petitioner’s contention that the whole shipment and as several cartons of conditions as they may deem convenient, provided
the death certificate is suspect because Dr. Gutierrez medicine were received in bad order condition, the they are not contrary to law, morals, good customs
was not present when Florence Pulido died, and consignee filed the corresponding claim in the and public policy. A stipulation fixing or limiting the
knew of Florence’s death only through Ramon amount of P1,109.67 representing the C.I.F. value of sum that may be recovered from the carrier on the
Piganto, does not merit a conclusion of fraud. No the damaged drum and cartons of medicine with the loss or deterioration of the goods is valid, provided it
motive was imputed to Dr. Gutierrez for seeking to carrier and the Manila Port Service. However, both is (a) reasonable and just under the circumstances,
perpetuate a falsity in public records. Petitioner was refused to pay such claim. Consequently, the and (b) has been fairly and freely agreed upon. In
likewise unable to make out any clear motive as to consignee filed its claim with the insurer, St. Paul the case at bar, the liabilities of the defendants-
why Ramon Piganto would purposely lie. Mere Fire & Marine Insurance Co., the insurance company, appellees with respect to the lost or damaged
allegations of fraud could not substitute for the full on the basis of such claim, paid to the consignee the shipments are expressly limited to the C.I.F. value of
and convincing evidence that is required to prove it. insured value of the lost and damagcd goods, the goods as per contract of sea carriage embodied
A failure to do so would leave intact the presumption including other expenses in connection therewith, in in the bill of lading.
of good faith and regularity in the performance of the total amount of $1,134.46 U.S. currency.
INSURANCE Page
114

- The plaintiff-appellant, as insurer, after paying the rejected it and made a counter-offer for P4,000.00, will indemnify any authorized Driver who is driving
claim of the insured for damages under the but the Company did not accept it. the Motor Vehicle . . . "
insurance, is subrogated merely to the rights of the - On September 18, 1962, the Insured and Carlito's "Conditions
assured. As subrogee, it can recover only the parents filed a complaint against the Company to "7. In the event of death of any person entitled
amount that is recoverable by the latter. Since the collect the proceeds of the policy. In its answer, the to indemnify under this Policy, the Company will, in
right of the assured, in case of loss or damage to the Company admitted the existence thereof, but respect of the liability incurred by such person,
goods, is limited or restricted by the provisions in the pleaded lack of cause of action on the part of the indemnify his personal representatives in terms of
bill of lading, a suit by the insurer as subrogee plaintiffs. and subject to the limitations of this Policy, provided,
necessarily is subject to like limitations and - TC rendered a decision sentencing the Company to that such representatives shall, as though they were
restrictions. pay to the plaintiffs the sum of P4,000.00 and the the Insured, observe, fulfill and be subject to the
2. On the date of the discharge of the cargo. The costs. Hence, this appeal by the Company, which Terms of this Policy insofar as they can apply.
peso equivalent was based by the consignee on the contends that plaintiffs have no cause of action "8. The Company may, at its option, make
exchange rate of P2.015 to $1.00 which was the rate because: 1) the Coquias have no contractual relation indemnity payable directly to the claimants or heirs
existing at that time. with the Company; and 2) the Insured has not of claimants, with or without securing the consent of
complied with the provisions of the policy concerning or prior notification to the Insured, it being the true
PHILAM v. AUDITOR (supra p.59) arbitration. intention of this Policy to protect, to the extent
herein specified and subject always to the Terms of
FIELDMEN’S v. ASIAN SURETY (supra p.60) ISSUES this Policy, the liabilities of the Insured towards the
1. WON there was contractual relations between the passengers of the Motor Vehicle and the Public."
EQUITABLE v. RURAL INSURANCE (supra p.60) Coquias and the Company - Thus, the policy under consideration is typical of
2. WON the insured has not complied with the contracts pour autrui, this character being made
COQUIA v. FIELDMEN'S INSURANCE CO. INC. provisions of the policy concerning arbitration more manifest by the fact that the deceased driver
26 SCRA 178 paid fifty percent (50%) of the corresponding
CONCEPCION; November 29, 1968 HELD premiums, which were deducted from his weekly
1. Although, in general, only parties to a contract commissions. Under these conditions, it is clear that
NATURE may bring an action based thereon, this rule is the Coquias — who, admittedly, are the sole heirs of
Appeal from the decision of the CFI certified by CA subject to exceptions, one of which is found in the the deceased — have a direct cause of action against
Art 1311 CC, reading: the Company, and, since they could have maintained
FACTS "If a contract should contain some stipulation in this action by themselves, without the assistance of
- December 1, 1961, appellant Fieldmen's Insurance favor of a third person, he may demand its the Insured, it goes without saying that they could
Company, Inc. issued, in favor of the Manila Yellow fulfillment provided he communicated his acceptance and did properly join the latter in filing the complaint
Taxicab Co., Inc. a common carrier accident of the obligor before its revocation. A mere incidental herein.
insurance policy, covering the period from December benefit or interest of a person is not sufficient. The 2. Based upon Section 17 of the policy:
1, 1961 to December ,1962. It was stipulated in said contracting parties must have clearly and "If any difference or dispute shall arise with
policy that: deliberately conferred a favor upon a third person." respect to the amount of the Company's liability
"The Company will, subject to the Limits of Liability - Does the policy in question belong to such class of under this Policy, the same shall be referred to the
and under the Terms of this Policy, indemnify the contracts pour autrui? decision of a single arbitrator to be agreed upon by
Insured in the event of accident caused by or arising In this connection, said policy provides, inter alia: both parties or failing such agreement of a single
out of the use of Motor Vehicle against all sums "Section I — Liability to Passengers. 1. The arbitrator, to the decision of two arbitrators, one
which the Insured will become legally liable to pay in Company will, subject to the Limits of Liability and to be appointed in writing by each of the parties
respect of: Death or bodily injury to any fare-paying under the Terms of this Policy, indemnify the Insured within one calendar month after having been
passenger including the Driver, Conductor and/or in the event of accident caused by or arising out of required in writing so to do by either of the parties
Inspector who is riding in the Motor Vehicle insured the use of Motor Vehicle against all sums which the and in case of disagreement between the
at the time of accident or injury." Insured will become legally liable to pay in respect arbitrators, to the decision of an umpire who shall
- While the policy was in force, or on February 10, of: Death or bodily injury to any fare-paying have been appointed in writing by the arbitrators
1962, a taxicab of the Insured, driven by Carlito passenger including the Driver. . . who is riding in before entering on the reference and the costs of
Coquia, met a vehicular accident to which he died. the Motor Vehicle insured at the time of accident or and incidental to the reference shall be dealt with
The Insured filed therefor a claim for P5,000.00 to injury. in the Award. And it is hereby expressly stipulated
which the Company replied with an offer to pay "Section II. — Liability to the Public and declared that it shall be a condition precedent
P2,000.00, by way of compromise. The Insured "3. In terms of and subject to the limitations of to any right of action or suit upon this Policy that
and for the purposes of this Section, the Company the award by such arbitrator, arbitrators or umpire
INSURANCE Page
115

of the amount of the Company's liability hereunder forest, and that such loss was an excepted risk under - The petitioner relies on the Sworn Statements of
if disputed shall be first obtained." paragraph No. 6 of the policy conditions of Fire Jose Lomocso and Ernesto Urbiztondo as well as on
- The record shows that none of the parties to the Insurance Policy No. F-1397, which provides: the Spot Report of Pfc. Arturo V. Juarbal. A witness
contract invoked this section, or made any reference This insurance does not cover any loss or damage can testify only to those facts which he knows of his
to arbitration, during the negotiations preceding the occasioned by or through or in consequence, personal knowledge, which means those facts which
institution of the present case. In fact, counsel for directly or indirectly, of any of the following are derived from his perception. Consequently, a
both parties stipulated, in the trial court, that none occurrences, namely: witness may not testify as to what he merely learned
of them had, at any time during said negotiations, (d) Mutiny, riot, military or popular uprising, from others either because he was told or read or
even suggested the settlement of the issue between insurrection, rebellion, revolution, military or heard the same. Such testimony is considered
them by arbitration, as provided in said section. usurped power. hearsay and may not be received as proof of the
Their aforementioned acts or omissions had the Any loss or damage happening during the truth of what he has learned.
effect of a waiver of their respective right to demand existence of abnormal conditions (whether Disposition the appealed Decision is MODIFIED.
an arbitration. physical or otherwise) which are occasioned by or The rate of interest on the adjudged principal
Disposition The decision appealed from should be through or in consequence, directly or indirectly, amount of Two Hundred Thousand Pesos
as it is hereby affirmed in toto, with costs against the of any of said occurrences shall be deemed to be (P200,000.00) shall be six percent (6%) per annum
herein defendant-appellant, Fieldmen's Insurance loss or damage which is not covered by this computed from the date of filing of the Complaint in
Co., Inc. insurance, except to the extent that the Insured the trial court. The awards in the amounts of Fifty
shall prove that such loss or damage happened Thousand Pesos (P50,000.00) as actual damages,
COUNTRY BANKERS INSURANCE CORP v. independently of the existence of such abnormal Fifty Thousand Pesos (P50,000.00) as exemplary
LIANGA BAY conditions. damages, Five Thousand Pesos (P5,000.00) as
DE LEON; January 25, 2002 - Finding the denial of its claim unacceptable, Lianga litigation expenses, and Ten Thousand Pesos
Bay then instituted in the trial court the complaint (P10,000.00) as attorney?s fees are hereby
NATURE for recovery of "loss, damage or liability" against DELETED.
Petition for review on certiorari Country Bankers.
- RTC ruled in favor of the cooperative. CA affirmed. DBP POOL OF ACCREDITED INSURANCE v.
FACTS RADIO MINDANAO NETWORK
- Lianga Bay is a duly registered cooperative ISSUE 480 SCRA 314
judicially declared insolvent and is here represented WON the cause of the loss was an excepted risk MARTINEZ; January 27, 2006
by, Cornelio Jamero. Country Bankers Insurance under the terms of the fire insurance policy
and Lianga Bay entered into a contract of fire NATURE
insurance. Country Bankers insured the HELD Petition for certiorari
respondent’s stocks-in-trade against fire loss, - Where a risk is excepted by the terms of a policy
damage or liability during the period starting from which insures against other perils or hazards, loss FACTS
June 20, 1989 at 4:00 p.m. to June 20, 1990 at 4:00 from such a risk constitutes a defense which the - In the evening of July 27, 1988, the radio station of
p.m., for the sum of P200,000.00. insurer may urge, since it has not assumed that risk, Radio Mindanao Network located at the SSS Building
- On July 1, 1989, at or about 12:40 a.m., the and from this it follows that an insurer seeking to in Bacolod City was burned down causing damage in
respondent’s building at Barangay Diatagon, Lianga, defeat a claim because of an exception or limitation the amount of over one million pesos. Respondent
Surigao del Sur was gutted by fire, resulting in the in the policy has the burden of proving that the loss sought to recover under two insurance policies but
total loss of the respondent’s stocks-in-trade, pieces comes within the purview of the exception or the claims were denied on the basis that the case of
of furnitures and fixtures, equipments and records. limitation set up. If a proof is made of a loss the loss was an excepted risk under condition no. 6
- Due to the loss, the respondent filed an insurance apparently within a contract of insurance, the burden (c) and (d), to wit:
claim with the petitioner under its Fire Insurance is upon the insurer to prove that the loss arose from 6. This insurance does not cover any loss or damage
Policy, submitting: (a) the Spot Report of Pfc. Arturo a cause of loss which is excepted or for which it is occasioned by or through or in consequence, directly
V. Juarbal, INP Investigator, dated July 1, 1989; (b) not liable, or from a cause which limits its liability. or indirectly, of any of the following consequences,
the Sworn Statement of Jose Lomocso; and (c) the Stated else wise, since Country bank here is namely:
Sworn Statement of Ernesto Urbiztondo. defending on the ground of non-coverage and relying (c) War, invasion, act of foreign enemies, hostilities,
- The petitioner, however, denied the insurance upon an exemption or exception clause in the fire or warlike operations (whether war be declared or
claim on the ground that, based on the submitted insurance policy it has the burden of proving the not), civic war.
documents, the building was set on fire by 2 NPA facts upon which such excepted risk is based, by a (d) Mutiny, riot, military or popular uprising,
rebels who wanted to obtain canned goods, rice and preponderance of evidence. But petitioner failed to insurrection, rebellion, revolution, military or
medicines as provisions for their comrades in the do so. usurped power.
INSURANCE Page
116

- The insurers maintained that based on witnesses WON Lea Mer is liable for the loss of the cargo QUISUMBING; January 24, 2006
and evidence gathered at the site, the fire was
caused by the members of the Communist Party of HELD NATURE
the Philippines/New People’s Army. Hence the refusal YES Review on certiorari (1) the Decision dated October
to honor their obligations. - Common carriers are bound to observe 15, 2002 and (2) the Resolution dated February 27,
- The trial court and the CA found in favor of the extraordinary diligence in their vigilance over the 2003 of CA
respondent. In its findings, both courts mentioned goods and the safety of the passengers they
the fact that there was no credible evidence transport, as required by the nature of their business FACTS
presented that the CCP/NPA did in fact cause the fire and for reasons of public policy. Extraordinary - June 6, 1984 - Petitioner Loadstar Shipping Co.,
that gutted the radio station in Bacolod. diligence requires rendering service with the greatest Inc. (LOADSTAR), registered owner and operator of
skill and foresight to avoid damage and destruction the vessel M/V Weasel, entered into a voyage-
ISSUE to the goods entrusted for carriage and delivery. charter with Northern Mindanao Transport Company,
WON the insurance companies are liable to pay - Common carriers are presumed to have been at Inc. for the carriage of 65,000 bags of cement from
Radio Mindanao Network under the insurance policies fault or to have acted negligently for loss or damage Iligan City to Manila. The shipper was Iligan Cement
to the goods that they have transported. This Corporation, while the consignee in Manila was
HELD presumption can be rebutted only by proof that they Market Developers, Inc. (MARKET)
YES observed extraordinary diligence, or that the loss or - June 24, 1984 - 67,500 bags of cement were
- The Court will not disturb the factual findings of the damage was occasioned by any of the following loaded on board M/V Weasel and stowed in the cargo
appellant and trial courts absent compelling reason. causes: holds for delivery to the consignee. The shipment
Under this mode of review, the jurisdiction of the “(1) Flood, storm, earthquake, lightning, or other was covered by petitioner’s Bill of Lading dated June
court is limited to reviewing only errors of law. natural disaster or calamity; 23, 1984.
- Particularly in cases of insurance disputes with “(2) Act of the public enemy in war, whether - Prior to the voyage, the consignee insured the
regard to excepted risks, it is the insurance international or civil; shipment of cement with respondent Pioneer Asia
companies which have the burden to prove that the “(3) Act or omission of the shipper or owner of the Insurance Corporation (PIONEER) for P1,400,000, for
loss comes within the purview of the exception or goods; which it issued Marine Open Policy No. MOP-006
limitation set up. It is sufficient for the insured to “(4) The character of the goods or defects in the dated September 17, 1980, covering all shipments
prove the fact of damage or loss. Once the insured packing or in the containers; made on or after September 30, 1980
makes out a prima facie case in its favor, the duty or “(5) Order or act of competent public authority.” - June 25, 1984 - Captain Montera of M/V Weasel
burden of evidence shifts to the insurer to controvert - To excuse the common carrier fully of any liability, ordered the vessel to be forced aground which
said prima facie case. the fortuitous event must have been the proximate rendered the entire shipment of cement as good as
Disposition Petition dismissed. Decision of the CA is and only cause of the loss. It should have exercised gone due to exposure to sea water. LOASTAR thus
affirmed. due diligence to prevent or minimize the loss before, failed to deliver the goods to MARKET in Manila.
during and after the occurrence of the event. - MARKET demanded from LOADSTAR full
LEA MER INDUSTRIES v. MALAYAN INSURANCE - Petitioner bore the burden of proving that it had reimbursement of the cost of the lost shipment.
471 SCRA 698 exercised extraordinary diligence to avoid the loss, LOADSTAR refused to reimburse the MARKET despite
PANGANIBAN; September 30, 2005 or that the loss had been occasioned by a fortuitous repeated demands.
event -- an exempting circumstance. - March 11, 1985 – PIONEER paid the MARKET
NATURE - The evidence presented by petitioner in support of P1,400,000 plus an additional amount of P500,000,
Petition for Review its defense of fortuitous event was sorely the value of the lost shipment of cement. In return,
insufficient. It was not enough for the common the MARKET executed a Loss and Subrogation
FACTS carrier to show that there was an unforeseen or Receipt in favor of PIONEER concerning the latter’s
- Ilian Silica Mining entered into a contract of unexpected occurrence. subrogation rights against LOADSTAR.
carriage with Lea Mer Industries for the shipment of Disposition Petition is DENIED and the assailed - October 15, 1986 – PIONEER filed a complaint
900 metric tons of silica sand valued at P565,000. against LOADSTAR with the RTC Manila alleging
Consigned to Vulcan Industrial and Mining Decision and Resolution are AFFIRMED. Costs that: (1) the M/V Weasel was not seaworthy at the
Corporation, the cargo was to be transported from commencement of the voyage; (2) the weather and
Palawan to Manila. The silica sand was placed on against petitioner. sea conditions then prevailing were usual and
board Judy VII, a barge leased by Lea Mer, the expected for that time of the year and as such, was
vessel sank, resulting in the loss of the cargo. LOADSTAR SHIPPING CO INC v. PIONEER ASIA an ordinary peril of the voyage for which the M/V
INSURANCE CORP Weasel should have been normally able to cope with;
ISSUE GR No. 157481 and (3) LOADSTAR was negligent in the selection
INSURANCE Page
117

and supervision of its agents and employees then persons, provided the charter is limited to the ship
manning the M/V Weasel. only, as in the case of a time-charter or voyage-
- LOADSTAR alleged that no fault nor negligence charter. It is only when the charter includes both
could be attributed to it because it exercised due the vessel and its crew, as in a bareboat or demise
diligence to make the ship seaworthy, as well as that a common carrier becomes private, at least
properly manned and equipped and failure to deliver insofar as the particular voyage covering the charter-
was due to force majeure. party is concerned.
- February 15, 1993 - RTC decided in favor of 2. YES
PIONEER and that LOADSTAR , as a common carrier, - As a common carrier, LOADSTAR is required to
bears the burden of proving that it exercised observe extraordinary diligence in the vigilance over
extraordinary diligence in its vigilance over the goods the goods it transports. When the goods placed in its
it transported. The trial court explained that in case care are lost, LOADSTAR is presumed to have been
of loss or destruction of the goods, a statutory at fault or to have acted negligently. LOADSTAR has
presumption arises that the common carrier was the burden of proving that it observed extraordinary
negligent unless it could prove that it had observed diligence in order to avoid responsibility for the lost
extraordinary diligence. LOADSTAR’S defense of cargo.
force majeure was found bereft of factual basis as a - Compania Maritima V CA - It requires common
PAG-ASA report that at the time of the incident, carriers to render service with the greatest skill and
tropical storm “Asiang” had moved away from the foresight and “to use all reasonable means to
Philippines was presented. ascertain the nature and characteristics of goods
- October 15, 2002 – CA affirmed RTC Decision with tendered for shipment, and to exercise due care in
modification the handling and stowage, including such methods
as their nature requires.
ISSUES - A1734 CC enumerates the instances when a carrier
1. WON LOADSTAR is a common carrier under might be exempt from liability for the loss of the
Article 1732 CC goods.
2. Assuming it is a common carrier, WON proximate (1) Flood, storm, earthquake, lightning,
cause of the loss of cargo was not a fortuitous event or other natural disaster or calamity;
but was allegedly due to the failure of petitioner to (2) Act of the public enemy in war,
exercise extraordinary diligence whether international or civil;
(3) Act or omission of the shipper or
HELD owner of the goods;
1. YES (4) The character of the goods or defects
- A1732 CC defines a “common carrier” as in the packing or in the containers; and
follows: (5) Order or act of competent public
Common carriers are persons, corporations, firms authority
or associations engaged in the business of carrying - LOADSTAR claims that the loss of the goods was
or transporting passengers or goods or both, by due to a fortuitous event under paragraph 1. Yet, its
land, water, or air, for compensation, offering their claim is not substantiated. It is supported by
services to the public. evidence that the loss of the entire shipment of
- LOADSTAR is a corporation engaged in the cement was due to the gross negligence of
business of transporting cargo by water and for LOADSTAR
compensation, offering its services indiscriminately - Records show that in the evening of June 24, 1984,
to the public. Thus, without doubt, it is a common the sea and weather conditions in the vicinity of
carrier. Even if it entered into a voyage-charter Negros Occidental were calm. The records reveal
agreement with Northern Mindanao Transport that LOADSTAR took a shortcut route, instead of the
Company, Inc, it did not in any way convert the usual route, which exposed the voyage to
common carrier into a private carrier. unexpected hazard. LOADSTAR has only itself to
> Planters Products, Inc. v. CA - public carrier blame for its misjudgment.
shall remain as such, notwithstanding the charter of Disposition petition is DENIED
the whole or portion of a vessel by one or more
i

You might also like